Atty. Aquino Case Reviewers.docx

  • Uploaded by: Jed Sulit
  • 0
  • 0
  • November 2019
  • PDF

This document was uploaded by user and they confirmed that they have the permission to share it. If you are author or own the copyright of this book, please report to us by using this DMCA report form. Report DMCA


Overview

Download & View Atty. Aquino Case Reviewers.docx as PDF for free.

More details

  • Words: 131,035
  • Pages: 115
Pasei vs Drilon The petitioner, Philippine Association of Service Exporters, Inc. (PASEI, for short), a firm "engaged principally in the recruitment of Filipino workers, male and female, for overseas placement," 1 challenges the Constitutional validity of Department Order No. 1, Series of 1988, of the Department of Labor and Employment, in the character of "GUIDELINES GOVERNING THE TEMPORARY SUSPENSION OF DEPLOYMENT OF FILIPINO DOMESTIC AND HOUSEHOLD WORKERS," in this petition for certiorari and prohibition. Specifically, the measure is assailed for "discrimination against males or females;" 2 that it "does not apply to all Filipino workers but only to domestic helpers and females with similar skills;" 3 and that it is violative of the right to travel. The concept of police power is well-established in this jurisdiction. It has been defined as the "state authority to enact legislation that may interfere with personal liberty or property in order to promote the general welfare." 5 As defined, it consists of (1) an imposition of restraint upon liberty or property, (2) in order to foster the common good. It is not capable of an exact definition but has been, purposely, veiled in general terms to underscore its all-comprehensive embrace. "Its scope, ever-expanding to meet the exigencies of the times, even to anticipate the future where it could be done, provides enough room for an efficient and flexible response to conditions and circumstances thus assuring the greatest benefits." 6 It finds no specific Constitutional grant for the plain reason that it does not owe its origin to the Charter. Along with the taxing power and eminent domain, it is inborn in the very fact of statehood and sovereignty. It is a fundamental attribute of government that has enabled it to perform the most vital functions of governance. Marshall, to whom the expression has been credited, 7 refers to it succinctly as the plenary power of the State "to govern its citizens." Notwithstanding its extensive sweep, police power is not without its own limitations. For all its awesome consequences, it may not be exercised arbitrarily or unreasonably. Otherwise, and in that event, it defeats the purpose for which it is exercised, that is, to advance the public good. The petitioner has shown no satisfactory reason why the contested measure should be nullified. There is no question that Department Order No. 1 applies only to "female contract workers," 14 but it does not thereby make an undue discrimination between the sexes. It is wellsettled that "equality before the law" under the Constitution 15 does not import a perfect Identity of rights among all men and women. It admits of classifications, provided that (1) such classifications rest on substantial distinctions; (2) they are germane to the purposes of the law; (3) they are not confined to existing conditions; and (4) they apply equally to all members of the same class. 16 Ichong vs Hernandez Republic Act No. 1180 is entitled "An Act to Regulate the Retail Business." In effect it nationalizes the retail trade business. The main provisions of the Act are: (1) a prohibition against persons, not citizens of the Philippines, and against associations, partnerships, or corporations the capital of which are not wholly owned by citizens of the Philippines, from engaging directly or indirectly in the retail trade; (2) an exception from the above prohibition in favor of aliens actually engaged in said business on May 15, 1954, who are allowed to continue to engaged therein, unless their licenses are forfeited in accordance with the law, until their death or voluntary retirement in case of natural persons, and for ten years after the approval of the Act or until the expiration of term in case of juridical persons; (3) an exception therefrom in favor of citizens and juridical entities of the United States; (4) a provision for the forfeiture of licenses (to engage in the retail business) for violation of the laws on nationalization, control weights and measures and labor and other laws relating to trade, commerce and industry; (5) a prohibition against the establishment or opening by

aliens actually engaged in the retail business of additional stores or branches of retail business, (6) a provision requiring aliens actually engaged in the retail business to present for registration with the proper authorities a verified statement concerning their businesses, giving, among other matters, the nature of the business, their assets and liabilities and their offices and principal offices of judicial entities; and (7) a provision allowing the heirs of aliens now engaged in the retail business who die, to continue such business for a period of six months for purposes of liquidation. Petitioner attacks the constitutionality of the Act, contending that: (1) it denies to alien residents the equal protection of the laws and deprives of their liberty and property without due process of law ; (2) the subject of the Act is not expressed or comprehended in the title thereof; (3) the Act violates international and treaty obligations of the Republic of the Philippines; (4) the provisions of the Act against the transmission by aliens of their retail business thru hereditary succession, and those requiring 100% Filipino capitalization for a corporation or entity to entitle it to engage in the retail business, violate the spirit of Sections 1 and 5, Article XIII and Section 8 of Article XIV of the Constitution. It has been said the police power is so far - reaching in scope, that it has become almost impossible to limit its sweep. As it derives its existence from the very existence of the State itself, it does not need to be expressed or defined in its scope; it is said to be co-extensive with self-protection and survival, and as such it is the most positive and active of all governmental processes, the most essential, insistent and illimitable. b. Limitations on police power. — The basic limitations of due process and equal protection are found in the following provisions of our Constitution: SECTION 1.(1) No person shall be deprived of life, liberty or property without due process of law, nor any person be denied the equal protection of the laws. (Article III, Phil. Constitution) These constitutional guarantees which embody the essence of individual liberty and freedom in democracies, are not limited to citizens alone but are admittedly universal in their application, without regard to any differences of race, of color, or of nationality. ( c. The, equal protection clause. — The equal protection of the law clause is against undue favor and individual or class privilege, as well as hostile discrimination or the oppression of inequality. It is not intended to prohibit legislation, which is limited either in the object to which it is directed or by territory within which is to operate. It does not demand absolute equality among residents; it merely requires that all persons shall be treated alike, under like circumstances and conditions both as to privileges conferred and liabilities enforced. The equal protection clause is not infringed by legislation which applies only to those persons falling within a specified class, if it applies alike to all persons within such class, and reasonable grounds exists for making a distinction between those who fall within such class and those who do not. d. The due process clause. — The due process clause has to do with the reasonableness of legislation enacted in pursuance of the police power. Is there public interest, a public purpose; is public welfare involved? Is the Act reasonably necessary for the accomplishment of the legislature's purpose; is it not unreasonable, arbitrary or oppressive? Is there sufficient foundation or reason in connection with the matter involved; or has there not been a capricious use of the legislative power? Can the aims conceived be achieved by the means used, or is it not merely an unjustified interference with private interest? These are the questions that we ask when the due process test is applied.

The conflict, therefore, between police power and the guarantees of due process and equal protection of the laws is more apparent than real. Properly related, the power and the guarantees are supposed to coexist. The balancing is the essence or, shall it be said, the indispensable means for the attainment of legitimate aspirations of any democratic society. There can be no absolute power, whoever exercise it, for that would be tyranny. Yet there can neither be absolute liberty, for that would mean license and anarchy. There is a general feeling on the part of the public, which appears to be true to fact, about the controlling and dominant position that the alien retailer holds in the nation's economy. Food and other essentials, clothing, almost all articles of daily life reach the residents mostly through him. In big cities and centers of population he has acquired not only predominance, but apparent control over distribution of almost all kinds of goods, such as lumber, hardware, textiles, groceries, drugs, sugar, flour, garlic, and scores of other goods and articles. Lutz vs Araneta This case was initiated in the Court of First Instance of Negros Occidental to test the legality of the taxes imposed by Commonwealth Act No. 567, otherwise known as the Sugar Adjustment Act. Promulgated in 1940, the law in question opens (section 1) with a declaration of emergency, due to the threat to our industry by the imminent imposition of export taxes upon sugar as provided in the Tydings-McDuffe Act, and the "eventual loss of its preferential position in the United States market"; wherefore, the national policy was expressed "to obtain a readjustment of the benefits derived from the sugar industry by the component elements thereof" and "to stabilize the sugar industry so as to prepare it for the eventuality of the loss of its preferential position in the United States market and the imposition of the export taxes." Plaintiff, Walter Lutz, in his capacity as Judicial Administrator of the Intestate Estate of Antonio Jayme Ledesma, seeks to recover from the Collector of Internal Revenue the sum of P14,666.40 paid by the estate as taxes, under section 3 of the Act, for the crop years 1948-1949 and 1949-1950; alleging that such tax is unconstitutional and void, being levied for the aid and support of the sugar industry exclusively, which in plaintiff's opinion is not a public purpose for which a tax may be constitutioally levied. The action having been dismissed by the Court of First Instance, the plaintifs appealed the case directly to this Court (Judiciary Act, section 17). This Court can take judicial notice of the fact that sugar production is one of the great industries of our nation, sugar occupying a leading position among its export products; that it gives employment to thousands of laborers in fields and factories; that it is a great source of the state's wealth, is one of the important sources of foreign exchange needed by our government, and is thus pivotal in the plans of a regime committed to a policy of currency stability. Its promotion, protection and advancement, therefore redounds greatly to the general welfare. Hence it was competent for the legislature to find that the general welfare demanded that the sugar industry should be stabilized in turn; and in the wide field of its police power, the lawmaking body could provide that the distribution of benefits therefrom be readjusted among its components to enable it to resist the added strain of the increase in taxes that it had to sustain. Once it is conceded, as it must, that the protection and promotion of the sugar industry is a matter of public concern, it follows that the Legislature may determine within reasonable bounds what is necessary for its protection and expedient for its promotion. Here, the legislative discretion must be allowed fully play, subject only to the test of reasonableness; and it is not contended that the means provided in section 6 of the law (above quoted) bear no relation to the objective pursued or are oppressive in character. If objective and methods are alike constitutionally valid, no reason is seen why the state may not

levy taxes to raise funds for their prosecution and attainment. Taxation may be made the implement of the state's police power. Lozano vs Martinez The constitutionality of Batas Pambansa Bilang 22 (BP 22 for short), popularly known as the Bouncing Check Law, which was approved on April 3, 1979, is the sole issue presented by these petitions for decision. The question is definitely one of first impression in our jurisdiction. These petitions arose from cases involving prosecution of offenses under the statute. The defendants in those cases moved seasonably to quash the informations on the ground that the acts charged did not constitute an offense, the statute being unconstitutional. The language of BP 22 is broad enough to cover all kinds of checks, whether present dated or postdated, or whether issued in payment of pre-existing obligations or given in mutual or simultaneous exchange for something of value. BP 22 punishes a person "who makes or draws and issues any check on account or for value, knowing at the time of issue that he does not have sufficient funds in or credit with the drawee bank for the payment of said check in full upon presentment, which check is subsequently dishonored by the drawee bank for insufficiency of funds or credit or would have been dishonored for the same reason had not the drawer, without any valid reason, ordered the bank to stop payment." The penalty prescribed for the offense is imprisonment of not less than 30 days nor more than one year or a fine or not less than the amount of the check nor more than double said amount, but in no case to exceed P200,000.00, or both such fine and imprisonment at the discretion of the court. 3 An essential element of the offense is "knowledge" on the part of the maker or drawer of the check of the insufficiency of his funds in or credit with the bank to cover the check upon its presentment. Since this involves a state of mind difficult to establish, the statute itself creates a prima facie presumption of such knowledge where payment of the check "is refused by the drawee because of insufficient funds in or credit with such bank when presented within ninety (90) days from the date of the check. 5 To mitigate the harshness of the law in its application, the statute provides that such presumption shall not arise if within five (5) banking days from receipt of the notice of dishonor, the maker or drawer makes arrangements for payment of the check by the bank or pays the holder the amount of the check. BP 22 is aimed at putting a stop to or curbing the practice of issuing checks that are worthless, i.e. checks that end up being rejected or dishonored for payment. The practice, as discussed later, is proscribed by the state because of the injury it causes to t public interests. Before the enactment of BP 22, provisions already existed in our statute books which penalize the issuance of bouncing or rubber checks. Criminal law has dealth with the problem within the context of crimes against property punished as "estafa" or crimes involving fraud and deceit. The focus of these penal provisions is on the damage caused to the property rights of the victim. Among the constitutional objections raised against BP 22, the most serious is the alleged conflict between the statute and the constitutional provision forbidding imprisonment for debt. It is contended that the statute runs counter to the inhibition in the Bill of Rights which states, "No person shall be imprisoned for debt or non-payment of a poll tax." 16 Petitioners insist that, since the offense under BP 22 is consummated only upon the dishonor or non-payment of the check when it is presented to the drawee bank, the statute is really a "bad debt law" rather than a "bad check law." What it punishes is the nonpayment of the check, not the act of issuing it. The statute, it is claimed, is nothing more than a veiled device to coerce payment of a debt under the threat of penal sanction.

"The 'debt' intended to be covered by the constitutional guaranty has a well-defined meaning. Organic provisions relieving from imprisonment for debt, were intended to prevent commitment of debtors to prison for liabilities arising from actions ex contractu The inhibition was never meant to include damages arising in actions ex delicto, for the reason that damages recoverable therein do not arise from any contract entered into between the parties but are imposed upon the defendant for the wrong he has done and are considered as punishment, nor to fines and penalties imposed by the courts in criminal proceedings as punishments for crime." t "one of the purposes of the law is to suppress possible abuses on the part of the employers who hire laborers or employees without paying them the salaries agreed upon for their services, thus causing them financial difficulties. "The law was viewed not as a measure to coerce payment of an obligation, although obviously such could be its effect, but to banish a practice considered harmful to public welfare. The gravamen of the offense punished by BP 22 is the act of making and issuing a worthless check or a check that is dishonored upon its presentation for payment. It is not the non-payment of an obligation which the law punishes. The law is not intended or designed to coerce a debtor to pay his debt. The thrust of the law is to prohibit, under pain of penal sanctions, the making of worthless checks and putting them in circulation. Because of its deleterious effects on the public interest, the practice is proscribed by the law. The law punishes the act not as an offense against property, but an offense against public order. It may be constitutionally impermissible for the legislature to penalize a person for non-payment of a debt ex contractu But certainly it is within the prerogative of the lawmaking body to proscribe certain acts deemed pernicious and inimical to public welfare. Acts mala in se are not the only acts which the law can punish. An act may not be considered by society as inherently wrong, hence, not malum in se but because of the harm that it inflicts on the community, it can be outlawed and criminally punished as malum prohibitum. The state can do this in the exercise of its police power. DECS vs San Diego The issue before us is mediocrity. The question is whether a person who has thrice failed the National Medical Admission Test (NMAT) is entitled to take it again. The petitioner contends he may not, under its rule thath) A student shall be allowed only three (3) chances to take the NMAT. After three (3) successive failures, a student shall not be allowed to take the NMAT for the fourth time. The private respondent is a graduate of the University of the East with a degree of Bachelor of Science in Zoology. The petitioner claims that he took the NMAT three times and flunked it as many times.1 When he applied to take it again, the petitioner rejected his application on the basis of the aforesaid rule. He then went to the Regional Trial Court of Valenzuela, Metro Manila, to compel his admission to the test. In his original petition for mandamus, he first invoked his constitutional rights to academic freedom and quality education. After hearing, the respondent judge rendered a decision on July 4, 1989, declaring the challenged order invalid and granting the petition. Judge Teresita Dizon-Capulong held that the petitioner had been deprived of his right to pursue a medical education through an arbitrary exercise of the police power. 3 We cannot sustain the respondent judge. Her decision must be reversed. In Tablarin v. Gutierrez, 4 this Court upheld the constitutionality of the NMAT as a measure intended to limit the admission to medical schools

only to those who have initially proved their competence and preparation for a medical education. There is no need to redefine here the police power of the State. Suffice it to repeat that the power is validly exercised if (a) the interests of the public generally, as distinguished from those of a particular class, require the interference of the State, and (b) the means employed are reasonably necessary to the attainment of the object sought to be accomplished and not unduly oppressive upon individuals.5

In other words, the proper exercise of the police power requires the concurrence of a lawful subject and a lawful method. While every person is entitled to aspire to be a doctor, he does not have a constitutional right to be a doctor. This is true of any other calling in which the public interest is involved; and the closer the link, the longer the bridge to one's ambition. The State has the responsibility to harness its human resources and to see to it that they are not dissipated or, no less worse, not used at all. These resources must be applied in a manner that will best promote the common good while also giving the individual a sense of satisfaction. A person cannot insist on being a physician if he will be a menace to his patients. If one who wants to be a lawyer may prove better as a plumber, he should be so advised and adviced. Of course, he may not be forced to be a plumber, but on the other hand he may not force his entry into the bar. By the same token, a student who has demonstrated promise as a pianist cannot be shunted aside to take a course in nursing, however appropriate this career may be for others. The right to quality education invoked by the private respondent is not absolute. The Constitution also provides that "every citizen has the right to choose a profession or course of study, subject to fair, reasonable and equitable admission and academic requirements.6 The private respondent must yield to the challenged rule and give way to those better prepared. Where even those who have qualified may still not be accommodated in our already crowded medical schools, there is all the more reason to bar those who, like him, have been tested and found wanting. Restituto Ynot Vs IAC On January 13, 1984, the petitioner transported six carabaos in a pump boat from Masbate to Iloilo when the same was confiscated by the police station commander of Barotac Nuevo, Iloilo for the violation of E.O. 626-A. A case was filed by the petitioner questioning the constitutionality of executive order and the recovery of the carabaos. After considering the merits of the case, the confiscation was sustained and the court declined to rule on the constitutionality issue. The petitioner appealed the decision to the Intermediate Appellate Court but it also upheld the ruling of RTC. Issue: Is E.O. 626-A unconstitutional? Ruling: The Respondent contends that it is a valid exercise of police power to justify EO 626-A amending EO 626 in asic rule prohibiting the slaughter of carabaos except under certain conditions. The supreme court said that The reasonable connection between the means employed and the purpose sought to be achieved by the questioned measure is missing the Supreme Court do not see how the prohibition of the inter-provincial transport of carabaos can prevent their indiscriminate slaughter, considering that they can be killed anywhere, with no less difficulty in one province than in another. Obviously, retaining the carabaos in one province will not prevent their slaughter there, any more than moving them to another province will make it easier to kill them there The Supreme Court found E.O. 626-A unconstitutional. The executive act defined the prohibition, convicted the petitioner and immediately

imposed punishment, which was carried out forthright. Due process was not properly observed. In the instant case, the carabaos were arbitrarily confiscated by the police station commander, were returned to the petitioner only after he had filed a complaint for recovery and given a supersedeas bond of P12,000.00. The measure struck at once and pounced upon the petitioner without giving him a chance to be heard, thus denying due process.

There is no reasonable relation between the setting aside of at least six (6) percent of the total area of an private cemeteries for charity burial grounds of deceased paupers and the promotion of health, morals, good order, safety, or the general welfare of the people. The ordinance is actually a taking without compensation of a certain area from a private cemetery to benefit paupers who are charges of the municipal corporation. Instead of building or maintaining a public cemetery for this purpose, the city passes the burden to private cemeteries.

Quezon City vs Ericta This is a petition for review which seeks the reversal of the decision of the Court of First Instance of Rizal, Branch XVIII declaring Section 9 of Ordinance No. 6118, S-64, of the Quezon City Council null and void. Section 9 of Ordinance No. 6118, S-64, entitled "ORDINANCE REGULATING THE ESTABLISHMENT, MAINTENANCE AND OPERATION OF PRIVATE MEMORIAL TYPE CEMETERY OR BURIAL GROUND WITHIN THE JURISDICTION OF QUEZON CITY AND PROVIDING PENALTIES FOR THE VIOLATION THEREOF" provides: Sec. 9. At least six (6) percent of the total area of the memorial park cemetery shall be set aside for charity burial of deceased persons who are paupers and have been residents of Quezon City for at least 5 years prior to their death, to be determined by competent City Authorities. The area so designated shall immediately be developed and should be open for operation not later than six months from the date of approval of the application. Petitioners argue that the taking of the respondent's property is a valid and reasonable exercise of police power and that the land is taken for a public use as it is intended for the burial ground of paupers. They further argue that the Quezon City Council is authorized under its charter, in the exercise of local police power, " to make such further ordinances and resolutions not repugnant to law as may be necessary to carry into effect and discharge the powers and duties conferred by this Act and such as it shall deem necessary and proper to provide for the health and safety, promote the prosperity, improve the morals, peace, good order, comfort and convenience of the city and the inhabitants thereof, and for the protection of property therein." On the other hand, respondent Himlayang Pilipino, Inc. contends that the taking or confiscation of property is obvious because the questioned ordinance permanently restricts the use of the property such that it cannot be used for any reasonable purpose and deprives the owner of all beneficial use of his property. The power to regulate does not include the power to prohibit (People vs. Esguerra, 81 PhiL 33, Vega vs. Municipal Board of Iloilo, L-6765, May 12, 1954; 39 N.J. Law, 70, Mich. 396). A fortiori, the power to regulate does not include the power to confiscate. The ordinance in question not only confiscates but also prohibits the operation of a memorial park cemetery, because under Section 13 of said ordinance, 'Violation of the provision thereof is punishable with a fine and/or imprisonment and that upon conviction thereof the permit to operate and maintain a private cemetery shall be revoked or cancelled.' Police power is defined by Freund as 'the power of promoting the public welfare by restraining and regulating the use of liberty and property' (Quoted in Political Law by Tanada and Carreon, V-11, p. 50). It is usually exerted in order to merely regulate the use and enjoyment of property of the owner. If he is deprived of his property outright, it is not taken for public use but rather to destroy in order to promote the general welfare. In police power, the owner does not recover from the government for injury sustained in consequence thereof (12 C.J. 623). It has been said that police power is the most essential of government powers, at times the most insistent, and always one of the least limitable of the powers of government

The expropriation without compensation of a portion of private cemeteries is not covered by Section 12(t) of Republic Act 537, the Revised Charter of Quezon City which empowers the city council to prohibit the burial of the dead within the center of population of the city and to provide for their burial in a proper place subject to the provisions of general law regulating burial grounds and cemeteries. Association of Small Landowners vs Secretary of Agrarian Reform In ancient mythology, Antaeus was a terrible giant who blocked and challenged Hercules for his life on his way to Mycenae after performing his eleventh labor. The two wrestled mightily and Hercules flung his adversary to the ground thinking him dead, but Antaeus rose even stronger to resume their struggle. This happened several times to Hercules' increasing amazement. Finally, as they continued grappling, it dawned on Hercules that Antaeus was the son of Gaea and could never die as long as any part of his body was touching his Mother Earth. Thus forewarned, Hercules then held Antaeus up in the air, beyond the reach of the sustaining soil, and crushed him to death. Mother Earth. The sustaining soil. The giver of life, without whose invigorating touch even the powerful Antaeus weakened and died. These are four consolidated cases questioning the constitutionality of the Comprehensive Agrarian Reform Act (R.A. No. 6657 and related laws i.e., Agrarian Land Reform Code or R.A. No. 3844). Brief background: Article XIII of the Constitution on Social Justice and Human Rights includes a call for the adoption by the State of an agrarian reform program. The State shall, by law, undertake an agrarian reform program founded on the right of farmers and regular farmworkers, who are landless, to own directly or collectively the lands they till or, in the case of other farmworkers, to receive a just share of the fruits thereof. RA 3844 was enacted in 1963. P.D. No. 27 was promulgated in 1972 to provide for the compulsory acquisition of private lands for distribution among tenant-farmers and to specify maximum retention limits for landowners. In 1987, President Corazon Aquino issued E.O. No. 228, declaring full land ownership in favor of the beneficiaries of PD 27 and providing for the valuation of still unvalued lands covered by the decree as well as the manner of their payment. In 1987, P.P. No. 131, instituting a comprehensive agrarian reform program (CARP) was enacted; later, E.O. No. 229, providing the mechanics for its (PP131’s) implementation, was also enacted. Afterwhich is the enactment of R.A. No. 6657, Comprehensive Agrarian Reform Law in 1988. This law, while considerably changing the earlier mentioned enactments, nevertheless gives them suppletory effect insofar as they are not inconsistent with its provisions. [Two of the consolidated cases are discussed below] G.R. No. 78742: (Association of Small Landowners vs Secretary) The Association of Small Landowners in the Philippines, Inc. sought exception from the land distribution scheme provided for in R.A. 6657. The Association is comprised of landowners of ricelands and cornlands whose landholdings do not exceed 7 hectares. They invoke that since their landholdings are less than 7 hectares, they should not be forced to distribute their land to their tenants under R.A. 6657 for they themselves have shown willingness to till their own land. In short, they want to be exempted from agrarian reform program because they claim to belong to a different class.1

G.R. No. 79777: (Manaay vs Juico) Nicolas Manaay questioned the validity of the agrarian reform laws (PD 27, EO 228, and 229) on the ground that these laws already valuated their lands for the agrarian reform program and that the specific amount must be determined by the Department of Agrarian Reform (DAR). Manaay averred that this violated the principle in eminent domain which provides that only courts can determine just compensation. This, for Manaay, also violated due process for under the constitution, no property shall be taken for public use without just compensation. Manaay also questioned the provision which states that landowners may be paid for their land in bonds and not necessarily in cash. Manaay averred that just compensation has always been in the form of money and not in bonds. There are traditional distinctions between the police power and the power of eminent domain that logically preclude the application of both powers at the same time on the same subject. In the case of City of Baguio v. NAWASA, 24 for example, where a law required the transfer of all municipal waterworks systems to the NAWASA in exchange for its assets of equivalent value, the Court held that the power being exercised was eminent domain because the property involved was wholesome and intended for a public use. Property condemned under the police power is noxious or intended for a noxious purpose, such as a building on the verge of collapse, which should be demolished for the public safety, or obscene materials, which should be destroyed in the interest of public morals. The confiscation of such property is not compensable, unlike the taking of property under the power of expropriation, which requires the payment of just compensation to the owner. Every restriction upon the use of property imposed in the exercise of the police power deprives the owner of some right theretofore enjoyed, and is, in that sense, an abridgment by the State of rights in property without making compensation. But restriction imposed to protect the public health, safety or morals from dangers threatened is not a taking. The restriction here in question is merely the prohibition of a noxious use. The property so restricted remains in the possession of its owner. The state does not appropriate it or make any use of it. The state merely prevents the owner from making a use which interferes with paramount rights of the public. Whenever the use prohibited ceases to be noxious — as it may because of further changes in local or social conditions — the restriction will have to be removed and the owner will again be free to enjoy his property as heretofore. Eminent domain is an inherent power of the State that enables it to forcibly acquire private lands intended for public use upon payment of just compensation to the owner. Obviously, there is no need to expropriate where the owner is willing to sell under terms also acceptable to the purchaser, in which case an ordinary deed of sale may be agreed upon by the parties. 35 It is only where the owner is unwilling to sell, or cannot accept the price or other conditions offered by the vendee, that the power of eminent domain will come into play to assert the paramount authority of the State over the interests of the property owner. Private rights must then yield to the irresistible demands of the public interest on the time-honored justification, as in the case of the police power, that the welfare of the people is the supreme law. But for all its primacy and urgency, the power of expropriation is by no means absolute (as indeed no power is absolute). The limitation is found in the constitutional injunction that "private property shall not be taken for public use without just compensation" and in the abundant jurisprudence that has evolved from the interpretation of this principle. Basically, the requirements for a proper exercise of the power are: (1) public use and (2) just compensation. As held in Republic of the Philippines v. Castellvi, 42 there is compensable taking when the following conditions concur: (1) the

expropriator must enter a private property; (2) the entry must be for more than a momentary period; (3) the entry must be under warrant or color of legal authority; (4) the property must be devoted to public use or otherwise informally appropriated or injuriously affected; and (5) the utilization of the property for public use must be in such a way as to oust the owner and deprive him of beneficial enjoyment of the property. All these requisites are envisioned in the measures before us. Manila Memorial Park vs DSWD Before us is a Petition for Prohibition2 under Rule 65 of the Rules of Court filed by petitioners Manila Memorial Park, Inc. and La Funeraria Paz-Sucat, Inc., domestic corporations engaged in the business of providing funeral and burial services, against public respondents Secretaries of the Department of Social Welfare and Development (DSWD) and the Department of Finance (DOF). Petitioners assail the constitutionality of Section 4 of Republic Act (RA) No. 7432,3 as amended by RA 9257,4 and the implementing rules and regulations issued by the DSWD and DOF insofar as these allow business establishments to claim the 20% discount given to senior citizens as a tax deduction. Factual Antecedents On April 23, 1992, RA 7432 was passed into law, granting senior citizens the following privileges: SECTION 4. Privileges for the Senior Citizens. – The senior citizens shall be entitled to the following: a) the grant of twenty percent (20%) discount from all establishments relative to utilization of transportation services, hotels and similar lodging establishment[s], restaurants and recreation centers and purchase of medicine anywhere in the country: Provided, That private establishments may claim the cost as tax credit; b) a minimum of twenty percent (20%) discount on admission fees charged by theaters, cinema houses and concert halls, circuses, carnivals and other similar places of culture, leisure, and amusement; c) exemption from the payment of individual income taxes: Provided, That their annual taxable income does not exceed the property level as determined by the National Economic and Development Authority (NEDA) for that year; d) exemption from training fees for socioeconomic programs undertaken by the OSCA as part of its work; e) free medical and dental services in government establishment[s] anywhere in the country, subject to guidelines to be issued by the Department of Health, the Government Service Insurance System and the Social Security System; f) to the extent practicable and feasible, the continuance of the same benefits and privileges given by the Government Service Insurance System (GSIS), Social Security System (SSS) and PAG-IBIG, as the case may be, as are enjoyed by those in actual service. On August 23, 1993, Revenue Regulations (RR) No. 02-94 was issued to implement RA 7432. Sections 2(i) and 4 of RR No. 02-94 provide: Sec. 2. DEFINITIONS. – For purposes of these regulations: i. Tax Credit – refers to the amount representing the 20% discount granted to a qualified senior citizen by all establishments relative to their utilization of transportation services, hotels and similar lodging establishments, restaurants, drugstores, recreation centers, theaters, cinema houses, concert halls, circuses, carnivals and other similar places of culture, leisure and amusement, which discount shall be deducted by the said establishments from their gross income for income tax purposes and from their gross sales for value-added tax or other percentage tax purposes. x x x x Sec. 4.

RECORDING/BOOKKEEPING REQUIREMENTS FOR PRIVATE ESTABLISHMENTS. – Private establishments, i.e., transport services, hotels and similar lodging establishments, restaurants, recreation centers, drugstores, theaters, cinema houses, concert halls, circuses, carnivals and other similar places of culture[,] leisure and amusement, giving 20% discounts to qualified senior citizens are required to keep separate and accurate record[s] of sales made to senior citizens, which shall include the name, identification number, gross sales/receipts, discounts, dates of transactions and invoice number for every transaction. The amount of 20% discount shall be deducted from the gross income for income tax purposes and from gross sales of the business enterprise concerned for purposes of the VAT and other percentage taxes. FACTS: RA 7432 was passed into law (amended by RA 9257), granting senior citizens 20% discount on certain establishments. To implement the tax provisions of RA 9257, the Secretary of Finance and the DSWD issued its own Rules and Regulations.

Facts:1. Petitioner was the governor of Rizal, filed a petition assailing the validity of R.A. 920 which contains an item providing for an appropriation of P85,000.00 for the construction and repair of a feeder road in Pasig. The said law was passed in Congress and approved by the President. 2. The property over which the feeder road will be constructed is however owned by Sen. Zulueta. The property was to be donated to the local government, though the donation was made a few months after the appropriation was included in RA 920. The petition alleged that the said planned feeder road would relieve Zulueta the responsibility of improving the road which is inside a private subdivision. 3. The lower court (RTC) ruled that the petitioner has standing to assail the validity of RA 920, due to the public interest involved in the appropriation. However, he does not have a standing with respect to the donation since he does not have an interest that will be injured by said donation, hence it dismissed the petition. Issue: Whether or not the petitioner has the standing to file the petition

Hence, this petition. Petitioners are not questioning the 20% discount granted to senior citizens but are only assailing the constitutionality of the tax deduction scheme prescribed under RA 9257 and the implementing rules and regulations issued by the DSWD and the DOF. Petitioners posit that the tax deduction scheme contravenes Article III, Section 9 of the Constitution, which provides that: "private property shall not be taken for public use without just compensation."

YES. 1. Petitioner has standing. He is not merely a taxpayer but the governor of the province of Rizal which is considered one of the most populated biggest provinces during that time, its taxpayers bear a substantial portion of the burden of taxation in the country.

ISSUE: Whether the legally mandated 20% senior citizen discount is an exercise of police power or eminent domain.

2. Public funds can only be appropriated for a public purpose. The test of the constitutionality of a statute requiring the use of public funds is whether it is used to promote public interest. Moreover, the validity of a stature depends on the powers of the Congress at the time of its passage or approval, not upon events occurring, or acts performed subsequent thereto, unless it is an amendment of the organic law.

RULING: The 20% senior citizen discount is an exercise of police power.

legislature is without power to appropriate public revenue for anything but a public purpose

It may not always be easy to determine whether a challenged governmental act is an exercise of police power or eminent domain. The judicious approach, therefore, is to look at the nature and effects of the challenged governmental act and decide on the basis thereof.

a general rule that the legislature is without power to appropriate public revenue for anything but a public purpose. . . . It is the essential character of the direct object of the expenditure which must determine its validity as justifying a tax, and not the magnitude of the interest to be affected nor the degree to which the general advantage of the community, and thus the public welfare, may be ultimately benefited by their promotion. Incidental to the public or to the state, which results from the promotion of private interest and the prosperity of private enterprises or business, does not justify their aid by the use public money

Respondents maintain that the tax deduction scheme is a legitimate exercise of the State’s police power.

The 20% discount is intended to improve the welfare of senior citizens who, at their age, are less likely to be gainfully employed, more prone to illnesses and other disabilities, and, thus, in need of subsidy in purchasing basic commodities. It serves to honor senior citizens who presumably spent their lives on contributing to the development and progress of the nation. In turn, the subject regulation affects the pricing, and, hence, the profitability of a private establishment. The subject regulation may be said to be similar to, but with substantial distinctions from, price control or rate of return on investment control laws which are traditionally regarded as police power measures. The subject regulation differs there from in that (1) the discount does not prevent the establishments from adjusting the level of prices of their goods and services, and (2) the discount does not apply to all customers of a given establishment but only to the class of senior citizens. Nonetheless, to the degree material to the resolution of this case, the 20% discount may be properly viewed as belonging to the category of price regulatory measures which affect the profitability of establishments subjected thereto. On its face, therefore, the subject regulation is a police power measure. Pascual vs. Secretary of Public Works

In accordance with the rule that the taxing power must be exercised for public purposes only, discussed supra sec. 14, money raised by taxation can be expended only for public purposes and not for the advantage of private individuals. (85 C.J.S. pp. 645-646; emphasis supplied.) Explaining the reason underlying said rule, Corpus Juris Secundum states: Generally, under the express or implied provisions of the constitution, public funds may be used only for public purpose. The right of the legislature to appropriate funds is correlative with its right to tax, and, under constitutional provisions against taxation except for public purposes and prohibiting the collection of a tax for one purpose and the devotion thereof to another purpose, no appropriation of state funds can be made for other than for a public purpose. The test of the constitutionality of a statute requiring the use of public funds is whether the statute is designed to promote the public interest, as opposed to the furtherance of the advantage of individuals, although each advantage to individuals might incidentally serve the public.

Punsalan vs. Municipal Board of Manila [G.R. No. L-4817, May 26, 1954] This suit was commenced in the Court of First Instance of Manila by two lawyers, a medical practitioner, a public accountant, a dental surgeon and a pharmacist, purportedly "in their own behalf and in behalf of other professionals practising in the City of Manila who may desire to join it." Object of the suit is the annulment of Ordinance No. 3398 of the City of Manila together with the provision of the Manila charter authorizing it and the refund of taxes collected under the ordinance but paid under protest. The ordinance in question, which was approved by the municipal board of the City of Manila on July 25, 1950, imposes a municipal occupation tax on persons exercising various professions in the city and penalizes non-payment of the tax "by a fine of not more than two hundred pesos or by imprisonment of not more than six months, or by both such fine and imprisonment in the discretion of the court." Among the professions taxed were those to which plaintiffs belong. The ordinance was enacted pursuant to paragraph (1) of section 18 of the Revised Charter of the City of Manila (as amended by Republic Act No. 409), which empowers the Municipal Board of said city to impose a municipal occupation tax, not to exceed P50 per annum, on persons engaged in the various professions above referred to. Having already paid their occupation tax under section 201 of the National Internal Revenue Code, plaintiffs, upon being required to pay the additional tax prescribed in the ordinance, paid the same under protest and then brought the present suit for the purpose already stated. We do not think it is for the courts to judge what particular cities or municipalities should be empowered to impose occupation taxes in addition to those imposed by the National Government. That matter is peculiarly within the domain of the political departments and the courts would do well not to encroach upon it. Moreover, as the seat of the National Government and with a population and volume of trade many times that of any other Philippine city or municipality, Manila, no doubt, offers a more lucrative field for the practice of the professions, so that it is but fair that the professionals in Manila be made to pay a higher occupation tax than their brethren in the provinces. Double Taxation The argument against double taxation may not be invoked where one tax is imposed by the state and the other is imposed by the city (1 Cooley on Taxation, 4th ed., p. 492), it being widely recognized that there is nothing inherently obnoxious in the requirement that license fees or taxes be exacted with respect to the same occupation, calling or activity by both the state and the political subdivisions thereof Lladoc vs. Commissioner of Internal Revenue Sometime in 1957, the M.B. Estate, Inc., of Bacolod City, donated P10,000.00 in cash to Rev. Fr. Crispin Ruiz, then parish priest of Victorias, Negros Occidental, and predecessor of herein petitioner, for the construction of a new Catholic Church in the locality. The total amount was actually spent for the purpose intended.

among others, that at the time of the donation, he was not the parish priest in Victorias; that there is no legal entity or juridical person known as the "Catholic Parish Priest of Victorias," and, therefore, he should not be liable for the donee's gift tax. It was also asserted that the assessment of the gift tax, even against the Roman Catholic Church, would not be valid, for such would be a clear violation of the provisions of the Constitution. Tax Exemptions It is a cardinal rule in taxation that exemptions from payment thereof are highly disfavored by law, and the party claiming exemption must justify his claim by a clear, positive, or express grant of such privilege by law The phrase "exempt from taxation" as employed in Section 22(3), Article VI of the Constitution of the Philippines, should not be interpreted to mean exemption from all kinds of taxes. Statutes exempting charitable and religious property from taxation should be construed fairly though strictly and in such manner as to give effect to the main intent of the lawmakers. Section 22 (3), Art. VI of the Constitution of the Philippines, exempts from taxation cemeteries, churches and parsonages or convents, appurtenant thereto, and all lands, buildings, and improvements used exclusively for religious purposes. The exemption is only from the payment of taxes assessed on such properties enumerated, as property taxes, as contra distinguished from excise taxes. In the present case, what the Collector assessed was a donee's gift tax; the assessment was not on the properties themselves. It did not rest upon general ownership; it was an excise upon the use made of the properties, upon the exercise of the privilege of receiving the properties (Phipps vs. Com. of Int. Rec. 91 F 2d 627). Manifestly, gift tax is not within the exempting provisions of the section just mentioned. A gift tax is not a property tax, but an excise tax imposed on the transfer of property by way of gift inter vivos, the imposition of which on property used exclusively for religious purposes, does not constitute an impairment of the Constitution. As well observed by the learned respondent Court, the phrase "exempt from taxation," as employed in the Constitution (supra) should not be interpreted to mean exemption from all kinds of taxes. And there being no clear, positive or express grant of such privilege by law, in favor of petitioner, the exemption herein must be denied. Abra Valley College vs. Aquino Abra Valley College is an educational corporation and institution of higher learning in Bangued, Abra. In 1974, the CFI ordered for the seizure and sale of the subject school property for non-payment of real estate taxes and penalties. Private respondents stated that the college lot and building in question are not only used for educational purposes of the college, but also as the permanent residence of the President and Director, Mr. Pedro V. Borgonia, and his family including his in-laws and grandchildren; while the ground floor of the college building is being used and rented by a commercial establishment. Abra Valley College vs. Aquino [G.R. No. L-39086, June 15, 1988]

On March 3, 1958, the donor M.B. Estate, Inc., filed the donor's gift tax return. Under date of April 29, 1960, the respondent Commissioner of Internal Revenue issued an assessment for donee's gift tax against the Catholic Parish of Victorias, Negros Occidental, of which petitioner was the priest. The tax amounted to P1,370.00 including surcharges, interests of 1% monthly from May 15, 1958 to June 15, 1960, and the compromise for the late filing of the return.

Due to its time frame, the constitutional provision which finds application in the case at bar is Section 22, paragraph 3, Article VI, of the then 1935 Philippine Constitution, which expressly grants exemption from realty taxes for "Cemeteries, churches and parsonages or convents appurtenant thereto, and all lands, buildings, and improvements used exclusively for religious, charitable or educational purposes ...

Petitioner lodged a protest to the assessment and requested the withdrawal thereof. The protest and the motion for reconsideration presented to the Commissioner of Internal Revenue were denied. The petitioner appealed to the Court of Tax Appeals on November 2, 1960. In the petition for review, the Rev. Fr. Casimiro Lladoc claimed,

Relative thereto, Section 54, paragraph c, Commonwealth Act No. 470 as amended by Republic Act No. 409, otherwise known as the Assessment Law, provides:

The following are exempted from real property tax under the Assessment Law:

praphrase a recent decision, taxes being the lifeblood of the government, their prompt and certain availability is of the essence. 12

(c) churches and parsonages or convents appurtenant thereto, and all lands, buildings, and improvements used exclusively for religious, charitable, scientific or educational purposes.

Limitations

Taxation and Incidental Use Moreover, the exemption in favor of property used exclusively for charitable or educational purposes is 'not limited to property actually indispensable' therefor (Cooley on Taxation, Vol. 2, p. 1430), but extends to facilities which are incidental to and reasonably necessary for the accomplishment of said purposes, such as in the case of hospitals, "a school for training nurses, a nurses' home, property use to provide housing facilities for interns, resident doctors, superintendents, and other members of the hospital staff, and recreational facilities for student nurses, interns, and residents' The test of exemption from taxation is the use of the property for purposes mentioned in the Constitution (Apostolic Prefect v. City Treasurer of Baguio, 71 Phil, 547 [1941]). It must be stressed however, that while this Court allows a more liberal and non-restrictive interpretation of the phrase "exclusively used for educational purposes" as provided for in Article VI, Section 22, paragraph 3 of the 1935 Philippine Constitution, reasonable emphasis has always been made that exemption extends to facilities which are incidental to and reasonably necessary for the accomplishment of the main purposes. Otherwise stated, the use of the school building or lot for commercial purposes is neither contemplated by law, nor by jurisprudence. Thus, while the use of the second floor of the main building in the case at bar for residential purposes of the Director and his family, may find justification under the concept of incidental use, which is complimentary to the main or primary purpose—educational, the lease of the first floor thereof to the Northern Marketing Corporation cannot by any stretch of the imagination be considered incidental to the purpose of education. Sison vs. Ancheta The success of the challenge posed in this suit for declaratory relief or prohibition proceeding 1 on the validity of Section I of Batas Pambansa Blg. 135 depends upon a showing of its constitutional infirmity. The assailed provision further amends Section 21 of the National Internal Revenue Code of 1977, which provides for rates of tax on citizens or residents on (a) taxable compensation income, (b) taxable net income, (c) royalties, prizes, and other winnings, (d) interest from bank deposits and yield or any other monetary benefit from deposit substitutes and from trust fund and similar arrangements, (e) dividends and share of individual partner in the net profits of taxable partnership, (f) adjusted gross income. 2 Petitioner 3 as taxpayer alleges that by virtue thereof, "he would be unduly discriminated against by the imposition of higher rates of tax upon his income arising from the exercise of his profession vis-a-vis those which are imposed upon fixed income or salaried individual taxpayers. 4 He characterizes the above sction as arbitrary amounting to class legislation, oppressive and capricious in character 5 Power to Tax Chief Justice Makalintal thus: "The areas which used to be left to private enterprise and initiative and which the government was called upon to enter optionally, and only 'because it was better equipped to administer for the public welfare than is any private individual or group of individuals,' continue to lose their well-defined boundaries and to be absorbed within activities that the government must undertake in its sovereign capacity if it is to meet the increasing social challenges of the times." 11 Hence the need for more revenues. The power to tax, an inherent prerogative, has to be availed of to assure the performance of vital state functions. It is the source of the bulk of public funds. To

The power to tax moreover, to borrow from Justice Malcolm, "is an attribute of sovereignty. It is the strongest of all the powers of of government." 13 It is, of course, to be admitted that for all its plenitude 'the power to tax is not unconfined. There are restrictions. The Constitution sets forth such limits . Adversely affecting as it does properly rights, both the due process and equal protection clauses may properly be invoked, all petitioner does, to invalidate in appropriate cases a revenue measure. if it were otherwise, there would -be truth to the 1803 dictum of Chief Justice Marshall that "the power to tax involves the power to destroy." . It is undoubted that the due process clause may be invoked where a taxing statute is so arbitrary that it finds no support in the Constitution. An obvious example is where it can be shown to amount to the confiscation of property. That would be a clear abuse of power. It then becomes the duty of this Court to say that such an arbitrary act amounted to the exercise of an authority not conferred. That properly calls for the application of the Holmes dictum. It has also been held that where the assailed tax measure is beyond the jurisdiction of the state, or is not for a public purpose, or, in case of a retroactive statute is so harsh and unreasonable, it is subject to attack on due process grounds. 19 equal protection. The applicable standard to avoid the charge that there is a denial of this constitutional mandate whether the assailed act is in the exercise of the police power or the power of eminent domain is to demonstrated that the governmental act assailed, far from being inspired by the attainment of the common weal was prompted by the spirit of hostility, or at the very least, discrimination that finds no support in reason. It suffices then that the laws operate equally and uniformly on all persons under similar circumstances or that all persons must be treated in the same manner, the conditions not being different, both in the privileges conferred and the liabilities imposed. Favoritism and undue preference cannot be allowed. For the principle is that equal protection and security shall be given to every person under circumstances which if not Identical are analogous. If law be looked upon in terms of burden or charges, those that fall within a class should be treated in the same fashion, whatever restrictions cast on some in the group equally binding on the rest." 20 That same formulation applies as well to taxation measures remedies. The Constitution does not require things which are different in fact or opinion to be treated in law as though they were the same." 21 Hence the constant reiteration of the view that classification if rational in character is allowable. As a matter of fact, in a leading case of Lutz V. Araneta, 22 this Court, through Justice J.B.L. Reyes, went so far as to hold "at any rate, it is inherent in the power to tax that a state be free to select the subjects of taxation, and it has been repeatedly held that 'inequalities which result from a singling out of one particular class for taxation, or exemption infringe no constitutional limitation.'" 7. Petitioner likewise invoked the kindred concept of uniformity. According to the Constitution: "The rule of taxation shag be uniform and equitable." 24 This requirement is met according to Justice Laurel in Philippine Trust Company v. Yatco,25 decided in 1940, when the tax "operates with the same force and effect in every place where the subject may be found. " 26 He likewise added: "The rule of uniformity does not call for perfect uniformity or perfect equality, because this is hardly attainable." 27 The problem of classification did not present itself in that case. It did not arise until nine years later, when the Supreme Court held: "Equality and uniformity in taxation means that all taxable articles or kinds of property of the same class shall be taxed at the same rate. The taxing power has the authority to make reasonable and natural classifications for purposes of taxation, ... . 28 As clarified

by Justice Tuason, where "the differentiation" complained of "conforms to the practical dictates of justice and equity” Taxpayers may be classified into different categories There is no legal objection to a broader tax base or taxable income by eliminating all deductible items and at the same time reducing the applicable tax rate. Taxpayers may be classified into different categories. To repeat, it. is enough that the classification must rest upon substantial distinctions that make real differences. In the case of the gross income taxation embodied in Batas Pambansa Blg. 135, the, discernible basis of classification is the susceptibility of the income to the application of generalized rules removing all deductible items for all taxpayers within the class and fixing a set of reduced tax rates to be applied to all of them. Taxpayers who are recipients of compensation income are set apart as a class. As there is practically no overhead expense, these taxpayers are e not entitled to make deductions for income tax purposes because they are in the same situation more or less. On the other hand, in the case of professionals in the practice of their calling and businessmen, there is no uniformity in the costs or expenses necessary to produce their income. It would not be just then to disregard the disparities by giving all of them zero deduction and indiscriminately impose on all alike the same tax rates on the basis of gross income. There is ample justification then for the Batasang Pambansa to adopt the gross system of income taxation to compensation income, while continuing the system of net income taxation as regards professional and business income. Phil. Blooming Mills Employees v. Phil. Blooming Mills Co., Inc 51 SCRA 189 1) The petitioner Philippine Blooming Mills Employees Organization (PBMEO) is a legitimate labor union composed of the employees of the respondent Philippine Blooming Mills Co., Inc., and petitioners Nicanor Tolentino, Florencio Padrigano, Rufino Roxas, Mariano de Leon, Asencion Paciente, Bonifacio Vacuna, Benjamin Pagcu and Rodulfo Munsod are officers and members of the petitioner Union. PBMEO decided to stage a mass demonstration in front of Malacañang to express their grievances against the alleged abuses of the Pasig Police. 2) Petitioners claim that on March 1, 1969, they decided to stage a mass demonstration at Malacañang on March 4, 1969, in protest against alleged abuses of the Pasig police, to be participated in by the workers in the first shift (from 6 A.M. to 2 P.M.) as well as those in the regular second and third shifts (from 7 A.M. to 4 P.M. and from 8 A.M. to 5 P.M., respectively); and that they informed the respondent Company of their proposed demonstration. 3) The Philippine Blooming Mills Inc., called for a meeting with the leaders of the PBMEO after learning about the planned mass demonstration. During the meeting, the planned demonstration was confirmed by the union. But it was stressed out by the union that the demonstration was not a strike against the company but was in factual exercise of the laborers inalienable constitutional right to freedom of expression, freedom of speech and freedom for petition for redress of grievances. 4) The company asked them to cancel the demonstration for it would interrupt the normal course of their business which may result in the loss of revenue. This was backed up with the threat of the possibility that the workers would lose their jobs if they pushed through with the rally. 5) A second meeting took place where the company reiterated their appeal that while the workers may be allowed to participate, those from the 1st and regular shifts should not absent themselves to participate, otherwise, they would be dismissed. Since it was too late to cancel the plan, the rally took place and the officers of the PBMEO were eventually dismissed for a violation of the ‘No Strike and No Lockout’ clause of their Collective Bargaining Agreement.

6) The lower court decided in favour of Philippine Blooming Mills Co., Inc., and the officers of the PBMEO were found guilty of bargaining in bad faith. The PBMEO’s motion for reconsideration was subsequently denied by the Court of Industrial Relations for being filed two days late. Issue: Whether or not to regard the demonstration against police officers, not against the employer, as a violation of freedom expression in general and of their right of assembly and petition for redress of grievances Whether or not the collective bargaining agreement is an inhibition of the rights of free expression, free assembly and petition of the employers Held: 1) Property and property rights can be lost thru prescription; but human rights are imprescriptible. If human rights are extinguished by the passage of time, then the Bill of Rights is a useless attempt to limit the power of government and ceases to be an efficacious shield against the tyranny of officials, of majorities, of the influential and powerful, and of oligarchs — political, economic or otherwise. The demonstration held petitioners on March 4, 1969 before Malacañang was against alleged abuses of some Pasig policemen, not against their employer, herein private respondent firm, said demonstrate was purely and completely an exercise of their freedom expression in general and of their right of assembly and petition for redress of grievances in particular before appropriate governmental agency, the Chief Executive, again the police officers of the municipality of Pasig. They exercise their civil and political rights for their mutual aid protection from what they believe were police excesses. As matter of fact, it was the duty of herein private respondent firm to protect herein petitioner Union and its members from the harassment of local police officers. It was to the interest herein private respondent firm to rally to the defense of, and take up the cudgels for, its employees, so that they can report to work free from harassment, vexation or peril and as consequence perform more efficiently their respective tasks enhance its productivity as well as profits. 2) To regard the demonstration against police officers, not against the employer, as evidence of bad faith in collective bargaining and hence a violation of the collective bargaining agreement and a cause for the dismissal from employment of the demonstrating employees, stretches unduly the compass of the collective bargaining agreement, is “a potent means of inhibiting speech” and therefore inflicts a moral as well as mortal wound on the constitutional guarantees of free expression, of peaceful assembly and of petition. The collective bargaining agreement which fixes the working shifts of the employees, according to the respondent Court Industrial Relations, in effect imposes on the workers the “duty … to observe regular working hours.” The strain construction of the Court of Industrial Relations that a stipulated working shifts deny the workers the right to stage mass demonstration against police abuses during working hours, constitutes a virtual tyranny over the mind and life the workers and deserves severe condemnation. Renunciation of the freedom should not be predicated on such a slender ground. The respondent company is the one guilty of unfair labor practice. Because the refusal on the part of the respondent firm to permit all its employees and workers to join the mass demonstration against alleged police abuses and the subsequent separation of the eight (8) petitioners from the service constituted an unconstitutional restraint on the freedom of expression, freedom of assembly and freedom petition for redress of grievances, the respondent firm committed an unfair labor practice defined in Section 4(a-1) in relation to Section 3 of Republic Act No. 875, otherwise known as the Industrial Peace Act. Section 3 of Republic Act No. 8 guarantees to the employees the right “to engage

in concert activities for … mutual aid or protection”; while Section 4(a1) regards as an unfair labor practice for an employer interfere with, restrain or coerce employees in the exercise their rights guaranteed in Section Three.

* The action to foreclose a mortgage is said to be a proceeding quasi in rem, by which is expressed the idea that while it is not strictly speaking an action in rem yet it partakes of that nature and is substantially such.

The Supreme Court set aside as null and void the orders of Court of Industrial Relations. The Supreme Court also directed the reinstatement of the herein eight (8) petitioners, with full back pay from the date of their separation from the service until re-instated, minus one day’s pay and whatever earnings they might have realized from other sources during their separation from the service.

* DUE PROCESS IN FORECLOSURE PROCEEDINGS: Property is always assumed to be in the possession of its owner, in person or by agent; and he may be safely held, under certain conditions, to be affected with knowledge that proceedings have been instituted for its condemnation and sale.

Simon v. CHR GR No. 100150, Jan 5, 1994 G.R. No. 100150 January 05, 1994 Facts :Petitioner Mayor Simon asks to prohibit CHR from further hearing and investigating "demolition case" on vendors of North EDSA. Constitutional Issue : Whether the CHR is authorized to hear and decide on the "demolition case" and to impose a fine for contempt. Ruling : Section 18, Article XIII, of the 1987 Constitution empowered the CHR to investigate all forms of human rights violations involving civil and political rights. The demolition of stalls, sari-sari stores and carenderia cannot fall within the compartment of "human rights violations involving civil and political rights". Human rights are the basic rights which inhere in man by virtue of his humanity and are the same in all parts of the world. Human rights include civil rights (right to life, liberty and property; freedom of speech, of the press, of religion, academic freedom; rights of the accused to due process of law), political rights (right to elect public officials, to be elected to public office, and to form political associations and engage in politics), social rights (right to education, employment and social services. Human rights are entitlements that inhere in the individual person from the sheer fact of his humanity...Because they are inherent, human rights are not granted by the State but can only be recognized and protected by it. Human rights includes all the civil, political, economic, social and cultural rights defined in the Universal Declaration of Human Rights. Human rights are rights that pertain to man simply because he is human. They are part of his natural birth, right, innate and inalienable. CIVIL RIGHTS - are those that belong to every citizen and are not connected with the organization or administration of the government. POLITICAL RIGHTS - are rights to participate, directly or indirectly, in the establishment or administration of the government. Section I Procedural Due Process Judicial Proceedings El Banco Espanol-Filipino vs. Palanca G.R. No. L-11390, March 26, 1918 * JURISDICTION, HOW ACQUIRED: Jurisdiction over the property which is the subject of the litigation may result either from a seizure of the property under legal process, whereby it is brought into the actual custody of the law, or it may result from the institution of legal proceedings wherein, under special provisions of law, the power ogfdcsxazf the court over the property is recognized and made effective.

FACTS: Engracio Palanca Tanquinyeng y Limquingco mortgaged various parcels of real property in Manila to El Banco EspanolFilipino. Afterwards, Engracio returned to China and there he died on January 29, 1810 without returning again to the Philippines. The mortgagor then instituted foreclosure proceeding but since defendant is a non-resident, it was necessary to give notice by publication. The Clerk of Court was also directed to send copy of the summons to the defendant’s last known address, which is in Amoy, China. It is not shown whether the Clerk complied with this requirement. Nevertheless, after publication in a newspaper of the City of Manila, the cause proceeded and judgment by default was rendered. The decision was likewise published and afterwards sale by public auction was held with the bank as the highest bidder. On August 7, 1908, this sale was confirmed by the court. However, about seven years after the confirmation of this sale, a motion was made by Vicente Palanca, as administrator of the estate of the original defendant, wherein the applicant requested the court to set aside the order of default and the judgment, and to vacate all the proceedings subsequent thereto. The basis of this application was that the order of default and the judgment rendered thereon were void because the court had never acquired jurisdiction over the defendant or over the subject of the action. ISSUE: * Whether or not the lower court acquired jurisdiction over the defendant and the subject matter of the action * Whether or not due process of law was observed RULING: On Jurisdiction:The word “jurisdiction” is used in several different, though related, senses since it may have reference (1) to the authority of the court to entertain a particular kind of action or to administer a particular kind of relief, or it may refer to the power of the court over the parties, or (2) over the property which is the subject to the litigation. The sovereign authority which organizes a court determines the nature and extent of its powers in general and thus fixes its competency or jurisdiction with reference to the actions which it may entertain and the relief it may grant. How Jurisdiction is Acquired: Jurisdiction over the person is acquired by the voluntary appearance of a party in court and his submission to its authority, or it is acquired by the coercive power of legal process exerted over the person. Jurisdiction over the property which is the subject of the litigation may result either from a seizure of the property under legal process, whereby it is brought into the actual custody of the law, or it may result from the institution of legal proceedings wherein, under special provisions of law, the power of the court over the property is recognized and made effective. In the latter case the property, though at all times within the potential power of the court, may never be taken into actual custody at all. An illustration of the jurisdiction acquired by actual seizure is found in attachment proceedings, where the property is seized at the beginning of the action, or some subsequent stage of its progress, and held to abide the final event of the litigation. An illustration of what we term potential jurisdiction over the res, is found in the proceeding to register the title of land under our system for the registration of land. Here the court, without taking actual physical control over the property assumes, at the instance of some person

claiming to be owner, to exercise a jurisdiction in rem over the property and to adjudicate the title in favor of the petitioner against all the world. In the terminology of American law the action to foreclose a mortgage is said to be a proceeding quasi in rem, by which is expressed the idea that while it is not strictly speaking an action in rem yet it partakes of that nature and is substantially such. The expression "action in rem" is, in its narrow application, used only with reference to certain proceedings in courts of admiralty wherein the property alone is treated as responsible for the claim or obligation upon which the proceedings are based. The action quasi rem differs from the true action in rem in the circumstance that in the former an individual is named as defendant, and the purpose of the proceeding is to subject his interest therein to the obligation or lien burdening the property. All proceedings having for their sole object the sale or other disposition of the property of the defendant, whether by attachment, foreclosure, or other form of remedy, are in a general way thus designated. The judgment entered in these proceedings is conclusive only between the parties. It is true that in proceedings of this character, if the defendant for whom publication is made appears, the action becomes as to him a personal action and is conducted as such. This, however, does not affect the proposition that where the defendant fails to appear the action is quasi in rem; and it should therefore be considered with reference to the principles governing actions in rem. Galvez vs. Court of Appeals, G.R. No. 114046 October 24, 1994 Facts:On November 12, 1993, petitioners Honorato Galvez, the incumbent Mayor of San Ildefonso, Bulacan, and one Godofredo Diego were charged in three separate informations with homicide and two counts of frustrated homicide for allegedly shooting to death Alvin Calma Vinculado and seriously wounding Levi Calma Vinculado and Miguel Reyes Vinculado, Jr. On December 15, 1993, before petitioners could be arraigned, respondent prosecutor filed an Ex Parte Motion to Withdraw Informations of the original informations. This motion was granted by Judge Villajuan also on December 15, 1993 and the cases were considered withdrawn from the docket of the court. On the same day, Prosecutor Villa-Ignacio filed four new informations against herein petitioners for murder, two counts of frustrated murder, and violation of Presidential Decree No. 1866 for illegal possession of firearms. Thereafter, a Motion to Quash the new informations for lack of jurisdiction was filed by petitioners before Judge Pornillos on January 3, 1994. At the court session set for the arraignment of petitioners on January 24, 1994, Judge Pornillos issued an order denying the motion to quash. In the meantime, and prior to the arraignment of herein petitioners before Judge Pornillos, an order was issued on January 20, 1994 by Judge Villajuan granting the motion for reconsideration filed by petitioners, ordering the reinstatement of the original informations, and setting the arraignment of the accused therein for February 8, 1994. On said date, however, the arraignment was suspended and, in the meanwhile, petitioners filed a petition for certiorari, prohibition and mandamus with respondent Court of Appeals, assailing the order dated January 24, 1994 issued by Judge Pornillos which denied petitioners’ motion to quash filed for the new informations. As earlier stated, respondent court dismissed the petition in its questioned resolution of February 18, 1994, hence this petition. Issue: Whether the ex parte motion to withdraw the original informations is null and void on the ground that there was no notice and hearing as required by Sections 4, 5 and 6, Rule 15 of the Rules of Court. Held: No, considering that in the original cases before Branch 14 of the trial court petitioners had not yet been placed in jeopardy, and the

ex parte motion to withdraw was filed and granted before they could be arraigned, there would be no imperative need for notice and hearing thereof. In actuality, the real grievance of herein accused is not the dismissal of the original three informations but the filing of four new informations, three of which charge graver offenses and the fourth, an additional offense. Had these new informations not been filed, there would obviously have been no cause for the instant petition. Accordingly, their complaint about the supposed procedural lapses involved in the motion to dismiss filed and granted in Criminal Cases Nos. 3642-M-93 to 3644-M-93 does not impress us as a candid presentation of their real position. Petitioner’s contention that the dismissal of the original informations and the consequent filing of the new ones substantially affected their right to bail is too strained and tenuous an argument. They would want to ignore the fact that had the original informations been amended so as to charge the capital offense of murder, they still stood to likewise be deprived of their right to bail once it was shown that the evidence of guilt is strong. Petitioners could not be better off with amended informations than with the subsequent ones. It really made no difference considering that where a capital offense is charged and the evidence of guilt is strong, bail becomes a matter of discretion under either an amended or a new information. Contrary to petitioners’ submission, the absence of notice and hearing does not divest a trial court of authority to pass on the merits of the motion. It has been held that—“The order of the court granting the motion to dismiss despite absence of a notice of hearing, or proof of service thereof, is merely an irregularity in the proceedings. It cannot deprive a competent court of jurisdiction over the case. The court still retains its authority to pass on the merits of the motion. The remedy of the aggrieved party in such cases is either to have the order set aside or the irregularity otherwise cured by the court which dismissed the complaint or to appeal from the dismissal and not certiorari.” State Prosecutors v. Muro 236 SCRA 505 Facts: The state prosecutors who are members of the DOJ Panel of Prosecution filed a complaint against respondent Judge Muro on the ground of ignorance of the law, grave misconduct and violation of the provisions in the Code of Judicial Conduct. The case at bar involves the prosecution of the 11 charges against Imelda Marcos in violation of the Central Bank Foreign Exchange Restriction in the Central Bank Circular 960. The respondent judge dismissed all 11 cases solely on the basis of the report published from the 2 newspapers, which the judge believes to be reputable and of national circulation, that the Pres. of the Philippines lifted all foreign exchange restrictions. The respondent’s decision was founded on his belief that the reported announcement of the Executive Department in the newspaper in effect repealed the CB 960 and thereby divested the court of its jurisdiction to further hear the pending case thus motu propio dismissed the case. He further contends that the announcement of the President as published in the newspaper has made such fact a public knowledge that is sufficient for the judge to take judicial notice which is discretionary on his part.

The complainants contend that the respondent judge erred in taking judicial notice on matters he purported to be a public knowledge based merely on the account of the newspaper publication that the Pres. has lifted the foreign exchange restriction. It was also an act of inexcusable ignorant of the law not to accord due process to the prosecutors who were already at the stage of presenting evidence thereby depriving the government the right to be heard. The judge also exercised grave abuse of discretion by taking judicial notice on the published statement of the

Pres. In the newspaper which is a matter that has not yet been officially in force and effect of the law. Issue: Whether or not the respondent judge committed grave abuse of discretion in taking judicial notice on the statement of the president lifting the foreign exchange restriction published in the newspaper as basis for dismissing the case? Ruling: The Supreme Court held the respondent judge guilty for gross ignorance of the law. It cannot comprehend his assertion that there is no need to wait for the publication of the circular no. 1353 which is the basis of the President’s announcement in the newspaper, believing that the public announcement is absolute and without qualification and is immediately effective and such matter becomes a public knowledge which he can take a judicial notice upon in his discretion. It is a mandatory requirement that a new law should be published for 15 days in a newspaper of general circulation before its effectivity. When the President’s statement was published in the newspaper, the respondent admitted of not having seen the official text of CB circular 1353 thus it was premature for him to take judicial notice on this matter which is merely based on his personal knowledge and is not based on the public knowledge that the law requires for the court to take judicial notice of. For the court to take judicial notice, three material requisites should be present: (1) the matter must be one of common and general knowledge; (2) it must be well and authoritatively settled and not doubtful or uncertain; (3) it must be known to be within the limits of the jurisdiction of the court. The fact that should be assumed as judicially known must be on such notoriety that such fact cannot be disputed. Judicial notice is not judicial knowledge where the personal knowledge of the judge does not amount to the judicial notice of the court. The common knowledge contemplated by the law where the court can take judicial notice must come from the knowledge of men generally in the course of ordinary experiences that are accepted as true and one that involves unquestioned demonstration. The court ruled that the information he obtained from the newspaper is one of hearsay evidence. The judge erred in taking cognizant of a law that was not yet in force and ordered the dismissal of the case without giving the prosecution the right to be heard and of due process. The court ordered for the dismissal of the judge from service for gross ignorance of the law and grave abuse of discretion for dismissing the case motu proprio and for erring in exercising his discretion to take judicial notice on matters that are hearsay and groundless with a reminder the power to take judicial notice is to be exercised by the courts with caution at all times. Carvajal v. CA 280 SCRA 351 FACTS: Petitioner allegedly acquired portions of the parcel of land by inheritance from his father Felix Carvajal who came to possess the unregistered land in 1938, continuously, openly, adversely and peacefully in the concept of an owner up to the time of his death.

Petitioner further maintains that he was denied due process when he, as an applicant in a land registration case, was not able to take the witness stand. According to petitioner, even his counsel hardly participated in the proceeding except to propound clarificatory questions during the examination of Engineer Silverio Perez of the Land Registration Authority ISSUE: Whether or not the petitioners were denied due process of law HELD: No. While petitioner claims that he was denied due process because he was unable to take the witness stand. The court ruled that the essence of due process is the opportunity to be heard. It is the denial of this opportunity that is repugnant to due process.[19] In this case, petitioner was afforded an opportunity to present witnesses, and he did present three. However, petitioner did not invoke his right to take the witness stand even when the trial court ordered the submission of the parties’ memoranda which signified the termination of the proceedings. Because he acquiesced to the termination of the case, he forfeited his right to take the witness stand. Perez v. Estrada AM. No. OI-4-O3-SC June 29, 2001 and Sept. 13, 2001 FACTS: On March 13, 2001, the Kapisanan ng mga Brodkaster ng Pilipinas (KBP), an association representing duly franchised and authorized television and radio networks throughout the country, sent a letter requesting the Supreme Court to allow live media coverage of the anticipated trial of the plunder and other criminal cases filed against former President Joseph E. Estrada before the Sandiganbayan in order "to assure the public of full transparency in the proceedings of an unprecedented case in our history." The request was seconded by Mr. Cesar N. Sarino in his letter of 5 April 2001 to the Chief Justice and, still later, by Senator Renato Cayetano and Attorney Ricardo Romulo. On 17 April 2001, the Secretary of Justice Hernando Perez formally filed the petition. ISSUE: Whether or not media coverage be allowed to air Estrada’s trial to the public. HELD: No. In Estes v. Texas, US SC held that television coverage of judicial proceedings involves an inherent denial of due process rights of the criminal defendant: "Witnesses might be frightened, play to the cameras, become nervous. They are then subject to extraordinary outof-court influences that might affect their testimony. Telecasting increases the trial judge's responsibility to avoid actual prejudice to the defendant. For the defendant, telecasting is a form of mental harassment and subjects him to excessive public exposure and distracts him from an effective presentation of his defense. Finally, the television camera is a powerful weapon which intentionally or inadvertently can destroy an accused and his case in the eyes of the public." The right of people to information does not prescribe that TV cameras be installed in the courtroom. This right might be fulfilled by less distracting, degrading and more judicial means. In a criminal case, a life is at stake, and the due process rights of the accused shall take precedence over the people's right to information. The accused has the right to a public trial, and the exercise of such a right is his to make, because it is his life and liberty that is in the balance. A public trial is not the same as a publicized trial.

The latter court upheld the trial court LRC Case No. 414(-A), LRC Record No. N-60084 filed before Regional Trial Court, Antipolo City, Branch 71. In dismissing petitioner's application for registration of title of a parcel of land in Antipolo City. The Court recognized respondent Solid Homes, Inc. as the registered owner of a parcel of land covered by TCT No. N-7873, situated in Antipolo City.

IBP: "TV coverage can negate the rule on the exclusion of the witness intended to ensure a fair trial...could allow the 'hooting throng' to arrogate upon themselves the task of judging the guilt of the accused...will not subserve the ends of justice, but will only pander to the desire of publicity of a few grandstanding lawyers."

The Court of Appeals affirmed the dismissal of the application for registration, and denied the subsequent motion for reconsideration. Hence, this recourse to this Court via Rule 45 of the Rules of Court.

Court is not unmindful of the recent technological advances but to chance forthwith the life and liberty of any person in a hasty bid to use and apply them, even before ample safety nets are provided and the

concerns heretofore expressed are aptly addressed, is a price too high to pay.

The Supreme Court agreed with NLU. The Solicitor General, arguing for the CIR, filed a motion for reconsideration.

Perez v. Estrada

ISSUE: Whether or not the National Labor Union, Inc. is entitled to a new trial.

A.M. No. 01-4-03-SC September 13, 2001 FACTS: This is a motion for reconsideration of the decision denying petitioners’ request for permission to televise and broadcast live the trial of former President Estrada before the Sandiganbayan. The motion was filed by the Secretary of Justice, as one of the petitioners, who argues that there is really no conflict between the right of the people to public information and the freedom of the press, on the one hand, and, on the other, the right of the accused to a fair trial; that if there is a clash between these rights, it must be resolved in favor of the right of the people and the press because the people, as the repository of sovereignty, are entitled to information; and that live media coverage is a safeguard against attempts by any party to use the courts as instruments for the pursuit of selfish interests. On the other hand, former President Joseph E. Estrada reiterates his objection to the live TV and radio coverage of his trial on the ground that its allowance will violate the sub judice rule and that, based on his experience with the impeachment trial, live media coverage will only pave the way for so-called "expert commentary" which can trigger massive demonstrations aimed at pressuring the Sandiganbayan to render a decision one way or the other. Mr. Estrada contends that the right of the people to information may be served through other means less distracting, degrading, and prejudicial than live TV and radio coverage. ISSUE: Whether or not television and radio coverage of plunder case be allowed. HELD: No. The Court has considered the arguments of the parties on this important issue and, after due deliberation, finds no reason to alter or in any way modify its decision prohibiting live or real time broadcast by radio or television of the trial of the former president. By a vote of nine (9) to six (6) of its member, the Court denies the motion for reconsideration of the Secretary of Justice. In lieu of live TV and radio coverage of the trial, the Court, by the vote of eight (8) Justices, has resolved to order the audio-visual recording of the trial for documentary purposes. Seven (7) Justices vote against the audio-visual recording of the trial. Considering the significance of the trial before the Sandiganbayan of former President Estrada and the importance of preserving the records thereof, the Court believes that there should be an audio-visual recording of the proceedings. The recordings will not be for live or real time broadcast but for documentary purposes. Only later will they be available for public showing, after the Sandiganbayan shall have promulgated its decision in every case to which the recording pertains. The master film shall be deposited in the National Museum and the Records Management and Archives Office for historical preservation and exhibition pursuant to law. Ang Tibay v. CIR 69 Phil. 635 . Teodoro Toribio owns and operates Ang Tibay, a leather company which supplies the Philippine Army. Due to alleged shortage of leather, Toribio caused the lay off of a number of his employees. However, the National Labor Union, Inc. (NLU) questioned the validity of said lay off as it averred that the said employees laid off were members of NLU while no members of the rival labor union National Workers Brotherhood (NWB) were laid off. NLU claims that NWB is a company dominated union and Toribio was merely busting NLU. The case reached the Court of Industrial Relations (CIR) where Toribio and NWB won. Eventually, NLU went to the Supreme Court invoking its right for a new trial on the ground of newly discovered evidence.

HELD: Yes. The records show that the newly discovered evidence or documents obtained by NLU, which they attached to their petition with the SC, were evidence so inaccessible to them at the time of the trial that even with the exercise of due diligence they could not be expected to have obtained them and offered as evidence in the Court of Industrial Relations. Further, the attached documents and exhibits are of such farreaching importance and effect that their admission would necessarily mean the modification and reversal of the judgment rendered (said newly obtained records include books of business/inventory accounts by Ang Tibay which were not previously accessible but already existing). The SC also outlined that administrative bodies, like the CIR, although not strictly bound by the Rules of Court must also make sure that they comply to the requirements of due process. For administrative bodies, due process can be complied with by observing the following:

The right to a hearing which includes the right of the party interested or affected to present his own case and submit evidence in support thereof. Not only must the party be given an opportunity to present his case and to adduce evidence tending to establish the rights which he asserts but the tribunal must consider the evidence presented. While the duty to deliberate does not impose the obligation to decide right, it does imply a necessity which cannot be disregarded, namely, that of having something to support its decision. A decision with absolutely nothing to support it is a nullity, a place when directly attached. Not only must there be some evidence to support a finding or conclusion but the evidence must be “substantial.” Substantial evidence is more than a mere scintilla It means such relevant evidence as a reasonable mind might accept as adequate to support a conclusion. The decision must be rendered on the evidence presented at the hearing, or at least contained in the record and disclosed to the parties affected. The administrative body or any of its judges, therefore, must act on its or his own independent consideration of the law and facts of the controversy, and not simply accept the views of a subordinate in arriving at a decision. The administrative body should, in all controversial questions, render its decision in such a manner that the parties to the proceeding can know the various issues involved, and the reasons for the decisions rendered. The performance of this duty is inseparable from the authority conferred upon it. OCA v. Pascual 259 SCRA 604 Before any member of the Judiciary could be faulted, it should be only after due investigation and after presentation of competent evidence derived from direct knowledge, especially since the charge is penal in character. Administrative Case against JUDGE FILOMENO PASCUAL One CEFERINO TIGAS wrote a letter, addressed to OCA of SC, charging that irregularities and corruption were being committed by the RESPONDENT Presiding Judge of MTC

Letter was referred to NBI for “discreet investigation” of respondent.

Lumiqued appealed averring that his right to due process was violated as well as his right to security of tenure.

o Proceeded to Angat, Bulacan, in order to look for Ceferino Tigas, the letter writer but realized was a fictitious character.

ISSUE: Does the due process clause encompass the right to be assisted by counsel during an administrative inquiry?

o Proceeded to the residence of CANDIDO CRUZ, an accused in respondent’s sala.

HELD: No. The right to counsel, which cannot be waived unless the waiver is in writing and in the presence of counsel, is a right afforded a suspect or an accused during custodial investigation. It is not an absolute right and may, thus, be invoked or rejected in a criminal proceeding and, with more reason, in an administrative inquiry. In the case at bar, Lumiqued invoked the right of an accused in criminal proceedings to have competent and independent counsel of his own choice. Lumiqued, however, was not accused of any crime. The investigation conducted by the committee was for the purpose of determining if he could be held administratively liable under the law for the complaints filed against him. The right to counsel is not indispensable to due process unless required by the Constitution or the law.

In affidavit, Cruz declared that he was the accused in a criminal case for Frustrated Murder. o Respondent judge decided that the crime he committed was only physical injuries o Cruz made to understand that, in view of such action, Cruz was to give him P2,000 Respondent judge also believed to be a drunkard NBI entrapped Respondent judge with help of Cruz, for which reason, the judge was thought to have been caught in flagrante delicto. Result of investigation and Respondent referred to the Inquest Prosecutor of the Office of the Special Prosecutor, Ombudsman, with the recommendation that he be charged and prosecuted for Bribery (Art. 210 RPC) o Executive Judge NATIVIDAD G. DIZON submitted report and recommendation that Respondent judge be penalized for violation of Canons 2 and 3 of Code of Judicial Conduct (A Judge should avoid impropriety and the appearance of impropriety in all activities and a judge should perform official duties honestly, and with impartiality and diligence). ISSUES AND HOLDING 1. Whether or not the evidences presented against Judge Filomeno Pascual were strong enough to convict him. No Respondent was not afforded right to open trial wherein respondent can confront the witnesses against him and present evidence in his defense. Only bases for the Report and Recommendation submitted consist of: The Complaint, the Answer, the Memorandum of the respondent, and the transcript of stenographic notes of the hearing of the bribery case of respondent judge at the Sandiganbayan. Before any member of the Judiciary could be faulted, it should be only after due investigation and after presentation of competent evidence derived from direct knowledge, especially since the charge is penal in character. ADMINISTRATIVE CASE IS DISMISSED Lumiqued v. Exevea 282 SCRA 125 Arsenio Lumiqued was the Regional Director of DAR-CAR. He was charged by Jeannette Zamudio, the Regional Cashier, for dishonesty due to questionable gas expenses under his office. It was alleged that he was falsifying gas receipts for reimbursements and that he had an unliquidated cash advance worth P116,000.00. Zamudio also complained that she was unjustly removed by Lumiqued two weeks after she filed the two complaints. The issue was referred to the DOJ. Committee hearings on the complaints were conducted on July 3 and 10, 1992, but Lumiqued was not assisted by counsel. On the second hearing date, he moved for its resetting to July 17, 1992, to enable him to employ the services of counsel. The committee granted the motion, but neither Lumiqued nor his counsel appeared on the date he himself had chosen, so the committee deemed the case submitted for resolution. The Investigating Committee recommended the dismissal of Lumiqued. DOJ Sec Drilon adopted the recommendation. Fidel Ramos issued AO 52 dismissing Lumiqued.

“. . . There is nothing in the Constitution that says that a party in a non-criminal proceeding is entitled to be represented by counsel and that, without such representation, he shall not be bound by such proceedings. The assistance of lawyers, while desirable, is not indispensable. The legal profession was not engrafted in the due process clause such that without the participation of its members, the safeguard is deemed ignored or violated. The ordinary citizen is not that helpless that he cannot validly act at all except only with a lawyer at his side.” In administrative proceedings, the essence of due process is simply the opportunity to explain one’s side. Whatever irregularity attended the proceedings conducted by the committee was cured by Lumiqued’s appeal and his subsequent filing of motions for reconsideration. The Supreme Court also emphasized that the constitutional provision on due process safeguards life, liberty and property. Public office is a public trust. It is not a property guaranteed of due process. But when the dispute concerns one’s constitutional right to security of tenure, however, public office is deemed analogous to property in a limited sense; hence, the right to due process could rightfully be invoked. Nonetheless, the right to security of tenure is not absolute especially when it was proven, as in this case, that the public officer (Lumiqued) did not live up to the Constitutional precept i.e., that all public officers and employees must serve with responsibility, integrity, loyalty and efficiency. Fabella V. CA 282 SCRA 256 On September 17, 1990, DECS Secretary Carino issued a return-towork order to all public school teachers who had participated in walkouts and strikes on various dates during the period of September to October 1990. The mass action had been staged to demand payment of 13th month pay, allowances and passage of debt cap bill in Congress. On October 1990, Secretary Carino filed administrative cases against respondents, who are teachers of Mandaluyong High School. The charge sheets required respondents to explain in writing why they should not be punished for having taken part in the mass action in violation of civil service laws. Administrative hearings started on December 1990. Respondents, through counsel assailed the legality of the proceedings on the following due process grounds: first, they were not given copies of the guidelines adopted by the committee for the investigation and denied access to evidence; second, the investigation placed the burden of proof on respondents to prove their innocence; third, that the investigating body was illegally constituted, their composition and appointment violated Sec.9 of the Magna Carta for Public School Teachers. Pending the action assailing the validity of the administrative proceedings, the investigating committee rendered a decision finding the respondents guilty and ordered their immediate dismissal.

ISSUE: Whether or not private respondents were denied due process? HELD: YES. In administrative proceedings, due process has been recognized to include the following: (1) the right to actual or constructive notice of the institution of proceedings which may affect a respondent’s legal rights; (2) a real opportunity to be heard personally or with the assistance of counsel, to present witnesses and evidence in one’s favor, and to defend one’s rights; (3) a tribunal vested with competent jurisdiction and so constituted as to afford a person charged administratively a reasonable guarantee of honesty as well as impartiality; and (4) a finding by said tribunal which is supported by substantial evidence submitted for consideration during the hearing or contained in the records or made known to the parties affected. The legislature enacted a special law, RA 4670 known as the Magna Carta for Public School Teachers, which specifically covers administrative proceedings involving public schoolteachers. Section 9 of said law expressly provides that the committee to hear public schoolteachers’ administrative cases should be composed of the school superintendent of the division as chairman, a representative of the local or any existing provincial or national teachers’ organization and a supervisor of the division. In the present case, the various committees formed by DECS to hear the administrative charges against private respondents did not include “a representative of the local or, in its absence, any existing provincial or national teacher’s organization” as required by Section 9 of RA 4670. Accordingly, these committees were deemed to have no competent jurisdiction. Thus, all proceedings undertaken by them were necessarily void. They could not provide any basis for the suspension or dismissal of private respondents. The inclusion of a representative of a teachers’ organization in these committees was indispensable to ensure an impartial tribunal. It was this requirement that would have given substance and meaning to the right to be heard. Indeed, in any proceeding, the essence of procedural due process is embodied in the basic requirement of notice and a real opportunity to be heard. Other minor issues: Petitioners allege that Sec 9 of RA 4670 was complied with because the respondents are members of Quezon City Teachers Federation. We disagree. Mere membership of said teachers in their respective teachers’ organizations does not ipso facto make them authorized representatives of such organizations as contemplated by Section 9 of RA 4670. Under this section, the teachers’ organization possesses the right to indicate its choice of representative to be included by the DECS in the investigating committee. Such right to designate cannot be usurped by the secretary of education or the director of public schools or their underlings. In the instant case, there is no dispute that none of the teachers appointed by the DECS as members of its investigating committee was ever designated or authorized by a teachers’ organization as its representative in said committee. Sec 9 of RA 4670 was repealed by PD 807. Statcon principle, a subsequent general law cannot repeal a previous specific law, unless there is an express stipulation. Always interpret laws so as to harmonize them. Summary Dismissal vs Torcita FACTS: On 26 April 1994, a red Cortina Ford, driven by C/Insp. Lazaro Torcita, with his aide, PO2 Java, in the front seat and his wife with two ladies at the backseat, were overtaken by a Mazda pick-up owned by Congressman Manuel Puey and driven by one Reynaldo Consejo with four (4) passengers in the persons of Alex Edwin del Rosario, Rosita Bistal, Carmen Braganza and Cristina Dawa. After the Mazda pick-up has overtaken the red Cortina Ford, and after a vehicular collision almost took place, it accelerated speed and proceeded to Hacienda Aimee, a sugarcane plantation owned by the congressman. The red Cortina Ford followed also at high speed until it reached the hacienda where Torcita and Java alighted and the confrontation with del Rosario and Jesus Puey occurred. Torcita identified himself but the same had no effect. PO2 Java whispered to him that there are armed men around them and that it is dangerous for them to continue. That at this point, they radioed for back-up.

Torcita, upon the arrival of the back-up force of PNP Cadiz City, proceeded to the place where Capt. Jesus Puey and Alex Edwin del Rosario were. On 6 July 1994, 12 verified administrative complaints were filed against Torcita for Conduct Unbecoming of a Police Officer, Illegal Search, Grave Abuse of Authority and Violation of Domicile, and Abuse of Authority and Violation of COMELEC Gun Ban.The 12 administrative complaints were consolidated into 1 major complaint for conduct unbecoming of a police officer. The Summary Dismissal Board, however, did not find sufficient evidence to establish that Torcita threatened anybody with a gun, nor that a serious confrontation took place between the parties, nor that the urinating incident took place, and held that the charges of violation of domicile and illegal search were not proven. Still, while the Board found that Torcita was "in the performance of his official duties" when the incident happened, he allegedly committed a simple irregularity in performance of duty (for being in the influence of alcohol while in performance of duty) and was suspended for 20 days and salary suspended for the same period of time. Torcita appealed his conviction to the Regional Appellate Board of the Philippine National Police (PNP, Region VI, Iloilo City), but the appeal was dismissed for lack of jurisdiction. Whereupon, Torcita filed a petition for certiorari in the Regional Trial Court of Iloilo City (Branch 31), questioning the legality of the conviction of an offense for which he was not charged (lack of procedural due process of law). The Board filed a motion to dismiss, which was denied. The RTC granted the petition for certiorari and annulled the dispositive portion of the questioned decision insofar as it found Torcita guilty of simple irregularity in the performance of duty. The Board appealed from the RTC decision, by petition of review to the Court of Appeals, which affirmed the same for the reason that the respondent could not have been guilty of irregularity considering that the 12 cases were eventually dismissed. The Board filed the petition for review on certiorari before the Supreme Court. ISSUE: Whether Torcita may be proceeded against or suspended for breach of internal discipline, when the original charges against him were for Conduct Unbecoming of a Police Officer, Illegal Search, Grave Abuse of Authority and Violation of Domicile, and Abuse of Authority and Violation of COMELEC Gun Ban. HELD: NO. Notification of the charges contemplates that the respondent be informed of the specific charges against him. The absence of specification of the offense for which he was eventually found guilty is not a proper observance of due process. There can be no short-cut to the legal process. While the definition of the more serious offense is broad, and almost all-encompassing a finding of guilt for an offense, no matter how light, for which one is not properly charged and tried cannot be countenanced without violating the rudimentary requirements of due process. Herein, the 12 administrative cases filed against Torcita did not include charges or offenses mentioned or made reference to the specific act of being drunk while in the performance of official duty. There is no indication or warning at all in the summary dismissal proceedings that Torcita was also being charged with breach of internal discipline consisting of taking alcoholic drinks while in the performance of his duties. The omission is fatal to the validity of the judgment finding him guilty of the offense for which he was not notified nor charged. Further, the cursory conclusion of the Dismissal Board that Torcita "committed breach of internal discipline by taking drinks while in the performance of same" should have been substantiated by factual findings referring to this particular offense. Even if he was prosecuted for irregular performance of duty, he could not have been found to have the odor or

smell of alcohol while in the performance of duty because he was not on duty at the time that he had a taste of liquor because he was on a private trip fetching his wife. Hence , the decision of the petitioners Board was rendered without or in excess of jurisdiction. Petition dismissed. Office of the Ombudsman v. Coronel FACTS: Respondent Carmencita D. Coronel is a Senior Accounting Processor B with Salary Grade 10 of the Linamon Water District, Lanao del Norte. On September 26, 1997, the Board of Directors of Linamon Water District, by virtue of Resolution No. 056[,] Series of 1997, designated [respondent] as Officer-in-Charge, effective October 1, 1997 until a General Manager shall have been appointed. In the morning of October 14, 1998, respondent called for a meeting the officers of the different Water Districts in Lanao del Norte and Lanao del Sur, as well as their advisors from the Local Water Utilities Administration (LWUA). Since it was nearing lunchtime, the group opted to continue their meeting at Marvillas Store at Barangay Bunuun, Iligan City. The luncheon meeting, attended by more or less ten (10) persons, was presided over by Advisor Rhodora Gumban of the LWUA. As the host of the said meeting, [respondent] paid for the lunch in the amount of One Thousand Two [H]undred Thirteen [P]esos (P1,213.00), as shown in cash Invoice No. 0736 dated October 14, 1998. On November 13, 1998, respondent claimed for reimbursement of her expenses covered by Voucher No. 98-11-23, chargeable against the representation and entertainment account of her office. That very same day, the voucher was approved and [respondent] got her reimbursement in the amount of One Thousand Two Hundred Thirteen Pesos (P1,213.00). On November 17, 1998, Pedro C. Sausal, Jr. was appointed General Manager of Linamon Water District. On February 1999, he filed with the Office of the Ombudsman-Mindanao a sworn letter-complaint against herein [respondent] for dishonesty. The case was docketed as Case No. OMB-MIN-ADM 99-044. The complaint alleges that [respondent] falsified the cash invoice she submitted for reimbursement by making it appear that the luncheon bill was for One Thousand Two [H]undred Thirteen [P]esos (P1,213.00) when in fact, it was only Two [H]undred Thirteen [P]esos (P213.00), as reflected in the photocopy of the original duplicate of cash invoice No. 0736 dated October 14, 1998. On November 27, 2000, Grace H. Morales, Graft Investigation Officer I of the OMB-Mindanao, Davao City, rendered a decision holding that respondent CARMENCITA D. CORONEL is guilty of DISHONESTY and is hereby DISMISSED from the service, with forfeiture of all leave credits and retirement benefits, pursuant to Section 22 (a) in relation to Sec. 9 of Rule XIV of the Omnibus Rules Implementing Book V of the Administrative Code of 1987. She is disqualified from re-employment in the national and local governments, as well as in any agency, including government-owned or controlled corporations. Respondent filed for MR to the Graft Investigation Officer and such motion was granted. Herein, petitioner Ombudsman Aniano Desierto DISAPPROVED the above order with a marginal note, the original decision stands. The appellate court reinstated the decision of the Graft Investigation Officer where it nullified petitioner Ombudsman’s Disapproval Order. The appellate court said that the OMBs disapproval was tainted with grave abuse of discretion. First, petitioner ombudsman did not consider the credible evidence presented by respondent in her Motion for Reconsideration. Second, he did not give any justification for his disapproval of the investigating officers ruling. This shortcoming was in contravention of the constitutional mandate that all decisions, even if rendered by quasi-judicial and administrative bodies, should clearly and distinctly state the facts and the law on which they are based.

ISSUE: WON the Ombudsman committed grave injustice for not considering the evidence presented by respondent in her Motion for Reconsideration. HELD: YES. The SC find the evidence presented by the complainant insufficient to support his serious charge that she was dishonest. The original Decision of the ombudsman was not supported by the evidence, but was grounded entirely on speculations, surmises and conjectures. The evidence of the prosecution consisted merely of the original Official Receipt (OR) No. 0736, and a photocopy of the original duplicate of that receipt. The first was a bill for P1,213; and the latter, for P213. Both pieces of evidence refer to one and the same Official Receipt; yet, they contain different amounts. Obviously, one of them was falsified. Unfortunately, Complainant Sausal, Jr., failed to prove that it was indeed respondents receipt that was falsified. As it stands, we do not know for certain if the number 1 was inserted in OR No. 0736 or was deleted from the unauthenticated photocopy of the original duplicate. The evidence is equivocal. Besides, given that there were 8 to 10 attendees in the luncheon meeting, a bill of P1,213 for meals was not entirely improbable, even in 1998. In administrative cases, the quantum of proof necessary for a finding of guilt is substantial evidence; that is, such relevant evidence that a reasonable mind might accept as adequate to support a conclusion. In the instant case, the complainant did not present evidence to support his theory that the photocopy of the original duplicate reflected the true amount, or that OR No. 0736 had indeed been falsified. That oversight was fatal to the discharge of his burden of proof. A reasonable mind will not carelessly jump to the conclusion that respondent is the guilty party. The complainants evidence to prove falsification consisted of an unauthenticated photocopy of the original duplicate. He could have obtained an affidavit from the restaurant proprietor or employee who had issued the receipt, in order to attest to its due execution and authenticity. Absent any proof of due execution and authenticity, the alleged photocopy of the original duplicate of OR No. 0736 does not convince us that it is an accurate reflection of the actual bill incurred. While this Court adheres to a liberal view of the conduct of proceedings before administrative agencies, it also consistently requires some proof of authenticity or reliability as a condition for the admission of documents. Absent any such proof of authenticity, the photocopy of the original duplicate should be considered inadmissible and, hence, without probative value. Given the flimsy charge and the paucity of the evidence against respondent, there is no need for her to present additional evidence to vindicate herself. The Office of the Ombudsman should have dismissed the Administrative Complaint against her in the first place. Clearly, her guilt was not proven by substantial evidence. Thus, Respondent Carmencita D. Coronel is hereby EXONERATED of the charge against her for lack of substantial evidence.s Sec. of Justice vs Lantion FACTS: On 13 January 1977, then President Ferdinand E. Marcos issued Presidential Decree 1069 "Prescribing the Procedure for the Extradition of Persons Who Have Committed Crimes in a Foreign Country". On 13 November 1994, then Secretary of Justice Franklin M. Drilon, representing the Government of the Republic of the Philippines, signed in Manila the "Extradition Treaty Between the Government of the Republic of the Philippines and the Government of the United States of America. The Senate, by way of Resolution 11, expressed its concurrence in the ratification of the said treaty. It also expressed its concurrence in the Diplomatic Notes correcting Paragraph (5)(a), Article 7 thereof (on the

admissibility of the documents accompanying an extradition request upon certification by the principal diplomatic or consular officer of the requested state resident in the Requesting State). On 18 June 1999, the Department of Justice received from the Department of Foreign Affairs U. S. Note Verbale 0522 containing a request for the extradition of Mark Jimenez to the United States. Attached to the Note Verbale were the Grand Jury Indictment, the warrant of arrest issued by the U.S. District Court, Southern District of Florida, and other supporting documents for said extradition. Jimenez was charged in the United States for violation of (a) 18 USC 371 (Conspiracy to commit offense or to defraud the United States, 2 counts), (b) 26 USC 7201 (Attempt to evade or defeat tax, 4 counts), (c) 18 USC 1343 (Fraud by wire, radio, or television, 2 counts), (d) 18 USC 1001 (False statement or entries, 6 counts), and (E) 2 USC 441f (Election contributions in name of another; 33 counts). On the same day, the Secretary issued Department Order 249 designating and authorizing a panel of attorneys to take charge of and to handle the case. Pending evaluation of the aforestated extradition documents, Jimenez (on 1 July 1999 requested copies of the official extradition request from the US Government, as well as all documents and papers submitted therewith, and that he be given ample time to comment on the request after he shall have received copies of the requested papers. The Secretary denied the request.

On 6 August 1999, Jimenez filed with the Regional Trial Court a petition against the Secretary of Justice, the Secretary of Foreign Affairs, and the Director of the National Bureau of Investigation, l for mandamus (to compel the Justice Secretary to furnish Jimenez the extradition documents, to give him access thereto, and to afford him an opportunity to comment on, or oppose, the extradition request, and thereafter to evaluate the request impartially, fairly and objectively); l certiorari (to set aside the Justice Secretary’s letter dated 13 July 1999); and prohibition (to restrain the Justice Secretary from considering the extradition request and from filing an extradition petition in court; l and to enjoin the Secretary of Foreign Affairs and the Director of the NBI from performing any act directed to the extradition of Jimenez to the United States), with an application for the issuance of a temporary restraining order and a writ of preliminary injunction. The trial court ruled in favor of Jimenez. The Secretary filed a petition for certiorari before the Supreme Court. On 18 January 2000, by a vote of 9-6, the Supreme Court dismissed the petition and ordered the Justice Secretary to furnish Jimenez copies of the,extradition request and its supporting papers and to grant him a reasonable period within which to file his comment with supporting evidence. IN SUMMARY: The Department of Justice received from the Department of Foreign Affairs a request from the United States for the extradition of Mark Jimenez to the United States pursuant to PD No. 1609 prescribing the procedure for extradition of persons who have committed a crime in a foreign country. Jimenez requested for copies of the request and that he be given ample time to comment on said request. The petitioners denied the request pursuant to the RP-US Extradition Treaty.

ISSUE: Whether or not respondent’s entitlement to notice and hearing during the evaluation stage of the proceedings constitute a breach of the legal duties of the Philippine Government under the RP-US Extradition Treaty. HELD: NO. The human rights of person and the rights of the accused guaranteed in the Constitution should take precedence over treaty rights claimed by a contracting party, the doctrine of incorporation is applied whenever municipal tribunals are confronted with a situation where there is a conflict between a rule of the international law and the constitution. Efforts must first be made in order to harmonize the provisions so as to give effect to both but if the conflict is irreconcilable, the municipal law must be upheld. The fact that international law has been made part of the law of the land does not pertain to or imply the primacy of international law over the municipal law in the municipal sphere. In states where the constitution is the highest law of the land, both statutes and treaties may be invalidated if they are in conflict with the constitution. In the case at bar, private respondent does not only face a clear and present danger of loss of property or employment but of liberty itself, which may eventually lead to his forcible banishment to a foreign land. The convergence of petitioners favorable action on the extradition request and the deprivation of private respondents liberty is easily comprehensible. We have ruled time and again that this Courts equity jurisdiction, which is aptly described as "justice outside legality," may be availed of only in the absence of, and never against, statutory law or judicial pronouncements.The constitutional issue in the case at bar does not even call for "justice outside legality," since private respondents due process rights, although not guaranteed by statute or by treaty, are protected by constitutional guarantees. We would not be true to the organic law of the land if we choose strict construction over guarantees against the deprivation of liberty. That would not be in keeping with the principles of democracy on which our Constitution is premised. Thus, Petitioner is ordered to furnish private respondent copies of the extradition request and its supporting papers and to grant him a reasonable period within which to file his comment with supporting evidence. Gov’t of USA v. Puruganan FACTS: Petition is a sequel to the case “Sec. of Justice v. Hon. Lantion”. The Secretary was ordered to furnish Mr. Jimenez copies of the extradition request and its supporting papers and to grant the latter a reasonable period within which to file a comment and supporting evidence. But, on motion for reconsideration by the Sec. of Justice, it reversed its decision but held that the Mr. Jimenez was bereft of the right to notice and hearing during the evaluation stage of the extradition process. On May 18, 2001, the Government of the USA, represented by the Philippine Department of Justice, filed with the RTC, the Petition for Extradition praying for the issuance of an order for his “immediate arrest” pursuant to Sec. 6 of PD 1069 in order to prevent the flight of Jimenez. Before the RTC could act on the petition, Mr. Jimenez filed before it an “Urgent Manifestation/Ex-Parte Motion” praying for his application for an arrest warrant be set for hearing. After the hearing, as required by the court, Mr. Jimenez submitted his Memorandum. Therein seeking an alternative prayer that in case a warrant should issue, he be allowed to post bail in the amount of P100,000. The court ordered the issuance of a warrant for his arrest and fixing bail for his temporary liberty at P1M in cash. After he had surrendered his passport and posted the required cash bond, Jimenez was granted provisional liberty. Government of the USA filed a petition for Certiorari under Rule 65 of the Rules of Court to set aside the order for the issuance of a warrant for his arrest and fixing bail for his temporary liberty at P1M in cash which the court deems best to take cognizance as there is still no local jurisprudence to guide lower court.

ISSUES: i. Whether or NOT Hon. Purganan acted without or in excess of jurisdiction or with grave abuse of discretion amounting to lack or excess of jurisdiction in adopting a procedure of first hearing a potential extraditee before issuing an arrest warrant under Section 6 of PD No. 1069 ii. Whether or NOT Hon. Purganan acted without or in excess of jurisdiction or with grave abuse of discretion amounting to lack or excess of jurisdiction in granting the prayer for bail iii.

Whether or NOT there is a violation of due process

HELD: Petition is GRANTED. Bail bond posted is CANCELLED. Regional Trial Court of Manila is directed to conduct the extradition proceedings before it. i. YES. By using the phrase “if it appears,” the law further conveys that accuracy is not as important as speed at such early stage. From the knowledge and the material then available to it, the court is expected merely to get a good first impression or a prima facie finding sufficient to make a speedy initial determination as regards the arrest and detention of the accused. The prima facie existence of probable cause for hearing the petition and, a priori, for issuing an arrest warrant was already evident from the Petition itself and its supporting documents. Hence, after having already determined therefrom that a prima facie finding did exist, respondent judge gravely abused his discretion when he set the matter for hearing upon motion of Jimenez. The silence of the Law and the Treaty leans to the more reasonable interpretation that there is no intention to punctuate with a hearing every little step in the entire proceedings. It also bears emphasizing at this point that extradition proceedings are summary in nature. Sending to persons sought to be extradited a notice of the request for their arrest and setting it for hearing at some future date would give them ample opportunity to prepare and execute an escape which neither the Treaty nor the Law could have intended. Even Section 2 of Article III of our Constitution, which is invoked by Jimenez, does not require a notice or a hearing before the issuance of a warrant of arrest. To determine probable cause for the issuance of arrest warrants, the Constitution itself requires only the examination under oath or affirmation of complainants and the witnesses they may produce.1 The Proper Procedure to “Best Serve The Ends Of Justice” In Extradition Cases Upon receipt of a petition for extradition and its supporting documents, the judge must study them and make, as soon as possible, a prima facie finding whether a)

they are sufficient in form and substance

b) they show compliance with the Extradition Treaty and Law c)

the potential extraditee of the pendency of the petition, lest the latter be given the opportunity to escape and frustrate the proceedings.

ii. Yes.

The constitutional provision on bail on Article III, Section 13 of the Constitution, as well as Section 4 of Rule 114 of the Rules of Court, applies only when a person has been arrested and detained for violation of Philippine criminal laws. It does not apply to extradition proceedings, because extradition courts do not render judgments of conviction or acquittal. Moreover, the constitutional right to bail “flows from the presumption of innocence in favor of every accused who should not be subjected to the loss of freedom as thereafter he would be entitled to acquittal, unless his guilt be proved beyond reasonable doubt. In extradition, the presumption of innocence is not at issue. The provision in the Constitution stating that the “right to bail shall not be impaired even when the privilege of the writ of habeas corpus is suspended” finds application “only to persons judicially charged for rebellion or offenses inherent in or directly connected with invasion.”

That the offenses for which Jimenez is sought to be extradited are bailable in the United States is not an argument to grant him one in the present case. Extradition proceedings are separate and distinct from the trial for the offenses for which he is charged. He should apply for bail before the courts trying the criminal cases against him, not before the extradition court.

Exceptions to the “No Bail” Rule Bail is not a matter of right in extradition cases. It is subject to judicial discretion in the context of the peculiar facts of each case. Bail may be applied for and granted as an exception, only upon a clear and convincing showing 1) that, once granted bail, the applicant will not be a flight risk or a danger to the community; and 2) that there exist special, humanitarian and compelling circumstances including, as a matter of reciprocity, those cited by the highest court in the requesting state when it grants provisional liberty in extradition cases therein

Since this exception has no express or specific statutory basis, and since it is derived essentially from general principles of justice and fairness, the applicant bears the burden of proving the above two-tiered requirement with clarity, precision and emphatic forcefulness.

the person sought is extraditable

At his discretion, the judge may require the submission of further documentation or may personally examine the affiants and witnesses of the petitioner. If, in spite of this study and examination, no prima facie finding is possible, the petition may be dismissed at the discretion of the judge. On the other hand, if the presence of a prima facie case is determined, then the magistrate must immediately issue a warrant for the arrest of the extraditee, who is at the same time summoned to answer the petition and to appear at scheduled summary hearings. Prior to the issuance of the warrant, the judge must not inform or notify

It must be noted that even before private respondent ran for and won a congressional seat in Manila, it was already of public knowledge that the United States was requesting his extradition. Therefore, his constituents were or should have been prepared for the consequences of the extradition case. Thus, the court ruled against his claim that his election to public office is by itself a compelling reason to grant him bail.

Giving premium to delay by considering it as a special circumstance for the grant of bail would be tantamount to giving him the power to

grant bail to himself. It would also encourage him to stretch out and unreasonably delay the extradition proceedings even more. Extradition proceedings should be conducted with all deliberate speed to determine compliance with the Extradition Treaty and Law; and, while safeguarding basic individual rights, to avoid the legalistic contortions, delays and technicalities that may negate that purpose.

That he has not yet fled from the Philippines cannot be taken to mean that he will stand his ground and still be within reach of our government if and when it matters; that is, upon the resolution of the Petition for Extradition.

iii.

NO.

Potential extraditees are entitled to the rights to due process and to fundamental fairness. The doctrine of right to due process and fundamental fairness does not always call for a prior opportunity to be heard. A subsequent opportunity to be heard is enough. He will be given full opportunity to be heard subsequently, when the extradition court hears the Petition for Extradition. Indeed, available during the hearings on the petition and the answer is the full chance to be heard and to enjoy fundamental fairness that is compatible with the summary nature of extradition.

It is also worth noting that before the US government requested the extradition of respondent, proceedings had already been conducted in that country. He already had that opportunity in the requesting state; yet, instead of taking it, he ran away. Other Doctrines:

involve the determination of the guilt or innocence of an accused. His guilt or innocence will be adjudged in the court of the state where he will be extradited. b) An extradition proceeding is summary in nature while criminal proceedings involve a full-blown trial. c) In terms of the quantum of evidence to be satisfied, a criminal case requires proof “beyond reasonable doubt” for conviction while a fugitive may be ordered extradited “upon showing of the existence of a prima facie case” d) Unlike in a criminal case where judgment becomes executory upon being rendered final, in an extradition proceeding, our courts may adjudge an individual extraditable but the President has the final discretion to extradite him.

Extradition is merely a measure of international judicial assistance through which a person charged with or convicted of a crime is restored to a jurisdiction with the best claim to try that person. The ultimate purpose of extradition proceedings in court is only to determine whether the extradition request complies with the Extradition Treaty, and whether the person sought is extraditable.

4) Compliance Shall Be in Good Faith.

We are bound by pacta sunt servanda to comply in good faith with our obligations under the Treaty. Accordingly, the Philippines must be ready and in a position to deliver the accused, should it be found proper

Five Postulates of Extradition 1) Extradition Is a Major Instrument for the Suppression of Crime

In this era of globalization, easier and faster international travel, and an expanding ring of international crimes and criminals, we cannot afford to be an isolationist state. We need to cooperate with other states in order to improve our chances of suppressing crime in our own country.

5) There Is an Underlying Risk of Flight

Indeed, extradition hearings would not even begin, if only the accused were willing to submit to trial in the requesting country. Prior acts of herein respondent: a) leaving the requesting state right before the conclusion of his indictment proceedings there; and

2) The Requesting State Will Accord Due Process to the Accused

b) remaining in the requested state despite learning that the requesting state is seeking his return and that the crimes he is charged with are bailable

By entering into an extradition treaty, the Philippines is deemed to have reposed its trust in the reliability or soundness of the legal and judicial system of its treaty partner, as well as in the ability and the willingness of the latter to grant basic rights to the accused in the pending criminal case therein.

3) The Proceedings Are Sui Generis

Extradition is Essentially Executive Extradition is essentially an executive, not a judicial, responsibility arising out of the presidential power to conduct foreign relations and to implement treaties. Thus, the Executive Department of government has broad discretion in its duty and power of implementation. Gov’t of Hongkong v. Olalia FACTS:

An extradition proceeding is sui generis: a) It is not a criminal proceeding which will call into operation all the rights of an accused as guaranteed by the Bill of Rights. It does not

Private respondent Muñoz was charged before the Hong Kong Court with three (3) counts of the offense of "accepting an advantage as agent," in violation of Section 9 (1) (a) of the Prevention of Bribery Ordinance, Cap. 201 of Hong Kong. He also faces seven (7) counts of the offense of conspiracy to defraud, penalized by the common law of Hong Kong. Warrants of arrest were issued against him. If convicted, he faces a jail term of seven (7) to fourteen (14) years for each charge.

Section 2(a) of Presidential Decree (P.D.) No. 1069 (The Philippine Extradition Law) defines "extradition" as "the removal of an accused from the Philippines with the object of placing him at the disposal of foreign authorities to enable the requesting state or government to hold him in connection with any criminal investigation directed against him or the execution of a penalty imposed on him under the penal or criminal law of the requesting state or government."

On September 13, 1999, the DOJ received from the Hong Kong Department of Justice a request for the provisional arrest of private respondent. The RTC issued an Order of Arrest against private respondent. That same day, the NBI agents arrested and detained him.

Extradition is not a trial to determine the guilt or innocence of the potential extraditee. Nor is it a full-blown civil action, but one that is merely administrative in character. Its object is to prevent the escape of a person accused or convicted of a crime and to secure his return to the state from which he fled, for the purpose of trial or punishment. It does not necessarily mean that in keeping with its treaty obligations, the Philippines should diminish a potential extraditee’s rights to life, liberty, and due process. More so, where these rights are guaranteed, not only by our Constitution, but also by international conventions, to which the Philippines is a party. We should not, therefore, deprive an extraditee of his right to apply for bail, provided that a certain standard for the grant is satisfactorily met.

Private respondent filed a petition for bail which was opposed by petitioner. After hearing, Judge Bernardo, Jr. issued an Order denying the petition for bail, holding that there is no Philippine law granting bail in extradition cases and that private respondent is a high "flight risk." Judge Bernardo, Jr. inhibited himself from further hearing the case, it was then raffled off to Branch 8 presided by respondent judge. Private respondent filed a motion for reconsideration of the Order denying his application for bail and this was granted by respondent judge.

Petitioner filed an urgent motion to vacate the above Order, but it was denied by respondent judge. Hence, the instant petition.

ISSUE:

Whether or not respondent judge acted with grave abuse of discretion amounting to lack or excess of jurisdiction as there is no provision in the Constitution granting bail to a potential extraditee.

HELD:

No. Bearing in mind the purpose of extradition proceedings, the premise behind the issuance of the arrest warrant and the "temporary detention" is the possibility of flight of the potential extraditee. This is based on the assumption that such extraditee is a fugitive from justice. Given the foregoing, the prospective extraditee thus bears the onus probandi of showing that he or she is not a flight risk and should be granted bail.

The Philippines, along with the other members of the family of nations, committed to uphold the fundamental human rights as well as value the worth and dignity of every person. Clearly, the right of a prospective extraditee to apply for bail in this jurisdiction must be viewed in the light of the various treaty obligations of the Philippines concerning respect for the promotion and protection of human rights. Under these treaties, the presumption lies in favor of human liberty. Thus, the Philippines should see to it that the right to liberty of every individual is not impaired.

there is no showing that private respondent presented evidence to show that he is not a flight risk. Consequently, this case should be remanded to the trial court to determine whether private respondent may be granted bail on the basis of "clear and convincing evidence."

The time-honored principle of pacta sunt servanda demands that the Philippines honor its obligations under the Extradition Treaty it entered into with the Hong Kong Special Administrative Region. Failure to comply with these obligations is a setback in our foreign relations and defeats the purpose of extradition. However, it does not necessarily mean that in keeping with its treaty obligations, the Philippines should diminish a potential extraditee’s rights to life, liberty, and due process. More so, where these rights are guaranteed, not only by our Constitution, but also by international conventions, to which the Philippines is a party. This Court should not, therefore, deprive an extraditee of his right to apply for bail, provided that a certain standard for the grant is satisfactorily met.

An extradition proceeding being sui generis, the standard of proof required in granting or denying bail can neither be the proof beyond reasonable doubt in criminal cases nor the standard of proof of preponderance of evidence in civil cases. While administrative in character, the standard of substantial evidence used in administrative cases cannot likewise apply given the object of extradition law which is to prevent the prospective extraditee from fleeing our jurisdiction. In his Separate Opinion in Purganan, then Associate Justice, now Chief Justice Reynato S. Puno, proposed that a new standard which he termed "clear and convincing evidence" should be used in granting bail in extradition cases. According to him, this standard should be lower than proof beyond reasonable doubt but higher than preponderance of evidence. The potential extraditee must prove by "clear and convincing evidence" that he is not a flight risk and will abide with all the orders and processes of the extradition court. In this case, there is no showing that private respondent presented evidence to show that he is not a flight risk. Consequently, this case should be remanded to the trial court to determine whether private respondent may be granted bail on the basis of "clear and convincing evidence."

Ocampo vs. Abando

FACTS:A mass graveyard was found at Sitio Sapang Daco, Barangay Kaulisihan, Inopacan, Leyte by the43rd Infantry Brigade containing 67 skeletal remains of those believed to be victims of “Operation Venereal Disease (VD)” by the Communist Party of the Philippines/ New People’s Army/National Democratic Front (CPP/NPA/NPDF) of the Philippines. This was done to purge their ranks of suspected military informers. Members of the Scene of the Crime Operation team conducted forensic crime analysis to identify the bodies by way of DNA sample. The initial report of the PNP Crime Laboratory on their identities remained inconclusive, but, in a Special Report, the Case Secretariat of the Regional and National Inter-Agency Legal Action Group came up with ten names of possible victims after comparing the testimonies of relatives and witnesses. Police Chief Inspector George L. Almaden and Staff Judge Advocate Captain Allan Tiu sent undated letters to Pros. Vivero, requesting for legal action on the twelve attached complaint affidavits. These were from relatives of the alleged victims of Operation VD who all swore that their relatives had been abducted or last seen with members of the CPP/NPA/NDFP.

records were transmitted to RTC Manila. Echanis and Baylosis continued to seek relief from the Supreme Court in response to Judge Abando’s orders. Echanis also prayed for his release. Both Ocampo and Echanis were granted provisional release by the Supreme Court under cash bonds. As to Ladlad’s Motion to Quash, it was denied by respondent judge and the same happened to his Motion for Reconsideration. Ladlad sought to annul the latter’s orders by way of special civil action for certiorari under Rule 65. As to their bail, Ladlad filed an Urgent Motion to Fix Bail whereas Baylosis filed a Motion to Allow Petitioner to Post Bail which were granted, with no opposition from the OSG (bec. they’re consultants of the NDFP negotiating team, then having talks with the GRP peace panel). ISSUE:Whether or not the petitioners’ right to due process was violated. HELD:NO. Petitioners were accorded due process during preliminary investigation and in the issuance of the warrants of arrest.

Charging them with murder, the affidavits were directed to 71 named members of the group, including the petitioners. Namely, the petitioners were Ocampo, Echanis, Baylosis and Ladlad who were all pointed out to be members of the Central Committee that ordered the campaign to be carried out in 1985.

A preliminary investigation is "not a casual affair." It is conducted to protect the innocent from the embarrassment, expense and anxiety of a public trial. While the right to have a preliminary investigation before trial is statutory rather than constitutional, it is a substantive right and a component of due process in the administration of criminal justice.

On this basis, Pros. Vivero issued a subpoena requiring them to submit their counter-affidavits and Ocampo complied. However, Echanis and Baylosis did not do so because allegedly they were not served the copy of a subpoena. As for Ladlad, though his counsel made formal appearance during the preliminary investigation, he also did not submit for the same reason as the two.

In the context of a preliminary investigation, the right to due process of law entails the opportunity to be heard. It serves to accord an opportunity for the presentation of the respondent’s side with regard to the accusation. Afterwards, the investigating officer shall decide whether the allegations and defenses lead to a reasonable belief that a crime has been committed, and that it was the respondent who committed it. Otherwise, the investigating officer is bound to dismiss the complaint.

Pros. Vivero, in a resolution, directed the filing of information for 15 counts of multiple murder against the 54 named members, including the petitioners. He also caused some respondents to be used as state witnesses for their testimony is vital to the prosecution. Said information was filed before RTC Hilongos, Leyte branch 18 presided by Judge Abando. Prior to receiving the resolution, Ocampo filed an Ex Parte Motion to Set Case for Clarificatory Hearing. Judge Obando found probable cause and ordered the issuance of warrants of arrest against them with no recommended bail. Ocampo went to the Supreme Court by way of special civil action for certiorari and prohibition under Rule 65 and asked for the abovementioned order and the prosecutor’s resolution to be annulled. He said that a case for rebellion against him and 44 others was then already pending before RTC Makati and so, the crime of murder was absorbed by the rebellion in line with the political offense doctrine. The Court ordered the Solicitor General to comment on the issue and also ordered the parties to submit their memoranda. From the oral arguments, the Court found that the single Information charging them all of 15 counts of murder was defective. The prosecution moved to admit amended and new information, but Judge Abando suspended the proceedings during the pendency of the case before the Court. Meanwhile, Echanis was arrested and he, along with Baylosis, filed a Motion for Judicial Reinvestigation/ Determination of Probable Cause with Prayer to Dismiss the Case Outright and Alternative Prayer to Recall/ Suspend Service of Warrant, but it was dismissed by Judge Abando. Around this time, Ladlad filed a Motion to Quash/Dismiss with the RTC Manila. Echanis and Baylosis moved to reconsider but it was not acted because, as per request of the DOJ Secretary to change the venue of the trial, the

"The essence of due process is reasonable opportunity to be heard and submit evidence in support of one's defense." What is proscribed is lack of opportunity to be heard. Thus, one who has been afforded a chance to present one’s own side of the story cannot claim denial of due process. As to the claim of petitioners Echanis and Baylosis that they were denied due process, we quote the pertinent portion of Prosecutor Vivero’s Resolution, which states: In connection with the foregoing and pursuant to the Revised Rules of Criminal Procedure[,] the respondents were issued and served with Subpoena at their last known address for them to submit their counteraffidavits and that of their witnesses. Majority of the respondents did not submit their counter-affidavits because they could no longer be found in their last known address, per return of the subpoenas. On the other hand, Saturnino Ocampo Satur, Fides Lim, Maureen Palejaro and Ruben Manatad submitted their Counter-Affidavits. However, Vicente Ladlad and Jasmin Jerusalem failed to submit the required Counter Affidavits in spite entry of appearance by their respective counsels. Section 3(d), Rule 112 of the Rules of Court, allows Prosecutor Vivero to resolve the complaint based on the evidence before him if a respondent could not be subpoenaed. As long as efforts to reach a respondent were made, and he was given an opportunity to present countervailing evidence, the preliminary investigation remains valid. The rule was put in place in order to foil underhanded attempts of a respondent to delay the prosecution of offenses.In this case, the Resolution stated that efforts were undertaken to serve subpoenas on the named respondents at their last known addresses. This is sufficient for due process. It was only because a majority of them could no longer

be found at their last known addresses that they were not served copies of the complaint and the attached documents or evidence. Moreover, Petitioner Ladlad, through his counsel, had every opportunity to secure copies of the complaint after his counsel’s formal entry of appearance and, thereafter, to participate fully in the preliminary investigation. Instead, he refused to participate. We have previously cautioned that "litigants represented by counsel should not expect that all they need to do is sit back, relax and await the outcome of their case."106 Having opted to remain passive during the preliminary investigation, petitioner Ladlad and his counsel cannot now claim a denial of due process, since their failure to file a counteraffidavit was of their own doing. As to Ocampo’s claim that he was denied the right to file a motion for reconsideration or to appeal the Resolution of Prosecutor Vivero due to the 19-day delay in the service of the Resolution, it must be pointed out that the period for filing a motion for reconsideration or an appeal to the Secretary of Justice is reckoned from the date of receipt of the resolution of the prosecutor, not from the date of the resolution. This is clear from Section 3 of the 2000 National Prosecution Service Rule on Appeal: Sec. 3. Period to appeal. – The appeal shall be taken within fifteen (15) days from receipt of the resolution, or of the denial of the motion for reconsideration/ reinvestigation if one has been filed within fifteen (15) days from receipt of the assailed resolution. Only one motion for reconsideration shall be allowed. (Emphasis supplied) Thus, when petitioner Ocampo received the Resolution of Prosecutor Vivero on 12 March 2007,108 the former had until 27 March 2007 within which to file either a motion for reconsideration before the latter or an appeal before the Secretary of Justice. Instead, petitioner Ocampo chose to file the instant petition for certiorari directly before this Court on 16 March 2007.

Sen. Estrada claims that the denial of his Request for the counteraffidavits of his co-respondents violates his constitutional right to due process. Sen. Estrada, however, fails to specify a law or rule which states that it is a compulsory requirement of due process in a preliminary investigation that the Ombudsman furnish a respondent with the counter-affidavits of his co-respondents. Neither Section 3(b), Rule 112 of the Revised Rules of Criminal Procedure nor Section 4(c), Rule II of the Rules of Procedure of the Office of the Ombudsman supports Sen. Estrada’s claim. What the Rules of Procedure of the Office of the Ombudsman require is for the Ombudsman to furnish the respondent with a copy of the complaint and the supporting affidavits and documents at the time the order to submit the counter-affidavit is issued to the respondent. This is clear from Section 4(b), Rule II of the Rules of Procedure of the Office of the Ombudsman when it states, "[a]fter such affidavits [of the complainant and his witnesses] have been secured, the investigating officer shall issue an order, attaching thereto a copy of the affidavits and other supporting documents, directing the respondent to submit, within ten (10) days from receipt thereof, his counter-affidavits x x x." At this point, there is still no counter-affidavit submitted by any respondent. Clearly, what Section 4(b) refers to are affidavits of the complainant and his witnesses, not the affidavits of the co-respondents. Obviously, the counter-affidavits of the co-respondents are not part of the supporting affidavits of the complainant. No grave abuse of discretion can thus be attributed to the Ombudsman for the issuance of the 27 March 2014 Order which denied Sen. Estrada’s Request. Second, it should be underscored that the conduct of a preliminary investigation is only for the determination of probable cause, and “probable cause merely implies probability of guilt and should be determined in a summary manner. A preliminary investigation is not a part of the trial and it is only in a trial where an accused can demand the full exercise of his rights, such as the right to confront and crossexamine his accusers to establish his innocence.” Thus, the rights of a respondent in a preliminary investigation are limited to those granted by procedural law.

Estrada vs Ombudsman FACTS:Sometime in November and December 2013, the Ombudsman served on Sen. Estrada two (2) criminal complaints for plunder, among others. Eighteen (18) of Sen. Estrada’s co-respondents in the two complaints filed their counter-affidavits between 9 December 2013 and 14 March 2014. On 20 March 2014, Sen. Estrada filed his “Request to be Furnished with Copies of Counter-Affidavits of the Other Respondents, Affidavits of New Witnesses and Other Filings” (the “Request”). Sen. Estrada’s request was made “[p]ursuant to the right of a respondent ‘to examine the evidence submitted by the complainant which he may not have been furnished’ (Section 3[b], Rule 112 of the Rules of Court) and to ‘have access to the evidence on record’ (Section 4[c], Rule II of the Rules of Procedure of the Office of the Ombudsman).” The Ombudsman denied Sen. Estrada’s Request, which is not the subject of the present certiorari case. ISSUE:WON petitioner Estrada was denied due process of law HELD:NO. The denial did not violate Sen. Estrada’s constitutional right to due process. First. There is no law or rule which requires the Ombudsman to furnish a respondent with copies of the counter-affidavits of his corespondents. The SC cited in its decision Sections 3 and 4, Rule 112 of the Revised Rules of Criminal Procedure, as well as Rule II of Administrative Order No. 7, Rules of Procedure of the Office of the Ombudsman, for ready reference.

A preliminary investigation is defined as an inquiry or proceeding for the purpose of determining whether there is sufficient ground to engender a well-founded belief that a crime cognizable by the Regional Trial Court has been committed and that the respondent is probably guilty thereof, and should be held for trial. The quantum of evidence now required in preliminary investigation is such evidence sufficient to “engender a well-founded belief” as to the fact of the commission of a crime and the respondent’s probable guilt thereof. A preliminary investigation is not the occasion for the full and exhaustive display of the parties’ evidence; it is for the presentation of such evidence only as may engender a well-grounded belief that an offense has been committed and that the accused is probably guilty thereof. We are in accord with the state prosecutor’s findings in the case at bar that there exists prima facie evidence of petitioner’s involvement in the commission of the crime, it being sufficiently supported by the evidence presented and the facts obtaining therein. Third, the technical rules on evidence are not binding on the fiscal who has jurisdiction and control over the conduct of a preliminary investigation. If by its very nature a preliminary investigation could be waived by the accused, we find no compelling justification for a strict application of the evidentiary rules. Fourth, the quantum of evidence in preliminary investigations is not akin to those in administrative proceedings as laid down in the landmark doctrine of Ang Tibay. The quantum of evidence needed in Ang Tibay, as amplified in GSIS, is greater than the evidence needed in a preliminary investigation to establish probable cause, or to establish the existence of a prima facie case that would warrant the prosecution of a case. Ang Tibay refers to “substantial evidence,” while the establishment of probable cause needs “only more than ‘bare suspicion,’ or ‘less than evidence which would justify . . . conviction’.”

In the United States, from where we borrowed the concept of probable cause, the prevailing definition of probable cause is this: In dealing with probable cause, however, as the very name implies, we deal with probabilities. These are not technical; they are the factual and practical considerations of everyday life on which reasonable and prudent men, not legal technicians, act. The standard of proof is accordingly correlative to what must be proved. Thus, probable cause can be established with hearsay evidence, as long as there is substantial basis for crediting the hearsay. Hearsay evidence is admissible in determining probable cause in a preliminary investigation because such investigation is merely preliminary, and does not finally adjudicate rights and obligations of parties. However, in administrative cases, where rights and obligations are finally adjudicated, what is required is “substantial evidence” which cannot rest entirely or even partially on hearsay evidence. Substantial basis is not the same as substantial evidence because substantial evidence excludes hearsay evidence while substantial basis can include hearsay evidence. To require the application of Ang Tibay, as amplified in GSIS, in preliminary investigations will change the quantum of evidence required in determining probable cause from evidence of likelihood or probability of guilt to substantial evidence of guilt. Actually, the Ombudsman went beyond legal duty and even furnished Sen. Estrada with copies of the counter-affidavits of his co-respondents whom he specifically named, as well as the counter-affidavits of some of other co-respondents. In the 4 June 2014 Joint Order, the Ombudsman even held in abeyance the disposition of the motions for reconsideration because the Ombudsman granted Sen. Estrada five days from receipt of the 7 May 2014 Joint Order to formally respond to the claims made by his co-respondents. The Ombudsman faithfully complied with the existing Rules on preliminary investigation and even accommodated Sen. Estrada beyond what the Rules required. Thus, the Ombudsman could not be faulted with grave abuse of discretion. Since this is a Petition for Certiorari under Rule 65, the Petition fails in the absence of grave abuse of discretion on the part of the Ombudsman. The constitutional due process requirements mandated in Ang Tibay, as amplified in GSIS, are not applicable to preliminary investigations which are creations of statutory law giving rise to mere statutory rights. A law can abolish preliminary investigations without running afoul of the constitutional requirements of due process as prescribed in Ang Tibay, as amplified in GSIS. The present procedures for preliminary investigations do not comply and were never intended to comply, with Ang Tibay, as amplified in GSIS. Preliminary investigations do not adjudicate with finality rights and obligations of parties, while administrative investigations governed by Ang Tibay, as amplified in GSIS, so adjudicate. Ang Tibay, as amplified in GSIS, requires substantial evidence for a decision against the respondent in the administrative case.In preliminary investigations, only likelihood or probability of guilt is required. To apply Ang Tibay, as amplified in GSIS, to preliminary investigations will change the quantum of evidence required to establish probable cause. The respondent in an administrative case governed by Ang Tibay, as amplified in GSIS, has the right to an actual hearing and to cross-examine the witnesses against him. In preliminary investigations, the respondent has no such rights. Also, in an administrative case governed by Ang Tibay, as amplified in GSIS, the hearing officer must be impartial and cannot be the factfinder, investigator, and hearing officer at the same time. In preliminary investigations, the same public officer may be the investigator and hearing officer at the same time, or the fact-finder, investigator and hearing officer may be under the control and supervision of the same public officer, like the Ombudsman or Secretary of Justice. This explains why Ang Tibay, as amplified in GSIS, does not apply to preliminary investigations. To now declare that the guidelines in Ang Tibay, as amplified in GSIS, are fundamental and essential requirements in preliminary investigations

will render all past and present preliminary investigations invalid for violation of constitutional due process. This will mean remanding for reinvestigation all criminal cases now pending in all courts throughout the country. No preliminary investigation can proceed until a new law designates a public officer, outside of the prosecution service, to determine probable cause. Moreover, those serving sentences by final judgment would have to be released from prison because their conviction violated constitutional due process. Thus, petition dismissed for being premature and it constitutes forum shopping. C. Academic Discipline Guzman v. NU Facts: Petitioners Diosdado Guzman, Ulysses Urbiztondo and Ariel Ramacula, students of respondent National University, have come to this Court to seek relief from what they describe as their school's "continued and persistent refusal to allow them to enrol." In their petition "for extraordinary legal and equitable remedies with prayer for preliminary mandatory injunction" dated August 7, 1984, they alleged that they were denied due process due to the fact that they were active participants in peaceful mass actions within the premises of the University. The respondents on the other hand claimed that the petitioners’ failure to enroll for the first semester of the school year 1984-1985 is due to their own fault and not because of their alleged exercise of their constitutional and human rights. That as regards to Guzman, his academic showing was poor due to his activities in leading boycotts of classes. That Guzman “is facing criminal charges for malicious mischief before the Metropolitan Trial Court of Manila in connection with the destruction of properties of respondent University. The petitioners have “failures in their records, and are not of good scholastic standing.” Issue: WON the petitioners were denied due process. Held:Immediately apparent from a reading of respondents' comment and memorandum is the fact that they had never conducted proceedings of any sort to determine whether or not petitionersstudents had indeed led or participated "in activities within the university premises, conducted without prior permit from school authorities, that disturbed or disrupted classes therein" 3 or perpetrated acts of "vandalism, coercion and intimidation, slander, noise barrage and other acts showing disdain for and defiance of University authority." 4 Parenthetically, the pendency of a civil case for damages and a criminal case for malicious mischief against petitioner Guzman, cannot, without more, furnish sufficient warrant for his expulsion or debarment from re-enrollment. Also apparent is the omission of respondents to cite this Court to any duly published rule of theirs by which students may be expelled or refused re-enrollment for poor scholastic standing. The school had violated the Manual of Regulations for Private Schools that “no penalty shall be imposed upon any student except for cause as defined in the manual and/or in the school rules and regulations as duly promulgated and only after due investigation shall have been conducted. There are standards which must be met to satisfy the demands of procedural due process; and these are, that (1) the students must be informed in writing of the nature and cause of any accusation against them; SMBIW NCAT (2) they shall have the right to answer the charges against them, with the assistance of counsel, if desired; RTA CA WAC (3) they shall be informed of the evidence against them; SBI EAT

(4) they shall have the right to adduce evidence in their own behalf; and RAE ITOB (5) the evidence must be duly considered by the investigating committee or official designated by the school authorities to hear and decide the case. EMDC BTIC Non v. Dames Facts: Ariel Non, Rex Magana, Alvin Agura, Normandy Occiano, Jorge Dayaon, Lourdes Banares, Bartolome Ibasco, Emmanuel Barba, Sonny Moreno. Giovani Palma, Joselito Villalon, Luis Santos and Daniel Torres, students in Mabini Colleges, Inc. in Daet, Camarines Norte, were not allowed to re-enroll by the school for the academic year 1988-1989 for leading or participating in student mass actions against the school in the preceding semester. They thus filed a petition in the Regional Trial Court of Daet (Branch 38) seeking their readmission or re-enrollment to the school, but the trial court dismissed the petition in an order dated 8 August 1988. A motion for reconsideration was filed, but this was denied by the trial court on 24 February 1989; stating that they waived-their privilege to be admitted for re-enrollment with respondent college when they adopted, signed, and used its enrollment form for the first semester of school year 198889. In addition, for the same semester, they duly signed pledges "to abide and comply with all the rules and regulations laid down by competent authorities in the College Department or School in which I am enrolled." Hence, the affected students filed the petition for certiorari with prayer for preliminary mandatory injunction before the Supreme Court. Issue: Whether the school exclude students because of failing grades when the cause for the action taken against them relates to possible breaches of discipline. Held: The contract between the school and the student is not an ordinary contract. It is imbued with public interest, considering the high priority given by the Constitution to education and the grant to the State of supervisory and regulatory powers over all educational institutions. The authority for schools to refuse enrollment to a student on the ground that his contract, which has a term of one semester, has already expired, cannot be justified. Still, institutions' discretion on the admission and enrollment of students as a major component of the academic freedom guaranteed to institutions of higher learning. The right of an institution of higher learning to set academic standards, however, cannot be utilized to discriminate against students who exercise their constitutional rights to speech and assembly, for otherwise there will be a violation of their right to equal protection. Thus, an institution of learning has a contractual obligation to afford its students a fair opportunity to complete the course they seek to pursue. However, when a student commits a serious breach of discipline or fails to maintain the required academic standard, he forfeits his contractual right; and the court should not review the discretion of university authorities. Excluding students because of failing grades when the cause for the action taken against them undeniably related to possible breaches of discipline not only is a denial of due process but also constitutes a violation of the basic tenets of fair play. Further, the failures in one or two subjects by some cannot be considered marked academic deficiency. Neither can the academic deficiency be gauged from the academic standards of the school due to insufficiency of information. Herein, the students could have been subjected to disciplinary proceedings in connection with the mass actions, but the penalty that could have been imposed must be commensurate to the offense committed and it must be imposed only after the requirements of procedural due process have been complied with (Paragraph 145, Manual of Regulations for Private Schools). But this matter of disciplinary proceedings and the imposition of administrative sanctions have become moot and academic; as the students have been refused readmission or re-enrollment and have been effectively

excluded from for 4 semesters, have already been more than sufficiently penalized for any breach of discipline they might have committed when they led and participated in the mass actions that resulted in the disruption of classes. To still subject them to disciplinary proceedings would serve no useful purpose and would only further aggravate the strained relations between the students and the officials of the school which necessarily resulted from the heated legal battle. ADMU vs Capulong Facts: Leonardo H. Villa, a first year law student of Petitioner University, died of serious physical injuries at Chinese General Hospital after the initiation rites of Aquila Legis. Bienvenido Marquez was also hospitalized at the Capitol Medical Center. Petitioner Dean Cynthia del Castillo created a Joint Administration-Faculty-Student Investigating Committee which was tasked to investigate and submit a report within 72 hours on the circumstances surrounding the death of Lennie Villa. Said notice also required respondent students to submit their written statements within twenty-four (24) hours from receipt. Although respondent students received a copy of the written notice, they failed to file a reply. In themeantime, they were placed on preventive suspension. The Investigating Committee found a prima facie case against respondent students for violation of Rule 3 of the Law School Catalogue entitled "Discipline." Respondent students were then required to file their written answers to the formal charge. Petitioner Dean created a Disciplinary Board to hear the charges against respondent students. The Board found respondent students guilty of violating Rules on Discipline which prohibits participation in hazing activities. However, in view of the lack of unanimity among the members of the Board on the penalty of dismissal, the Board left the imposition of the penalty to the University Administration. Accordingly, Fr. Bernas imposed the penalty of dismissal on all respondent students. Respondent students filed with RTC Makati a TRO since they are currently enrolled. This was granted. A day after the expiration of the temporary restraining order, Dean del Castillo created a Special Board to investigate the charges of hazing against respondent students Abas and Mendoza. This was requested to be stricken out by the respondents and argued that the creation of the Special Board was totally unrelated to the original petition which alleged lack of due process. This was granted and reinstatement of the students was ordered. Issue: Was there denial of due process against the respondent students. Held: There was no denial of due process, more particularly procedural due process. The Dean of the Ateneo Law School, notified and required respondent students to submit their written statement on the incident. Instead of filing a reply, respondent students requested through their counsel, copies of the charges. The nature and cause of the accusation were adequately spelled out in petitioners' notices. Present is the twin elements of notice and hearing. The Minimum standards to be satisfied in the imposition of disciplinary sanctions in academic institutions, such as petitioner university herein, thus: (1) the students must be informed in WRITING of the nature and cause of any accusation against them; (2) that they shall have the right to answer the charges against them with the assistance of counsel, if desired: (3) they shall be informed of the evidence against them (4) they shall have the right to adduce evidence in their own behalf; and (5) the evidence must be duly considered by the investigating committee or official designated by the school authorities to hear and decide the case.

It cannot seriously be asserted that the above requirements were not met. When, in view of the death of Leonardo Villa, petitioner Cynthia del Castillo, as Dean of the Ateneo Law School, notified and required respondent students on February 11, 1991 to submit within twenty-four hours their written statement on the incident, the records show that instead of filing a reply, respondent students requested through their counsel, copies of the charges. While of the students mentioned in the February 11, 1991 notice duly submitted written statements, the others failed to do so. Thus, the latter were granted an extension of up to February 18, 1991 to file their statements . Indubitably, the nature and cause of the accusation were adequately spelled out in petitioners' notices dated February 14 and 20, 1991. 30 It is to be noted that the February 20, 1991 letter which quoted Rule No. 3 of its Rules of Discipline as contained in the Ateneo Law School Catalogue was addressed individually to respondent students. Petitioners' notices/letters dated February 11, February 14 and 20 clearly show that respondent students were given ample opportunity to adduce evidence in their behalf and to answer the charges leveled against them. The requisite assistance of counsel was met when, from the very start of the investigations before the Joint Administration Faculty- Student Committee, the law firm of Gonzales Batiler and Bilog and Associates put in its appearance and filed pleadings in behalf of respondent students. Respondent students may not use the argument that since they were not accorded the opportunity to see and examine the written statements which became the basis of petitioners' February 14, 1991 order, they were denied procedural due process. Granting that they were denied such opportunity, the same may not be said to detract from the observance of due process, for disciplinary cases involving students need not necessarily include the right to cross examination. An ADMINISTRATIVE PROCEEDING conducted to investigate students' participation in a hazing activity need not be clothed with the attributes of a judicial proceeding. A closer examination of the March 2, 1991 hearing which characterized the rules on the investigation as being summary in nature and that respondent students have no righ t to examine affiants-neophytes, reveals that this is but a reiteration of our previous ruling in Alcuaz . Respondent students' contention that the investigating committee failed to consider their evidence is far from the truth because the February 14, 1992 ordered clearly states that it was reached only after receiving the written statements and hearing the testimonies of several witnesses. Similarly, the Disciplinary Board's resolution dated March 10, 1991 was preceded by a hearing on March 2, 1991 wherein respondent students were summoned to answer clarificatory questions. UP v. Ligot-Telan Facts: In an effort to make the University of the Philippines (U.P.) truly the University of the People, U.P. administration conceptualized and implemented the socialized scheme of tuition fee payments through the Socialized Tuition Fee and Assistance Program (STFAP), popularly known as the "Iskolar ng Bayan" program. After broad consultations with the various university constituencies, U.P. President Jose V. Abueva, the U.P. Board of Regents issued on April 28, 1988 a Resolution establishing the STFAP. A year later, it was granted official recognition when the Congress of the Philippines allocated a portion of the National Budget for the implementation of the program. In the interest of democratizing admission to the State University, all students are entitled to apply for STFAP benefits which include reduction in fees, living and book subsidies and student assistantships which give undergraduate students the opportunity to earn P12.00 per hour by working for the University. Applicants are required to accomplish a questionnaire where, among others, they state the amount and source of the annual income of the family, their real and personal properties and special circumstances from which the

University may evaluate their financial status and need on the basis of which they are categorized into brackets. To further insure the integrity of the program, a random sampling scheme of verification of data indicated in a student's application form is undertaken. Among those who applied for STFAP benefits for School Year 1989-90 was Ramon P. Nadal, a student enrolled in the College of Law. A team composed of Arsenio L. Dona and Jose Carlo Manalo conducted a home investigation at the residence of Nadal. Ms. Cristeta Packing, Nadal's aunt, was interviewed and the team submitted a home visit report. Consolacion Urbino, Scholarship Affairs Officer II, found discrepancies between the report and Nadal's application form. Forthwith, she and Bella M. Villanueva, head of the Office of Scholarships and Student Services, presented the matter to the Diliman Committee on Scholarships and Financial Assistance. In compliance with the said Committee's directive, Bella Villanueva wrote Nadal informing him that the investigation showed that he had failed to declare, not only the fact that he had been maintaining a 1977 Corolla car which was owned by his brother but also the income of his mother who was supporting his brothers Antonio and Federico. Nadal was likewise informed that the Diliman Committee had reclassified him to Bracket 9 (from Bracket 4), retroactive to June 1989, unless he could submit "proofs to the contrary." Nadal was required "to pay back the equivalent amount of full school fees" with "interest based on current commercial rates." Failure to settle his account would mean the suspension of his registration privileges and the withholding of clearance and transcript of records. He was also warned that his case might be referred to the Student Disciplinary Tribunal for further investigation. commercial rates." Failure to settle his account would mean the suspension of his registration privileges and the withholding of clearance and transcript of records. He was also warned that his case might be referred to the Student Disciplinary Tribunal for further investigation. Nadal issued a certification stating, among other things, that his mother migrated to the United States in 1981 but because her residency status had not yet been legalized, she had not been able to find a "stable, regular, well-paying employment." U.P. charged Nadal before the Student Disciplinary Tribunal (SDT) that he committed acts which find him guilty of willfully and deliberately withholding information about the income of his mother, who is living abroad and that he was maintaining a Toyota Corolla car. As such, the SDT imposed upon Nadal the penalty of expulsion from the University and required him to reimburse all STFAP benefits he had received but if he does not voluntarily make reimbursement, it shall be "effected” by the University thru outside legal action. The SDT decision was thereafter automatically elevated to the Executive Committee of U.P. Diliman for review pursuant to Sec. 20 of the U.P. Rules on Student Conduct and Discipline. Board of regents modified the penalty from Expulsion to One Year- Suspension, effective immediately, plus reimbursement of all benefits received from the STFAP, with legal interest. However the BOR also decided against giving Nadal, a certification of good moral character. Nadal forthwith filed a motion for reconsideration of the BOR decision, in the next BOR meeting Regent Antonio T. Carpio raised the "material importance" of the truth of Nadal's claim that earlier, he was a beneficiary of a scholarship and financial aid from the Ateneo de Manila University (AdeMU). Learning that the "certification issued by the AdeMU that it had not given Nadal financial aid while he was a student there was made through a telephone call," Regent Carpio declared that there was as yet "no direct evidence in the records to substantiate the charge." According to Carpio, if it should be disclosed that Nadal falsely stated that he received such financial aid, it would be a clear case of gross and material misrepresentation that would even warrant the penalty of expulsion. Hence, he cast a conditional vote that would depend on the verification of Nadal's claim on the matter. U.P. President and concurrently Regent Jose V. Abueva countered by stating that "a decision should not be anchored solely on one piece of information which he considered irrelevant, and which would ignore the whole pattern of the respondent's dishonesty and deception

from 1989 which had been established in the investigation and the reviews."In the morning of March 29, 1993, the AdeMU issued a certification to the effect that Nadal was indeed a recipient of a scholarship grant from 1979 to 1983. That evening, the BOR met again at a special meeting, according to Regent Carpio, in executive session, the BOR found Nadal "guilty." However, on April 22, 1993, Nadal filed with the Regional Trial Court of Quezon City a petition for mandamus with preliminary injunction and prayer for a temporary restraining order against President Abueva, the BOR, Oscar M. Alfonso, Cesar A. Buenaventura, Armand V. Fabella and Olivia C. Caoili. Issue: Whether or not the Board of Regent violated Nadal's right to due process when it rendered a decision finding Nadal guilty of the charges against him" during the March 29, 1993 meeting. Held: With respect to the March 29, 1993 meeting, respondent considers the same as "unquestionably void for lack of due process" inasmuch as he was not sent a notice of said meeting, that imposition of sanctions on students requires "observance of procedural due process," the phrase obviously referring to the sending of notice of the meeting. However BOR ruled that in any event it is gross error to equate due process in the instant case with the sending of notice of the March 29, 1993 BOR meeting to respondent. University rules do not require the attendance in BOR meetings of individuals whose cases are included as items on the agenda of the Board. This is not exclusive of students whose disciplinary cases have been appealed to the Board of Regents as the final review body. At no time did respondent complain of lack of notice given to him to attend any of the regular and special BOR meetings where his case was up for deliberation. Counsel for Nadal charged before the lower court that Nadal was not given due process in the March 29 meeting because the ground upon which he was again convicted was not the same as the original charge. Obviously, he was referring to the basis of the conditional votes on March 28. Whether or not Nadal was telling the truth when he claimed that he received a scholarship grant from the AdeMU. However, Regent Carpio himself testified that the charge considered was "exactly the same charge" of withholding information on the income of Nadal's mother. It should be stressed that the reason why Regent Carpio requested a verification of Nadal's claim that he was a scholar at the AdeMU was that Regent Carpio was not "morally convinced" yet as to the guilt of Nadal. In other words, he sought additional insights into the character of Nadal through the information that would be obtained from the AdeMU. The Court in this regard find such information to be irrelevant and a mere superfluity. In his July, 12, 1991 certification aforementioned, Nadal admitted, although inconsistently, that his mother was a "TNT" who could not find a "stable, regular, well-paying employment" but that she was supporting the education of his brothers with the help of another son. The court constitutes this as a sufficient admission that Nadal withheld information on the income, however measly and irregular, of his mother. The court also sighted that respondent aspires to join the ranks of the professionals who would uphold truth at all costs so that justice may prevail. In those who exhibit duplicity in their student days, one spots the shady character who is bound to sow the seeds of chicanery in the practice of his profession. With this the court ruled that it sufficiently shown that respondent has committed an act of dishonesty in withholding vital information in connection with his application for STFAP benefits, all in blatant violation of the Rules and Regulations on Student Conduct and Discipline of petitioner University, the latter's inherent power and authority to impose disciplinary sanction may be invoked and rightfully exercised. Therefore deciding that the BOR did not violate Nadal’s right of due process. The lower court is hereby ordered to DISMISS the petition for mandamus.

Lao Gi v. CA Facts: Filomeno Chia Jr. was made a Filipino citizen by virtue of Opinion 191 by the Secretary of justice. However, this was revoked when his father’s citizenship was cast aside due to fraud and misrepresentation. Charges of deportation were filed against the Chias. Charges also alleged that they refused to register as aliens and that they committed acts of undesirability. The Chias said that the CID has no authority to deport them which was denied by the CID. They filed a petition with the Supreme Court for a writ of preliminary injunction which was dismissed for lack of merit. Their MFR was also denied. Earlier, Manuel Chia’s case of falsification of public documents in alleging he was a Filipino citizen. He was alleged to have done this for the sale of real property. The trial court acquitted him by saying that Opinion 191 was res judicata and cant be contravened by Opinion 147. The CID set the hearing for the deportation case against the Chias and told them to register as aliens. The Chias tooks further action. Their petition for injunctive relief was denied by the CFI of Manila. They also lost the appeal in the CA. The Chias was denied. In their SC petition, they seek to set aside the CA decision. They argued that they weren’t subject to immediate deportation, the presence of fraud in the citizenship, the CA’s overstepping of appellate jurisdiction, and the resolution of the SC didn’t make a ruling that the petitioner entered the Philippines by false pretenses. Issue: Does the CID have the jurisdiction to determine the deportation? Held: Yes. Petition granted Hearing must be continued to determine if they are really aliens. Section 37 of the Immigration act states: SEC. 37. (a) The following aliens shall be arrested upon the warrant of the Commissioner of Immigration or of any other officer designated by him for the purpose and deported upon the warrant of the Commissioner of Immigration after a determination by the Board of Commissioners of the existence of the ground for deportation as charged against the alien: (1) Any alien who enters the Philippines after the effective date of this Act by means of false and misleading statements or without inspection and admission by the immigration authorities at a designated port of entry or at any place other than at a designated port of entry. (As amended by Sec. 13, Rep. Act No. 503.) ... There must be a determination of the existence of the ground charged, particularly illegal entry into the country. Only after the hearing can the alien be deported. Also, there must be appositive finding from the CID that they are aliens before compelling them to register as such. This power is the police power to protect the state from undesirable aliens injurious to the public good. Since the deportation is a harsh process, due process must be observed. In the same law, it is provided that: No alien shall be deported without being informed of the specific grounds for deportation nor without being given a hearing under rules of procedure to be prescribed by the Commissioner of Immigration. The acts or omissions that they are charged of must be in ordinary language for the person to be informed and for the CID to make a proper judgment. Also, the warrants of arrewst must be in accordance with the rules on criminal procedure. On the information of a private prosecutor in the case: Deportation is the sole concern of the state. There is no justification for a private party to intervene.

Domingo v. Scheer FACTS: Herbert Markus Emil Scheer, a German, was given permanent status to reside in the Philippines on July 18, 1986. He married a Filipina and have 3 children. He also opened a restaurant in Puerto Prinsesa. One day, the Bureau of Immigration and Deportation (BID) received information that Scheer was wanted by the German Federal Police that a warrant of arrest had been issued against him. The BOC thereafter issued a Summary Deportation against Scheer. In issuing the said order, the BOC relied on the correspondence from the German Vice-Consul on its speculation that it was unlikely that the German Embassy will issue a new passport to the respondent; on the warrant of arrest issued by the District Court of Germany against the respondent for insurance fraud; and on the alleged illegal activities of the respondent in Palawan. The BOC concluded that the respondent was not only an undocumented but an undesirable alien as well. When the respondent was apprised of the deportation order, he forthwith aired his side to then BID Commissioner Leandro T. Verceles. The Commissioner allowed the respondent to remain in the Philippines, giving the latter time to secure a clearance and a new passport from the German Embassy. Respondent filed an MR. However, the BOC did not resolve the respondent's motion. The respondent was neither arrested nor deported. Meanwhile, on February 15, 1996, the District Court of Straubing rendered a Decision dismissing the criminal case against the respondent for physical injuries. The German Embassy in Manila, thereafter, issued a temporary passport to the respondent. In a Letter dated March 1, 1996, the respondent informed Commissioner Verceles that his passport had been renewed following the dismissal of the said criminal case. He reiterated his request for the cancellation of the Summary Deportation Order dated September 27, 1995, and the restoration of his permanent resident status.[19] Subsequently, on March 12, 1996, the German Embassy issued to the respondent a regular passport, to expire on March 11, 2006. The BOC still failed to resolve the respondent's Urgent Motion for Reconsideration. Commissioner Verceles did not respond to the respondents March 1, 1996, Letter. The respondent remained in the Philippines and maintained his business in Palawan. On March 20, 1997, the Department of Labor and Employment approved his application for Alien Employment Registration Certificate as manager of the Bavaria Restaurant in Puerto Princesa City. In the meantime, petitioner Immigration Commissioner Andrea T. Domingo assumed office. She wrote the German Embassy and inquired if the respondent was wanted by the German police. On April 12, 2002, the German Embassy replied that the respondent was not so wanted. At about midnight on June 6, 2002, Marine operatives and BID agents apprehended the respondent in his residence on orders of the petitioner. He was whisked to the BID Manila Office and there held in custody while awaiting his deportation. Despite entreaties from the respondent's wife and his employees, the petitioner refused to release the respondent. Shocked at the sudden turn of events, the respondent promptly communicated with his lawyer. The latter filed with the BID a motion for bail to secure the respondent's temporary liberty.

On June 11, 2002, the respondent's counsel filed with the Court of Appeals a petition for certiorari, prohibition, and mandamus with a prayer for temporary restraining order and writ of preliminary injunction, to enjoin the petitioner from proceeding with the respondent's deportation. The respondent (petitioner therein) alleged, inter alia, that his arrest and detention were premature, unjust, wrongful, illegal and unconstitutional, effected without sufficient cause and without jurisdiction or with grave abuse of discretion. He asserted that there was no speedy remedy open to him in the ordinary course of law and that his Urgent Motion for Reconsideration of the Summary Deportation Order of the BOC had not yet been resolved despite the lapse of more than six years. The respondent averred that he was a fully documented alien, a permanent resident and a law-abiding citizen. CA issued a status quo order restraining the petitioner from deporting the respondent on a bond of P100,000.00. BOC issued an Omnibus Resolution dated June 14, 2002, pendente lite denying the respondents Urgent Motion for Reconsideration, Motion for Bail/Recognizance. On August 20, 2002, the Court of Appeals rendered a Decision in favor of the respondent granting his petition for certiorari and prohibition and permanently enjoining the petitioner from deporting the respondent. ISSUE: 1. WON the Board of Commissioners is an indispensable party. 2. WON the the Non-joinder of an Indispensable Party is a Ground for the Dismissal of the Petition 3. WON The CA had Jurisdiction Over the Petition for Certiorari, Prohibition, and Mandamus HELD: 2. YES. The BOC is an Indispensable Party. We agree with the petitioner's contention that the BOC was an indispensable party to the respondents' petition for certiorari, prohibition, and mandamus in the Court of Appeals. The respondent was arrested and detained on the basis of the Summary Deportation Order of the BOC. The petitioner caused the arrest of the respondent in obedience to the said Deportation Order. The respondent, in his Memorandum, prayed that the CA annul not only the Summary Deportation Order of the BOC but also the latter's Omnibus Resolution, and, thus, order the respondents immediate release. The respondent also prayed that the CA issue a writ of mandamus for the immediate resolution of his Urgent Motion for Reconsideration. The said motion had to be resolved by the BOC as the order sought to be resolved and reconsidered was issued by it and not by the petitioner alone. The powers and duties of the BOC may not be exercised by the individual members of the Commission. 2. NO. Section 7, Rule 3 of the Rules of Court, as amended, requires indispensable parties to be joined as plaintiffs or defendants. The joinder of indispensable parties is mandatory. Without the presence of indispensable parties to the suit, the judgment of the court cannot attain real finality. Strangers to a case are not bound by the judgment rendered by the court. The absence of an indispensable party renders all subsequent actions of the court null and void. Lack of authority to act not only of the absent party but also as to those present. The responsibility of impleading all the indispensable parties rests on the petitioner/plaintiff. However, the non-joinder of indispensable parties is not a ground for the dismissal of an action. Parties may be added by order of the court on motion of the party or on its own initiative at any stage of the action and/or such times as are just. If the petitioner/plaintiff refuses to implead an indispensable party despite the order of the court, the latter may dismiss the complaint/petition for the petitioner/plaintiffs failure

to comply therefor.The remedy is to implead the non-party claimed to be indispensable. In this case, the CA did not require the respondent (petitioner therein) to implead the BOC as respondent but merely relied on the rulings of the Court in Vivo v. Arca, and Vivo v. Cloribel. The CAs reliance on the said rulings is, however, misplaced. The acts subject of the petition in the two cases were those of the Immigration Commissioner and not those of the BOC; hence, the BOC was not a necessary nor even an indispensable party in the aforecited cases. 3. YES. The settled rule is that the authority to exclude or expel aliens by a power affecting international relation is vested in the political department of the government, and is to be regulated by treaty or by an act of Congress, and to be executed by the executive authority according to the regulations so established, except in so far as the judicial department has been authorized by treaty or by statute, or is required by the Constitution to intervene. The judicial department cannot properly express an opinion upon the wisdom or the justice of the measures executed by Congress in the exercise of the power conferred on it, by statute or as required by the Constitution. Congress may, by statute, allow the decision or order of the Immigration Commissioner or the BOC to be reviewed by the President of the Philippines or by the courts, on the grounds and in the manner prescribed by law. Article VIII, Section 1 of the Constitution has vested judicial power in the Supreme Court and the lower courts such as the Court of Appeals, as established by law. Although the courts are without power to directly decide matters over which full discretionary authority has been delegated to the legislative or executive branch of the government and are not empowered to execute absolutely their own judgment from that of Congress or of the President, the Court may look into and resolve questions of whether or not such judgment has been made with grave abuse of discretion, when the act of the legislative or executive department violates the law or the Constitution. In Harvy Bridges v. I.F. Wixon, the United States Federal Supreme Court reversed an Order of Deportation made by the Attorney General for insufficiency of evidence and for improper admission of evidence. In Nging v. Nagh,the United States Court of Appeals (9th Circuit Court) held that conclusions of administrative offices on the issues of facts are invulnerable in courts unless when they are not rendered by fairminded men; hence, are arbitrary. In Toon v. Stump, the Court ruled that courts may supervise the actions of the administrative offices authorized to deport aliens and reverse their rulings when there is no evidence to sustain them. When acts or omissions of a quasi-judicial agency are involved, a petition for certiorari or prohibition may be filed in the Court of Appeals as provided by law or by the Rules of Court, as amended. In this case, the respondent alleges that the petitioner acted arbitrarily, contrary to law and with grave abuse of discretion in causing his arrest and detention at a time when his Urgent Motion for Reconsideration of the BOCs Summary Deportation Order had yet to be resolved. There was no factual or legal basis for his deportation considering that he was a documented alien and a law-abiding citizen; the respondent, thus, prayed for a writ of mandamus to compel the petitioner, the Chairperson of the BOC, to resolve the said motion. The petition before the CA did not involve the act or power of the President of the Philippines to deport or exclude an alien from the country. This being so, the petition necessarily did not call for a substitution of the Presidents discretion on the matter of the deportation of the respondent with that of the judgment of the CA. Philcomsat vs Alcuaz 180 SCRA 218 Facts: By virtue of Republic Act No. 5514, the Philippine Communications Satellite Corporation (PHILCOMSAT) was granted the authority to “construct and operate such ground facilities as needed to deliver telecommunications services from the communications

satellite system and ground terminal or terminals” in the Philippines. PHILCOMSAT provides satellite services to companies like Globe Mackay (now Globe) and PLDT. Under Section 5 of the same law, PHILCOMSAT was exempt from the jurisdiction, control and regulation of the Public Service Commission later known as the National Telecommunications Commission (NTC). However, Executive Order No. 196 was later promulgated and the same has placed PHILCOMSAT under the jurisdiction of the NTC. Consequently, PHILCOMSAT has to acquire permit to operate from the NTC in order to continue operating its existing satellites. NTC gave the necessary permit but it however directed PHILCOMSAT to reduce its current rates by 15%. NTC based its power to fix the rates on EO 546. PHILCOMSAT now sues NTC and its commissioner (Jose Luis Alcuaz) assailed the said directive and holds that the enabling act (EO 546) of the NTC, empowering it to fix rates for public service communications, does not provide the necessary standards which were constitutionally required, hence, there is an undue delegation of legislative power, particularly the adjudicatory powers of NTC. PHILCOMSAT asserts that nowhere in the provisions of EO 546, providing for the creation of NTC and granting its rate-fixing powers, nor of EO 196, placing PHILCOMSAT under the jurisdiction of NTC, can it be inferred that NTC is guided by any standard in the exercise of its rate-fixing and adjudicatory powers. PHILCOMSAT subsequently clarified its said submission to mean that the order mandating a reduction of certain rates is undue delegation not of legislative but of quasi-judicial power to NTC, the exercise of which allegedly requires an express conferment by the legislative body. ISSUE: Whether or not there is an undue delegation of power. HELD: No. There is no undue delegation. The power of the NTC to fix rates is limited by the requirements of public safety, public interest, reasonable feasibility and reasonable rates, which conjointly more than satisfy the requirements of a valid delegation of legislative power. Fundamental is the rule that delegation of legislative power may be sustained only upon the ground that some standard for its exercise is provided and that the legislature in making the delegation has prescribed the manner of the exercise of the delegated power. Therefore, when the administrative agency concerned, NTC in this case, establishes a rate, its act must both be non-confiscatory and must have been established in the manner prescribed by the legislature; otherwise, in the absence of a fixed standard, the delegation of power becomes unconstitutional. In case of a delegation of rate-fixing power, the only standard which the legislature is required to prescribe for the guidance of the administrative authority is that the rate be reasonable and just. However, it has been held that even in the absence of an express requirement as to reasonableness, this standard may be implied. However, in this case, it appears that the manner of fixing the rates was done without due process since no hearing was made in ascertaining the rate imposed upon PHILCOMSAT. Globe Telecom vs NTC FACTS: On 4 June 1999, Smart filed a Complaint with public respondent NTC,praying that NTC order the immediate interconnection of Smarts and Globes GSM networks. Smart alleged that Globe, with evident bad faith and malice, refused to grant Smarts request for the interconnection of SMS. Globe filed its Answer with Motion to Dismiss on 7 June 1999, interposing grounds that the Complaint was premature, Smarts failure to comply with the conditions precedent required in Section 6 of NTC Memorandum Circular 9-7-93,19 and its omission of the mandatory Certification of Non-Forum Shopping.

On 19 July 1999, NTC issued the Order now subject of the present petition. Both Smart and Globe were equally blameworthy for their lack of cooperation in the submission of the documentation required for interconnection and for having unduly maneuvered the situation into the present impasse. NTC held that since SMS falls squarely within the definition of value-added service or enhanced-service given 3333 in NTC Memorandum Circular No. 8-9-95 (MC No. 8-9-95) their implementation of SMS interconnection is mandatory.The NTC also declared that both Smart and Globe have been providing SMS without authority from it. Globe filed with the Court of Appeals a Petition for Certiorari and Prohibition to nullify and set aside the Order and to prohibit NTC from taking any further action in the case. Globe reiterated its previous arguments that the complaint should have been dismissed for failure to comply with conditions precedent and the non-forum shopping rule.They claimed that NTC acted without jurisdiction in declaring that it had no authority to render SMS, pointing out that the matter was not raised as an issue before it at all.They alleged that the Order is a patent nullity as it imposed an administrative penalty for an offense for which neither it nor Smart was sufficiently charged nor heard on in violation of their right to due process. The CA issued a TRO on 31 Aug 1999. In its Memorandum, Globe called the attention of the CA in an earlier NTC decision regarding Islacom, holding that SMS is a deregulated special feature and does not require the prior approval of the NTC. ISSUE: Whether or not the NTC’s order is not supported by substantial evidence. HELD: There is no legal basis under the PTA or the memorandum circulars promulgated by the NTC to denominate SMS as VAS, and any subsequent determination by the NTC on whether SMS is VAS should be made with proper regard for due process and in conformity with the PTA; the assailed Order violates due process for failure to sufficiently explain the reason for the decision rendered, for being unsupported by substantial evidence, and for imputing violation to, and issuing a corresponding fine on, Globe despite the absence of due notice and hearing which would have afforded Globe the right to present evidence on its behalf. CORONA VS. UHPAP FACTS: The Philippine Ports Authority [PPA] was created through PD 505, as amended by PD857 to “control, regulate, supervise pilots and the pilot age profession”. After hearing from relevant government agencies, pursuant to said charter, PPA General Manager Rogelio A. Dayan issued Administrative Order 04-92 [PPA-AO 04-92] and corresponding Memorandum Order in 1992, stating that all existing regular appointments which have been previously issued shall remain valid up to 31 December 1992 only and that all appointments to harbor pilot positions in all pilot age districts shall, henceforth, be only for a term of one year from date of effectivity subject to yearly renewal or cancellation by the Authority after conduct of a rigid evaluation of performance” to regulate and improve pilot services by instilling discipline and give better protection to port users. PPA-AO 04-92 replaces PPA-AO 03-85 which succinctly provides that, aspiring pilots must have a license and train as probationary pilots, and only upon satisfactory performance, are given permanent and regular appointments by the PPA itself and to exercise harbor pilot age until they reach the age of 70.Upon learning of PPA-AO 04-92 only after publication in the newspaper, the United Harbor Pilots Association of the Philippines: (a) questioned said PPA-AO twice before the DOTC, which Secretary Garcia said twice that only the PPA Board of Directors [as governing body] has exclusive jurisdiction to review, recall or annul PPA-AOs, (b) appealed to the Office of the President, which first issued a restraining order to the PPA on the implementation of the PPA-AO, and after PPA’s answer, then dismissed the

appeal/petition and lifted said order, stating, through Assistant Executive Secretary for Legal Affairs Renato C. Corona, that the PPAAO (i) merely implements PPA Charter, (ii) issuance is an act of PPA, not of its General Manager, (iii) merely regulates, not forbids practice of the profession, recognizing that such exercise is property right, and (iv) sufficiently complied with the requirement in the PD to consult only with relevant Government Agencies and (d) finally finding affirmative relief with Manila RTC Br. 6. Court, which ruled that (i) said PPA-AO is null and void (ii) PPA acted in excess of jurisdiction with grave abuse of discretion and (iii) imposed a permanent restraining order on PPA on its implementation.Assistant Executive Secretary Corona thus filed petition for review [of the Manila RTC Decision] to the Supreme Court. ISSUE: Whether or not the respondents have acted in excess of jurisdiction. Whether or not the Philippine Ports Authority (PPA) violate respondents’ right to exercise their profession and their right to due process of law. HELD: WHEREFORE, for all the foregoing, this Court hereby rules that: Respondents (herein petitioners) have acted in excess of jurisdiction and with grave abuse of discretion and in a capricious, whimsical and arbitrary manner in promulgating PPA Administrative Order 04-92 including all its implementing Memoranda, Circulars and Orders; PPA Administrative Order 04-92 and its implementing Circulars and Orders are declared null and void; The respondents are permanently enjoined from implementing PPA Administrative Order 04-92 and its implementing Memoranda, Circulars and Orders. No costs. SO ORDERED NPC vs. Zozobrado FACTS: Agustin A. Zozobrado, herein respondent, is a permanent employee of petitioner National Power Corporation (NPC) assigned as Pilot in the aviation group, received a letter from NPC President Frederico C. Puno, informing him that he was being dropped from the rolls. Zozobrado filed an appeal before the CSC questioning NPCs implementation of dropping him from the rolls. CSC dismissed Zobrado’s appeal and MR. Respondent filed with the Court of Appeals a Petition for Review on Certiorari under Rule 43 of the Rules of Court and was granted. Petitioner filed for MR but was denied. Respondent had been dropped by petitioner from the rolls due to Unsatisfactory or Poor Performance. CA ruled in favor of herein respondents finding that the separation was made with utter lack of due process. ISSUE: WON due process was followed in dropping respondent from the rolls. HELD: NO. Both the substantive and procedural aspect of due process were violated by petitioner in dismissing respondent. As to the procedural aspect, not even one requisites laid down by Memorandum Circular No. 12 has been complied with. Respondent was never notified in writing of his Unsatisfactory rating within 30

days from the end of the semester when the Unsatisfactory rating was given. Respondent was never warned in writing that a succeeding Unsatisfactory performance shall warrant his separation from the service. Even the allegation of the oral notice itself (that petitioner claims and respondent categorically denies) is clearly an afterthought, having been utilized for the first time in the Motion for Reconsideration of the assailed Court of Appeals decision and was never used as an argument in the administrative proceedings. The proof of such notice, a self-serving affidavit of the very individual who unilaterally gave the apparently groundless rating, deserves scant consideration. As to the substantive aspect, evidence shows that petitioner never denied that respondents unsatisfactory rating was due to respondents testimony in court concerning the graft charges against NPC employees. On the day respondent was supposed to testify in court under pain of contempt, Gen. Lagera suddenly sent him to fly the NPC President despite the fact that another pilot was assigned to such mission. Moreover, Gen. Lagera’s ill motive is further proved by the fact that respondent was kept in the dark as to the status of his employment even though the same had already been terminated two months earlier. It appears that the sad news was relayed to respondent only on his natal day affair. We can see no reason for the two months delay other than the devastation Gen. Lagera expected to cause by imparting the shocking news on respondents birth anniversary, during a celebration and in front of other people. As further found by the Court of Appeals, when respondent brought to the Grievance Committee the matter of his unsatisfactory ratings, the Grievance Committee recommended a review thereof to take into account respondents quantity of flying hours. Pilots have traditionally been rated by the number of flying hours spent in their career, and respondent had more than double the flying hours of the two other pilots of the Aviation Group combined. However, Gen. Lagera blocked such review, claiming that he had already considered the same, albeit minimally. This is a clear indication that Gen. Lagera really wanted to take it upon himself to solely give the Unsatisfactory ratings to respondent, in violation of the approved Performance Appraisal System (PAS) of the NPC. Salaw v. NLRC FACTS:Espero Santos Salaw was a credit investigator-appraiser of hxerein respondent Associated Bank. His duties included inspecting, investigating, appraising, and identifying the company's foreclosed assets; giving valuation to its real properties and verifying the genuineness and encumbrances of the titles of properties mortgaged to the respondents. Salaw and a fellow employee were alleged to have conspired in selling twenty (20) sewing machines and electric generators which had been foreclosed by the respondent bank from Worldwide Garment and L.P. Money Garment, for P60,000.00, and divided the proceeds thereof in equal shares of P30,000.00 between the two of them. The Criminal Investigation Service (CIS) of the Philippine Constabulary extracted Sworn Statement from them without the assistance of a counsel. Rollie Tuazon, the bank manager, requested petitioner to appear before the bank's Personnel Discipline and Investigation Committee (PDIC) which petitioner attended and 3 months after, his termination became effective for alleged serious misconduct or willful disobedience and fraud or willful breach of the trust reposed on him by the private respondents.

Petitioner filed an illegal dismissal case against respondent and likewise submitted an affidavit recanting his Sworn Statement before the CIS. The labor arbiter ruled in favor of the petitioner. Private respondents appealed to the NLRC and reversed the LA’s decision. Petitioner’s MR was denied. Hence, this petition. ISSUE: WON petitioner’s dismissal was legally justified. HELD: NO. Under the Labor Code, as amended, the requirements for the lawful dismissal of an employee by his employer are two-fold: the substantive and the procedural. Not only must the dismissal be for a valid or authorized cause as provided by law (Articles 279, 281, 282284, New Labor Code), but the rudimentary requirements of due process — notice and hearing — must also be observed before an employee may be dismissed. One does not suffice; without their concurrence, the terminate would, in the eyes of the law, be illegal. As to the LA’s finding, petitioner was terminated without the benefit of due process of law. The respondents' initial act in convening their Personnel Discipline and Investigation Committee (PDIC) to investigate complainant (after the CIS experience) would have complied with the demands of due process had complainant been given the opportunity to present his own defense and confront the witnesses, if any, and examine the evidence against him. But as the records clearly show, the complainant was denied that constitutional right when his subsequent request refute the allegations against him was granted and a hearing was set "without counsel or representative. The investigation of petitioner Salaw by the respondent Bank' investigating committee violated his constitutional right to due process, in as much as he was not given a chance to defend himself, as provided in Rule XIV, Book V of the Implementing Rules and Regulations of the Labor Code governing the dismissal of employees. Section 5 of the said Rule requires that "the employer shall afford the worker ample opportunity to be heard and to defend himself with the assistance of his representative if he so desires."11 (Emphasis supplied.) Here petition was perfunctorily denied the assistance of counsel during the investigation to be conducted by the PDIC. No reasons preferred which vitiated the denial with irregularity and unfairness. As aptly observed by the labor arbiter, the respondents premised their action in dismissing the complainant on his supposed admission of the offense imputed to him by the Criminal Investigation Service (CIS) in its interrogation in November 1984. The said admission was carried in a three-page Sworn Statement signed by the complainant. Aside from this Statement, other evidence was presented by the respondents to establish the culpability of the complainant in the fraudulent sale of respondents' foreclosed properties. Even the minutes of proceeding taken during the investigation conducted by respondents were not presented. ... This is a glaring denial of due process. Considering further that the admission by the petitioner which was extracted from him by the Criminal Investigate Service of the Philippine Constabulary (National Capital Region) without the assistance of counsel and which was made the sole basis for his dismissal, cannot be admitted in evidence against him, then, the finding of guilt of the PDIC, which was affirmed by the public respondent NLRC; has no more leg stand on. A decision with absolutely nothing to support it is a nullity. Significantly, the dismissal of the petitioner from his employment was characterized by undue haste. The law is clear that even in the disposition of labor cases, due process must not be subordinated to

expediency or dispatch. Otherwise, the dismissal of the employee will be tainted with illegality. Castillo Co vs Barbers FACTS: Petitioner Josie Castillo-Co is a Governor of Quirino. Congressman Junie Cua filed a complaint before the Office of the Ombudsman against Governor Castillo-Co and Provincial Engineer Virgilio Ringor for alleged fraud against the public treasury and malversation. (Sections 3(e) and 3(g) of the Anti-Graft and Corrupt Practices Act, as amended, and Articles 213 and 217 of the Revised Penal Code.) Congressman Cua charged that the equipment purchased was reconditioned instead of brand new as required by resolutions of the provinces Sanggunian authorizing such purchase. Other irregularities claimed to have been committed included overpricing, lack of public bidding, lack of inspection, advance payment prior to delivery in violation of Section 338 of the Local Government Code, and an attempt to cover up such irregularities. A week after the complaint was filed, Governor Castillo-Co and Provincial Engineer Ringor were placed under preventive suspension for a period of six (6) months. Said order was signed by Emilio A. Gonzalez III, Director, and approved by Jesus Guerrero, Deputy Ombudsman for Luzon. Petitioners thereafter filed separate motions for reconsideration. Both motions were denied in a Joint Order signed by Director Gonzales and approved by Deputy Ombudsman Guerrero. Petitioner file for special civil action for certiorari and prohibition, with a prayer for temporary restraining order/writ of preliminary injunction, seeks to nullify the Order of the Deputy Ombudsman directing her preventive suspension and claiming that the Deputy Ombudsman has no authority to issue the preventive suspension against them. ISSUE: WON the Deputy Ombudsman has authority to issue the preventive suspension WON petitioner was denied due process because she was not afforded the opportunity to controvert the evidence against her before the order of preventive suspension was issued. HELD: 1. YES. Petitioner claims that under Republic Act No. 7975, only the Ombudsman has the authority to sign the order placing officials with a 27 salary grade or above, like petitioner-governor, under preventive suspension. In this case, the suspension order was neither signed nor approved by Ombudsman Aniano Desierto. Rather, said order was signed by Director Emilio Gonzales III and approved by Deputy Ombudsman for Luzon Jesus Guerrero. There is nothing in RA 7975, however, that would remotely suggest that only the Ombudsman, and not his Deputy, may sign an order preventively suspending officials occupying positions classified as grade 27 or above. Moreover, Section 24 of Republic Act No. 6770 and Section 9, Rule III of the Rules of Procedure of the Office of Ombudsman similarly provides: SEC. 24. Preventive Suspension. -- The Ombudsman or his Deputy may preventively suspend any officer or employee under his authority pending an investigation, if in his judgment, the evidence of guilt is strong, and (a) the charge against such officer or employee involves dishonesty, oppression or gross misconduct, or neglect in the performance of duty; or (b) the charge would warrant removal from the service; or (c) the respondents continued stay in office may prejudice the case filed against him. Xxx

SEC. 9. Preventive suspension. Pending investigation, the respondent may be preventively suspended without pay for a period of not more than six (6) months, if, in the judgment of the Ombudsman or his proper deputy, the evidence of guilt is strong, and (a) the charge against such officer or employee involves dishonesty, oppression or gross misconduct, or neglect in the performance of duty, (b) the charge would warrant removal from the service; or (c) the respondents continued stay in office may prejudice the case filed against him. xxx Under these provisions, there cannot be any doubt that the Ombudsman or his Deputy may preventively suspend an officer or employee, where appropriate, as indicated by the word or between the Ombudsman and his Deputy. The word or is a disjunctive term signifying disassociation and independence of one thing from each of the other things enumerated. The law does not require that only the Ombudsman himself may sign the order of suspension. 2. NO. A preventive suspension, however, can be decreed on an official under investigation after charges are brought and even before the charges are heard since the same is not in the nature of a penalty,[13] but merely a preliminary step in an administrative investigation. In connection with the suspension of petitioner before he could file his answer to the administrative complaint, suffice it to say that the suspension was not a punishment or penalty for the acts of dishonesty and misconduct in office, but only as a preventive measure. A suspension is a preliminary step in an administrative investigation. If after such investigation, the charges are established and the person investigated is found guilty of acts warranting his removal, then he is removed or dismissed. This is the penalty. There is, therefore, nothing improper in suspending an officer pending his investigation and before the charges against him are heard and be given an opportunity to prove his innocence. The fact that the said order was issued seven days after the complaint was filed did not constitute grave abuse of discretion. The immediate issuance of such order is required in order to prevent the subject of the suspension from committing further irregularities. Such prompt action, moreover, is in consonance with Section 15 of RA 6770 which exhorts the Ombudsman to: xxx give priority to complaints filed against high ranking government officials and/or those occupying supervisory positions, complaints involving grave offenses as well as complaints involving large sums of money and/or properties. American Inter-Fashion v. OP Facts: Glorious Sun Fashion was found guilty by Garments and Textile Export Board GTEB of dollar salting and mis-declaration of importations. As a result, its export quotas were cancelled. After GTEB rendered its decision, Glorious filed with the Court a petition for certiorari and prohibition contending that its right to due process of law was violated and that GTEB decision was not supported by substantial evidence. The Court then issued a resolution ordering GTEB to conduct further proceedings. However, on July 25, 1984, Glorious filed a manifestation of its intention to withdraw the petition. The Court granted the motion for withdrawal. Glorious filed another motion to dismiss with prejudice which the Court duly noted. After two years, Glorious filed with GTEB a petition for restitution of its export quota allocation and requested for a reconsideration of the GTEB decision dated April 27, 1984. Glorious once again alleged that the charges against them were not supported by evidence. Moreover, it alleged that the GTEB decision cancelling its export quota was rendered as a result of duress, threats, intimidation and undue influence exercised by former Minister Roberto V. Ongpin in order to transfer Glorious export quotas to “Marcos crony-owned” corporations. Glorious further alleged that it was coerced by Mr. Roberto Ongpin to withdraw its petition in G.R. No. 67180 and to enter into joint venture

agreements paving the way for the creation of De Soleil Apparel and AIFC. On Sept. 4, 1987, GTEB denied the petition of Glorious. An appeal was then taken on Oct. 5, 1987 to the Office of the President. AIFC filed its opposition to Glorious’ appeal claiming that the GTEB decision dated April 27, 1984 has long been final. The Office of the President ruled in favor of Glorious and remanded the case to GTEB for further proceedings. The motion for reconsideration of AIFC was subsequently denied. Hence, this petition. Issue: 1. W/N the previous GTEB decision constituted res judicata to the instant case on the ground that the former decision was a final judgment on the merits. – NO 2. W/N Glorious was accorded due process in relation to the 1984 GTEB decision. – NO Held:The petitioner contends that in entertaining the appeal of private respondent Glorious, the Office of the President “had unwittingly made itself a tool in a cunning move to resurrect a decision which had become final and executory more than three years earlier. The petitioner asseverates resolution dismissing G.R. No. 67180 was res judicata on the matter. The Supreme Court said that one of the requirements for a judgment to be a bar to a subsequent case is that the it must be a judgment on the merits. A judgment is upon the merits when it amounts to a declaration of the law as to the respective rights and duties of the parties, based upon the ultimate fact or state of facts disclosed by the pleadings and evidence, and upon which the right of recovery depends, irrespective of formal, technical or dilatory objection or contentions. Certainly, the dismissal of G.R. No. 67180 cannot be categorized as a judgment on the merits. The action in 1984 did not resolve anything. In fact, when the court heard the parties during the oral arguments, GTEB was not able to present any showing of mis-declaration if imports. The motion to withdraw te petition arose from the fears of Mr. Nemesio Co that not only Glorious Sun but his other businesses would be destroyed by the martial law regime. The resolution dismissing G.R. No. 67180 was based solely on the notice of withdrawal by the private respondent. The dismissal of the petition was clearly based on a technical matter rather than on the merits of the petition. Hence, it cannot constitute res judicata. With regards to the second issue, the Petitioner contend that Glorious Sun was not denied due process. Although AIFC admits that the 1984 GTEB decision failed to disclose to Glorious vital evidence used by GTEB in arriving at its conclusion that Glorious was guilty of dollarsalting, it contends that the subsequent disclosure in 1987, where relevant documents were given to Glorious and that the latter was given an opportunity to comment thereon, cured the defect. This contention by AIFC, the court holds, is MISLEADING. The SC recognized that the instant petition involves the 1984 resolution of the GTEB. AIFC cannot use as an excuse the subsequent disclosure of the evidence used by the GTEB to Glorious in 1987 to justify the 1984 GTEB resolution. The glaring fact is that Glorious was denied due process when GTEB failed to disclose evidence used by it in rendering a resolution against Glorious. Moreover, the documents disclosed to Glorious by GTEB in 1987 enhanced the charge that the former was denied due process. Attention was also brought to the Puno affidavit, wherein Puno, the Chairman of the Investigating Panel created by the Ministry of Trade and Industry admitted that he was pressured by Minister Ongpin to look for ways and means to remove the quotas from Glorious. AIFC claims that it is an inconsequential matter in that the GTEB Board did not give credence to it and also, none of the members of the committee would agree that there was any pressure or instruction from Minister Ongpin.

The Supreme Court said that the fact that the other members would not agree that there was pressure from Ongpin does not mean that Puno was not telling the truth. Mr. Puno stated that he was pressured. He did not state that the members of the investigating panel were pressured. Mr. Puno was the Chairman of the Investigating Panel. Hence, it is plausible that in view of his position he was the one pressured by Minister Ongpin. There is every reason to suspect that even before Glorious Sun was investigated, a decision to strip it of its quotas and to award them to friends of their administration had already been made. The Supreme Court also held that although factual findings of administrative agencies are generally accorded respect, such factual findings may be disregarded if they are not supported by evidence; where the findings are initiated by fraud, imposition or collusion; where the procedures which lead to the factual findings are irregular; when palpable errors are committed; or when grave abuse of discretion arbitrariness or capriciousness is manifest. Contrary to the petitioners posture, the record clearly manifests that in cancelling the export quotas of the private respondent GTEB violated the private respondent’s constitutional right to due process. Before the cancellation in 1984, Glorious had been enjoying export quotas granted to it since 1977. In effect the private respondent’s export quota allocation which initially was a privilege evolved into some form of property right which should not be removed from it arbitrarily and without due process only to hurriedly confer it on another. The motion for reconsideration was GRANTED. The instant petition is DISMISSED. United States v. Toribio 15 Phil. 85 Facts: Sometime in the 1900s, in the town of Carmen, province of Bohol, Toribio applied for a license to have his carabao be slaughtered. His request was denied because his carabao is found to be fit for agricultural work. Even so, he still slaughtered his carabao for the purpose of human consumption. The trial court of Bohol found that the respondent slaughtered or caused to be slaughtered a carabao without a permit from the municipal treasurer of the municipality, in violation of Sections 30 and 33 of Act No. 1147, an Act regulating the registration, branding, and slaughter of Large Cattle. The act prohibits the slaughter of large cattle fit for agricultural work or other draft purposes for human consumption. The counsel for appellant contended that the provisions of Act No. 1147 do not prohibit nor penalize the slaughter of large cattle without a permit of the municipal treasure if the slaughtering of large cattle happened outside the municipal slaughterhouse. They said that the prohibition and penalty is limited only to the large cattle slaughtered at the municipal slaughterhouse for the prohibition contained in section 30 and the penalty imposed in section 33 stated only the phrase “at the municipal slaughterhouse”. They also contended that the act constitutes a taking of property for public use in the exercise of the right of eminent domain without providing for the compensation of owners, and it is an undue and unauthorized exercise of police power of the state for it deprives them of the enjoyment of their private property. Ruling: The extent and limits of what is known as the police power have been a fruitful subject of discussion in the appellate courts of nearly every State in the Union. It is universally conceded to include everything essential to the public safely, health, and morals, and to justify the destruction or abatement, by summary proceedings, of whatever may be regarded as a public nuisance. Under this power it has been held that the State may order the destruction of a house falling to decay or otherwise endangering the lives of passers‐by; the demolition of such as are in the path of a conflagration; the slaughter of diseased cattle; the destruction of decayed or unwholesome food; the prohibition of wooden buildings in cities; the regulation of railways and other means of public conveyance, and of interments in burial

grounds; the restriction of objectionable trades to certain localities; the compulsary vaccination of children; the confinement of the insane or those afficted with contagious deceases; the restraint of vagrants, beggars, and habitual drunkards; the suppression of obscene publications and houses of ill fame; and the prohibition of gambling houses and places where intoxicating liquors are sold. Beyond this, however, the State may interfere wherever the public interests demand it, and in this particular a large discretion is necessarily vested in the legislature to determine, not only what the interests of the public require, but what measures are necessary for the protection of such interests. (Barbier vs. Connolly, 113 U. S., 27; Kidd vs. Pearson, 128 U. S., 1.) To justify the State in thus interposing its authority in behalf of the public, it must appear, first, that the interests of the public generally, as distinguished from those of a particular class, require such interference; and, second, that the means are reasonably necessary for the accomplishment of the purpose, and not unduly oppressive upon individuals. The legislature may not, under the guise of protecting the public interests, arbitrarily interfere with private business, or impose unusual and unnecessary restrictions upon lawful occupations. In other words, its determination as to what is a proper exercise of its police powers is not final or conclusive, but is subject to the supervision of the court. From what has been said, we think it is clear that the enactment of the provisions of the statute under consideration was required by "the interests of the public generally, as distinguished from those of a particular class;" and that the prohibition of the slaughter of carabaos for human consumption, so long as these animals are fit for agricultural work or draft purposes was a "reasonably necessary" limitation on private ownership, to protect the community from the loss of the services of such animals by their slaughter by improvident owners, tempted either by greed of momentary gain, or by a desire to enjoy the luxury of animal food, even when by so doing the productive power of the community may be measurably and dangerously affected. Chief Justice Redfield, in Thorpe vs. Rutland & Burlington R. R. Co. (27 Vt., 140), said (p. 149) that by this "general police power of the State, persons and property are subjected to all kinds of restraints and burdens, in order to secure the general comfort, health, and prosperity of the State; of the perfect right in the legislature to do which no question ever was, or, upon acknowledge and general principles, ever can be made, so far as natural persons are concerned." Ynot vs. Intermediate Appellate Court [G.R. No. 74457, March 20, 1987] FACTS: Petitioner in this case transported six carabaos in a pump boat from Masbate to Iloilo on January 13, 1984, when they were confiscated by the police station commander of Barotac Nuevo, Iloilo for the violation of E.O. No. 626-A which prohibits the slaughter of carabaos except under certain conditions. Petitioner sued for recovery, and the trial Court of Iloilo issued a writ of replevin upon his filing of a supersedeas bond of twelve thousand pesos (P 12, 000.00). After considering the merits of the case, the court sustained the confiscation of the said carabaos and, since they could no longer be produced, ordered the confiscation of the bond. The court also declined to rule on the constitutionality of the E.O, as raised by the petitioner, for lack of authority and also for its presumed validity. Issue: Whether or not the said Executive Order is unconstitutional. Ruling: The Respondent contends that it is a valid exercise of police power to justify EO 626-A amending EO 626 in asic rule prohibiting the slaughter of carabaos except under certain conditions. The supreme court said that The reasonable connection between the means employed and the purpose sought to be achieved by the questioned measure is missing the Supreme Court do not see how the prohibition of the inter-provincial transport of carabaos can prevent their indiscriminate slaughter, considering that they can be killed anywhere, with no less difficulty in one province than in another. Obviously,

retaining the carabaos in one province will not prevent their slaughter there, any more than moving them to another province will make it easier to kill them there The Supreme Court found E.O. 626-A unconstitutional. The executive act defined the prohibition, convicted the petitioner and immediately imposed punishment, which was carried out forthright. Due process was not properly observed. In the instant case, the carabaos were arbitrarily confiscated by the police station commander, were returned to the petitioner only after he had filed a complaint for recovery and given a supersedeas bond of P12,000.00. The measure struck at once and pounced upon the petitioner without giving him a chance to be heard, thus denying due process. though police power was invoked by the government in this case for the reason that the present condition demand that the carabaos and the buffaloes be conserved for the benefit of the small farmers who rely on them for energy needs, it does not however, comply with the second requisite for a valid exercise of the said power which is, "that there be a lawful method." The reasonable connection between the means employed and the purpose sought to be achieved by the questioned measure is missing. The challenged measure is an invalid exercise of Police power because the method employed to conserve the carabaos is not reasonably necessary to the purpose of the law and, worse, is unduly oppressive. To justify the State in the imposition of its authority in behalf of the public, it must be: 1) The interest of the public generally, as distinguished from those of a particular class, require such interference; 2) that the means employed are reasonably necessary for the accomplishment of the purpose, and not unduly oppressive upon individuals. Churchill v. Rafferty 32 Phil. 580 Substantive due process -- whether an act of government has su fficient justification for depriving a person of life, liberty, or propert y. FACTS: This is an appeal of a judgement rendered perpetually res training defendant and his deputies from collecting and enforcing ag ainst plaintiffs and their property the annual tax prescribed in section 100 of Act No. 2339, from destroying or removing any sign on the property of the plaintiffs for the sole reason that it may be offensive to sight, and cancelling the bond given by the plaintiffs to secure the issuance of the preliminary injunction granted after the comme ncement of the action. The Act allows the Collector of Internal Re venue to decide whether a sign or billboard displayed to the public is offensive to sight or is otherwise a nuisance and empowers hi m to order its removal. Plaintiffs billboards are posted on privat e lands in Rizal. The Act also expressly forbids the use of an i njunction to stay the collection of any internal revenue tax, and prov ides a remedy for any wrong in connection to such taxes. This reme dy was intended to be exclusive, thereby precluding the remedy by injunction which should not issue as of course, but is granted only upon the oath of a party and when there is no adequate remedy at la w. These sections take away the preventive remedy of injunction, leaving ordinary remedial actions available to taxpayers. The Attorney-General contends that there is no provisions of the paramount law which prohibit such a course, while the plaintiff s urge that the two sections are unconstitutional because they att empt to deprive aggrieved taxpayers of all substantial remedies for the protection of their property, depriving them of it without due proc ess. ISSUES: 1.Whether or not portions of section 100 of Act No. 2 339, empowering the Collector of Internal Revenue to remove bil

lboards as nuisances if objectionable to sight, is unconstitutional as it is a deprivation of property without due process of law. HELD: 1.No. Offensive noises and smells have been for a long tim e considered susceptible of suppression in thickly populated districts . “Ostensibly located on private property, the real and sole value of the billboard is its proximity to the public thoroughfares. Hence, we conceive that the regulation of billboards and their restriction i s not so much a regulation of private property as it is a regulation of the use of the streets and other public thoroughfares.” The inquir y is limited to the question of whether the enactment assailed by the plaintiffs was a legitimate exercise of the police power of the Go vernment, for all property is held subject to that power. The Court s tates that the exercise of police power belongs to the Legislature, a nd that power is limited only by the Acts of Congress and those principles which are the foundation of all republican forms of g overnment, and where the Act is reasonably within the proper consi deration of and care for the public health, safety, or comfort, it should not be disturbed by the courts. People. v. Fajardo 104 SCRA 443 Facts: Aug. 15, 1950 - Juan Fajardo was the mayor of Baoo, Camarines Sur. During his term the municipal council passed Ordinance No. 7 which prohibited the construction or repair of any building without a written permit from the mayor prior to construction or repairing. 1954 - Fajardo and Babillonia (Fajardo’s son-in-law) applied for a permit to construct a building adjacent to their gas station, still on Fajardo’s private land, separated from public plaza by a creek. Jan. 16, 1954 – request denied because it would destroy the view of the public plaza. o Applicants appealed but were turned down again on Jan. 18, 1954. Fajardo and Babillonia proceeded to construct even without a permit because they claimed that they needed a residence badly due to a typhoon destroying their previous place of residence Feb. 26, 1954 – Fajardo et at., were charged and convicted by peace court of Baoo for violating Ordinance no. 7 o

CFI – Affirmed 

CA forwarded the case to the SC because “the appeal attacks the constitutionality of the ordinance in question.”

Issue: Is the ordinance constitutional? Held: No, petition granted. The ordinance doesn’t state any standard that limits the grant of power to the mayor. It is an arbitrary and unlimited conferment. Ordinances which thus invest a city council with a discretion which is purely arbitrary, and which may be exercised in the interest of a favored few, are unreasonable and invalid. The ordinance should have established a rule by which its impartial enforcement could be secured. All of the authorities cited above sustain this conclusion. The ordinance is unreasonable and oppressive, in that it operates to permanently deprive appellants of the right to use their own property; hence, it oversteps the bounds of police power, and amounts to a taking of appellants property without just compensation. While property may be regulated to the interest of the general welfare, and the state may eliminate structures offensive to the sight, the state may not permanently divest owners of the beneficial use of their property and practically confiscate them solely to preserve or assure the aesthetic appearance of the community.

Fajardo would be constrained to let the land be fallow and not be used for urban purposes. To do this legally, there must be just compensation and they must be given an opportunity to be heard. An ordinance which permanently so restricts the use of property that it can not be used for any reasonable purpose goes, it is plain, beyond regulation and must be recognized as a taking of the property. The validity was also refuted by the Admin Code which states: SEC. 2243. Certain legislative powers of discretionary character. — The municipal council shall have authority to exercise the following discretionary powers: xxx

xxx

xxx

(c) To establish fire limits in populous centers, prescribe the kinds of buildings that may be constructed or repaired within them, and issue permits for the creation or repair thereof, charging a fee which shall be determined by the municipal council and which shall not be less than two pesos for each building permit and one peso for each repair permit issued. The fees collected under the provisions of this subsection shall accrue to the municipal school fund. Since, there was absolutely no showing in this case that the municipal council had either established fire limits within the municipality or set standards for the kind or kinds of buildings to be constructed or repaired within them before it passed the ordinance in question, it is clear that said ordinance was not conceived and promulgated under the express authority of sec. 2243 (c) Ermita-Manila Hotel 7 Motel Operator v. City of Manila 20 SCRA 849 Facts: Ermita-Malate Hotel and Motel Operators Association, and one of its members Hotel del Mar Inc. petitioned for the prohibition of Ordinance 4670 on June 14, 1963 to be applicable in the city of Manila. They claimed that the ordinance was beyond the powers of the Manila City Board to regulate due to the fact that hotels were not part of its regulatory powers. They also asserted that Section 1 of the challenged ordinance was unconstitutional and void for being unreasonable and violative of due process insofar because it would impose P6,000.00 license fee per annum for first class motels and P4,500.00 for second class motels; there was also the requirement that the guests would fill up a form specifying their personal information. There was also a provision that the premises and facilities of such hotels, motels and lodging houses would be open for inspection from city authorites. They claimed this to be violative of due process for being vague. The law also classified motels into two classes and required the maintenance of certain minimum facilities in first class motels such as a telephone in each room, a dining room or, restaurant and laundry. The petitioners also invoked the lack of due process on this for being arbitrary. It was also unlawful for the owner to lease any room or portion thereof more than twice every 24 hours. There was also a prohibition for persons below 18 in the hotel. The challenged ordinance also caused the automatic cancellation of the license of the hotels that violated the ordinance. The lower court declared the ordinance unconstitutional. Hence, this appeal by the city of Manila.

Issue:Whether Ordinance No. 4760 of the City of Manila is violative of the due process clause?

individual and for the greater good of the peace and order of society and the general well-being.

Held: No. Judgment reversed.

Laurel- The citizen should achieve the required balance of liberty and authority in his mind through education and personal discipline, so that there may be established the resultant equilibrium, which means peace and order and happiness for all.

Ratio:"The presumption is towards the validity of a law.” However, the Judiciary should not lightly set aside legislative action when there is not a clear invasion of personal or property rights under the guise of police regulation. O'Gorman & Young v. Hartford Fire Insurance Co- Case was in the scope of police power. As underlying questions of fact may condition the constitutionality of legislation of this character, the resumption of constitutionality must prevail in the absence of some factual foundation of record for overthrowing the statute." No such factual foundation being laid in the present case, the lower court deciding the matter on the pleadings and the stipulation of facts, the presumption of validity must prevail and the judgment against the ordinance set aside.” There is no question but that the challenged ordinance was precisely enacted to minimize certain practices hurtful to public morals, particularly fornication and prostitution. Moreover, the increase in the licensed fees was intended to discourage "establishments of the kind from operating for purpose other than legal" and at the same time, to increase "the income of the city government." Police power is the power to prescribe regulations to promote the health, morals, peace, good order, safety and general welfare of the people. In view of the requirements of due process, equal protection and other applicable constitutional guaranties, however, the power must not be unreasonable or violative of due process. There is no controlling and precise definition of due process. It has a standard to which the governmental action should conform in order that deprivation of life, liberty or property, in each appropriate case, be valid. What then is the standard of due process which must exist both as a procedural and a substantive requisite to free the challenged ordinance from legal infirmity? It is responsiveness to the supremacy of reason, obedience to the dictates of justice. Negatively put, arbitrariness is ruled out and unfairness avoided. Due process is not a narrow or "technical conception with fixed content unrelated to time, place and circumstances," decisions based on such a clause requiring a "close and perceptive inquiry into fundamental principles of our society." Questions of due process are not to be treated narrowly or pedantically in slavery to form or phrase. Nothing in the petition is sufficient to prove the ordinance’s nullity for an alleged failure to meet the due process requirement. Cu Unjieng case: Licenses for non-useful occupations are also incidental to the police power and the right to exact a fee may be implied from the power to license and regulate, but in fixing amount of the license fees the municipal corporations are allowed a much wider discretion in this class of cases than in the former, and aside from applying the well-known legal principle that municipal ordinances must not be unreasonable, oppressive, or tyrannical, courts have, as a general rule, declined to interfere with such discretion. Eg. Sale of liquors. Lutz v. Araneta- Taxation may be made to supplement the state’s police power. In one case- “much discretion is given to municipal corporations in determining the amount," here the license fee of the operator of a massage clinic, even if it were viewed purely as a police power measure. On the impairment of freedom to contract by limiting duration of use to twice every 24 hours- It was not violative of due process. 'Liberty' as understood in democracies, is not license; it is 'liberty regulated by law.' Implied in the term is restraint by law for the good of the

The freedom to contract no longer "retains its virtuality as a living principle, unlike in the sole case of People v Pomar. The policy of laissez faire has to some extent given way to the assumption by the government of the right of intervention even in contractual relations affected with public interest. What may be stressed sufficiently is that if the liberty involved were freedom of the mind or the person, the standard for the validity of governmental acts is much more rigorous and exacting, but where the liberty curtailed affects at the most rights of property, the permissible scope of regulatory measure is wider. On the law being vague on the issue of personal information, the maintenance of establishments, and the “full rate of payment”Holmes- “We agree to all the generalities about not supplying criminal laws with what they omit but there is no canon against using common sense in construing laws as saying what they obviously mean." White Light Corp. vs. City of Manila, G.R. No. 122846, January 20, 2009 Facts: On December 3, 1992, City Mayor Alfredo S. Lim signed into law Manila City Ordinance No. 7774 entitled “An Ordinance Prohibiting Short-Time Admission, Short-Time Admission Rates, and Wash-Up Rate Schemes in Hotels, Motels, Inns, Lodging Houses, Pension Houses, and Similar Establishments in the City of Manila” (the Ordinance).” The ordinance sanctions any person or corporation who will allow the admission and charging of room rates for less than 12 hours or the renting of rooms more than twice a day. The petitioners White Light Corporation (WLC), Titanium Corporation (TC), and Sta. Mesa Tourist and Development Corporation (STDC), who own and operate several hotels and motels in Metro Manila, filed a motion to intervene and to admit attached complaint-in-intervention on the ground that the ordinance will affect their business interests as operators. The respondents, in turn, alleged that the ordinance is a legitimate exercise of police power. RTC declared Ordinance No. 7774 null and void as it “strikes at the personal liberty of the individual guaranteed and jealously guarded by the Constitution.” Reference was made to the provisions of the Constitution encouraging private enterprises and the incentive to needed investment, as well as the right to operate economic enterprises. Finally, from the observation that the illicit relationships the Ordinance sought to dissuade could nonetheless be consummated by simply paying for a 12-hour stay, When elevated to CA, the respondents asserted that the ordinance is a valid exercise of police power pursuant to Section 458 (4)(iv) of the Local Government Code which confers on cities the power to regulate the establishment, operation and maintenance of cafes, restaurants, beerhouses, hotels, motels, inns, pension houses, lodging houses and other similar establishments, including tourist guides and transports. Also, they contended that under Art III Sec 18 of Revised Manila Charter, they have the power to enact all ordinances it may deem necessary and proper for the sanitation and safety, the furtherance of the prosperity and the promotion of the morality, peace, good order, comfort, convenience and general welfare of the city and its inhabitants and to fix penalties for the violation of ordinances. Petitioners argued that the ordinance is unconstitutional and void since it violates the right to privacy and freedom of movement; it is an

invalid exercise of police power; and it is unreasonable and oppressive interference in their business. CA, in turn, reversed the decision of RTC and affirmed the constitutionality of the ordinance. First, it held that the ordinance did not violate the right to privacy or the freedom of movement, as it only penalizes the owners or operators of establishments that admit individuals for short time stays. Second, the virtually limitless reach of police power is only constrained by having a lawful object obtained through a lawful method. The lawful objective of the ordinance is satisfied since it aims to curb immoral activities. There is a lawful method since the establishments are still allowed to operate. Third, the adverse effect on the establishments is justified by the well-being of its constituents in general. Hence, the petitioners appeared before the SC. Issue: Whether Ordinance No. 7774 is a valid exercise of police power of the State. Held: No. Ordinance No. 7774 cannot be considered as a valid exercise of police power, and as such, it is unconstitutional. The facts of this case will recall to mind not only the recent City of Manila v Laguio Jr ruling, but the 1967 decision in Ermita-Malate Hotel and Motel Operations Association, Inc., v. Hon. City Mayor of Manila. The common thread that runs through those decisions and the case at bar goes beyond the singularity of the localities covered under the respective ordinances. All three ordinances were enacted with a view of regulating public morals including particular illicit activity in transient lodging establishments. This could be described as the middle case, wherein there is no wholesale ban on motels and hotels but the services offered by these establishments have been severely restricted. At its core, this is another case about the extent to which the State can intrude into and regulate the lives of its citizens The test of a valid ordinance is well established. A long line of decisions including City of Manila has held that for an ordinance to be valid, it must not only be within the corporate powers of the local government unit to enact and pass according to the procedure prescribed by law, it must also conform to the following substantive requirements: (1) must not contravene the Constitution or any statute; (2) must not be unfair or oppressive; (3) must not be partial or discriminatory; (4) must not prohibit but may regulate trade; (5) must be general and consistent with public policy; and (6) must not be unreasonable. The ordinance in this case prohibits two specific and distinct business practices, namely wash rate admissions and renting out a room more than twice a day. The ban is evidently sought to be rooted in the police power as conferred on local government units by the Local Government Code through such implements as the general welfare clause. Police power is based upon the concept of necessity of the State and its corresponding right to protect itself and its people. Police power has been used as justification for numerous and varied actions by the State. The apparent goal of the ordinance is to minimize if not eliminate the use of the covered establishments for illicit sex, prostitution, drug use and alike. These goals, by themselves, are unimpeachable and certainly fall within the ambit of the police power of the State. Yet the desirability of these ends do not sanctify any and all means for their achievement. Those means must align with the Constitution. SC contended that if they were to take the myopic view that an ordinance should be analyzed strictly as to its effect only on the petitioners at bar, then it would seem that the only restraint imposed by the law that they were capacitated to act upon is the injury to property sustained by the petitioners. Yet, they also recognized the capacity of the petitioners to invoke as well the constitutional rights of their

patrons – those persons who would be deprived of availing short time access or wash-up rates to the lodging establishments in question. The rights at stake herein fell within the same fundamental rights to liberty. Liberty as guaranteed by the Constitution was defined by Justice Malcolm to include “the right to exist and the right to be free from arbitrary restraint or servitude. The term cannot be dwarfed into mere freedom from physical restraint of the person of the citizen, but is deemed to embrace the right of man to enjoy the facilities with which he has been endowed by his Creator, subject only to such restraint as are necessary for the common welfare, Indeed, the right to privacy as a constitutional right must be recognized and the invasion of it should be justified by a compelling state interest. Jurisprudence accorded recognition to the right to privacy independently of its identification with liberty; in itself it is fully deserving of constitutional protection. Governmental powers should stop short of certain intrusions into the personal life of the citizen. An ordinance which prevents the lawful uses of a wash rate depriving patrons of a product and the petitioners of lucrative business ties in with another constitutional requisite for the legitimacy of the ordinance as a police power measure. It must appear that the interests of the public generally, as distinguished from those of a particular class, require an interference with private rights and the means must be reasonably necessary for the accomplishment of the purpose and not unduly oppressive of private rights. It must also be evident that no other alternative for the accomplishment of the purpose less intrusive of private rights can work. More importantly, a reasonable relation must exist between the purposes of the measure and the means employed for its accomplishment, for even under the guise of protecting the public interest, personal rights and those pertaining to private property will not be permitted to be arbitrarily invaded. Lacking a concurrence of these requisites, the police measure shall be struck down as an arbitrary intrusion into private rights. The behavior which the ordinance seeks to curtail is in fact already prohibited and could in fact be diminished simply by applying existing laws. Less intrusive measures such as curbing the proliferation of prostitutes and drug dealers through active police work would be more effective in easing the situation. So would the strict enforcement of existing laws and regulations penalizing prostitution and drug use. These measures would have minimal intrusion on the businesses of the petitioners and other legitimate merchants. Further, it is apparent that the ordinance can easily be circumvented by merely paying the whole day rate without any hindrance to those engaged in illicit activities. Moreover, drug dealers and prostitutes can in fact collect “wash rates” from their clientele by charging their customers a portion of the rent for motel rooms and even apartments. SC reiterated that individual rights may be adversely affected only to the extent that may fairly be required by the legitimate demands of public interest or public welfare. The State is a leviathan that must be restrained from needlessly intruding into the lives of its citizens. However well¬-intentioned the ordinance may be, it is in effect an arbitrary and whimsical intrusion into the rights of the establishments as well as their patrons. The ordinance needlessly restrains the operation of the businesses of the petitioners as well as restricting the rights of their patrons without sufficient justification. The ordinance rashly equates wash rates and renting out a room more than twice a day with immorality without accommodating innocuous intentions. Balacuit v. CFI 163 SCRA 182 Facts:Petitioners, theater owners, assailed the constitutionality of Ordinance No. 640 passed by the Municipal Board of the City of Butuan on April 21, 1969. This called for a reduction to ½ of the ticket price given to minors from 7-12 years old. There was a fine from 200600 pesos or a 2-6 month imprisonment

The complaint was issued in the trial court. A TRO was then issued to prevent the law from being enforced. The respondent court entered its decision declaring the law valid. Petitioners attack the validity and constitutionality of Ordinance No. 640 on the grounds that it is ultra vires and an invalid exercise of police power. Petitioners contend that Ordinance No. 640 is not within the power of’ the Municipal Board to enact as provided for in Section 15(n) of Republic Act No. 523 where it states that the Muncipal board can only fix license fees for theaters and not admission rates. The respondent attempts to justify the enactment of the ordinance by invoking the general welfare clause embodied in Section 15 (nn) of the cited law. Issue:W/N Ordinance 640 – prohibiting payment on theater tickets for children below seven (7) is constitutional? Ruling: NO, because it infringes theater owners’ right to property. There is nothing pernicious in demanding equal price for both children and adults. The petitioners are merely conducting their legitimate businesses. The object of every business entrepreneur is to make a profit out of his venture. There is nothing immoral or injurious in charging the same price for both children and adults. In fact, no person is under compulsion to purchase a ticket. It is a totally voluntary act on the part of the purchaser if he buys a ticket to such performances. Such ticket represents a right, Positive or conditional, as the case may be, according to the terms of the original contract of sale. This right is clearly a right of property. The ticket which represents that right is also, necessarily, a species of property. As such, the owner thereof, in the absence of any condition to the contrary in the contract by which he obtained it, has the clear right to dispose of it, to sell it to whom he pleases and at such price as he can obtain. So that an act prohibiting the sale of tickets to theaters or other places of amusement at more than the regular price was held invalid as conflicting with the state constitution securing the right of property. For the assailed ordinance be held constitutional it must pass the test of police power. To invoke the exercise the police power, it must be for the interest of the public without interfering with private rights and adoptive means must be reasonably necessary for the accomplishment of the purpose and not unduly oppressive upon individuals. While it is true that a business may be regulated, it is equally true that such regulation must be within the bounds of reason, that is, the regulatory ordinance must be reasonable, and its provisions cannot be oppressive amounting to an arbitrary interference with the business or calling subject of regulation. The right of the owner to fix a price at which his property shall be sold or used is an inherent attribute of the property itself and, as such, within the protection of the due process clause. Hence, the proprietors of a theater have a right to manage their property in their own way, to fix what prices of admission they think most for their own advantage, and that any person who did not approve could stay away. Carlos Superdrug v. DSWD GR No. 166494, June 29 2007 FACTS:Petitioners, belonging to domestic corporations and proprietors operating drugstores in the Philippines, are praying for preliminary injunction assailing the constitutionality of Section 4(a) of Republic Act (R.A.) No. 9257, otherwise known as the “Expanded Senior Citizens Act of 2003.” On February 26, 2004, R.A. No. 9257, amending R.A. No. 7432, was signed into law by President Gloria Macapagal-Arroyo and it became effective on March 21, 2004. Section 4(a) of the Act states: SEC. 4. Privileges for the Senior Citizens. – The senior citizens shall be entitled to the following: (a) the grant of twenty percent (20%) discount from all establishments relative to the utilization of services in hotels and similar lodging

establishments, restaurants and recreation centers, and purchase of medicines in all establishments for the exclusive use or enjoyment of senior citizens, including funeral and burial services for the death of senior citizens; Ÿ The establishment may claim the discounts granted under (a), (f), (g) and (h) as tax deduction based on the net cost of the goods sold or services rendered: Provided, That the cost of the discount shall be allowed as deduction from gross income for the same taxable year that the discount is granted. Provided, further, That the total amount of the claimed tax deduction net of value added tax if applicable, shall be included in their gross sales receipts for tax purposes and shall be subject to proper documentation and to the provisions of the National Internal Revenue Code, as amended. Ÿ The DSWD, on May 8, 2004, approved and adopted the Implementing Rules and Regulations of RA No. 9275, Rule VI, Article 8 which contains the proviso that the implementation of the tax deduction shall be subject to the Revenue Regulations to be issued by the BIR and approved by the DOF. With the new law, the Drug Stores Association of the Philippines wanted a clarification of the meaning of tax deduction. The DOF clarified that under a tax deduction scheme, the tax deduction on discounts was subtracted from Net Sales together with other deductions which are considered as operating expenses before the Tax Due was computed based on the Net Taxable Income. On the other hand, under a tax credit scheme, the amount of discounts which is the tax credit item, was deducted directly from the tax due amount. Ÿ The DOH issued an Administrative Order that the twenty percent discount shall include both prescription and non-prescription medicines, whether branded or generic. It stated that such discount would be provided in the purchase of medicines from all establishments supplying medicines for the exclusive use of the senior citizens. Ÿ Drug store owners assail the law with the contention that granting the discount would result to loss of profit and capital especially that such law failed to provide a scheme to justly compensate the discount. ISSUE: WON Section 4(a) of the Expanded Senior Citizens Act is unconstitutional or not violative of Article 3 Section 9 of the Constitution which provides that private property shall not be taken for public use without just compensation and the equal protection clause of Article 3 Section 1. HELD: The permanent reduction in their total revenues is a forced subsidy corresponding to the taking of private property for public use or benefit. This constitutes compensable taking for which petitioners would ordinarily become entitled to a just compensation. Just compensation is defined as the full and fair equivalent of the property taken from its owner by the expropriator. The measure is not the taker’s gain but the owner’s loss. The word just is used to intensify the meaning of the word compensation, and to convey the idea that the equivalent to be rendered for the property to be taken shall be real, substantial, full and ample. Ÿ The law grants a twenty percent discount to senior citizens for medical and dental services, and diagnostic and laboratory fees; admission fees charged by theaters, concert halls, circuses, carnivals, and other similar places of culture, leisure and amusement; fares for domestic land, air and sea travel; utilization of services in hotels and similar lodging establishments, restaurants and recreation centers; and purchases of medicines for the exclusive use or enjoyment of senior citizens. As a form of reimbursement, the law provides that business establishments extending the twenty percent discount to senior citizens may claim the discount as a tax deduction. Ÿ The law is a legitimate exercise of police power which, similar to the power of eminent domain, has general welfare for its object. Police power is not capable of an exact definition, but has been purposely

veiled in general terms to underscore its comprehensiveness to meet all exigencies and provide enough room for an efficient and flexible response to conditions and circumstances, thus assuring the greatest benefits. Accordingly, it has been described as “the most essential, insistent and the least limitable of powers, extending as it does to all the great public needs.” It is “[t]he power vested in the legislature by the constitution to make, ordain, and establish all manner of wholesome and reasonable laws, statutes, and ordinances, either with penalties or without, not repugnant to the constitution, as they shall judge to be for the good and welfare of the commonwealth, and of the subjects of the same.” For this reason, when the conditions so demand as determined by the legislature, property rights must bow to the primacy of police power because property rights, though sheltered by due process, must yield to general welfare.[ National Development Corporation v. PVB 192 SCRA 257 Facts: The particular enactment in question is Presidential Decree No. 1717, which ordered the rehabilitation of the Agrix Group of Companies to be administered mainly by the National Development Company. The law outlined the procedure for filling claims against the Agrix Companies and created a claims committee to process these claims. Especially relevant to this case, and noted at the outset, is section 4(1) thereof providing that “all mortgages and other liens presently attaching to any of the assets of the dissolved corporations are hereby extinguished.” Earlier, the Agrix Marketing Inc. had executed in favor of private respondent Philippine Veterans Bank a real estate mortgage dated July 7, 1978 over three parcels of land situated in Los Baños, Laguna. During the existence of the mortgage, Agrix went bankrupt. It was the expressed purpose of salvaging this and the other Agrix companies that the aforementioned decree was issued by President Marcos. A claim for the payment of its loan credit was filed by PNB against herein petitioner, however the latter alleged and invoked that the same was extinguished by PD 1717. Issue: Whether or not Philippine Veterans Bank as creditor of Agrix is still entitled for payment without prejudice to PD 1717. Held: Yes. A mortgage lien is a property right derived from contract and so comes under the protection of Bill of rights so do interests on loans, as well s penalties and charges, which are also vested rights once they accrue. Private property cannot simply be taken by law from one person and given to another without just compensation and any known public purpose. This is plain arbitrariness and is not permitted under the constitution. The court also feels that the decree impairs the obligation of the contract between Agrix and the private respondent without justification. While it is true that the police power is superior to the impairment clause, the principle will apply only where the contract is so related to the public welfare that it will be considered congenitally susceptible to change by the legislature in the interest of greater number. Our finding in sum, is that PD 1717 is an invalid exercise of the police power, not being in conformity with the traditional requirements of a lawful subject and a lawful method. The extinction of the mortgage and other liens and of the interest and other charges pertaining to the legitimate creditors of Agrix constitutes taking without due process of law, and this is compounded by the reduction of the secured creditors to the category of unsecured creditors in violation of the equal protection clause. Moreover, the new corporation being neither owned nor controlled by the government, should have been created only by general and not special law. And in so far as the decree also interferes with purely private agreements without any demonstrated connection with the public interest, there is likewise an impairment of the obligation of the contract.

AgustIn v. Edu 88 SCRA 195 Facts: President Marcos issued the Letter of Instruction No. 229 which states that all owners, users or drivers shall have at all times one pair of early warning devise (EWD) in their cars acquire from any source depending on the owner’s choice. The Letter of Instruction was assailed by petitioner Leovillo Agustin to have violated the constitution guarantee of due process against Hon Edu, Land Transportation Commissioner, Hon. Juan Ponce Enrile, Minister of national Defense, Hon. Juinio, Minister of Public Works, Transportation and Communication and Hon. Aquino, Minister of Public Highways. Because of such contentions, the Implementing Rules and Regulation was ordered to be suspended for a period of 6 months. Petitioner alleges that EWD are not necessary because vehicles already have hazard lights (blinking lights) that can be use as a warning device. Also petitioner contest that the letter of instruction violates the delegation of police power because it is deemed harsh, oppressive and unreasonable for the motorists and those dealers of EWD will become instant millionaires because of such law. Issue: Whether or not Petitioner’s contentions possess merit. Held: Petitioner’s contentions are without merit because the exercise of police power may interfere with personal liberty or property to ensure and promote the safety, health and prosperity of the State. Also, such letter of instruction is intended to promote public safety and it is indeed a rare occurrence that such contention was alleged in a instruction with such noble purpose. Petitioner also failed to present the factual foundation that is necessary to invalidate the said letter of instruction. In cases where there is absence in the factual foundation, it should be presumed that constitutionality shall prevail. Pres. Marcos on the other hand possesses vital statistics that will justify the need for the implementation of this instruction. As signatory to the 1968 Vienna Conventions on Road Signs and Signals, our country must abide with the standards given as stated in our Constitution that “the Philippines adopts the generally accepted principles of International Law as part of the law of the land. In the case at bar, the Vienna Convention also requires the use of EWD. Vehicle owners are not obliged to buy an EDW. They can personally create a EWD provided that it is in accordance to the specifications provided by law. Petitioner’s allegation against the manufacturers of EDW being millionaires is deemed to be an unfounded speculation. Wherefore, the petition is dismissed. The restraining order regarding the implementation of the Reflector Law is lifted making the said law immediately executory. Magtajas v. Pryce Properties 234 SCRA 255 Facts: The Philippine Amusement and Gaming Corporation (PAGCOR) is a corporation created directly by P.D. 1869 to help centralize and regulate all games of chance, including casinos on land and sea within the territorial jurisdiction of the Philippines. In Basco v. Philippine Amusements and Gaming Corporation, this Court sustained the constitutionality of the decree and even cited the benefits of the entity to the national economy as the third highest revenueearner in the government. PAGCOR decided to expand its operations to Cagayan de Oro City by leasing a portion of a building belonging to Pryce Properties Corporation Inc. for its casino. On December 7, 1992, Sangguniang Panlungsod of CDO enacted ordinance 3353, prohibiting the issuance of business permit and cancelling existing business permit to any establishment for the using and allowing to be used its premises or portion thereof for the operation of a casino. On January 4, 1993, it enacted Ordinance 3375-93, prohibiting the operation of casino and providing penalty for violation therefore. Pryce assailed the ordinances before the CA, where it was joined by PAGCOR as intervenor.

The Court found the ordinances invalid and issued the writ prayed for to prohibit their enforcement. CDO City and its mayor filed a petition for review under Rules of Court with the Supreme Court. Issue: WON the Sangguniang Panlungsod can prohibit the establishment of casino operated by PAGCOR through an ordinance or resolution. Held: No. Gambling is not illegal per se. While it is generally considered inimical to the interests of the people, there is nothing in the Constitution categorically proscribing or penalizing gambling or, for that matter, even mentioning it at all. In the exercise of its own discretion, the Congress may prohibit gambling altogether or allow it without limitation or it may prohibit some forms of gambling and allow others for whatever reasons it may consider sufficient. Under Sec. 458 of the Local Government Code, local government units are authorized to prevent or suppress, among others, “gambling and other prohibited games of chance.” Ordinances should not contravene a statue as municipal governments are only agents of the national government. Local councils exercise only delegated powers conferred on them by Congress as the national lawmaking body. The delegate cannot be superior to the principal or exercise powers higher than those of the latter. The tests of a valid ordinance are well established. A long line of decisions has held that to be valid, an ordinance must conform to the following substantive requirements: CUOPDPRGCU 1) It must not contravene the constitution or any statute. 2) It must not be unfair or oppressive. 3) It must not be partial or discriminatory. 4) It must not prohibit but may regulate trade. 5) It must be general and consistent with public policy. 6) It must not be unreasonable. Dans v. People 285 SCRA 504 While respondent Court was duty-bound to be just and impartial, it failed to give petitioner a fair trial, who was thereby denied due process of law. Respondent Court was plainly biased against, if not downright hostile to, petitioner; it unfairly allied itself with the prosecution, which made it prosecutor and judge at the same time.

Board of Directors of the PGH Foundation, who had participated in the collective acts, thereby singling Petitioner and her companion for discriminatory prosecution, in violation of her right to Equal Protection of the Laws, which violation existed from the filing of the information and cannot be cured by post hoc proceedings. E. The questioned Decision is a nullity, because of the participation therein of Mr. Justice Garchitorena, whose long-standing bias and hostility towards President Marcos and Petitioner Imelda R. Marcos prevented him from having the requisite cold neutrality of an impartial judge, in violation of her right as an accused person to Procedural Due Process of Law. F. The questioned Decision is a nullity because Petitioner was denied of her Constitutional Right to counsel. 1. Facts of record showing that Petitioner was deprived of and denied her Right to Counsel. 2. Under the circumstances of record, the absence of counsel resulting from imposition of suspension from the practice of law upon her retained counsel, constituted deprivation of or denial of the Right to Counsel. 3. Facts of record showing legal representation of Petitioner Imelda Marcos was not adequate. G. The questioned Decision is premature and had disregarded the constitutional right of the Petitioner to present evidence in her behalf. Her right to testify in her own behalf is a guaranteed right, the exercise of which is her personal choice alone, and which counsel had no authority to waive in her behalf. Besides, counsel being suspended, he could not have made a waiver. This constitutional right to be heard by himself and counsel she is invoking now, as part of her right to due process (Sec. 14 (1) and (2), Bill of Rights). H. The questioned Decision is a nullity for it was rendered in derogation of Petitioners subsisting right to be heard and to submit evidence in her defense. The finding of waiver is a prejudicial error. The evidence thereof on the record is tenuous. A waiver by an accused person of the right to be heard in her defense, including her right to testify in her own behalf must be indubitable, and is valid only if personally exercised through her own manifestation in open court. I. The questioned Decision is a nullity because the crime charged was not proven beyond a reasonable doubt, and the presumption of innocence was not overcome, which is required by Due Process. Corona v. UHPAP 283 SCRA 31

VII. Aside from the foregoing, the appealed decision is flawed by fatal infirmities which have effectively denied petitioner due process of law. G.R. No. 126995 A. The questioned Decision is a nullity because Section 3 (g) of the Anti-Graft and Corrupt Practices Act (RA 3019, as amended) is unconstitutional for being, on its face, void for vagueness. B. The questioned Decision is a nullity because Section 3 (g) of the Anti-Graft and Corrupt Practices Act (RA 3019, as amended) is unconstitutional for being a rider. C. The questioned Decision is a nullity because the Informations in SB Criminal Cases Nos. 17450 and 17453 did not state all the essential facts constituting the offense but instead stated conclusions of law, thereby denying the Petitioner her constitutional right to be informed of the nature and the cause of the accusation against her (Sec. 14 (2), Bill of Rights). D. The questioned Decision is a nullity because the Information in said SB Criminal Cases Nos. 17450 and 17453 charged only two of the total number of members in the Board of Directors of the LRTA and the

FACTS: The Philippine Ports Authority [PPA] was created through PD 505, as amended by PD857 to “control, regulate, supervise pilots and the pilot age profession”. After hearing from relevant government agencies, pursuant to said charter, PPA General Manager Rogelio A. Dayan issued Administrative Order 04-92 [PPA-AO 04-92] and corresponding Memorandum Order in 1992, stating that all existing regular appointments which have been previously issued shall remain valid up to 31 December 1992 only and that all appointments to harbor pilot positions in all pilot age districts shall, henceforth, be only for a term of one year from date of effectivity subject to yearly renewal or cancellation by the Authority after conduct of a rigid evaluation of performance” to regulate and improve pilot services by instilling discipline and give better protection to port users. PPA-AO 04-92 replaces PPA-AO 03-85 which succinctly provides that, aspiring pilots must have a license and train as probationary pilots, and only upon satisfactory performance, are given permanent and regular appointments by the PPA itself and to exercise harbor pilot age until they reach the age of 70.Upon learning of PPA-AO 04-92 only after publication in the newspaper, the United Harbor Pilots Association of the Philippines: (a) questioned said PPA-AO twice before the DOTC,

which Secretary Garcia said twice that only the PPA Board of Directors [as governing body] has exclusive jurisdiction to review, recall or annul PPA-AOs, (b) appealed to the Office of the President, which first issued a restraining order to the PPA on the implementation of the PPA-AO, and after PPA’s answer, then dismissed the appeal/petition and lifted said order, stating, through Assistant Executive Secretary for Legal Affairs Renato C. Corona, that the PPAAO (i) merely implements PPA Charter, (ii) issuance is an act of PPA, not of its General Manager, (iii) merely regulates, not forbids practice of the profession, recognizing that such exercise is property right, and (iv) sufficiently complied with the requirement in the PD to consult only with relevant Government Agencies and (d) finally finding affirmative relief with Manila RTC Br. 6. Court, which ruled that (i) said PPA-AO is null and void (ii) PPA acted in excess of jurisdiction with grave abuse of discretion and (iii) imposed a permanent restraining order on PPA on its implementation.Assistant Executive Secretary Corona thus filed petition for review [of the Manila RTC Decision] to the Supreme Court. ISSUE: Whether or not the respondents have acted in excess of jurisdiction. Whether or not the Philippine Ports Authority (PPA) violate respondents’ right to exercise their profession and their right to due process of law. HELD: the Court is convinced that PPA-AO No. 04-92 was issued in stark disregard of respondents' right against deprivation of property without due process of law. The Supreme Court said that In order to fall within the aegis of this provision, two conditions must concur, namely, that there is a deprivation and that such deprivation is done without proper observance of due process. As a general rule, notice and hearing, as the fundamental requirements of procedural due process, are essential only when an administrative body exercises its quasijudicial function. In the performance of its executive or legislative functions, such as issuing rules and regulations, an administrative body need not comply with the requirements of notice and hearing There is no dispute that pilotage as a profession has taken on the nature of a property right. It is readily apparent that PPA-AO No. 04-92 unduly restricts the right of harbor pilots to enjoy their profession before their compulsory retirement People v. De la Piedra 350 SCRA 163 FACTS:Accused-appellant Carol M. dela Piedra was charged of illegal recruitment in large scale by promising an employment abroad Maria Lourdes Modesto y Gadrino, Nancy Araneta y Aliwanag and Jennelyn Baez y Timbol, a job to Singapore without having previously obtained from the Philippine Overseas Employment Administration, a license or authority to engage in recruitment and overseas placement of workers. In fact said Maria Lourdes Modesto had already advanced the amount of P2,000.00 to the accused for and in consideration of the promised employment which did not materialize. Thus causing damage and prejudice to the latter in the said sum. Erlie Ramos, Attorney II of the Philippine Overseas Employment Agency (POEA), received a telephone call from an unidentified woman inquiring about the legitimacy of the recruitment conducted by a certain Mrs. Carol Figueroa. Ramos. An entrapment was then planned by the Criminal Investigation Service (CIS) headed by Capt. Mendoza and successfully arrested the accused-appellant. Later on, in the course of their investigation, the CIS discovered that Carol Figueroa had many aliases, among them, Carol Llena and Carol dela Piedra. At the trial, the prosecution presented five (5) witnesses, namely, Erlie Ramos, SPO2 Erwin Manalopilar, Eileen Fermindoza, Nancy Araneta and Lourdes Modesto and all of them positively testified that the accused offer them a job to Singapore.

The trial found the accused-appellant guilty of beyond reasonable doubt of Illegal Recruitment committed in a large scale. ISSUE: WON Article 13 (b) of the Labor Code defining recruitment and placement is void for vagueness and, thus, violates the due process clause. HELD: NO. Article 13 (b) of the Labor Code is not a vague provision. As a rule, a statute or act may be said to be vague when it lacks comprehensible standards that men of common intelligence must necessarily guess at its meaning and differ as to its application. It is repugnant to the Constitution in two respects: (1) it violates due process for failure to accord persons, especially the parties targeted by it, fair notice of the conduct to avoid; and (2) it leaves law enforcers unbridled discretion in carrying out its provisions and become an arbitrary flexing of the Government muscle. The court cannot sustain the Appellant argument that the acts that constitute recruitment and placement suffer from overbreadth since by merely referring a person for employment, a person may be convicted of illegal recruitment. Evidently,appellant has taken the penultimate paragraph in the excerpt quoted above out of context. The Court, in Panis case, merely bemoaned the lack of records that would help shed light on the meaning of the proviso. The absence of such records notwithstanding, the Court was able to arrive at a reasonable interpretation of the proviso by applying principles in criminal law and drawing from the language and intent of the law itself. Section 13 (b), therefore, is not a perfectly vague act whose obscurity is evident on its face. If at all, the proviso therein is merely couched in imprecise language that was salvaged by proper construction. It is not void for vagueness. An act will be declared void and inoperative on the ground of vagueness and uncertainty, only upon a showing that the defect is such that the courts are unable to determine, with any reasonable degree of certainty, what the legislature intended. x x x. In this connection we cannot permit reference to the rule that legislation should not be held invalid on the ground of uncertainty if susceptible of any reasonable construction that will support and give it effect. An Act will not be declared inoperative and ineffectual on the ground that it furnishes no adequate means to secure the purpose for which it is passed, if men of common sense and reason can devise and provide the means, and all the instrumentalities necessary for its execution are within the reach of those entrusted therewith. That Section 13 (b) encompasses what appellant apparently considers as customary and harmless acts such as labor or employment referral (referring an applicant, according to appellant, for employment to a prospective employer) does not render the law over broad. Evidently, appellant misapprehends concept of over breadth. A statute may be said to be over broad where it operates to inhibit the exercise of individual freedoms affirmatively guaranteed by the Constitution, such as the freedom of speech or religion. A generally worded statute, when construed to punish conduct which cannot be constitutionally punished is unconstitutionally vague to the extent that it fails to give adequate warning of the boundary between the constitutionally permissible and the constitutionally impermissible applications of the statute In Blo Umpar Adiong vs. Commission on Elections,for instance, we struck down as void for overbreadth provisions prohibiting the posting of election propaganda in any place including private vehicles other than in the common poster areas sanctioned by the COMELEC. We held that the challenged provisions not only deprived the owner of the vehicle the use of his property but also deprived the citizen of his right to free speech and information. The prohibition in Adiong, therefore, was so broad that it covered even constitutionally guaranteed rights and, hence, void for over breadth.

In the present case, however, appellant did not even specify what constitutionally protected freedoms are embraced by the definition of recruitment and placement that would render the same constitutionally over broad. Estrada vs Sandiganbayan Former President Estrada and co-accused were charged for Plunder under RA 7080 (An Act Defining and Penalizing the Crime of Plunder), as amended by RA 7659. On the information, it was alleged that Estrada have received billions of pesos through any or a combination or a series of overt or criminal acts, or similar schemes or means thereby unjustly enriching himself or themselves at the expense and to the damage of the Filipino people and the Republic of the Philippines. Estrada questions the constitutionality of the Plunder Law since for him: 1. it suffers from the vice of vagueness 2. it dispenses with the "reasonable doubt" standard in criminal prosecutions 3. it abolishes the element of mens rea in crimes already punishable under The Revised Penal Code. Office of the Ombudsman filed before the Sandiganbayan 8 separate Informations against petitioner. Estrada filed an Omnibus Motion on the grounds of lack of preliminary investigation, reconsideration/reinvestigation of offenses and opportunity to prove lack of probable cause but was denied. Later on, the Sandiganbayan issued a Resolution in Crim. Case No. 26558 finding that a probable cause for the offense of plunder exists to justify the issuance of warrants for the arrest of the accused. Estrada moved to quash the Information in Criminal Case No. 26558 on the ground that the facts alleged therein did NOT constitute an indictable offense since the law on which it was based was unconstitutional for vagueness and that the Amended Information for Plunder charged more than one offense. Same was denied. The questioned provisions of the petitioners are Secs. 1, par. (d), 2 and 4 of the Plunder Law which states that: Section 1. x x x x (d) "Ill-gotten wealth" means any asset, property, business, enterprise or material possession of any person within the purview of Section Two (2) hereof, acquired by him directly or indirectly through dummies, nominees, agents, subordinates and/or business associates by any combination or series of the following means or similar schemes: (1) Through misappropriation, conversion, misuse, or malversation of public funds or raids on the public treasury; (2) By receiving, directly or indirectly, any commission, gift, share, percentage, kickbacks or any other form of pecuniary benefit from any person and/or entity in connection with any government contract or project or by reason of the office or position of the public office concerned; (3) By the illegal or fraudulent conveyance or disposition of assets belonging to the National Government or any of its subdivisions, agencies or instrumentalities, or government owned or controlled corporations and their subsidiaries; (4) By obtaining, receiving or accepting directly or indirectly any shares of stock, equity or any other form of interest or participation including the promise of future employment in any business enterprise or undertaking;

(5) By establishing agricultural, industrial or commercial monopolies or other combinations and/or implementation of decrees and orders intended to benefit particular persons or special interests; or (6) By taking advantage of official position, authority, relationship, connection or influence to unjustly enrich himself or themselves at the expense and to the damage and prejudice of the Filipino people and the Republic of the Philippines. Section 2. Definition of the Crime of Plunder, Penalties. - Any public officer who, by himself or in connivance with members of his family, relatives by affinity or consanguinity, business associates, subordinates or other persons, amasses, accumulates or acquires ill-gotten wealth through a combination or series of overt or criminal acts as described in Section 1 (d) hereof, in the aggregate amount or total value of at least fifty million pesos (P50,000,000.00) shall be guilty of the crime of plunder and shall be punished by reclusion perpetua to death. Any person who participated with the said public officer in the commission of an offense contributing to the crime of plunder shall likewise be punished for such offense. In the imposition of penalties, the degree of participation and the attendance of mitigating and extenuating circumstances as provided by the Revised Penal Code shall be considered by the court. The court shall declare any and all ill-gotten wealth and their interests and other incomes and assets including the properties and shares of stocks derived from the deposit or investment thereof forfeited in favor of the State (underscoring supplied). Section 4. Rule of Evidence. - For purposes of establishing the crime of plunder, it shall not be necessary to prove each and every criminal act done by the accused in furtherance of the scheme or conspiracy to amass, accumulate or acquire ill-gotten wealth, it being sufficient to establish beyond reasonable doubt a pattern of overt or criminal acts indicative of the overall unlawful scheme or conspiracy (underscoring supplied). ISSUE: WON the crime of plunder is unconstitutional for being vague? HELD: NO. As long as the law affords some comprehensible guide or rule that would inform those who are subject to it what conduct would render them liable to its penalties, its validity will be sustained. The amended information itself closely tracks the language of the law, indicating w/ reasonable certainty the various elements of the offense w/c the petitioner is alleged to have committed. We discern nothing in the foregoing that is vague or ambiguous that will confuse petitioner in his defense. Petitioner, however, bewails the failure of the law to provide for the statutory definition of the terms “combination” and “series” in the key phrase “a combination or series of overt or criminal acts. These omissions, according to the petitioner, render the Plunder Law unconstitutional for being impermissibly vague and overbroad and deny him the right to be informed of the nature and cause of the accusation against him, hence violative of his fundamental right to due process. A statute is not rendered uncertain and void merely because general terms are used herein, or because of the employment of terms without defining them. A statute or act may be said to be vague when it lacks comprehensible standards that men of common intelligence most necessarily guess at its meaning and differ in its application. In such instance, the statute is repugnant to the Constitution in two (2) respects – it violates due process for failure to accord persons, especially the parties targeted by it, fair notice of what conduct to avoid; and, it leaves law enforcers unbridled discretion in carrying out its provisions and becomes an arbitrary flexing of the Government muscle.

A facial challenge is allowed to be made to vague statute and to one which is overbroad because of possible “chilling effect” upon protected speech. The possible harm to society in permitting some unprotected speech to go unpunished is outweighed by the possibility that the protected speech of others may be deterred and perceived grievances left to fester because of possible inhibitory effects of overly broad statutes. But in criminal law, the law cannot take chances as in the area of free speech.

expected to approach the same problems from the national perspective. Both views are thereby made to bear on the enactment of such laws.

ABAKADA v. Ermita 469 SCRA 1

The equal protection clause under the Constitution means that “no person or class of persons shall be deprived of the same protection of laws which is enjoyed by other persons or other classes in the same place and in like circumstances.”

Facts: Petitioners ABAKADA GURO Party List challenged the constitutionality of R.A. No. 9337 particularly Sections 4, 5 and 6, amending Sections 106, 107 and 108, respectively, of the National Internal Revenue Code (NIRC). These questioned provisions contain a uniform proviso authorizing the President, upon recommendation of the Secretary of Finance, to raise the VAT rate to 12%, effective January 1, 2006, after any of the following conditions have been satisfied, to wit: . . . That the President, upon the recommendation of the Secretary of Finance, shall, effective January 1, 2006, raise the rate of value-added tax to twelve percent (12%), after any of the following conditions has been satisfied: (i) Value-added tax collection as a percentage of Gross Domestic Product (GDP) of the previous year exceeds two and four-fifth percent (2 4/5%); or (ii) National government deficit as a percentage of GDP of the previous year exceeds one and one-half percent (1 ½%). Petitioners argue that the law is unconstitutional, as it constitutes abandonment by Congress of its exclusive authority to fix the rate of taxes under Article VI, Section 28(2) of the 1987 Philippine Constitution. They further argue that VAT is a tax levied on the sale or exchange of goods and services and cannot be included within the purview of tariffs under the exemption delegation since this refers to customs duties, tolls or tribute payable upon merchandise to the government and usually imposed on imported/exported goods. They also said that the President has powers to cause, influence or create the conditions provided by law to bring about the conditions precedent. Moreover, they allege that no guiding standards are made by law as to how the Secretary of Finance will make the recommendation. They claim, nonetheless, that any recommendation of the Secretary of Finance can easily be brushed aside by the President since the former is a mere alter ego of the latter, such that, ultimately, it is the President who decides whether to impose the increased tax rate or not. Issues: Whether or not R.A. No. 9337 has violated the provisions in Article VI, Section 24, and Article VI, Section 26 (2) of the Constitution. Whether or not there was an undue delegation of legislative power in violation of Article VI Sec 28 Par 1 and 2 of the Constitution. Whether or not there was a violation of the due process and equal protection under Article III Sec. 1 of the Constitution. Discussions: Basing from the ruling of Tolentino case, it is not the law, but the revenue bill which is required by the Constitution to “originate exclusively” in the House of Representatives, but Senate has the power not only to propose amendments, but also to propose its own version even with respect to bills which are required by the Constitution to originate in the House. the Constitution simply means is that the initiative for filing revenue, tariff or tax bills, bills authorizing an increase of the public debt, private bills and bills of local application must come from the House of Representatives on the theory that, elected as they are from the districts, the members of the House can be expected to be more sensitive to the local needs and problems. On the other hand, the senators, who are elected at large, are

In testing whether a statute constitutes an undue delegation of legislative power or not, it is usual to inquire whether the statute was complete in all its terms and provisions when it left the hands of the legislature so that nothing was left to the judgment of any other appointee or delegate of the legislature.

Rulings: R.A. No. 9337 has not violated the provisions. The revenue bill exclusively originated in the House of Representatives, the Senate was acting within its constitutional power to introduce amendments to the House bill when it included provisions in Senate Bill No. 1950 amending corporate income taxes, percentage, excise and franchise taxes. Verily, Article VI, Section 24 of the Constitution does not contain any prohibition or limitation on the extent of the amendments that may be introduced by the Senate to the House revenue bill. There is no undue delegation of legislative power but only of the discretion as to the execution of a law. This is constitutionally permissible. Congress does not abdicate its functions or unduly delegate power when it describes what job must be done, who must do it, and what is the scope of his authority; in our complex economy that is frequently the only way in which the legislative process can go forward. Supreme Court held no decision on this matter. The power of the State to make reasonable and natural classifications for the purposes of taxation has long been established. Whether it relates to the subject of taxation, the kind of property, the rates to be levied, or the amounts to be raised, the methods of assessment, valuation and collection, the State’s power is entitled to presumption of validity. As a rule, the judiciary will not interfere with such power absent a clear showing of unreasonableness, discrimination, or arbitrariness. GSIS V. Monteclaros 434 SCRA 441 FACTS:Nicolas Montesclaros, a 72-year-old widower married Milagros Orbiso, who was then 43 years old, on 10 July 1983. Nicolas filed with the GSIS an application for retirement benefits under the Revised Government Insurance Act of 1977. In his retirement application, he designated his wife as his sole beneficiary. GSIS approved Nicolas’ application for retirement effective 17 February 1984, granting a lump sum payment of annuity for the first five years and a monthly annuity after. Nicolas died on 22 April 1992. Milagros filed with the GSIS a claim for survivorship pension under PD 1146 but was denied the claim because, under section 18 of PD 1146, the surviving spouse has no right to survivorship pension if the surviving spouse contracted the marriage with the pensioner within three years before the pensioner qualified for the pension. Nicolas wed Milagros on 10 July 1983, less than one year from his date of retirement on 17 February 1984. Milagros filed with the trial court a special civil action for declaratory relief questioning the validity of Sec. 18 of PD 1146. The trial court rendered judgment declaring Milagros eligible for survivorship pension and ordered GSIS to pay Milagros the benefits including interest. Citing Articles 115and 117 of the Family Code, the trial court held that retirement benefits, which the pensioner has earned for services rendered and for which the pensioner has contributed through monthly salary deductions, are onerous acquisitions. Since retirement benefits are property the pensioner acquired through labor, such benefits are conjugal property. The trial court held that the

prohibition in Section 18 of PD 1146 is deemed repealed for being inconsistent with the Family Code, a later law. The Family Code has retroactive effect if it does not prejudice or impair vested rights. The trial court held that Section 18 of PD 1146 was repealed by the Family Code, a later law. GSIS appealed to the Court of Appeals, which affirmed the trial court’s decision. Hence, this appeal. In a letter dated 10 January 2003, Milagros informed the Court that she has accepted GSIS’ decision disqualifying her from receiving survivorship pension and that she is no longer interested in pursuing the case. However, the Court will still resolve the issue despite the manifestation of Milagros because social justice and public interest demand the resolution of the constitutionality of the proviso. ISSUE:Whether the proviso in Section 18 of PD 1146 is constitutional. HELD:NO. The sole proviso Sec. 18 of PD 1146 is unconstitutional. Under Section 18 of PD 1146, it prohibits the dependent spouse from receiving survivorship pension if such dependent spouse married the pensioner within three years before the pensioner qualified for the pension. The Court holds that such proviso is discriminatory and denies equal protection of the law. The proviso is contrary to Section 1, Article III of the Constitution, which provides that [n]o person shall be deprived of life, liberty, or property without due process of law, nor shall any person be denied the equal protection of the laws. The proviso is unduly oppressive in outrightly denying a dependent spouses claim for survivorship pension if the dependent spouse contracted marriage to the pensioner within the three-year prohibited period. There is outright confiscation of benefits due the surviving spouse without giving the surviving spouse an opportunity to be heard. The proviso undermines the purpose of PD 1146, which is to assure comprehensive and integrated social security and insurance benefits to government employees and their dependents in the event of sickness, disability, death, and retirement of the government employees. A statute based on reasonable classification does not violate the constitutional guaranty of the equal protection of the law. The requirements for a valid and reasonable classification are: (1) it must rest on substantial distinctions; (2) it must be germane to the purpose of the law; (3) it must not be limited to existing conditions only; and (4) it must apply equally to all members of the same class. Thus, the law may treat and regulate one class differently from another class provided there are real and substantial differences to distinguish one class from another. The proviso in question does not satisfy these requirements. The proviso discriminates against the dependent spouse who contracts marriage to the pensioner within three years before the pensioner qualified for the pension. Under the proviso, even if the dependent spouse married the pensioner more than three years before the pensioners death, the dependent spouse would still not receive survivorship pension if the marriage took place within three years before the pensioner qualified for pension. The object of the prohibition is vague. There is no reasonable connection between the means employed and the purpose intended. The law itself does not provide any reason or purpose for such a prohibition. If the purpose of the proviso is to prevent deathbed marriages, then we do not see why the proviso reckons the three-year prohibition from the date the pensioner qualified for pension and not from the date the pensioner died. The classification does not rest on substantial distinctions.

Worse, the classification lumps all those marriages contracted within three years before the pensioner qualified for pension as having been contracted primarily for financial convenience to avail of pension benefits. Indeed, the classification is discriminatory and arbitrary. This is probably the reason Congress deleted the proviso in Republic Act No. 8291 (RA 8291), otherwise known as the Government Service Insurance Act of 1997, the law revising the old charter of GSIS (PD 1146). Under the implementing rules of RA 8291, the surviving spouse who married the member immediately before the members death is still qualified to receive survivorship pension unless the GSIS proves that the surviving spouse contracted the marriage solely to receive the benefit. Thus, the present GSIS law does not presume that marriages contracted within three years before retirement or death of a member are sham marriages contracted to avail of survivorship benefits. The present GSIS law does not automatically forfeit the survivorship pension of the surviving spouse who contracted marriage to a GSIS member within three years before the members retirement or death. The law acknowledges that whether the surviving spouse contracted the marriage mainly to receive survivorship benefits is a matter of evidence. The law no longer prescribes a sweeping classification that unduly prejudices the legitimate surviving spouse and defeats the purpose for which Congress enacted the social legislation. Wherefore, the proviso in Section 18 of Presidential Decree No. 1146 is void for being violative of the constitutional guarantees of due process and equal protection of the law. Mirasol v. DPWH 490 SCRA 318 FACTS:Petitioners filed before the court a petition for declaratory judgment with application for temporary restraining order and injunction. It seeks the declaration of nullification of administrative issuances for being inconsistent with the provisions of Republic Act 2000 (Limited Access Highway Act) which was enacted in 1957. Previously, pursuant to its mandate under RA 2000, DPWH issued on June 25, 1998 Dept. Order no. 215 declaring the Manila Cavite (Coastal Road) Toll Expressway as limited access facilities. Petitioners filed an Amended Petition on February 8, 2001 wherein petitioners sought the declaration of nullity of the aforesaid administrative issuances. Petitioner assailed the constitutionality of an administrative regulation banning the use of motorcycles at the toll way on the ground that it is baseless and unwarranted for failure to provide scientific and objective data on the dangers of motorcycles plying the highways. Respondent avers that the toll ways were not designed to accommodate motorcycles and that their presence in the toll ways will compromise safety and traffic considerations. The petitioners prayed for the issuance of a temporary restraining order to prevent the enforcement of the total ban on motorcycles along NLEX, SLEX, Manila-Cavite (Coastal Road) toll Expressway under DO 15. RTC, after due hearing, granted the petitioner’s application for preliminary injunction conditioned upon petitioner’s filing of cash bond in the amount of P100, 000 which petitioners complied. DPWH issued an order (DO 123) allowing motorcycles with engine displacement of 400 cubic centimeters inside limited access facilities (toll ways). Upon assumption of Hon. Presiding Judge Cornejo, both the petitioners and respondents were required to file their Memoranda.

The court issued an order dismissing the petition but declaring invalid DO 123.The petitioners moved for reconsideration but it was denied. RTC ruled that DO 74 is valid but DO 123 is invalid being violative of the equal protection clause of the Constitution ISSUE:Whether RTC’s decision is barred by res judicata? Whether DO 74, DO 215 and the TRB regulation contravene RA 2000. Whether AO 1 is unconstitutional. HELD:1. NO. The petitioners are mistaken because they rely on the RTC’s Order granting their prayer for a writ of preliminary injunction. Since petitioners did not appeal from that order, the petitioners presumed that the order became a final judgment on the issues. The order granting the prayer is not an adjudication on the merits of the case that would trigger res judicata. A preliminary injunction does not serve as a final determination of the issues, it being a provisional remedy. 2. YES. The petitioners claimed that DO 74, DO 215 and TRB’s rules and regulation issued under them unduly expanded the power of the DPWH in sec. 4 of RA 2000 to regulate toll ways. They contend that DPWH’s regulatory authority is limited to acts like redesigning curbings or central dividing sections. They claim that DPWH is only allowed to redesign the physical structure of toll ways and not to determine “who or what can be qualifies as toll ways user”. The court ruled that DO 74 and DO 215 are void because the DPWH has no authority to declare certain expressways as limited access facilities. Under the law, it is the DOTC which is authorized to administer and enforce all laws, rules and regulations in the field of transportation and to regulate related activities. Since the DPWH has no authority to regulate activities relative to transportation, the Toll Regulatory Board (TRB) cannot derive its power from the DPWH to issue regulations governing limited access facilities. The DPWH cannot delegate a power or function which it does not possess in the first place. 3. NO. The Court emphasized that the secretary of the then Department of Public Works and Communications had issued AO 1 in February 1968, as authorized under Section 3 of Republic Act 2000, prior to the splitting of the department and the eventual devolution of its powers to the DOTC. Because administrative issuances had the force and effect of law, AO 1 enjoyed the presumption of validity and constitutionality. The burden to prove its unconstitutionality rested on the party assailing it, more so when police power was at issue and passed the test of reasonableness. The Administrative Order was not oppressive, as it did not impose unreasonable restrictions or deprive petitioners of their right to use the facilities. It merely set rules to ensure public safety and the uninhibited flow of traffic within those limited-access facilities. The right to travel did not mean the right to choose any vehicle in traversing a tollway. Petitioners were free to access the tollway as much as the rest of the public. However, the mode in which they wished to travel, pertaining to their manner of using the tollway, was a subject that could validly be limited by regulation. There was no absolute right to drive; on the contrary, this privilege was heavily regulated. EQUAL PROTECTION CLAUSE People v. Cayat 68 Phil. 12 DOCTRINE: Protection of laws is not violated by a legislation based on reasonable classification. The classification to be reasonable, (1) must rest on substantial distinctions; (2) must be germane to the

purposes of the law; (3) must not be limited to existing conditions only; (4) must apply equally to all members of the same class. FACTS: Respondent Cayat, native of Baguio, Benguet and a member of the non-Christian tribe was found guilty of violating sections 2 and 3 of Act No. 1639 for possessing an intoxicating liquor (one bottle of gin) which is not a native wine. Section 2 of the said act prohibits any native of the Philippines who is a member of the non-Christian tribe to buy, receive and possess any intoxicating liquor other than their so-called native wines. Consequently, Section 3 thereof provides for its punishment. Cayat challenges the constitutionality of Act No. 1639 on the grounds that it is discriminatory and denies the equal protection of the laws, violative of the due process and it is an improper exercise of police power. ISSUES: Whether the Act No. 1639 violates the equal protection clause? RULING: No, the Act No. 1639 is not violative of the equal protection clause. Equal protection of the laws is not violated by a legislation based on reasonable classifications. The classification to be reasonable, (1) must rest on substantial distinctions; (2) must be germane to the purposes of the law; (3) must not be limited to existing conditions only; (4) must apply equally to all members of the same class. Act No. 1639 satisfies these requirements. On the first requisite, the classification rests on real and substantial distinctions. The nonChristian tribes refer not to the religious belief, but in a way to the geographical and more directly to the natives of the Philippines of a low grade of civilization. Second, Act No. 1639 was designed to insure peace and order among the non-Christian tribes. The experience of the past and the lower court observed that the use of highly intoxicating liquors by the non-Christian tribes often resulted in lawlessness and crimes, which hamper the efforts of the Government to raise their standard of life and civilization. Third, the said act is intended to apply for all times as long as the conditions exist. Legislature understood that civilization of a people is a slow process and that hand in hand with it must go measures of protection and security. Fourth, the act applies equally to all members of same class. Ichong vs. Hernandez [G.R. No. L-7995, May 31, 1957] Facts: Petitioner, for and in his own behalf and on behalf of other alien residents corporations and partnerships adversely affected by the provisions of Republic Act. No. 1180, “An Act to Regulate the Retail Business,” filed to obtain a judicial declaration that said Act is unconstitutional contending that: (1) it denies to alien residents the equal protection of the laws and deprives of their liberty and property without due process of law ; (2) the subject of the Act is not expressed or comprehended in the title thereof; (3) the Act violates international and treaty obligations of the Republic of the Philippines; (4) the provisions of the Act against the transmission by aliens of their retail business thru hereditary succession, and those requiring 100% Filipino capitalization for a corporation or entity to entitle it to engage in the retail business, violate the spirit of Sections 1 and 5, Article XIII and Section 8 of Article XIV of the Constitution. Issue: Whether RA 1180 denies to alien residents the equal protection of the laws and deprives of their liberty and property without due process of law Held: No. The equal protection of the law clause is against undue favor and individual or class privilege, as well as hostile discrimination or the oppression of inequality. It is not intended to prohibit legislation, which is limited either in the object to which it is directed or by territory within which is to operate. It does not demand absolute equality among residents; it merely requires that all persons shall be

treated alike, under like circumstances and conditions both as to privileges conferred and liabilities enforced. The equal protection clause is not infringed by legislation which applies only to those persons falling within a specified class, if it applies alike to all persons within such class, and reasonable grounds exists for making a distinction between those who fall within such class and those who do not. (2 Cooley, Constitutional Limitations, 824-825.)

This petition seeks to prohibit the implementation of Proc. No. 131 and E.O. No. 229. They contend that taking must be simultaneous with payment of just compensation as it is traditionally understood, i.e., with money and in full, but no such payment is contemplated in Section 5 of the E.O. No. 229.

The due process clause has to do with the reasonableness of legislation enacted in pursuance of the police power. Is there public interest, a public purpose; is public welfare involved? Is the Act reasonably necessary for the accomplishment of the legislature’s purpose; is it not unreasonable, arbitrary or oppressive? Is there sufficient foundation or reason in connection with the matter involved; or has there not been a capricious use of the legislative power? Can the aims conceived be achieved by the means used, or is it not merely an unjustified interference with private interest? These are the questions that we ask when the due process test is applied.

The petitioner argues that E.O. Nos. 228 and 229 are violative of the constitutional provision that no private property shall be taken without due process or just compensation.

The conflict, therefore, between police power and the guarantees of due process and equal protection of the laws is more apparent than real. Properly related, the power and the guarantees are supposed to coexist. The balancing is the essence or, shall it be said, the indispensable means for the attainment of legitimate aspirations of any democratic society. There can be no absolute power, whoever exercise it, for that would be tyranny. Yet there can neither be absolute liberty, for that would mean license and anarchy. So the State can deprive persons of life, liberty and property, provided there is due process of law; and persons may be classified into classes and groups, provided everyone is given the equal protection of the law. The test or standard, as always, is reason. The police power legislation must be firmly grounded on public interest and welfare, and a reasonable relation must exist between purposes and means. And if distinction and classification has been made, there must be a reasonable basis for said distinction. The law does not violate the equal protection clause of the Constitution because sufficient grounds exist for the distinction between alien and citizen in the exercise of the occupation regulated, nor the due process of law clause, because the law is prospective in operation and recognizes the privilege of aliens already engaged in the occupation and reasonably protects their privilege; that the wisdom and efficacy of the law to carry out its objectives appear to us to be plainly evident — as a matter of fact it seems not only appropriate but actually necessary — and that in any case such matter falls within the prerogative of the Legislature, with whose power and discretion the Judicial department of the Government may not interfere; that the provisions of the law are clearly embraced in the title, and this suffers from no duplicity and has not misled the legislators or the segment of the population affected; and that it cannot be said to be void for supposed conflict with treaty obligations because no treaty has actually been entered into on the subject and the police power may not be curtailed or surrendered by any treaty or any other conventional agreement. Association of Small Landowners vs. Secretary of Agrarian Reform [G.R. No. 78742, July 14, 1989] Facts: These are consolidated cases which involve common legal, including serious challenges to the constitutionality of the several measures such as P.D. No. 27, E.O. No. 228, Presidential Proclamation No. 131, E.O. No. 229, and R.A. No. 6657. G.R. No. 79777 The petitioners are questioning P.D. No. 27 and E.O. Nos. 228 and 229 on grounds inter alia of separation of powers, due process, equal protection and the constitutional limitation that no private property shall be taken for public use without just compensation. G.R. No. 79310 G.R. No. 79310

G.R. No. 79744

G.R. No. 78742 Petitioners claim they cannot eject their tenants and so are unable to enjoy their right of retention because the Department of Agrarian Reform has so far not issued the implementing rules required under the above-quoted decree. Held: The argument of the small farmers that they have been denied equal protection because of the absence of retention limits has also become academic under Section 6 of R.A. No. 6657. Significantly, they too have not questioned the area of such limits. There is also the complaint that they should not be made to share the burden of agrarian reform, an objection also made by the sugar planters on the ground that they belong to a particular class with particular interests of their own. However, no evidence has been submitted to the Court that the requisites of a valid classification have been violated. Classification has been defined as the grouping of persons or things similar to each other in certain particulars and different from each other in these same particulars. 31 To be valid, it must conform to the following requirements: (1) it must be based on substantial distinctions; (2) it must be germane to the purposes of the law; (3) it must not be limited to existing conditions only; and (4) it must apply equally to all the members of the class. 32 The Court finds that all these requisites have been met by the measures here challenged as arbitrary and discriminatory. Equal protection simply means that all persons or things similarly situated must be treated alike both as to the rights conferred and the liabilities imposed. 33 The petitioners have not shown that they belong to a different class and entitled to a different treatment. The argument that not only landowners but also owners of other properties must be made to share the burden of implementing land reform must be rejected. There is a substantial distinction between these two classes of owners that is clearly visible except to those who will not see. There is no need to elaborate on this matter. In any event, the Congress is allowed a wide leeway in providing for a valid classification. Its decision is accorded recognition and respect by the courts of justice except only where its discretion is abused to the detriment of the Bill of Rights. It is worth remarking at this juncture that a statute may be sustained under the police power only if there is a concurrence of the lawful subject and the lawful method. Put otherwise, the interests of the public generally as distinguished from those of a particular class require the interference of the State and, no less important, the means employed are reasonably necessary for the attainment of the purpose sought to be achieved and not unduly oppressive upon individuals. 34 As the subject and purpose of agrarian reform have been laid down by the Constitution itself, we may say that the first requirement has been satisfied. What remains to be examined is the validity of the method employed to achieve the constitutional goal. One of the basic principles of the democratic system is that where the rights of the individual are concerned, the end does not justify the means. It is not enough that there be a valid objective; it is also necessary that the means employed to pursue it be in keeping with the Constitution. Mere expediency will not excuse constitutional shortcuts. There is no question that not even the strongest moral conviction or the most urgent public need, subject only to a few notable exceptions, will excuse the bypassing of an individual's rights. It is no exaggeration to

say that a, person invoking a right guaranteed under Article III of the Constitution is a majority of one even as against the rest of the nation who would deny him that right. That right covers the person's life, his liberty and his property under Section 1 of Article III of the Constitution. With regard to his property, the owner enjoys the added protection of Section 9, which reaffirms the familiar rule that private property shall not be taken for public use without just compensation. Villegas v. Hiu Chiong Tsai Pao Ho 86 SCRA 270 FACTS: This case involves an ordinance prohibiting aliens from being employed or engage or participate in any position or occupation or business enumerated therein, whether permanent, temporary or casual, without first securing an employment permit from the Mayor of Manila and paying the permit fee of P50.00. Private respondent Hiu Chiong Tsai Pao Ho who was employed in Manila, filed a petition to stop the enforcement of such ordinance as well as to declare the same null and void. Trial court rendered judgment in favor of the petitioner, hence this case. ISSUE: WON said Ordinance violates due process of law and equal protection rule of the Constitution. Held: The P50.00 fee is unreasonable not only because it is excessive but because it fails to consider valid substantial differences in situation among individual aliens who are required to pay it. Although the equal protection clause of the Constitution does not forbid classification, it is imperative that the classification should be based on real and substantial differences having a reasonable relation to the subject of the particular legislation. The same amount of P50.00 is being collected from every employed alien whether he is casual or permanent, part time or full time or whether he is a lowly employee or a highly paid executive Ordinance No. 6537 does not lay down any criterion or standard to guide the Mayor in the exercise of his discretion. It has been held that where an ordinance of a municipality fails to state any policy or to set up any standard to guide or limit the mayor's action, expresses no purpose to be attained by requiring a permit, enumerates no conditions for its grant or refusal, and entirely lacks standard, thus conferring upon the Mayor arbitrary and unrestricted power to grant or deny the issuance of building permits, such ordinance is invalid, being an undefined and unlimited delegation of power to allow or prevent an activity per se lawful. Dumlao v. COMELEC 96 SCRA 392 Facts: Petitioner Dumlao questions the constitutionality of Sec. 4 of Batas Pambansa Blg 52 as discriminatory and contrary to equal protection and due process guarantees of the Constitution. Sec. 4 provides that any retired elective provicial or municipal official who has received payments of retirement benefits and shall have been 65 years of age at the commencement of the term of office to which he seeks to be elected, shall not be qualified to run for the same elective local office from which he has retired. According to Dumlao, the provision amounts to class legislation. Petitioners Igot and Salapantan Jr. also assail the validity of Sec. 4 of Batas Pambansa Blg 52, which states that any person who has committed any act of disloyalty to the State, including those amounting to subversion, insurrection, rebellion, or other similar crimes, shall not be qualified for any of the offices covered by the act, or to participate in any partisan activity therein: provided that a judgment of conviction of those crimes shall be conclusive evidence of such fact and the filing of charges for the commission of such crimes before a civil court or military tribunal after preliminary investigation shall be prima facie evidence of such fact. Issue: Whether or not the aforementioned statutory provisions violate the Constitution and thus, should be declared null and void

Held: In regards to the unconstitutionality of the provisions, Sec. 4 of BP Blg 52 remains constitutional and valid. The constitutional guarantee of equal protection of the laws is subject to rational classification. One class can be treated differently from another class. In this case, employees 65 years of age are classified differently from younger employees. The purpose of the provision is to satisfy the “need for new blood” in the workplace. In regards to the second paragraph of Sec. 4, it should be declared null and void for being violative of the constitutional presumption of innocence guaranteed to an accused. “Explicit is the constitutional provision that, in all criminal prosecutions, the accused shall be presumed innocent until the contrary is proved, and shall enjoy the right to be heard by himself and counsel (Article IV, section 19, 1973 Constitution). An accusation, according to the fundamental law, is not synonymous with guilt. The challenged proviso contravenes the constitutional presumption of innocence, as a candidate is disqualified from running for public office on the ground alone that charges have been filed against him before a civil or military tribunal. It condemns before one is fully heard. In ultimate effect, except as to the degree of proof, no distinction is made between a person convicted of acts of dislotalty and one against whom charges have been filed for such acts, as both of them would be ineligible to run for public office. A person disqualified to run for public office on the ground that charges have been filed against him is virtually placed in the same category as a person already convicted of a crime with the penalty of arresto, which carries with it the accessory penalty of suspension of the right to hold office during the term of the sentence (Art. 44, Revised Penal Code).” And although the filing of charges is considered as but prima facie evidence, and therefore, may be rebutted, yet. there is "clear and present danger" that because of the proximity of the elections, time constraints will prevent one charged with acts of disloyalty from offering contrary proof to overcome the prima facie evidence against him. Additionally, it is best that evidence pro and con of acts of disloyalty be aired before the Courts rather than before an administrative body such as the COMELEC. A highly possible conflict of findings between two government bodies, to the extreme detriment of a person charged, will thereby be avoided. Furthermore, a legislative/administrative determination of guilt should not be allowed to be substituted for a judicial determination. Being infected with constitutional infirmity, a partial declaration of nullity of only that objectionable portion is mandated. It is separable from the first portion of the second paragraph of section 4 of Batas Pambansa Big. 52 which can stand by itself. Wherefore, the first paragraph of section 4 of Batas pambansa Bilang 52 is hereby declared valid and that portion of the second paragraph of section 4 of Batas Pambansa Bilang 52 is hereby declared null and void, for being violative of the constitutional presumption of innocence guaranteed to an accused. Ormoc Sugar Central v. Ormoc City L-23794; Feb 17 1968 FACTS: The Municipal Board of Ormoc City passed Ordinance No. 4, imposing "on any and all productions of sugar milled at petitioner's, municipal tax of 1% per export sale. Petitioner paid but were under protest. Petitioner filed before the CFI contending that the ordinance is unconstitutional for being in violation of the equal protection clause and the rule of uniformity of taxation, aside from being an export tax forbidden under Section 2287 of the Revised Administrative Code. It further alleged that the tax is neither a production nor a license tax which Ormoc City its charter and under Section 2 of Republic Act 2264, or the Local Autonomy Act, is authorized to impose; that it also violates RA 2264 because the tax is on both the sale and export of sugar.

ISSUE: Whether the ordinance is valid. Held: A perusal of the requisites instantly shows that the questioned ordinance does not meet them (requisites of classification), for it taxes only centrifugal sugar produced and exported by the Ormoc Sugar Company, Inc. and none other. At the time of the taxing ordinance's enactment, Ormoc Sugar Company, Inc., it is true, was the only sugar central in the city of Ormoc. Still, the classification, to be reasonable, should be in terms applicable to future conditions as well. The taxing ordinance should not be singular and exclusive as to exclude any subsequently established sugar central, of the same class as plaintiff, for the coverage of the tax. As it is now, even if later a similar company is set up, it cannot be subject to the tax because the ordinance expressly points only to Ormoc City Sugar Company, Inc. as the entity to be levied upon. Basco v. PAGCOR 197 SCRA 52 Facts: A TV ad proudly announces: “The New PAGCOR – Responding Through Responsible Gaming.” But the petitioners think otherwise, that is why, they filed the instant petition seeking to annul the PAGCOR charter – PD 1869, because it is allegedly contrary to morals, public policy and order, and because – a. It constitutes a waiver of a right prejudicial to a third person with a right recognized by law. It waived the Manila city government’s right to impose taxes and license fees, which is recognized by law; b. For the same reason stated in the immediately preceeding paragraph, the law has intruded into the local government’s right to impose local taxes and license fees. This, in contravention of the constitutionally enshrined principle of local autonomy;

If the law presumably hits the evil where it is most felt, it is not to be overthrown because there are other instances to which it might have been applied. (Gomez v. Palomar, 25 SCRA 827) The equal protection clause of the 14th Amendment does not mean that all occupations called by the same name must be treated the same way; the state may do what it can to prevent which is deemed as evil and stop short of those cases in which harm to the few concerned is not less than the harm to the public that would insure if the rule laid down were made mathematically exact. (Dominican Hotel v. Arizona, 249 US 2651). Binay v. Domingo 201 SCRA 508 Facts: Petitioner Municipality of Makati, through its Council, approved Resolution No. 60 which extends P500 burial assistance to bereaved families whose gross family income does not exceed P2,000.00 a month. The funds are to be taken out of the unappropriated available funds in the municipal treasury. The Metro Manila Commission approved the resolution. Thereafter, the municipal secretary certified a disbursement of P400,000.00 for the implementation of the program. However, the Commission on Audit disapproved said resolution and the disbursement of funds for the implementation thereof for the following reasons: (1) the resolution has no connection to alleged public safety, general welfare, safety, etc. of the inhabitants of Makati; (2) government funds must be disbursed for public purposes only; and, (3) it violates the equal protection clause since it will only benefit a few individuals. Issues:1. Whether Resolution No. 60 is a valid exercise of the police power under the general welfare clause 2. Whether the questioned resolution is for a public purpose

c. It violates the equal protection clause of the constitution in that it legalizes PAGCOR – conducted gambling, while most other forms of gambling are outlawed, together with prostitution, drug trafficking and other vices;

3. Whether the resolution violates the equal protection clause

d. It violates the avowed trend of the Cory government away from the monopolistic and crony economy, and toward free enterprise and privatization.

Held: 1. The police power is a governmental function, an inherent attribute of sovereignty, which was born with civilized government. It is founded largely on the maxims, "Sic utere tuo et ahenum non laedas and "Salus populi est suprema lex. Its fundamental purpose is securing the general welfare, comfort and convenience of the people.

HELD: Petitioners next contend that P.D. 1869 violates the equal protection clause of the Constitution, because "it legalized PAGCOR — conducted gambling, while most gambling are outlawed together with prostitution, drug trafficking and other vices" (p. 82, Rollo).

Police power is inherent in the state but not in municipal corporations. Before a municipal corporation may exercise such power, there must be a valid delegation of such power by the legislature which is the repository of the inherent powers of the State.

We, likewise, find no valid ground to sustain this contention. The petitioners' posture ignores the well-accepted meaning of the clause "equal protection of the laws." The clause does not preclude classification of individuals who may be accorded different treatment under the law as long as the classification is not unreasonable or arbitrary (Itchong v. Hernandez, 101 Phil. 1155). A law does not have to operate in equal force on all persons or things to be conformable to Article III, Section 1 of the Constitution (DECS v. San Diego, G.R. No. 89572, December 21, 1989).

Municipal governments exercise this power under the general welfare clause. Pursuant thereto they are clothed with authority to "enact such ordinances and issue such regulations as may be necessary to carry out and discharge the responsibilities conferred upon it by law, and such as shall be necessary and proper to provide for the health, safety, comfort and convenience, maintain peace and order, improve public morals, promote the prosperity and general welfare of the municipality and the inhabitants thereof, and insure the protection of property therein.

The "equal protection clause" does not prohibit the Legislature from establishing classes of individuals or objects upon which different rules shall operate (Laurel v. Misa, 43 O.G. 2847). The Constitution does not require situations which are different in fact or opinion to be treated in law as though they were the same (Gomez v. Palomar, 25 SCRA 827).

2. Police power is not capable of an exact definition but has been, purposely, veiled in general terms to underscore its all comprehensiveness. Its scope, over-expanding to meet the exigencies of the times, even to anticipate the future where it could be done, provides enough room for an efficient and flexible response to conditions and circumstances thus assuring the greatest benefits.

Just how P.D. 1869 in legalizing gambling conducted by PAGCOR is violative of the equal protection is not clearly explained in the petition. The mere fact that some gambling activities like cockfighting (P.D 449) horse racing (R.A. 306 as amended by RA 983), sweepstakes, lotteries and races (RA 1169 as amended by B.P. 42) are legalized under certain conditions, while others are prohibited, does not render the applicable laws, P.D. 1869 for one, unconstitutional.

The police power of a municipal corporation is broad, and has been said to be commensurate with, but not to exceed, the duty to provide for the real needs of the people in their health, safety, comfort, and convenience as consistently as may be with private rights. It extends to all the great public needs, and, in a broad sense includes all legislation and almost every function of the municipal government. It covers a wide scope of subjects, and, while it is especially occupied with whatever affects the peace, security, health, morals, and general

welfare of the community, it is not limited thereto, but is broadened to deal with conditions which exists so as to bring out of them the greatest welfare of the people by promoting public convenience or general prosperity, and to everything worthwhile for the preservation of comfort of the inhabitants of the corporation. Thus, it is deemed inadvisable to attempt to frame any definition which shall absolutely indicate the limits of police power. Public purpose is not unconstitutional merely because it incidentally benefits a limited number of persons. As correctly pointed out by the Office of the Solicitor General, "the drift is towards social welfare legislation geared towards state policies to provide adequate social services, the promotion of the general welfare, social justice as well as human dignity and respect for human rights." The care for the poor is generally recognized as a public duty. The support for the poor has long been an accepted exercise of police power in the promotion of the common good. 3. There is no violation of the equal protection clause. Paupers may be reasonably classified. Different groups may receive varying treatment. Precious to the hearts of our legislators, down to our local councilors, is the welfare of the paupers. Thus, statutes have been passed giving rights and benefits to the disabled, emancipating the tenant-farmer from the bondage of the soil, housing the urban poor, etc. Resolution No. 60, re-enacted under Resolution No. 243, of the Municipality of Makati is a paragon of the continuing program of our government towards social justice. The Burial Assistance Program is a relief of pauperism, though not complete. The loss of a member of a family is a painful experience, and it is more painful for the poor to be financially burdened by such death. Resolution No. 60 vivifies the very words of the late President Ramon Magsaysay 'those who have less in life, should have more in law." This decision, however must not be taken as a precedent, or as an official go-signal for municipal governments to embark on a philanthropic orgy of inordinate dole-outs for motives political or otherwise. (Binay vs Domingo, G.R. No. 92389, September 11, 1991)

"b)Those who shall attain the age of fifty-nine (59) on the second year of the effectivity of this Act. "c)Those who shall attain the age of fifty-eight (58) on the third year of the effectivity of this Act. "d)Those who shall attain the age of fifty-seven (57) on the fourth year of the effectivity of this Act." Respondents added that the term "INP" includes both the former members of the Philippine Constabulary and the local police force who were earlier constituted as the Integrated National Police (INP) by virtue of PD 765 in 1975. On the other hand, it is the belief of petitioners that the 4-year transition period provided in Section 89 applies only to the local police forces who previously retire, compulsorily, at age sixty (60) for those in the ranks of Police/Fire Lieutenant or higher, while the retirement age for the PC had already been set at fifty-six (56) under the AFP law. Respondent judge De Guzman issued a restraining order followed by a writ of injunction. He declared that the term "INP" in Section 89 of the PNP Law includes all members of the present Philippine National police, irrespective of the original status of the present members of the Philippine National police before its creation and establishment, and that Section 39 thereof shall become operative after the lapse of the four-year transition period. Thus, the preliminary injunction issued is made permanent. Moreover, he observed, among others, that it may have been the intention of Congress to refer to the local police forces as the "INP" but the PNP Law failed to define who or what constituted the INP. The natural recourse of the court is to trace the source of the "INP" as courts are permitted to look to prior laws on the same subject and to investigate the antecedents involved ISSUE: Whether or not Section 89 of the PNP Law includes all members of the present

NPC v. De Guzman 229 SCRA 801

Philippine National police, irrespective of the original status of its present members and that

FACTS: RA 6975, otherwise known as "An Act Establishing the Philippine National Police Under a Reorganized Department of the Interior and Local Government", took effect on January 2, 1991.

Section 39 of RA 6975 shall become applicable to petitioners only after the lapse of the four-year

RA 6975 provides for a uniform retirement system for PNP members. Section 39 reads:

transition period.

"SEC. 39.Compulsory Retirement. — Compulsory retirement, for officer and non-officer, shall be upon the attainment of age fifty-six (56); Provided, That, in case of any officer with the rank of chief superintendent, director or deputy director general, the Commission may allow his retention in the service for an unextendible period of one (1) year.

HELD: From a careful review of Sections 23 and 85 of RA 6975, it appears that the use of the term INP is not synonymous with the PC. Had it been otherwise, the statute could have just made a uniform reference to the members of the whole Philippine National police (PNP) for retirement purposes and not just the INP. The law itself distinguishes INP from the PC and it cannot be construed that "INP" as used in Sec. 89 includes the members of the PC.

Based on the above provision, petitioners sent notices of retirement to private respondents who are all members of the defunct Philippine Constabulary and have reached the age of fifty-six.

Contrary to the pronouncement of respondent judge that “the law failed to define who constitutes the INP”, Sec. 90 of RA 6975 has in fact defined the same. Thus,

Private respondents filed a complaint for declaratory relief with prayer for the issuance of an ex parte restraining order and/or injunction before the RTC of Makati. They aver that the age of retirement set at fifty-six (56) by Section 39 of RA 6975 cannot be applied to them since they are also covered by Sec. 89 thereof which provides:

"SEC. 90. Status of Present NAPOLCOM, PC-INP. — Upon the effectivity of this Act, the present National police Commisdion and the Philippine Constabulary-Integrated

"Any provision hereof to the contrary notwithstanding, and within the transition period of four (4) years following the effectively of this Act, the following members of the INP shall be considered compulsorily retired: "a)Those who shall attain the age of sixty (60) on the first year of the effectivity of this Act.

National police shall cease to exist. The Philippine Constabulary, which is the nucleus of the Philippine Constabulary-Integrated National police shall cease to be a major service of the Armed Forces of the Philippines. The Integrated National police, which is the civilian component of the Philippine Constabulary-Integrated National police, shall cease to be the national police force and lieu thereof, a new police force shall be establish and constituted pursuant to this Act." It is not altogether correct to state, therefore, that the legislature failed to define who the members of the INP are. In this regard, it is of no

moment that the legislature failed to categorically restrict the application of the transition period in Sec. 89 specifically in favor of the local police forces for it would be a mere superfluity as the PC component of the INP was already retirable at age fifty-six (56). Having defined the meaning of INP, the trial court need not have belabored on the supposed dubious meaning of the term. Nonetheless, if confronted with such a situation, courts are not without recourse in determining the construction of the statute with doubtful meaning for they may avail themselves of the actual proceedings of the legislative body. In case of doubt as to what a provision of a statute means, the meaning put to the provision during the legislative deliberations may be adopted. Courts should not give a literal interpretation to the letter of the law if it runs counter to the legislative intent. The legislative intent to classify the INP in such manner that Section 89 of R.A. 6975 is applicable only to the local police force is clear. The question now is whether the classification is valid. The test for this is reasonableness such that it must conform to the following requirements: (1) It must be based upon substantial distinctions; (2) It must be germane to the purpose of the law; (3) It must not be limited to existing conditions only; (4) It must apply equally to all members of the same class Himagan v. People 237 SCRA 538 FACTS: Himagan is a policeman assigned in Camp Catititgan, Davao City. He was charged for the murder of and attempted murder. Pursuant to Sec 47 of RA 6975, Himagan was placed into suspension pending the murder case. The law provides that “Upon the filing of a complaint or information sufficient in form and substance against a member of the PNP for grave felonies where the penalty imposed by law is six (6) years and one (1) day or more, the court shall immediately suspend the accused from office until the case is terminated. Such case shall be subject to continuous trial and shall be terminated within ninety (90) days from arraignment of the accused. Himagan assailed the suspension averring that Sec 42 of PD 807 of the Civil Service Decree, that his suspension should be limited to ninety (90) days. He claims that an imposition of preventive suspension of over 90 days is contrary to the Civil Service Law and would be a violation of his constitutional right to equal protection of laws. ISSUE: Whether or not Sec 47, RA 6975 violates equal protection guaranteed by the Constitution. HELD: No. The reason why members of the PNP are treated differently from the other classes of persons charged criminally or administratively insofar as the application of the rule on preventive suspension is concerned is that policemen carry weapons and the badge of the law which can be used to harass or intimidate witnesses against them, as succinctly brought out in the legislative discussions. If a suspended policeman criminally charged with a serious offense is reinstated to his post while his case is pending, his victim and the witnesses against him are obviously exposed to constant threat and thus easily cowed to silence by the mere fact that the accused is in uniform and armed. The imposition of preventive suspension for over 90 days under Sec 47 of RA 6975 does not violate the suspended policeman’s constitutional right to equal protection of the laws.

5 March 1987, Tablarin, et. al., in behalf of applicants for admission into the Medical Colleges who have not taken up or successfully hurdled the NMAT, filed with the Regional Trial Court (RTC), National Capital Judicial Region, a Petition for Declaratory Judgment and Prohibition with a prayer for Temporary Restraining Order (TRO) and Preliminary Injunction, to enjoin the Secretary of Education, Culture and Sports, the Board of Medical Education and the Center for Educational Measurement from enforcing Section 5 (a) and (f) of Republic Act 2382, as amended, and MECS Order 52 (series of 1985), dated 23 August 1985 [which established a uniform admission test (NMAT) as an additional requirement for issuance of a certificate of eligibility for admission into medical schools of the Philippines, beginning with the school year 1986-1987] and from requiring the taking and passing of the NMAT as a condition for securing certificates of eligibility for admission, from proceeding with accepting applications for taking the NMAT and from administering the NMAT as scheduled on 26 April 1987 and in the future. After hearing on the petition for issuance of preliminary injunction, the trial court denied said petition on 20 April 1987. The NMAT was conducted and administered as previously scheduled. Tablarin, et. al. accordingly filed a Special Civil Action for Certiorari with the Supreme Court to set aside the Order of the RTC judge denying the petition for issuance of a writ of preliminary injunction. Petitioners have contended, finally, that MECS Order No. 52, s. 1985, is in conflict with the equal protection clause of the Constitution. More specifically, petitioners assert that that portion of the MECS Order which provides that "the cutoff score for the successful applicants, based on the scores on the NMAT, shall be determined every year by the Board of Medical Education after consultation with the Association of Philippine Medical Colleges." (Underscoring supplied)It infringes the requirements of equal protection. They assert, in other words, that students seeking admission during a given school year, e.g., 19871988, when subjected to a different cutoff score than that established for an, e.g., earlier school year, are discriminated against and that this renders the MECS Order "arbitrary and capricious". Held: Different cutoff scores for different school years may be dictated by differing conditions obtaining during those years. Thus, the appropriate cutoff score for a given year may be a function of such factors as the number of students who have reached the cutoff score established the preceding year; the number of places available in medical schools during the current year; the average score attained during the current year; the level of difficulty of the test given during the current year, and so forth. To establish a permanent and immutable cutoff score regardless of changes in circumstances from year to year, may well result in an unreasonable rigidity. The above language in MECS Order No. 52, far from being arbitrary or capricious, leaves the Board of Medical Education with the measure of flexibility needed to meet circumstances as they change. We conclude that prescribing the NMAT and requiring certain minimum scores therein as a condition for admission to medical schools in the Philippines, do not constitute an unconstitutional imposition. Lim v. Pacquing 240 SCRA 649 FACTS:The Charter of the City of Manila was enacted by Congress on 18 June 1949 (R.A. No. 409).

Tablarin v. Gutierrez 152 SCRA 730

On 1 January 1951, Executive Order No. 392 was issued transferring the authority to regulate jai-alais from local government to the Games and Amusements Board (GAB).

Facts:Teresita Tablarin, Ma. Luz Ciriaco, Ma. Nimfa B. Rovira, and Evangelina S. Labao sought admission into colleges or schools of medicine for the school year 1987-1988. However, they either did not take or did not successfully take the National Medical Admission Test (NMAT) required by the Board of Medical Education and administered by the Center for Educational Measurement (CEM). On

On 07 September 1971, however, the Municipal Board of Manila nonetheless passed Ordinance No. 7065 entitled “An Ordinance Authorizing the Mayor To Allow And Permit The Associated Development Corporation To Establish, Maintain And Operate A JaiAlai In The City Of Manila, Under Certain Terms And Conditions And For Other Purposes.”

On 20 August 1975, Presidential Decree No. 771 was issued by then President Marcos. The decree, entitled “Revoking All Powers and Authority of Local Government(s) To Grant Franchise, License or Permit And Regulate Wagers Or Betting By The Public On Horse And Dog Races, Jai-Alai Or Basque Pelota, And Other Forms Of Gambling”, in Section 3 thereof, expressly revoked all existing franchises and permits issued by local governments. In May 1988, Associated Development Corporation (ADC) tried to operate a Jai-Alai. The government through Games and Amusement Board intervened and invoked Presidential Decree No. 771 which expressly revoked all existing franchises and permits to operate all forms of gambling facilities (including Jai-Alai) by local governments. ADC assails the constitutionality of P.D. No. 771. ISSUE: Whether or not P.D. No. 771 is violative of the equal protection and non-impairment clauses of the Constitution. HELD:NO. P.D. No. 771 is valid and constitutional. On the alleged violation of the non-impairment and equal protection clauses of the Constitution, it should be remembered that a franchise is not in the strict sense a simple contract but rather it is more importantly, a mere privilege specially in matters which are within the government's power to regulate and even prohibit through the exercise of the police power. Thus, a gambling franchise is always subject to the exercise of police power for the public welfare. In RCPI v. NTC (150 SCRA 450), we held that: A franchise started out as a "royal privilege or (a) branch of the King's prerogative, subsisting in the hands of a subject." This definition was given by Finch, adopted by Blackstone, and accepted by every authority since . . . Today, a franchise being merely a privilege emanating from the sovereign power of the state and owing its existence to a grant, is subject to regulation by the state itself by virtue of its police power through its administrative agencies. There is a stronger reason for holding ADC's permit to be a mere privilege because jai-alai, when played for bets, is pure and simple gambling. To analogize a gambling franchise for the operation of a public utility, such as public transportation company, is to trivialize the great historic origin of this branch of royal privilege. As earlier noted, ADC has not alleged ever applying for a franchise under the provisions of PD No. 771. and yet, the purpose of PD No. 771 is quite clear from its provisions, i.e., to give to the national government the exclusive power to grant gambling franchises. Thus, all franchises then existing were revoked but were made subject to reissuance by the national government upon compliance by the applicant with government-set qualifications and requirements. There was no violation by PD No. 771 of the equal protection clause since the decree revoked all franchises issued by local governments without qualification or exception. ADC cannot allege violation of the equal protection clause simply because it was the only one affected by the decree, for as correctly pointed out by the government, ADC was not singled out when all jai-alai franchises were revoked. Besides, it is too late in the day for ADC to seek redress for alleged violation of its constitutional rights for it could have raised these issues as early as 1975, almost twenty 920) years ago. RATIO:Presumption against unconstitutionality. There is nothing on record to show or even suggest that PD No. 771 has been repealed, altered or amended by any subsequent law or presidential issuance (when the executive still exercised legislative powers). Neither can it be tenably stated that the issue of the continued existence of ADC’s franchise by reason of the unconstitutionality of PD No. 771 was settled in G.R. No. 115044, for the decision of the Court’s First Division in said case, aside from not being final, cannot have the effect of nullifying PD No. 771 as unconstitutional, since only the Court En

Banc has that power under Article VIII, Section 4(2) of the Constitution. And on the question of whether or not the government is estopped from contesting ADC’s possession of a valid franchise, the well-settled rule is that the State cannot be put in estoppel by the mistakes or errors, if any, of its officials or agents. (Republic v. Intermediate Appellate Court, 209 SCRA 90) Philippine Judges Association v. Prado 227 SCRA 703 Facts: Section 35 of RA 7354, the law creating the Philippine Postal Corporation, withdraws the franking privileges from the judiciary, along with certain other government offices. he Philippine Postal Corporation issued circular No. 92-28 to implement Section 35 of RA 7354 withdrawing the franking privilege from the SC, CA, RTCs, MeTCs, MTCs and Land Registration Commission and with certain other government offices. It is alleged that RA 7354 is discriminatory becasue while withdrawing the franking privilege from judiciary, it retains the same for the President & Vice-President of the Philippines, Senator & members of the House of Representatives, COMELEC, National Census & Statistics Office and the general public. The respondents counter that there is no discrimination because the law is based on a valid classification in accordance with the equal protection clause. Issue: Whether or Not Section 35 of RA 7354 is constitutional. Held: The equal protection of the laws is embraced in the concept of due process, as every unfair discrimination offends the requirements of justice and fair play. It has nonetheless been embodied in a separate clause in Article III Section 1 of the Constitution to provide for amore specific guarantee against any form of undue favoritism or hostility from the government. Arbitrariness in general may be challenged on the basis of the due process clause. But if the particular act assailed partakes of an unwarranted partiality or prejudice, the sharper weapon to cut it down is the equal protection clause. Equal protection simply requires that all persons or things similarly situated should be treated alike, both as to rights conferred and responsibilities imposed. What the clause requires is equality among equals as determined according to a valid classification. Section 35 of RA 7354 is declared unconstitutional. Circular No. 92-28 is set aside insofar In lumping the Judiciary with the other offices from which the franking privilege has been withdrawn, Section 35 has placed the courts of justice in a category to which it does not belong. If it recognizes the need of the President of the Philippines and the members of Congress for the franking privilege, there is no reason why it should not recognize a similar and in fact greater need on the part of the Judiciary for such privilege. Sison v. Ancheta 130 SCRA 654 Facts: Petitioners challenged the constitutionality of Section 1 of Batas Pambansa Blg. 135. It amended Section 21 of the National Internal Revenue Code of 1977, which provides for rates of tax on citizens or residents on (a) taxable compensation income, (b) taxable net income, (c) royalties, prizes, and other winnings, (d) interest from bank deposits and yield or any other monetary benefit from deposit substitutes and from trust fund and similar arrangements, (e) dividends and share of individual partner in the net profits of taxable partnership, (f) adjusted gross income. Petitioner as taxpayer alleged that "he would be unduly discriminated against by the imposition of higher rates of tax upon his income arising from the exercise of his profession vis-a-vis those which are imposed upon fixed income or salaried individual taxpayers." He characterizes the above section as arbitrary amounting to class legislation, oppressive and capricious in character.

For petitioner, therefore, there is a transgression of both the equal protection and due process clauses of the Constitution as well as of the rule requiring uniformity in taxation. Issue: Whether the imposition of a higher tax rate on taxable net income derived from business or profession than on compensation is constitutionally infirm. (WON there is a transgression of both the equal protection and due process clauses of the Constitution as well as of the rule requiring uniformity in taxation) Held: No. Petition dismissed Ratio: The need for more revenues is rationalized by the government's role to fill the gap not done by public enterprise in order to meet the needs of the times. It is better equipped to administer for the public welfare. The power to tax, an inherent prerogative, has to be availed of to assure the performance of vital state functions. It is the source of the bulk of public funds. The power to tax is an attribute of sovereignty and the strongest power of the government. There are restrictions, however, diversely affecting as it does property rights, both the due process and equal protection clauses may properly be invoked, as petitioner does, to invalidate in appropriate cases a revenue measure. If it were otherwise, taxation would be a destructive power.

The petitioner failed to prove that the statute ran counter to the Constitution. He used arbitrariness as basis without a factual foundation. This is merely to adhere to the authoritative doctrine that where the due process and equal protection clauses are invoked, considering that they are not fixed rules but rather broad standards, there is a need for proof of such persuasive character as would lead to such a conclusion.

protection of the laws,' and laws are not abstract propositions. They do not relate to abstract units A, B and C, but are expressions of policy arising out of specific difficulties, addressed to the attainment of specific ends by the use of specific remedies. The Constitution does not require things which are different in fact or opinion to be treated in law as though they were the same. Lutz v Araneta- it is inherent in the power to tax that a state be free to select the subjects of taxation, and it has been repeatedly held that 'inequalities which result from a singling out of one particular class for taxation, or exemption infringe no constitutional limitation. Petitioner- kindred concept of uniformity- Court- Philippine Trust Company- The rule of uniformity does not call for perfect uniformity or perfect equality, because this is hardly attainable Equality and uniformity in taxation means that all taxable articles or kinds of property of the same class shall be taxed at the same rate. The taxing power has the authority to make reasonable and natural classifications for purposes of taxation There is quite a similarity then to the standard of equal protection for all that is required is that the tax "applies equally to all persons, firms and corporations placed in similar situation" There was a difference between a tax rate and a tax base. There is no legal objection to a broader tax base or taxable income by eliminating all deductible items and at the same time reducing the applicable tax rate. The discernible basis of classification is the susceptibility of the income to the application of generalized rules removing all deductible items for all taxpayers within the class and fixing a set of reduced tax rates to be applied to all of them. As there is practically no overhead expense, these taxpayers are not entitled to make deductions for income tax purposes because they are in the same situation more or less.Taxpayers who are recipients of compensation income are set apart as a class.

It is undoubted that the due process clause may be invoked where a taxing statute is so arbitrary that it finds no support in the Constitution. An obvious example is where it can be shown to amount to the confiscation of property. That would be a clear abuse of power.

On the other hand, in the case of professionals in the practice of their calling and businessmen, there is no uniformity in the costs or expenses necessary to produce their income. It would not be just then to disregard the disparities by giving all of them zero deduction and indiscriminately impose on all alike the same tax rates on the basis of gross income.

It has also been held that where the assailed tax measure is beyond the jurisdiction of the state, or is not for a public purpose, or, in case of a retroactive statute is so harsh and unreasonable, it is subject to attack on due process grounds.

There was a lack of a factual foundation, the forcer of doctrines on due process and equal protection, and he reasonableness of the distinction between compensation and taxable net income of professionals and businessmen not being a dubious classification.

For equal protection, the applicable standard to determine whether this was denied in the exercise of police power or eminent domain was the presence of the purpose of hostility or unreasonable discrimination.

Telebap v. COMELEC 289 SCRA 337

It suffices then that the laws operate equally and uniformly on all persons under similar circumstances or that all persons must be treated in the same manner, the conditions not being different, both in the privileges conferred and the liabilities imposed. Favoritism and undue preference cannot be allowed. For the principle is that equal protection and security shall be given to every person under circumstances, which if not identical are analogous. If law be looks upon in terms of burden or charges, those that fall within a class should be treated in the same fashion, whatever restrictions cast on some in the group equally binding on the rest. The equal protection clause is, of course, inspired by the noble concept of approximating the ideal of the laws's benefits being available to all and the affairs of men being governed by that serene and impartial uniformity, which is of the very essence of the idea of law. The equality at which the 'equal protection' clause aims is not a disembodied equality. The Fourteenth Amendment enjoins 'the equal

Facts: Petitioner Telecommunications and Broadcast Attorneys of the Philippines, Inc. (TELEBAP) is an organization of lawyers of radio and television broadcasting companies. Section 92 of Batas Pambansa (BP) Blg. 881, as amended, reads as follows: Sec. 92. Comelec time. — The commission shall procure radio and television time to be known as “Comelec Time” which shall be allocated equally and impartially among the candidates within the area of coverage of all radio and television stations. For this purpose, the franchise of all radio broadcasting and television stations are hereby amended so as to provide radio or television time, free of charge, during the period of the campaign. Petitioners contend that §92 of BP Blg. 881 violates the due process clause and the eminent domain provision of the Constitution by taking airtime from radio and television broadcasting stations without payment of just compensation. 1) have taken properties without due process of law and without just compensation; (2) it denied the radio and television broadcast companies the equal protection of the laws; and (3) that it is in excess of the power given to the Comelec to regulate the operation of media communication or information during election

period. Petitioners claim that the primary source of revenue of the radio and television stations is the sale of airtime to advertisers and that to require these stations to provide free airtime is to authorize a taking which is not “a de minimis temporary limitation or restraint upon the use of private property.” According to petitioners, in 1992, the GMA Network, Inc. lost P22,498,560.00 in providing free airtime of one (1) hour every morning from Mondays to Fridays and one (1) hour on Tuesdays and Thursdays from 7:00 to 8:00 p.m. (prime time) and, in this year’s elections, it stands to lose P58,980,850.00 in view of COMELEC’s requirement that radio and television stations provide at least 30 minutes of prime time daily for the COMELEC Time. Held:Petitioners' argument is without merit, All broadcasting, whether by radio or by television stations, is licensed by the government. Airwave frequencies have to be allocated as there are more individuals who want to broadcast than there are frequencies to assign. 9 A franchise is thus a privilege subject, among other things, to amended by Congress in accordance with the constitutional provision that "any such franchise or right granted . . . shall be subject to amendment, alteration or repeal by the Congress when the common good so requires." Indeed, provisions for COMELEC Time have been made by amendment of the franchises of radio and television broadcast stations and, until the present case was brought, such provisions had not been thought of as taking property without just compensation. Art. XII, §11 of the Constitution authorizes the amendment of franchises for "the common good." What better measure can be conceived for the common good than one for free air time for the benefit not only of candidates but even more of the public, particularly the voters, so that they will be fully informed of the issues in an election? "[I]t is the right of the viewers and listeners, not the right of the broadcasters, which is paramount." Nor indeed can there be any constitutional objection to the requirement that broadcast stations give free air time. Even in the United States, there are responsible scholars who believe that government controls on broadcast media can constitutionally be instituted to ensure diversity of views and attention to public affairs to further the system of free expression. For this purpose, broadcast stations may be required to give free air time to candidates in an election. In truth, radio and television broadcasting companies, which are given franchises, do not own the airwaves and frequencies through which they transmit broadcast signals and images. They are merely given the temporary privilege of using them. Since a franchise is a mere privilege, the exercise of the privilege may reasonably be burdened with the performance by the grantee of some form of public service. On the other hand, the transistor radio is found everywhere. The television set is also becoming universal. Their message may be simultaneously received by a national or regional audience of listeners including the indifferent or unwilling who happen to be within reach of a blaring radio or television set. The materials broadcast over the airwaves reach every person of every age, persons of varying susceptibilities to persuasion, persons of different I.Q.s and mental capabilities, persons whose reactions to inflammatory or offensive speech would be difficult to monitor or predict. The impact of the vibrant speech is forceful and immediate. Unlike readers of the printed work, the radio audience has lesser opportunity to cogitate, analyze, and reject the utterance.[30] Petitioners assertion therefore that 92 of B.P. Blg. 881 denies them the equal protection of the law has no basis. In addition, their plea that 92 (free air time) and 11(b) of R.A. No. 6646 (ban on paid political ads) should be invalidated would pave the way for a return to the old regime where moneyed candidates could monopolize media advertising to the disadvantage of candidates with less resources. That is what Congress tried to reform in 1987 with the enactment of R.A. No. 6646. We are not free to set aside the judgment of Congress, especially in light of the

recent failure of interested parties to have the law repealed or at least modified. Tiu v. CA 301 S

`1CRA 278

Facts: On March 13, 1992, Congress, with the approval of the President, passed into law RA 7227. This was for the conversion of former military bases into industrial and commercial uses. Subic was one of these areas. It was made into a special economic zone. In the zone, there were no exchange controls. Such were liberalized. There was also tax incentives and duty free importation policies under this law. On June 10, 1993, then President Fidel V. Ramos issued Executive Order No. 97 (EO 97), clarifying the application of the tax and duty incentives. It said that On Import Taxes and Duties. — Tax and duty-free importations shall apply only to raw materials, capital goods and equipment brought in by business enterprises into the SSEZ On All Other Taxes. — In lieu of all local and national taxes (except import taxes and duties), all business enterprises in the SSEZ shall be required to pay the tax specified in Section 12(c) of R.A. No. 7227. Nine days after, on June 19, 1993, the President issued Executive Order No. 97-A (EO 97-A), specifying the area within which the tax-andduty-free privilege was operative. Section 1.1. The Secured Area consisting of the presently fenced-in former Subic Naval Base shall be the only completely tax and dutyfree area in the SSEFPZ. Business enterprises and individuals (Filipinos and foreigners) residing within the Secured Area are free to import raw materials, capital goods, equipment, and consumer items tax and duty-free. Petitioners challenged the constitutionality of EO 97-A for allegedly being violative of their right to equal protection of the laws. This was due to the limitation of tax incentives to Subic and not to the entire area of Olongapo. The case was referred to the Court of Appeals. The appellate court concluded that such being the case, petitioners could not claim that EO 97-A is unconstitutional, while at the same time maintaining the validity of RA 7227. The court a quo also explained that the intention of Congress was to confine the coverage of the SSEZ to the "secured area" and not to include the "entire Olongapo City and other areas mentioned in Section 12 of the law. Hence, this was a petition for review under Rule 45 of the Rules of Court. Issue: Whether the provisions of Executive Order No. 97-A confining the application of R.A. 7227 within the secured area and excluding the residents of the zone outside of the secured area is discriminatory or not owing to a violation of the equal protection clause. Held. No. Petition dismissed. Ratio: Citing Section 12 of RA 7227, petitioners contend that the SSEZ encompasses (1) the City of Olongapo, (2) the Municipality of Subic in Zambales, and (3) the area formerly occupied by the Subic Naval Base. However, they claimed that the E.O. narrowed the application to the naval base only. OSG- The E.O. Was a valid classification. Court- The fundamental right of equal protection of the laws is not absolute, but is subject to reasonable classification. If the groupings are

characterized by substantial distinctions that make real differences, one class may be treated and regulated differently from another. The classification must also be germane to the purpose of the law and must apply to all those belonging to the same class.

Inchong v Hernandez- Equal protection does not demand absolute equality among residents; it merely requires that all persons shall be treated alike, under like circumstances and conditions both as to privileges conferred and liabilities enforced.

Classification, to be valid, must (1) rest on substantial distinctions, (2) be germane to the purpose of the law, (3) not be limited to existing conditions only, and (4) apply equally to all members of the same class.

RA 7227 aims primarily to accelerate the conversion of military reservations into productive uses. This was really limited to the military bases as the law's intent provides. Moreover, the law tasked the BCDA to specifically develop the areas the bases occupied.

Among such enticements are: (1) a separate customs territory within the zone, (2) tax-and-duty-free importations, (3) restructured income tax rates on business enterprises within the zone, (4) no foreign exchange control, (5) liberalized regulations on banking and finance, and (6) the grant of resident status to certain investors and of working visas to certain foreign executives and workers. The target of the law was the big investor who can pour in capital.e Even more important, at this time the business activities outside the "secured area" are not likely to have any impact in achieving the purpose of the law, which is to turn the former military base to productive use for the benefit of the Philippine economy. Hence, there was no reasonable basis to extend the tax incentives in RA 7227. It is well-settled that the equal-protection guarantee does not require territorial uniformity of laws. As long as there are actual and material differences between territories, there is no violation of the constitutional clause. Besides, the businessmen outside the zone can always channel their capital into it. RA 7227, the objective is to establish a "self-sustaining, industrial, commercial, financial and investment center”. There will really be differences between it and the outside zone of Olongapo. The classification of the law also applies equally to the residents and businesses in the zone. They are similarly treated to contribute to the end gaol of the law. Lacson v. Executive Secretary 301 SCRA 298 Facts: Eleven persons believed to be members of the Kuratong Baleleng gang, an organized crime syndicate involved in bank robberies, were slain by elements of the Anti-Bank Robbery andIntelligence Task Group (ABRITG). Among those included in the ABRITG were petitioners and petitioner-intervenors. Acting on a media expose of SPO2 Eduardo delos Reyes, a member of the Criminal Investigation Command, that what actually transpired was a summary execution and not a shoot-out between the Kuratong Baleleng gang members and the ABRITG, Ombudsman Aniano Desiertoformed a panel of investigators to investigate the said incident. Said panel found the incident as a legitimate police operation. However, a review board modified the panel’s finding and recommended the indictment for multiple murder against twenty-six

respondents including herein petitioner, charged as principal, and herein petitioner-intervenors, charged as accessories. After a reinvestigation, the Ombudsman filed amended informations before the Sandiganbayan, where petitioner was charged only as an accessory. The accused filed separate motions questioning the jurisdiction of the Sandiganbayan, asserting that under the amended informations, the cases fall within the jurisdiction of the Regional Trial Court pursuant to Section 2 of R.A. 7975. They contend that the said law limited the jurisdiction of the Sandiganbayan to cases where one or ore of the “principal accused” are government officals with Salary Grade 27 or higher, or PNP officials with rank of Chief Superintendent or higher. Thus, they did not qualify under said requisites. However, pending resolution of their motions, R.A. 8249 was approved amending the jurisdiction of the Sandiganbayan by deleting the word “principal” from the phrase “principal accused” in Section 2 of R.A. 7975. Petitioner questions the constitutionality of Section 4 of R.A. 8249, including Section 7 which provides that the said law shall apply to all cases pending in any court over which trial has not begun as of the approval hereof. Issues: (1) Whether or not Sections 4 and 7 of R.A. 8249 violate the petitioners’ right to due process and the equal protection clause of the Constitution as the provisions seemed to have been introduced for the Sandiganbayan to continue to acquire jurisdiction over the Kuratong Baleleng case. (2) Whether or not said statute may be considered as an ex-post facto statute. (3) Whether or not the multiple murder of the alleged members of the Kuratong Baleleng was committed in relation to the office of the accused PNP officers which is essential to the determination whether the case falls within the Sandiganbayan’s or Regional Trial Court’s jurisdiction. RULING: Petitioner and intervenors’ posture that Sections 4 and 7 of R.A. 8249 violate their right to equal protection of the law is too shallow to deserve merit. No concrete evidence and convincing argument were presented to warrant such a declaration. Every classification made by the law is presumed reasonable and the party who challenges the law must present proof of arbitrariness. The classification is reasonable and not arbitrary when the following concur: (1) it must rest on substantial distinction; (2) it must be germane to the purpose of the law; (3) must not be limited to existing conditions only, and (4) must apply equally to all members of the same class; all of which are present in this case. Paragraph a of Section 4 provides that it shall apply “to all cases involving” certain public officials and under the transitory provision in Section 7, to “all cases pending in any court.” Contrary to petitioner and intervenors’ argument, the law is not particularly directed only to the Kuratong Baleleng cases. The transitory provision does not only cover cases which are in the Sandiganbayan but also in “any court.” There is nothing ex post facto in R.A. 8249. Ex post facto law, generally, provides retroactive effect of penal laws. R.A. 8249 is not apenal law. It is a substantive law on jurisdiction which is not penal in character. Penal laws are those acts of the Legislature which prohibit certain acts and establish penalties for their violations or those that define crimes and provide for their punishment. R.A. 7975, as regards the Sandiganbayan’s jurisdiction, its mode of appeal and other procedural matters, has been declared by the Court as not a penal law, but clearly a procedural statute, one which prescribes rules of procedure by which courts applying laws of all kinds can properly administer justice. Not being a penal law, the retroactive application of R.A. 8249 cannot be challenged as unconstitutional.

In People vs. Montejo, it was held that an offense is said to have been committed in relation to the office if it is intimately connected with the office of the offender and perpetrated while he was in the performance of his official functions. Such intimate relation must be alleged in the information which is essential in determining the jurisdiction of the Sandiganbayan. However, upon examination of the amended information, there was no specific allegation of facts that the shooting of the victim by the said principal accused was intimately related to the discharge of their official duties as police officers. Likewise, the amended information does not indicate that the said accused arrested and investigated the victim and then killed the latter while in their custody. The stringent requirement that the charge set forth with such particularity as will reasonably indicate the exact offense which the accused is alleged to have committed in relation to his office was not established. Consequently, for failure to show in the amended informations that the charge of murder was intimately connected with the discharge of official functions of the accused PNP officers, the offense charged in the subject criminal cases is plain murder and, therefore, within the exclusive original jurisdiction of the Regional Trial Court and not the Sandiganbayan. Soriano v. CA, GR 123936, March 4, 1999 Facts: Petitioner Ronald Soriano was convicted of the crime of Reckless Imprudence resulting to homicide, serious physical injuries and damage to property on December 7, 1993.[2] His application for probation was granted on March 8, 1994, and among the terms and conditions imposed by the trial court were the following:[3] "x x x x x x x x x 7. He shall meet his family responsibilities. 8. He shall devote himself to a specific employment and shall not change employment without prior notice to the supervising officer; and/or shall pursue a prescribed secular study or vocational training.

xxxxxxxxx

11. He is to indemnify the heirs of the victim Isidrino Daluyong in the amount of P98,560.00 as ordered by the Court.

xxxxxxxxx

On April 26, 1994, Assistant Prosecutor Benjamin A. Fadera filed a motion to cancel petitioners probation due to his failure to satisfy his civil liability to the heirs of the victim, and a supplemental motion alleging petitioners commission of another crime for which at that time he was awaiting arraignment. The Zambales Parole and Probation Office filed a comment recommending that petitioner be allowed to continue with his probation and that he be required instead to submit a program of payment of his civil liability. Thereafter, probation officer Nelda Da Maycong received information that petitioners father, who owned the vehicle involved in the accident which killed Daluyong, received P16,500.00 as insurance payment. This amount was not turned over to the heirs of Daluyong and Da Maycong considered this a violation of the terms and conditions of the probation. She submitted a manifestation to the trial court praying that petitioner be made to explain his non-compliance with the courts order of June 20, 1994, or that he be cited for contempt for such noncompliance. Da Maycong also asked that petitioner be made to submit

a program of payment as soon as possible. The trial court granted her prayers in an order dated August 15, 1994. Petitioner was once again ordered to submit his program of payment. Petitioner instead filed a motion for reconsideration explaining that he did not receive any notice of the order dated June 20, 1994. His counsel received a copy of said order on June 23, 1994 but failed to notify petitioner. Thus, the latter failed to comply with said order. On October 4, 1994, the trial court issued an order declaring petitioner in contempt of court for his failure to comply with its orders of June 20, 1994 and August 15, 1994. The court likewise revoked the grant of probation to petitioner and ordered that he be arrested to serve the sentence originally imposed upon him. Petitioner avers that to require him to satisfy his civil liability in order to continue to avail of the benefits of probation is to violate the constitutional proscription against unequal protection of the law. He says only moneyed probationers will be able to benefit from probation if satisfaction of civil liability is made a condition. Held: Contrary to his assertion, this requirement is not violative of the equal protection clause of the Constitution. Note that payment of the civil liability is not made a condition precedent to probation. If it were, then perhaps there might be some basis to petitioners assertion that only moneyed convicts may avail of the benefits of probation. In this case, however, petitioners application for probation had already been granted. Satisfaction of his civil liability was not made a requirement before he could avail of probation, but was a condition for his continued enjoyment of the same. The trial court could not have done away with imposing payment of civil liability as a condition for probation, as petitioner suggests. This is not an arbitrary imposition but one required by law. It is a consequence of petitioners having been convicted of a crime,[12] and petitioner is bound to satisfy this obligation regardless of whether or not he is placed under probation. We fail to see why petitioner cannot comply with a simple order to furnish the trial court with a program of payment of his civil liability. He may, indeed, be poor, but this is precisely the reason why the trial court gave him the chance to make his own program of payment. Knowing his own financial condition, he is in the best position to formulate a program of payment that fits his needs and capacity. Loong v. COMELEC 305 SCRA 832 PUTANGINA NETO Automated elections were held in arm, however the ballots failed to correctly read the ballots in pata thus a manual count was ordered by the Comelec. After the manual count was made, Tan was proclaimed as governor-elect and Loong was 3rd and Jakiri was 2nd. Loong and Jakiri questioned the results and claimed denial of due process and the illegality of the manual count in light of RA 8346. They seeked special re-elections. Issue:w/n a re-election may be held Ruling: NO. it cannot be held. Under the Omnibus code a special election can only be ordered upon special circumstances such as force majeure, terrorism, fraud and the like which only the COMELEC can decide on. In the case at bar the people of Sulu were able to cast their votes fairly and freely. A special election would also violate the equal protection of Tan. The records show that all the elected officials were now discharging their duties and that they were proclaimed on the same basis of the manually counted votes. If manual counting is illegal then those who were discharging their duties cannot hold office.

Intemational School v. Quisumbing 333 SCRA 13 FACTS: International School Alliance of Educators (the School) hires both foreign and local teachers as members of its faculty, classifying the same into two: (1) foreign-hires and (2) local-hires. In which, the School grants foreign-hires certain benefits not accorded local-hires including housing, transportation, shipping costs, taxes, home leave travel allowance and a salary rate 25% more than local hires based on “significant economic disadvantages” The labor union and the collective bargaining representative of all faculty members of the School, contested the difference in salary rates between foreign and local-hires. The Union claims that the point-of-hire classification employed by the School is discriminatory to Filipinos and that the grant of higher salaries to foreign-hires constitutes racial discrimination. ISSUE: Whether or not the Union can invoke the equal protection clause to justify its claim of parity. RULING: Yes. The Labor Code’s and the Constitution’s provisions impregnably institutionalize in this jurisdiction the long honored legal truism of "equal pay for equal work." Persons who work with substantially equal qualifications, skill, effort and responsibility, under similar conditions, should be paid similar salaries. If an employer accords employees the same position and rank, the presumption is that these employees perform equal work. If the employer pays one employee less than the rest, it is not for that employee to explain why he receives less or why the others receive more. That would be adding insult to injury. The employer in this case has failed to discharge this burden. There is no evidence here that foreign-hires perform 25% more efficiently or effectively than the local-hires. Both groups have similar functions and responsibilities, which they perform under similar working conditions. Hence, the Court finds the point-of-hire classification employed by respondent School to justify the distinction in the salary rates of foreign-hires and local hires to be an invalid classification. There is no reasonable distinction between the services rendered by foreign-hires and local-hires. De Guzman v COMELEC, GR 129113,. July 19 2000 FACTS: This is a petition for certiorari and prohibition with urgent prayer for the issuance of a writ of preliminary injunction and temporary restraining order, assailing the validity of Section 44 of Republic Act No. 8189 (RA 8189) otherwise known as "The Voters Registration Act of 1996". SEC. 44. Reassignment of Election Officers. - No Election Officer shall hold office in a particular city or municipality for more than four (4) years. Any election officer who, either at the time of the approval of this Act or subsequent thereto, has served for at least four (4) years in a particular city or municipality shall automatically be reassigned by the Commission to a new station outside the original congressional district. Petitioners, who are either City or Municipal Election Officers, were reassigned to different stations by the COMELEC. Petitioners contend that the said law is unconstitutional because it violates the equal protection clause guaranteed by the 1987 Constitution because it singles out the City and Municipal Election Officers of the COMELEC as prohibited from holding office in the same city or municipality for more than four (4) years. They maintain that there is no substantial distinction between them and other COMELEC officials, and therefore, there is no valid classification to justify the objective of the provision of law under attack.

ISSUE: Whether or not Section 44 of RA 8189 violates the equal protection clause. HELD:No. The singling out of election officers in order to "ensure the impartiality of election officials by preventing them from developing familiarity with the people of their place of assignment" does not violate the equal protection clause of the Constitution. Lutz vs. Araneta: "the legislature is not required by the Constitution to adhere to a policy of all or none". This is so for underinclusiveness is not an argument against a valid classification. It may be true that all the other officers of COMELEC referred to by petitioners are exposed to the same evils sought to be addressed by the statute. However, in this case, it can be discerned that the legislature thought the noble purpose of the law would be sufficiently served by breaking an important link in the chain of corruption than by breaking up each and every link thereof. Verily, under Section 3(n) of RA 8189, election officers are the highest officials or authorized representatives of the COMELEC in a city or municipality. It is safe to say that without the complicity of such officials, large-scale anomalies in the registration of voters can hardly be carried out. The petition is dismissed and upheld the constitutionality of Section 44 of RA 8189. Dimaporo v. HRET , GR 158359, Mar. 23, 2004 DIMAPORO V. HRETFACTS: This is a petition brought by Congressman Dimaporo seeking tonullify the twin Resolutions of the HRET which denied his Motion for Technical Evaluation of the Thumbmarks and Signatures Affixed in theVoters Registration Records and Motion for Reconsideration of ResolutionDenying the Motion for Technical Examination of Voting Records.Pursuant to the 1998 HRET Rules Congressionalcandidate Mangotara Petition of Protest (Ad Cautelam) seeking thetechnical examination of the signatures and thumb the protested precinctsof the municipality of Sultan Naga Dimaporo (SND). Mangotara alleged thatthe massive substitution of voters and other electoral irregularitiesperpetrated by Dimaporo’s supporters will be uncovered and proven. Fromthis and other premises, he concluded that he is the duly-electedrepresentative of the 2ndDistrict of Lanao del Norte.Noting that “the Tribunal cannot evaluate the questionedballots because there are no ballots but only election documents toconsider” HRET granted Mangotara's motion and permitted the latter toengage an expert to assist him in prosecution of the case, NBI conductedthe technical examination. ISSUE: 1. W/N Dimaporo was deprived by HRET of Equal Protection whenthe latter denied his motion for technicalexamination.2. W/N Dimaporo was deprived of procedural due processor the right to present scientific evidence to show themassive substitute voting committed in counter protested precincts. RULING: 1. Resolution of HRET did not offend equal protectionclause. Equal protection simply means that all persons and thingssimilarly situated must be treated alike both as to the rights conferredand the liabilities imposed. It follows that the existence of a valid andsubstantial distinction justifies divergent treatment. According to Dimaporo since the ballot boxes subject of hispetition and that of Mangotara were both unavailable for revision, hismotion,

like Mangotara’s, should be granted.The argument fails to take into account the distinctionsextant in Mangotara’s protest vis-à-vis Dimaporo’s counter-protest whichvalidate the grant of Mangotara’s motion and the denial of Dimaporo’s. First. The election results in SND were the sole subjects of Mangotara’s protest. The opposite is true with regard to Dimaporo’scounter-protest as he contested the election results in all municipalities butSND.Significantly, the results of the technical examination of the election recordsof SND are determinative of the final outcome of the election protestagainst Dimaporo. The same cannot be said of the precincts subject of Dimaporo’s motion. It should be emphasized that the grant of a motion for technical examination is subject to the sound discretion of the HRET.In this case, the Tribunal deemed it useful in the conduct of therevision proceedings to grant Mangotara’s motion for technicalexamination. Conversely, it found Dimaporo’s motion unpersuasiveand accordingly denied the same. In so doing, the HRET merely actedwithin the bounds of its Constitutionally-granted jurisdiction. After all,the Constitution confers full authority on the electoral tribunals of theHouse of Representatives and the Senate as the sole judges of allcontests relating to the election, returns, and qualifications of their respective members. Such jurisdiction is original and exclusive.

from further implementing the last proviso in Sec. 15, Art. II of the New Central Bank Act (i.e., the exemption from the Salary Standardization Law (SSL) of all employees with salary grade of 19 and the non-exemption of those having a salary grade under 19). They alleged its constitutionality for being an invalid “class legislation”. Petitioner’s Contentions: The said proviso violates equal protection clause because only the officers of the BSP (those holding the salary grade of 19 and up) are exempted from the SSL. Those belonging from 19 and up and those 19 below do not really differ from one other in terms of the nature of work and expertise. Other GFIs, which are the same as the BSP, exempt all their rankand-file personnel from SSL without any distinction. BSP’s contention: The proviso is not unconstitutional as it can stand the constitutional test, provided it is construed in harmony with other provisions of the same law, such as the mandate of the Monetary Board to “establish professionalism and excellence at all levels in accordance with sound principles of management.” Solicitor General, on behalf of respondent Executive Secretary: The proviso is not unconstitutional as the classification is based on actual and real differentiation, even as it adheres to the enunciated policy of the new SB Act to establish professionalism and excellence within the BSP subject to prevailing laws and policies of the national government.

2. Anent Dimaporo’s contention that the assailed Resolutions denied him the right to procedural due process and to present evidence tosubstantiate his claim of massive substitute voting committed in thecounter-protested precincts, suffice it to state that the HRET itself mayascertain the validity of Dimaporo’s allegations without resort to technicalexamination. To this end, the Tribunal declared that the ballots, electiondocuments and other election paraphernalia are still subject to its scrutinyin the appreciation of evidence.It should be noted that the records are replete with evidence,documentary and testimonial, presented by Dimaporo. Dimaporo’sallegation of denial of due process is an indefensible pretense.The instant petition is DISMISSED for lack of merit Central Bank Employees vs Bangko Sentral, G.R. No. 148208, December 15 2004 Doctrines: Elements of valid class legislation: (1) must rest on substantial distinctions; (2) must be germane to the purposes of the law; (3) must not be limited to existing conditions only; (4) must apply equally to all members of the same class Relative Constitutionality. The fact that a statute is constitutional at first does not mean it is constitutional forever. The subsequent changes in the original circumstance surrounding the law would affect its validity.

Facts: The new Central Bank Act took effect and gave way for the creation of Bangko Sentral ng Pilipinas. Other Governmental Financial Institutions (GFIs) also amended their charters. After almost 8 years following the amendment of the GFIs’ charters, BSP’s employees, through petitioner, filed a petition for prohibition against the BSP and the Executive Secretary to restrain the respondents

Issue: WON the proviso is unconstitutional for being violative of equal protection clause. Held: YES, the proviso is unconstitutional for being violative of the equal protection clause. Equal protection clause does not prevent the Legislature from establishing classes of individuals or objects upon which different rules shall operate – so long as the classification is not unreasonable. Equality of operation of statutes does not mean indiscriminate operation on persons themselves, but on persons according to the circumstances surrounding them. It guarantees equality, not identity of rights. In the case at bar, it is clear in the legislative deliberations that the exemption of officers (SG 20 and above) from the SSL was intended to address the BSP’s lack of competitiveness in terms of attracting competent officers and executives. It was not intended to discriminate against the rank-and-file and the resulting discrimination or distinction has a rational basis and is not palpably, purely, and entirely arbitrary in the legislative sense. However, in the subsequent passages of the amendment on the charters of other GFI, the surrounding circumstances of the case changed. The subsequent amendments of the other GFIs’ charter (i.e., express authorization to determine and institute its own compensation and wage structure, and explicit exemption – without distinction as to salary grade or position – all employees of the GFI from the SSL) resulted to the oppressive results of Congress’ inconsistent and unequal policy towards the BSP rank-and-file and those of the seven other GFI. In the case at bar, it is precisely the fact that as regards the exemption from the SSL, there are no characteristics peculiar only to the seven GFIs or their rank-and-file so as to justify the exemption which BSP rank-and-file employees were denied (not to mention the anomaly of the SEC getting one). The distinction made by the law is not only superficial, but also arbitrary. It is not based on substantial distinctions that make real differences between the BSP rank-and-file and the seven other GFIs.

The subsequent grant to the rank-and-file of the seven other GFIs and continued denial to the BSP rank-and-file employees of the exemption from SSL breached the latter’s right to equal protection.

petitioner’s “[b]asic salary, US$1,400.00/month + US$700.00/month, fixed overtime pay, + US$490.00/month, vacation leave pay = US$2,590.00/compensation per month.”

The equal protection clause does not demand absolute equality but it requires that all persons shall be treated alike, under like circumstances and conditions both as to privileges conferred and liabilities enforced.

Respondents appealed to the National Labor Relations Commission (NLRC) to question the finding of the LA that petitioner was illegally dismissed.

Serrano vs. Gallant Maritime Service, G.R. No. 167614, March 24 2009 FACTS: Petitioner was hired by Gallant Maritime Services, Inc. and Marlow Navigation Co., Ltd. (respondents) under a Philippine Overseas Employment Administration (POEA)-approved Contract of Employment with the following terms and conditions: Duration of contract 12 months Position Chief Officer Basic monthly salary US$1,400.00 Hours of work 48.0 hours per week Overtime US$700.00 per month

The NLRC modified the LA Decision and corrected the LA’s computation of the lump-sum salary awarded to petitioner by reducing the applicable salary rate from US$2,590.00 to US$1,400.00 because R.A. No. 8042 “does not provide for the award of overtime pay, which should be proven to have been actually performed, and for vacation leave pay. Petitioner filed a Motion for Partial Reconsideration, but this time he questioned the constitutionality of the subject clause. The NLRC denied the motion. Petitioner filed a Petition for Certiorari with the CA, reiterating the constitutional challenge against the subject clause. After initially dismissing the petition on a technicality, the CA eventually gave due course to it, as directed by this Court in its Resolution which granted the petition for certiorari,filed by petitioner. The CA affirmed the NLRC ruling on the reduction of the applicable salary rate; however, the CA skirted the constitutional issue raised by petitioner.

Vacation leave with pay 7.00 days per month On March 19, 1998, the date of his departure, petitioner was constrained to accept a downgraded employment contract for the position of Second Officer with a monthly salary of US$1,000.00, upon the assurance and representation of respondents that he would be made Chief Officer by the end of April 1998. Respondents did not deliver on their promise to make petitioner Chief Officer. Hence, petitioner refused to stay on as Second Officer and was repatriated to the Philippines on May 26, 1998. Petitioner’s employment contract was for a period of 12 months or from March 19, 1998 up to March 19, 1999, but at the time of his repatriation on May 26, 1998, he had served only two (2) months and seven (7) days of his contract, leaving an unexpired portion of nine (9) months and twenty-three (23) days. Petitioner filed with the Labor Arbiter (LA) a Complaint against respondents for constructive dismissal and for payment of his money claims in the total amount of US$26,442.73. The LA rendered a Decision dated July 15, 1999, declaring the dismissal of petitioner illegal and awarding him monetary benefits, to wit: WHEREFORE, premises considered, judgment is hereby rendered declaring that the dismissal of the complainant (petitioner) by the respondents in the above-entitled case was illegal and the respondents are hereby ordered to pay the complainant [petitioner], jointly and severally, in Philippine Currency, based on the rate of exchange prevailing at the time of payment, the amount of EIGHT THOUSAND SEVEN HUNDRED SEVENTY U.S. DOLLARS (US $8,770.00), representing the complainant’s salary for three (3) months of the unexpired portion of the aforesaid contract of employment. The claims of the complainant for moral and exemplary damages are hereby DISMISSED for lack of merit. In awarding petitioner a lump-sum salary of US$8,770.00, the LA based his computation on the salary period of three months only — rather than the entire unexpired portion of nine months and 23 days of petitioner’s employment contract – applying the subject clause. However, the LA applied the salary rate of US$2,590.00, consisting of

His Motion for Reconsideration having been denied by the CA, petitioner brings his cause to this Court on the following grounds: The Court of Appeals and the labor tribunals have decided the case in a way not in accord with applicable decision of the Supreme Court involving similar issue of granting unto the migrant worker back wages equal to the unexpired portion of his contract of employment instead of limiting it to three (3) months. Even without considering the constitutional limitations [of] Sec. 10 of Republic Act No. 8042, the Court of Appeals gravely erred in law in excluding from petitioner’s award the overtime pay and vacation pay provided in his contract since under the contract they form part of his salary.

The Court now takes up the full merit of the petition mindful of the extreme importance of the constitutional question raised therein.

ISSUES: Whether Section 10 (par 5) of RA 8042 is unconstitutional Proper computation of the Lump-sum salary to be awarded to petitioner by reason of his illegal dismissal Whether the overtime and leave pay should form part of the salary basis in the computation of his monetary award The unanimous finding of the LA, NLRC and CA that the dismissal of petitioner was illegal is not disputed. Likewise not disputed is the salary differential of US$45.00 awarded to petitioner in all three fora. Applying the subject clause, the NLRC and the CA computed the lump-sum salary of petitioner at the monthly rate of US$1,400.00 covering the period of three months out of the unexpired portion of nine months and 23 days of his employment contract or a total of US$4,200.00. Impugning the constitutionality of the subject clause, petitioner contends that, in addition to the US$4,200.00 awarded by the NLRC

and the CA, he is entitled to US$21,182.23 more or a total of US$25,382.23, equivalent to his salaries for the entire nine months and 23 days left of his employment contract, computed at the monthly rate of US$2,590.00.31 Arguments of the Petitioner For Antonio Serrano (petitioner), a Filipino seafarer, the last clause in the 5th paragraph of Section 10, Republic Act (R.A.) No. 8042, violates the OFWs’ constitutional rights in that it impairs the terms of their contract, deprives them of equal protection and denies them due process. The Arguments of Respondents Respondents contend that the constitutional issue should not be entertained, for this was belatedly interposed by petitioner in his appeal before the CA, and not at the earliest opportunity, which was when he filed an appeal before the NLRC.40 The Arguments of the Solicitor General The Solicitor General (OSG)41 points out that as R.A. No. 8042 took effect on July 15, 1995, its provisions could not have impaired petitioner’s 1998 employment contract. Rather, R.A. No. 8042 having preceded petitioner’s contract, the provisions thereof are deemed part of the minimum terms of petitioner’s employment, especially on the matter of money claims, as this was not stipulated upon by the parties. The Court’s Ruling: First Issue Does the subject clause violate Section 1, Article III of the Constitution, and Section 18, Article II and Section 3, Article XIII on Labor as protected sector? The answer is in the affirmative. Section 1, Article III of the Constitution guarantees: No person shall be deprived of life, liberty, or property without due process of law nor shall any person be denied the equal protection of the law. Section 18, Article II and Section 3, Article XIII accord all members of the labor sector, without distinction as to place of deployment, full protection of their rights and welfare. To Filipino workers, the rights guaranteed under the foregoing constitutional provisions translate to economic security and parity: all monetary benefits should be equally enjoyed by workers of similar category, while all monetary obligations should be borne by them in equal degree; none should be denied the protection of the laws which is enjoyed by, or spared the burden imposed on, others in like circumstances. Imbued with the same sense of “obligation to afford protection to labor,” the Court in the present case also employs the standard of strict judicial scrutiny, for it perceives in the subject clause a suspect classification prejudicial to OFWs. Upon cursory reading, the subject clause appears facially neutral, for it applies to all OFWs. However, a closer examination reveals that the subject clause has a discriminatory intent against, and an invidious impact on OFWs

Second Issue It is plain that prior to R.A. No. 8042, all OFWs, regardless of contract periods or the unexpired portions thereof, were treated alike in terms of the computation of their monetary benefits in case of illegal dismissal. Their claims were subjected to a uniform rule of computation: their basic salaries multiplied by the entire unexpired portion of their employment contracts. The enactment of the subject clause in R.A. No. 8042 introduced a differentiated rule of computation of the money claims of illegally dismissed OFWs based on their employment periods, in the process singling out one category whose contracts have an unexpired portion of one year or more and subjecting them to the peculiar disadvantage of having their monetary awards limited to their salaries for 3 months or for the unexpired portion thereof, whichever is less, but all the while sparing the other category from such prejudice, simply because the latter’s unexpired contracts fall short of one year. Prior to R.A. No. 8042, a uniform system of computation of the monetary awards of illegally dismissed OFWs was in place. This uniform system was applicable even to local workers with fixed-term employment. The subject clause does not state or imply any definitive governmental purpose; and it is for that precise reason that the clause violates not just petitioner’s right to equal protection, but also her right to substantive due process under Section 1, Article III of the Constitution. The subject clause being unconstitutional, petitioner is entitled to his salaries for the entire unexpired period of nine months and 23 days of his employment contract, pursuant to law and jurisprudence prior to the enactment of R.A. No. 8042. Third Issue Petitioner contends that his overtime and leave pay should form part of the salary basis in the computation of his monetary award, because these are fixed benefits that have been stipulated into his contract. Petitioner is mistaken. The word salaries in Section 10(5) does not include overtime and leave pay. For seafarers like petitioner, DOLE Department Order No. 33, series 1996, provides a Standard Employment Contract of Seafarers, in which salary is understood as the basic wage, exclusive of overtime, leave pay and other bonuses; whereas overtime pay is compensation for all work “performed” in excess of the regular eight hours, and holiday pay is compensation for any work “performed” on designated rest days and holidays. In the same vein, the claim for the day’s leave pay for the unexpired portion of the contract is unwarranted since the same is given during the actual service of the seamen. WHEREFORE, the Court GRANTS the Petition. The subject clause “or for three months for every year of the unexpired term, whichever is less” in the 5th paragraph of Section 10 of Republic Act No. 8042 is DECLARED UNCONSTITUTIONAL; and the December 8, 2004 Decision and April 1, 2005 Resolution of the Court of Appeals are MODIFIED to the effect that petitioner is AWARDED his salaries for the entire unexpired portion of his employment contract consisting of nine months and 23 days computed at the rate of US$1,400.00 per month. Dlmayuga vs OMB 495 SCRA 461

The subject clause does not state or imply any definitive governmental purpose; and it is for that precise reason that the clause violates not just petitioner’s right to equal protection, but also her right to substantive due process under Section 1, Article III of the Constitution.

Disini vs. Secretary of Justice, G.R.No. 203335, February 18 2014 Facts: Petitioners assail the validity of several provision of the Republic Act (R.A.) 10175, the Cybercrime Prevention Act of 2012.

Petitioners claim that the means adopted by the cybercrime law for regulating undesirable cyberspace activities violate certain of their constitutional rights. The government of course asserts that the law merely seeks to reasonably put order into cyberspace activities, punish wrongdoings, and prevent hurtful attacks on the system. Held: Section 4(a)(6) of the Cybercrime Law Section 4(a)(6) provides: (6) Cyber-squatting. The acquisition of domain name over the internet in bad faith to profit, mislead, destroy the reputation, and deprive others from registering the same, if such a domain name is: (i) Similar, identical, or confusingly similar to an existing trademark registered with the appropriate government agency at the time of the domain name registration; (ii) Identical or in any way similar with the name of a person other than the registrant, in case of a personal name; and (iii) Acquired without right or with intellectual property interests in it. Petitioners claim that Section 4(a)(6) or cyber-squatting violates the equal protection clausein that, not being narrowly tailored, it will cause a user using his real name to suffer the same fate as those who use aliases or take the name of another in satire, parody, or any other literary device. The law is reasonable in penalizing the offender for acquiring the domain name in bad faith to profit, mislead, destroy reputation, or deprive others who are not ill-motivated of the rightful opportunity of registering the same.1 Hence, valid and constitutional. Biraogo vs. Philippine Truth Commission, G.R. No. 193036, December 7 2010 FACTS: Pres. Aquino signed E. O. No. 1 establishing Philippine Truth Commission of 2010 (PTC) dated July 30, 2010. PTC is a mere ad hoc body formed under the Office of the President with the primary task to investigate reports of graft and corruption committed by third-level public officers and employees, their coprincipals, accomplices and accessories during the previous administration, and to submit its finding and recommendations to the President, Congress and the Ombudsman. PTC has all the powers of an investigative body. But it is not a quasi-judicial body as it cannot adjudicate, arbitrate, resolve, settle, or render awards in disputes between contending parties. All it can do is gather, collect and assess evidence of graft and corruption and make recommendations. It may have subpoena powers but it has no power to cite people in contempt, much less order their arrest. Although it is a fact-finding body, it cannot determine from such facts if probable cause exists as to warrant the filing of an information in our courts of law. Petitioners asked the Court to declare it unconstitutional and to enjoin the PTC from performing its functions. They argued that: (a) E.O. No. 1 violates separation of powers as it arrogates the power of the Congress to create a public office and appropriate funds for its operation. (b) The provision of Book III, Chapter 10, Section 31 of the Administrative Code of 1987 cannot legitimize E.O. No. 1 because the delegated authority of the President to structurally reorganize the Office of the President to achieve economy, simplicity and efficiency does not include the power to create an entirely new public office which was hitherto inexistent like the “Truth Commission.” (c) E.O. No. 1 illegally amended the Constitution and statutes when it vested the “Truth Commission” with quasi-judicial powers

duplicating, if not superseding, those of the Office of the Ombudsman created under the 1987 Constitution and the DOJ created under the Administrative Code of 1987. (d) E.O. No. 1 violates the equal protection clause as it selectively targets for investigation and prosecution officials and personnel of the previous administration as if corruption is their peculiar species even as it excludes those of the other administrations, past and present, who may be indictable. Respondents, through OSG, questioned the legal standing of petitioners and argued that: 1] E.O. No. 1 does not arrogate the powers of Congress because the President’s executive power and power of control necessarily include the inherent power to conduct investigations to ensure that laws are faithfully executed and that, in any event, the Constitution, Revised Administrative Code of 1987, PD No. 141616 (as amended), R.A. No. 9970 and settled jurisprudence, authorize the President to create or form such bodies. 2] E.O. No. 1 does not usurp the power of Congress to appropriate funds because there is no appropriation but a mere allocation of funds already appropriated by Congress. 3] The Truth Commission does not duplicate or supersede the functions of the Ombudsman and the DOJ, because it is a fact-finding body and not a quasi-judicial body and its functions do not duplicate, supplant or erode the latter’s jurisdiction. 4] The Truth Commission does not violate the equal protection clause because it was validly created for laudable purposes. ISSUES:1. WON the petitioners have legal standing to file the petitions and question E. O. No. 1; 2. WON E. O. No. 1 violates the principle of separation of powers by usurping the powers of Congress to create and to appropriate funds for public offices, agencies and commissions; 3. WON E. O. No. 1 supplants the powers of the Ombudsman and the DOJ; 4. WON E. O. No. 1 violates the equal protection clause. RULING:The power of judicial review is subject to limitations, to wit: (1) there must be an actual case or controversy calling for the exercise of judicial power; (2) the person challenging the act must have the standing to question the validity of the subject act or issuance; otherwise stated, he must have a personal and substantial interest in the case such that he has sustained, or will sustain, direct injury as a result of its enforcement; (3) the question of constitutionality must be raised at the earliest opportunity; and (4) the issue of constitutionality must be the very lis mota of the case. 1. The petition primarily invokes usurpation of the power of the Congress as a body to which they belong as members. To the extent the powers of Congress are impaired, so is the power of each member thereof, since his office confers a right to participate in the exercise of the powers of that institution. Legislators have a legal standing to see to it that the prerogative, powers and privileges vested by the Constitution in their office remain inviolate. Thus, they are allowed to question the validity of any official action which, to their mind, infringes on their prerogatives as legislators. With regard to Biraogo, he has not shown that he sustained, or is in danger of sustaining, any personal and direct injury attributable to the implementation of E. O. No. 1.

Locus standi is “a right of appearance in a court of justice on a given question.” In private suits, standing is governed by the “real-parties-in interest” rule. It provides that “every action must be prosecuted or defended in the name of the real party in interest.” Real-party-in interest is “the party who stands to be benefited or injured by the judgment in the suit or the party entitled to the avails of the suit.” Difficulty of determining locus standi arises in public suits. Here, the plaintiff who asserts a “public right” in assailing an allegedly illegal official action, does so as a representative of the general public. He has to show that he is entitled to seek judicial protection. He has to make out a sufficient interest in the vindication of the public order and the securing of relief as a “citizen” or “taxpayer. The person who impugns the validity of a statute must have “a personal and substantial interest in the case such that he has sustained, or will sustain direct injury as a result.” The Court, however, finds reason in Biraogo’s assertion that the petition covers matters of transcendental importance to justify the exercise of jurisdiction by the Court. There are constitutional issues in the petition which deserve the attention of this Court in view of their seriousness, novelty and weight as precedents The Executive is given much leeway in ensuring that our laws are faithfully executed. The powers of the President are not limited to those specific powers under the Constitution. One of the recognized powers of the President granted pursuant to this constitutionally-mandated duty is the power to create ad hoc committees. This flows from the obvious need to ascertain facts and determine if laws have been faithfully executed. The purpose of allowing ad hoc investigating bodies to exist is to allow an inquiry into matters which the President is entitled to know so that he can be properly advised and guided in the performance of his duties relative to the execution and enforcement of the laws of the land. 2. There will be no appropriation but only an allotment or allocations of existing funds already appropriated. There is no usurpation on the part of the Executive of the power of Congress to appropriate funds. There is no need to specify the amount to be earmarked for the operation of the commission because, whatever funds the Congress has provided for the Office of the President will be the very source of the funds for the commission. The amount that would be allocated to the PTC shall be subject to existing auditing rules and regulations so there is no impropriety in the funding. 3. PTC will not supplant the Ombudsman or the DOJ or erode their respective powers. If at all, the investigative function of the commission will complement those of the two offices. The function of determining probable cause for the filing of the appropriate complaints before the courts remains to be with the DOJ and the Ombudsman. PTC’s power to investigate is limited to obtaining facts so that it can advise and guide the President in the performance of his duties relative to the execution and enforcement of the laws of the land. 4. Court finds difficulty in upholding the constitutionality of Executive Order No. 1 in view of its apparent transgression of the equal protection clause enshrined in Section 1, Article III (Bill of Rights) of the 1987 Constitution. Equal protection requires that all persons or things similarly situated should be treated alike, both as to rights conferred and responsibilities imposed. It requires public bodies and institutions to treat similarly situated individuals in a similar manner. The purpose of the equal protection clause is to secure every person within a state’s jurisdiction against intentional and arbitrary discrimination, whether occasioned by the express terms of a statue or by its improper execution through the state’s duly constituted authorities. There must be equality among equals as determined according to a valid classification. Equal protection clause permits classification. Such classification, however, to be valid must pass the test of

reasonableness. The test has four requisites: (1) The classification rests on substantial distinctions; (2) It is germane to the purpose of the law; (3) It is not limited to existing conditions only; and (4) It applies equally to all members of the same class. The classification will be regarded as invalid if all the members of the class are not similarly treated, both as to rights conferred and obligations imposed. Executive Order No. 1 should be struck down as violative of the equal protection clause. The clear mandate of truth commission is to investigate and find out the truth concerning the reported cases of graft and corruption during the previous administration only. The intent to single out the previous administration is plain, patent and manifest. Arroyo administration is but just a member of a class, that is, a class of past administrations. It is not a class of its own. Not to include past administrations similarly situated constitutes arbitrariness which the equal protection clause cannot sanction. Such discriminating differentiation clearly reverberates to label the commission as a vehicle for vindictiveness and selective retribution. Superficial differences do not make for a valid classification. The PTC must not exclude the other past administrations. The PTC must, at least, have the authority to investigate all past administrations. The Constitution is the fundamental and paramount law of the nation to which all other laws must conform and in accordance with which all private rights determined and all public authority administered. Laws that do not conform to the Constitution should be stricken down for being unconstitutional. Garcia vs. Drilon, G.R. No. 179267, June 25 2013 Facts: Private respondent Rosalie filed a petition before the RTC of Bacolod City a Temporary Protection Order against her husband, Jesus, pursuant to R.A. 9262, entitled “An Act Defining Violence Against Women and Their Children, Providing for Protective Measures for Victims, Prescribing Penalties Therefor, and for Other Purposes.” She claimed to be a victim of physical, emotional, psychological and economic violence, being threatened of deprivation of custody of her children and of financial support and also a victim of marital infidelity on the part of petitioner. The TPO was granted but the petitioner failed to faithfully comply with the conditions set forth by the said TPO, private-respondent filed another application for the issuance of a TPO ex parte. The trial court issued a modified TPO and extended the same when petitioner failed to comment on why the TPO should not be modified. After the given time allowance to answer, the petitioner no longer submitted the required comment as it would be an “axercise in futility.” Petitioner filed before the CA a petition for prohibition with prayer for injunction and TRO on, questioning the constitutionality of the RA 9262 for violating the due process and equal protection clauses, and the validity of the modified TPO for being “an unwanted product of an invalid law.” The CA issued a TRO on the enforcement of the TPO but however, denied the petition for failure to raise the issue of constitutionality in his pleadings before the trial court and the petition for prohibition to annul protection orders issued by the trial court constituted collateral attack on said law. Petitioner filed a motion for reconsideration but was denied. Thus, this petition is filed. Issues: WON the CA erred in dismissing the petition on the theory that the issue of constitutionality was not raised at the earliest opportunity and that the petition constitutes a collateral attack on the validity of the law.

SECTION 2 - SEARCHES AND SEIZURES WON the CA committed serious error in failing to conclude that RA 9262 is discriminatory, unjust and violative of the equal protection clause.

Section 2 - Searches and Seizures

WON the CA committed grave mistake in not finding that RA 9262 runs counter to the due process clause of the Constitution

Facts: On 20 January 1987, the National Capital Region District Command (NCRDC) was activated pursuant to Letter of Instruction 02/87 of the Philippine General Headquarters, AFP, with the mission of conducting security operations within its area of responsibility and peripheral areas, for the purpose of establishing an effective territorial defense, maintaining peace and order, and providing an atmosphere conducive to the social, economic and political development of the National Capital Region. As part of its duty to maintain peace and order, the NCRDC installed checkpoints in various parts of Valenzuela, Metro Manila.

WON the CA erred in not finding that the law does violence to the policy of the state to protect the family as a basic social institution WON the CA seriously erredin declaring RA 9262 as invalid and unconstitutional because it allows an undue delegation of judicial power to Brgy. Officials. Decision: 1. Petitioner contends that the RTC has limited authority and jurisdiction, inadequate to tackle the complex issue of constitutionality. Family Courts have authority and jurisdiction to consider the constitutionality of a statute. The question of constitutionality must be raised at the earliest possible time so that if not raised in the pleadings, it may not be raised in the trial and if not raised in the trial court, it may not be considered in appeal. 2. RA 9262 does not violate the guaranty of equal protection of the laws. Equal protection simply requires that all persons or things similarly situated should be treated alike, both as to rights conferred and responsibilities imposed. In Victoriano v. Elizalde Rope Workerkers’ Union, the Court ruled that all that is required of a valid classification is that it be reasonable, which means that the classification should be based on substantial distinctions which make for real differences; that it must be germane to the purpose of the law; not limited to existing conditions only; and apply equally to each member of the class. Therefore, RA9262 is based on a valid classification and did not violate the equal protection clause by favouring women over men as victims of violence and abuse to whom the Senate extends its protection. 3. RA 9262 is not violative of the due process clause of the Constitution. The essence of due process is in the reasonable opportunity to be heard and submit any evidence one may have in support of one’s defense. The grant of the TPO exparte cannot be impugned as violative of the right to due process.

4. The non-referral of a VAWC case to a mediator is justified. Petitioner’s contention that by not allowing mediation, the law violated the policy of the State to protect and strengthen the family as a basic autonomous social institution cannot be sustained. In a memorandum of the Court, it ruled that the court shall not refer the case or any issue therof to a mediator. This is so because violence is not a subject for compromise.

5. There is no undue delegation of judicial power to Barangay officials. Judicial power includes the duty of the courts of justice to settle actual controversies involving rights which are legally demandable and enforceable and to determine whether or not there has been a grave abuse of discretion amounting to lack or excess of jurisdiction on any part of any branch of the Government while executive power is the power to enforce and administer the laws. The preliminary investigation conducted by the prosecutor is an executive, not a judicial, function. The same holds true with the issuance of BPO. Assistance by Brgy. Officials and other law enforcement agencies is consistent with their duty executive function.

Valmonte v. General de Villa, 178 SCRA 211 and 185 SCRA 665

Petitioners aver that because of the institution of said checkpoints, the Velanzuela residents are worried of being harassed and of their safety being placed at the arbitrary, capricious and whimsical disposition of the military manning the checkpoints considering that their cars and vehicles are being subjected to regular searches and check-ups especially at night or at dawn without a search warrant or a court order. Their alleged fear for their safety increased when Benjamin Parpon was gunned down allegedly in cold blood by members of the NCRDC for ignoring and/or continuing to speed off inspite of warning shots fired in the air. Petitioners Atty. Ricardo Valmonte, who is a resident of Valenzuela, Metro Manila, and the Union of Lawyers and Advocates For People’s Rights (ULAP) sought the declaration of checkpoints in Valenzuela, Metro Manila and elsewhere as unconstitutional. In the alternative, they prayed that respondents Renato De Villa and the National Capital Region District Command (NCRDC) be directed to formulate guidelines in the implementation of checkpoints for the protection of the people. Petitioners contended that the checkpoints gave the respondents blanket authority to make searches and seizures without search warrant or court order in violation of the Constitution. Held: Not all searches and seizures are prohibited. Those which are reasonable are not forbidden. A reasonable search is not to be determined by any fixed formula but is to be resolved according to the facts of each case. Where, for example, the officer merely draws aside the curtain of a vacant vehicle which is parked on the public fair grounds, or simply looks into a vehicle, or flashes a light therein, these do not constitute unreasonable search. The setting up of the questioned checkpoints in Valenzuela (and probably in other areas) may be considered as a security measure to enable the NCRDC to pursue its mission of establishing effective territorial defense and maintaining peace and order for the benefit of the public. Checkpoints may also be regarded as measures to thwart plots to destabilize the government, in the interest of public security. In this connection, the Court may take judicial notice of the shift to urban centers and their suburbs of the insurgency movement, so clearly reflected in the increased killings in cities of police and military men by NPA “sparrow units,” not to mention the abundance of unlicensed firearms and the alarming rise in lawlessness and violence in such urban centers, not all of which are reported in media, most likely brought about by deteriorating economic conditions – which all sum up to what one can rightly consider, at the very least, as abnormal times. Between the inherent right of the state to protect its existence and promote public welfare and an individual's right against a warrantless search which is however reasonably conducted, the former should prevail. True, the manning of checkpoints by the military is susceptible of abuse by the men in uniform, in the same manner that all governmental power is susceptible of abuse. But, at the cost of occasional

inconvenience, discomfort and even irritation to the citizen, the checkpoints during these abnormal times, when conducted within reasonable limits, are part of the price we pay for an orderly society and a peaceful community. Guazon v. De Villa, 181 SCRA 623 FACTS: This is a petition for prohibition with preliminary injunction to prohibit the military and police officers represented by public respondents from conducting "Areal Target Zonings" or "Saturation Drives" in Metro Manila. The 41 petitioners alleged that the "saturation drive" or "areal target zoning" that were conducted in their place (Tondo Manila) were unconstitutional. The alleged acts committed during the raid are the following: 1. Petitioners alleged that there is no specific target house to search and that there is no search warrant or warrant of arrest served. 2. Most of the policemen are in their civilian clothes and without nameplates or identification cards. 3. The residents were rudely roused from their sleep by banging on the walls and windows of their houses. 4. The residents were at the point of high-powered guns and herded like cows. 5. Men were ordered to strip down to their briefs for the police to examine their tattoo marks. 6. The residents complained that they're homes were ransacked, tossing their belongings and destroying their valuables. Some of their money and valuables had disappeared after the operation. 7. The residents also reported incidents of maulings, spot-beatings, and maltreatment. Those who were detained also suffered mental and physical torture to extract confessions and tactical information. l The respondents said that such accusations mentioned above were total lies. l Respondents contends that the Constitution grants to the government the power to seek and cripple subversive movements for the maintenance of peace in the state. l The aerial target zoning was intended to flush out subversives and criminal elements coddled by the communities were the said drives were conducted. l Respondents averred that they have intelligently and carefully planned months ahead for the actual operation and that local and foreign media joined the operation to witness and record such event. ISSUE:WON the areal target zoning and the saturation drive is legal HELD:YES. The conduct of areal target zoning or saturation drive is a valid exercise of the military powers of the President. According to the Supreme Court, everything stated before them consists only of allegations. According to petitioners, more than 3,407 persons were arrested in the saturation drives covered by the petition. No estimates are given for the drives in Block 34, Dagat-dagatan, Navotas; Apelo Cruz Compound, Pasig; and Sun Valley Drive near the Manila International Airport area. Not one of the several thousand persons treated in the illegal and inhuman manner described by the petitioners appears as a petitioner or has come before a trial court to present the kind of evidence admissible in courts of justice. Moreover, there must have been tens of thousands of nearby residents who were inconvenienced in addition to the several thousand allegedly arrested.

None of those arrested has apparently been charged and none of those affected has apparently complained. The areal target zonings in this petition were intended to flush out subversives and criminal elements particularly because of the blatant assassinations of public officers and police officials by elements supposedly coddled by the communities where the "drives" were conducted. Moreover, there is nothing in the Constitution which denies the authority of the Chief Executive, to order police actions to stop unabated criminality, rising lawlessness, and alarming communist activities. Where there is large scale mutiny or actual rebellion, the police or military may go in force to the combat areas, enter affected residences or buildings, round up suspected rebels and otherwise quell the mutiny or rebellion without having to secure search warrants and without violating the Bill of Rights. The Constitution grants the Government the power to seek and cripple subversive movements which would bring down constituted authority and substitute a regime where individual liberties are suppressed as a matter of policy in the name of security of the State. People vs. Andre Marti, 193 SCRA 57 (1991) FACTS: On 14 August 1987, appellant Andre Marti and his commonlaw wife went to the booth of the Manila Packing and Export Forwarders to send four (4) packages to a fiend in Zurich, Switzerland. When asked if the packages could be examined and inspected, appellant refused, assuring that they were simply gifts of books cigars, and gloves. The packages were then placed in a box and was sealed with masking tape for shipment. As a standard operating procedure before delivering packages to the Bureau of Customs and/or Burueau of Posts, the proprietor of the forwarding agency opened the box for final inspection. A peculiar odor emitted therefrom and he found dried leaves inside. He brought samples to NBI, and informed them that the rest of the shipment was still in his office. Agents of the NBI went to his office and found the shipment containing bricks of dried marijuana leaves, some of which were packed inside the gloves and neatly stocked underneath tabacalera cigars. Thereafter, an information was filed against the appellant in violation of RA 6425 (Dangerous Drugs Act), for which he was found guilty. Appellant assailed the decision, claiming that the evidence was obtained in violation of his constitutional rights against unreasonable search and seizure, and further, that the court erred in admitting in evidence the illegally searched and seized packages. ISSUE:May an act of a private individual, allegedly in violation of appellant’s constitutional rights be invoked against the State? HELD:No. As the Court held in several other cases, the liberties guaranteed by the Constitution cannot be invoked against the State in the absence of governmental interference. This constitutional right (against unreasonable search and seizure) refers to the immunity of one’s person, whether citizen or alien, from interference by government; and the search and seizure clauses are restraints upon the government and its agents, not upon private individuals. In the present case, it was the proprietor of the forwarding agency who made search/inspection of the packages and the contraband came into possession of the Government without the latter transgressing appellant’s rights against unreasonable search and seizure. The NBI agents made no search and seizure, much less an illegal one. Thus, the alleged act of the private individual in violation of a constitutional right cannot be invoked against the State. NB:The constitutional proscription against unlawful searches and seizures therefore applies as a restraint directed only against the government and its agencies tasked with the enforcement of the law.

Thus, it could only be invoked against the State to whom the restraint against arbitrary and unreasonable exercise of power is imposed.

that a corporation is not entitled to immunity, against unreasonable searches and seizures.

If the search is made upon the request of law enforcers, a warrant must generally be first secured if it is to pass the test of constitutionality. However, if the search is made at the behest or initiative of the proprietor of a private establishment for its own and private purposes, as in the case at bar, and without the intervention of police authorities, the right against unreasonable search and seizure cannot be invoked for only the act of private individual, not the law enforcers, is involved. In sum, the protection against unreasonable searches and seizures cannot be extended to acts committed by private individuals so as to bring it within the ambit of alleged unlawful intrusion by the government.

The petition for Certiorari, prohibition and Mandamus are granted. Search Warrant No. 2-M-70 issued by respondent Judge is declared null and void.

Bache and Co. v. Ruiz, 37 SCRA 323 (1971) Fact: Respondent Commissioner, wrote a letter to respondent Judge Ruiz requesting the issuance of a search warrant against petitioners for violation of the Sec. 46 (a) National Internal Revenue Code, in relation to all other pertinent provisions thereof, and authorizing a Revenue Examiner to make and file the application for search warrant which was attached to the letter. In the afternoon of the following day, respondent De Leon and his witness, respondent Logronio, went to the Court of First Instance of Rizal. They brought with them the following papers: respondent Vera’s aforesaid letter-request; an application for search warrant already filled up but still unsigned by respondent De Leon; an affidavit of respondent Logronio subscribed before respondent De Leon; a deposition in printed form of respondent Logronio already accomplished and signed by him but not yet subscribed; and a search warrant already accomplished but still unsigned by respondent Judge. At that time respondent Judge was hearing a certain case; so, by means of a note, he instructed his Deputy Clerk of Court to take the depositions of respondents De Leon and Logronio. After the session had adjourned, respondent Judge was informed that the depositions had already been taken. The stenographer, upon request of respondent Judge, read to him her stenographic notes; and thereafter, respondent Judge asked respondent Logronio to take the oath and warned him that if his deposition was found to be false and without legal basis, he could be charged for perjury. Respondent Judge signed respondent de Leon’s application for search warrant and respondent Logronio’s deposition, Search Warrant was then sign by respondent Judge and accordingly issued. Three days later, the BIR agents served the search warrant petitioners at the offices of petitioner corporation. Petitioners’ lawyers protested the search on the ground that no formal complaint or transcript of testimony was attached to the warrant. The agents nevertheless proceeded with their search which yielded six boxes of documents. Petitioners filed a petition with the Court of First Instance of Rizal praying that the search warrant be quashed, dissolved or recalled, that preliminary prohibitory and mandatory writs of injunction be issued, that the search warrant be declared null and void, and that the respondents be ordered to pay petitioners, jointly and severally, damages and attorney’s fees. On March 18, 1970, the respondents, thru the Solicitor General, filed an answer to the petition. After hearing, the court, presided over by respondent Judge, issued on July 29, 1970, an order dismissing the petition for dissolution of the search warrant. Hence, Petitioners came to this Court. Held: A corporation is, after all, but an association of individuals under an assumed name and with a distinct legal entity. In organizing itself as a collective body it waives no constitutional immunities appropriate to such body. Its property cannot be taken without compensation. It can only be proceeded against by due process of law, and is protected against unlawful discrimination. we are of the opinion that an officer of a corporation which is charged with a violation of a statute of the state of its creation, or of an act of Congress passed in the exercise of its constitutional powers, cannot refuse to produce the books and papers of such corporation, we do not wish to be understood as holding

RATIONALE: Personal examination by the Judge of the complainant and the witnesses is necessary to enable him to determine the existence or nonexistence of a probable cause, the determination of whether or not a probable cause exists calls for the exercise of judgment after a judicial appraisal of facts and should not be allowed to be delegated in the absence of any rule to the contrary. In this case at bar, no personal examination at all was conducted by respondent Judge of the complainant and his witnesses. The participation of respondent Judge in the proceedings which led to the issuance of Search Warrant 2-M-70 was thus limited to listening to the stenographer’s readings of her notes to a few words of warning against the commission of perjury, and to administering the oath to the complainant and his witness. This cannot be considered a personal examination. If there was an examination at all of the complainant and his witness, it was one conducted by the Deputy Clerk of Court Section 4 (Examination of the Applicant): The Judge or Justice of the peace must, before issuing the warrant, personally examine on oath or affirmation the complaint and any witnesses he may produce and take their depositions in writing, and attached them to the record, in addition to any affidavits presented to him. Stonehill v. Diokno, 20 SCRA 383 (1967) FACTS: Upon application of the officers of the government, Respondents-Judges — issued on different dates a total of 42 search warrants against petitioners herein and/or the corporations of which they were officers, directed to the any peace officer, to search the persons above-named and/or the premises of their offices, warehouses and/or residences, and to seize and take possession of the following personal property to wit: Books of accounts, financial records, vouchers, correspondence, receipts, ledgers, journals, portfolios, credit journals, typewriters, and other documents and/or papers showing all business transactions including disbursements receipts, balance sheets and profit and loss statements and Bobbins (cigarette wrappers) as "the subject of the offense; stolen or embezzled and proceeds or fruits of the offense," or "used or intended to be used as the means of committing the offense," which is described in the applications adverted to above as "violation of Central Bank Laws, Tariff and Customs Laws, Internal Revenue (Code) and the Revised Penal Code." Alleging that the aforementioned search warrants are null and void, as contravening the Constitution and the Rules of Court — because, inter alia: (1) they do not describe with particularity the documents, books and things to be seized; (2) cash money, not mentioned in the warrants, were actually seized; (3) the warrants were issued to fish evidence against the aforementioned petitioners in deportation cases filed against them; (4) the searches and seizures were made in an illegal manner; and (5) the documents, papers and cash money seized were not delivered to the courts that issued the warrants, to be disposed of in accordance with law, the said petitioners filed with the Supreme Court this original action for certiorari, prohibition, mandamus and injunction, and prayed that, pending final disposition of the present case, a writ of preliminary injunction be issued restraining Respondents-Prosecutors, their agents and /or representatives from using the effects seized as aforementioned or any copies thereof, in the deportation cases already adverted to, and that, in due course, thereafter, decision be rendered quashing the contested search warrants

and declaring the same null and void, and commanding the respondents, their agents or representatives to return to petitioners herein, in accordance with Section 3, Rule 67, of the Rules of Court, the documents, papers, things and cash moneys seized or confiscated under the search warrants in question.

lacks a sufficiently particularized description of the person or thing to be seized or the place to be searched.

In their answer, respondents-prosecutors alleged, (1) that the contested search warrants are valid and have been issued in accordance with law; (2) that the defects of said warrants, if any, were cured by petitioners' consent; and (3) that, in any event, the effects seized are admissible in evidence against herein petitioners, regardless of the alleged illegality of the aforementioned searches and seizures.

● The provisions of the constitution require that there be not only probable cause before the issuance of a search warrant but that the search warrant must be based upon an application supported by oath of the applicant and the witnesses he may produce.

ISSUE: WON the search warrants issued are valid. HELD: With regard the search issued in the corporation – valid; with regard the search in the houses – void. RATIO: As regards the first group(In the offices), we hold that petitioners herein have no cause of action to assail the legality of the contested warrants and of the seizures made in pursuance thereof, for the simple reason that said corporations have their respective personalities, separate and distinct from the personality of herein petitioners, regardless of the amount of shares of stock or of the interest of each of them in said corporations, and whatever the offices they hold therein may be.8 Indeed, it is well settled that the legality of a seizure can be contested only by the party whose rights have been impaired thereby,9 and that the objection to an unlawful search and seizure is purely personal and cannot be availed of by third parties. 10 Consequently, petitioners herein may not validly object to the use in evidence against them of the documents, papers and things seized from the offices and premises of the corporations adverted to above, since the right to object to the admission of said papers in evidence belongs exclusively to the corporations, to whom the seized effects belong, and may not be invoked by the corporate officers in proceedings against them in their individual capacity. Second in their houses: Indeed, the same were issued upon applications stating that the natural and juridical person therein named had committed a "violation of Central Bank Laws, Tariff and Customs Laws, Internal Revenue (Code) and Revised Penal Code." In other words, no specific offense had been alleged in said applications. The averments thereof with respect to the offense committed were abstract. As a consequence, it was impossible for the judges who issued the warrants to have found the existence of probable cause, for the same presupposes the introduction of competent proof that the party against whom it is sought has performed particular acts, or committed specific omissions, violating a given provision of our criminal laws. As a matter of fact, the applications involved in this case do not allege any specific acts performed by herein petitioners. It would be the legal heresy, of the highest order, to convict anybody of a "violation of Central Bank Laws, Tariff and Customs Laws, Internal Revenue (Code) and Revised Penal Code," — as alleged in the aforementioned applications — without reference to any determinate provision of said laws. the warrants authorized the search for and seizure of records pertaining to all business transactions of petitioners herein, regardless of whether the transactions were legal or illegal. The warrants sanctioned the seizure of all records of the petitioners and the aforementioned corporations, whatever their nature, thus openly contravening the explicit command of our Bill of Rights — that the things to be seized be particularly described — as well as tending to defeat its major objective: the elimination of general warrants. Notes: SCATTER-SHOT WARRANT- It is a warrant of arrest that is issued for more than one offense. It is void for the law requires that a warrant of arrest should only be issued in connection with one specific offense. GENERAL WARRANT- A general warrant refers to a warrant providing a law-enforcement officer with broad discretion or authority to search and seize unspecified places or persons. A general warrant

Probable Cause Alvarez v. CFI, 64 Phil. 33

● It is not mandatory to present affidavits of witnesses to corroborate the applicant or a complainant in cases where the latter has personal knowledge of the facts. ● A detailed description of the person and place to be searched and the articles to be seized is necessary, but where, by the nature of the articles to be seized, their description must be rather general, it is not required that a technical description be given, as this would mean that no warrant could issue. Facts: Mariano Almeda, the chief of the secret service of the AntiUsury Board, applied for a search warrant and presented to Judge David of the Court of First Instance of Tayabas an affidavit alleging that according to reliable information, petitioner Alvarez kept in his house in Infanta, Tayabas, books, documents, receipts, lists, chits and other papers used by him in connection with his activities as a moneylender charging usurious rates of interest in violation of the law. In his oath at the end of the affidavit, Almeda stated that his answers to the questions were correct to the best of his knowledge and belief. He did not swear to the truth of his statements upon his own knowledge of the facts but upon the information received by him from a reliable person. On the basis of such affidavit, Judge David issued a warrant ordering the search of the petitioner’s house at any time of the day or night, the seizure of the books and documents above-mentioned and the immediate delivery thereof to him to be disposed of in accordance with the law. With said warrant, several agents of the Anti-Usury Board entered the petitioner’s store and residence at 7:00 pm and seized articles such as internal revenue license, ledger, journals, cash bonds, check stubs, memorandums, blackboards, contracts, inventories, bill of lading, credit receipts, correspondence, receipt books, promissory notes and checks. Alvarez filed a petition praying that the search warrant be declared illegal and that all the articles in question be returned to him. He claimed that the search warrant was illegal for the following reasons: (a) it was based solely upon the affidavit of the petitioner who had no personal knowledge of the facts of probable cause; (b) it was not supported by other affidavits aside from that made by the applicant; (c) it authorized its execution at night; (d) lack of an adequate description of the books and documents to be seized; and (e) it was issued for the sole purpose of seizing evidence which would later be used in the criminal proceedings that might be instituted against him for violation of the Anti-Usury Law. The Anti-Usury Board insinuates in its answer that the petitioner cannot now question the validity of the search warrant or the proceedings had subsequent to the issuance thereof, because he has waived his constitutional rights in proposing a compromise whereby he agreed to pay a fine of P200 for the purpose of evading the criminal proceeding or proceedings. The CFI ruled against the Alvarez and upheld the validity of the search warrant. Issues:1. Is the search warrant illegal when the affidavit is based on hearsay?

2. Is there a need for affidavit of another witnesses to support the application for search warrant? 3. Can the search warrant be executed at night? 4. Did the search warrant satisfy the particularity of description as required by the law? 5. Is fishing evidence is valid? 6. Did Alvarez waived his constitutional right to question the validity of the search warrant or the proceedings when he offered a compromise or settlement of the case? Held: 1. Yes. The provisions of the constitution require that there be not only probable cause before the issuance of a search warrant but that the search warrant must be based upon an application supported by oath of the applicant and the witnesses he may produce. The oath required must refer to the truth of the facts within the personal knowledge of the petitioner or his witnesses, because the purpose thereof is to convince the committing magistrate, not the individual making the affidavit and seeking the issuance of the warrant, of the existence of probable cause. The true test of sufficiency of an affidavit to warrant issuance of a search warrant is whether it has been drawn in such a manner that perjury could be charged thereon and affiant be held liable for damages caused. It appears that the affidavit, which served as the exclusive basis of the search warrant, is insufficient and fatally defective by reason of the manner in which the oath was made, and therefore, it is hereby held that the search warrant in question and the subsequent seizure of the books, documents and other papers are illegal and do not in any way warrant the deprivation to which the petitioner was subjected. 2. No. It is not mandatory to present affidavits of witnesses to corroborate the applicant or a complainant in cases where the latter has personal knowledge of the facts. However, when the applicant's or complainant's knowledge of the facts is merely hearsay, it is the duty of the judge to require affidavits of other witnesses so that he may determine whether probable cause exists. 3. Yes. The search be made at night when it is positively asserted in the affidavits that the property is on the person or in the place ordered to be searched. In this case, it does not appear positively in the affidavit that the articles were in the possession of the petitioner and in the place indicated; hence, the search and seizure could not be made at night. 4. Yes. A detailed description of the person and place to be searched and the articles to be seized is necessary, but where, by the nature of the articles to be seized, their description must be rather general, it is not required that a technical description be given, as this would mean that no warrant could issue. The only description of the articles given in the affidavit presented to the judge was as follows: "that there are being kept in said premises books, documents, receipts, lists, chits and other papers used by him in connection with his activities as moneylender, charging a usurious rate of interest, in violation of the law." Taking into consideration the nature of the article so described, it is clear that no other more adequate and detailed description could have been given, particularly because it is difficult to give a particular description of the contents thereof. The description so made substantially complies with the legal provisions because the officer of the law who executed the warrant was thereby placed in a position enabling him to identify the articles, which he did. 5. At the hearing of the incidents of the case raised before the court it clearly appeared that the books and documents had really been seized to enable the Anti-Usury Board to conduct an investigation and later use all or some of the articles in question as evidence against the petitioner in the criminal cases that may be filed against him. The seizure of books and documents by means of a search warrant, for the purpose of using them as evidence in a criminal case against the person

in whose possession they were found, is unconstitutional because it makes the warrant unreasonable, and it is equivalent to a violation of the constitutional provision prohibiting the compulsion of an accused to testify against himself. Therefore, it appearing that at least nineteen of the documents in question were seized for the purpose of using them as evidence against the petitioner in the criminal proceeding or proceedings for violation against him, we hold that the search warrant issued is illegal and that the documents should be returned to him. 6. No. The petitioner did not waive his constitutional rights because the offer of compromise or settlement attributed to him, does not mean, if so made, that he voluntarily tolerated the search and seizure. The waiver would have been a good defense for the respondents had the petitioner voluntarily consented to the search and seizure of the articles in question, but such was not the case because the petitioner protested from the beginning and stated his protest in writing in the insufficient inventory furnished him by the agents. Notes: Probable Cause (Search Warrant)- Probable cause for a search is defined as such facts and circumstances which would lead a reasonably discreet and prudent man to believe that an offense has been committed and that the objects sought in connection with the offense are in the place sought to be searched. Burgos v. Chief of Staff, 133 SCRA 800 (1984) Facts: Petitioners assail the validity of 2 search warrants issued on December 7, 1982 by respondent Judge Cruz-Pano of the then Court of First Instance of Rizal, under which the premises known as No. 19, Road 3, Project 6, Quezon City, and 784 Units C & D, RMS Building, Quezon Avenue, Quezon City, business addresses of the "Metropolitan Mail" and "We Forum" newspapers, respectively, were searched, and office and printing machines, equipment, paraphernalia, motor vehicles and other articles used in the printing, publication and distribution of the said newspapers, as well as numerous papers, documents, books and other written literature alleged to be in the possession and control of petitioner Jose Burgos, Jr. publisher-editor of the "We Forum" newspaper, were seized. As a consequence of the search and seizure, these premises were padlocked and sealed, with the further result that the printing and publication of said newspapers were discontinued. Respondents contend that petitioners should have filed a motion to quash said warrants in the court that issued them before impugning the validity of the same before this Court. Respondents also assail the petition on ground of laches (Failure or negligence for an unreasonable and unexplained length of time to do that which, by exercising due diligence, could or should have been done earlier. It is negligence or omission to assert a right within a reasonable time, warranting a presumption that the party entitled to assert it either has abandoned it or declined to assert it). Respondents further state that since petitioner had already used as evidence some of the documents seized in a prior criminal case, he is stopped from challenging the validity of the search warrants. Petitioners submit the following reasons to nullify the questioned warrants: 1. Respondent Judge failed to conduct an examination under oath or affirmation of the applicant and his witnesses, as mandated by the above-quoted constitutional provision as well as Sec. 4, Rule 126 of the Rules of Court. 2. The search warrants pinpointed only one address which would be the former abovementioned address. 3. Articles belonging to his co-petitioners were also seized although the warrants were only directed against Jose Burgos, Jr. 4. Real properties were seized.

5. The application along with a joint affidavit, upon which the warrants were issued, from the Metrocom Intelligence and Security Group could not have provided sufficient basis for the finding of a probable cause upon which a warrant may be validly issued in accordance with Section 3, Article IV of the 1973 Constitution.

the request of military authorities to sequester the property seized from petitioners. The closure of the premises subjected to search and seizure is contrary to the freedom of the press as guaranteed in our fundamental law. The search warrants are declared null and void. Personal determination

Respondents justify the continued sealing of the printing machines on the ground that they have been sequestered under Section 8 of Presidential Decree No. 885, as amended, which authorizes sequestration of the property of any person engaged in subversive activities against the government in accordance with implementing rules and regulations as may be issued by the Secretary of National Defense. Issue: Whether or Not the 2 search warrants were validly issued and executed. Held: In regard to the quashal of warrants that petitioners should have initially filed to the lower court, this Court takes cognizance of this petition in view of the seriousness and urgency of the constitutional Issue raised, not to mention the public interest generated by the search of the "We Forum" offices which was televised in Channel 7 and widely publicized in all metropolitan dailies. The existence of this special circumstance justifies this Court to exercise its inherent power to suspend its rules. With the contention pertaining to laches, the petitioners gave an explanation evidencing that they have exhausted other extra-judicial efforts to remedy the situation, negating the presumption that they have abandoned their right to the possession of the seized property. On the enumerated reasons: 1. This objection may properly be considered moot and academic, as petitioners themselves conceded during the hearing on August 9, 1983, that an examination had indeed been conducted by respondent judge of Col. Abadilla and his witnesses. 2. The defect pointed out is obviously a typographical error. Precisely, two search warrants were applied for and issued because the purpose and intent were to search two distinct premises. It would be quite absurd and illogical for respondent judge to have issued two warrants intended for one and the same place. 3. Section 2, Rule 126, of the Rules of Court, does not require that the property to be seized should be owned by the person against whom the search warrant is directed. It may or may not be owned by him. 4. Petitioners do not claim to be the owners of the land and/or building on which the machineries were placed. This being the case, the machineries in question, while in fact bolted to the ground, remain movable property susceptible to seizure under a search warrant. 5. The broad statements in the application and joint affidavit are mere conclusions of law and does not satisfy the requirements of probable cause. Deficient of such particulars as would justify a finding of the existence of probable cause, said allegation cannot serve as basis for the issuance of a search warrant and it was a grave error for respondent judge to have done so. In Alvarez v. Court of First Instance, this Court ruled that "the oath required must refer to the truth of the facts within the personal knowledge of the petitioner or his witnesses, because the purpose thereof is to convince the committing magistrate, not the individual making the affidavit and seeking the issuance of the warrant, of the existence of probable cause." Another factor which makes the search warrants under consideration constitutionally objectionable is that they are in the nature of general warrants. The description of the articles sought to be seized under the search warrants in question are too general. With regard to the respondents invoking PD 885, there is an absence of any implementing rules and regulations promulgated by the Minister of National Defense. Furthermore, President Marcos himself denies

Soliven v. Makasiar, 167 SCRA 394 (1988) Facts: Soliven broadcasted the statement that President Aquino hid under her bed during a coup d' etat. The case at bar is a petition raised by one of the petitioners, Beltran, who wants to call for an interpretation of the constitutional provision on the issuance of warrants of arrest. The petitioner assailed that his constitutional right was violated when respondent RTC judge issued a warrant for his arrest without personally examining the complainant and the witnesses, if any, to determine probable cause. Beltran's interpretation of the words "determined personally" convinced him that the judge is solely responsible to personally examine the complainant and his witnesses in his determination of probable cause for the issuance of warrants of arrest. ISSUE: Whether or not respondent committed a grave abuse of discretion amounting to lack or excess of jurisdiction when the warrant of arrest was issued. HELD: No. The Court did not find any grave abuse of discretion amounting to lack or excess of jurisdiction on the part of the respondent judge. Article III, Section 2 of the 1987 Constitution The right of the people to be secure in their persons, houses, papers and effects against unreasonable searches and seizures of whatever nature and for any purpose shall be inviolable, and no search warrant or warrant of arrest shall issue except upon probable cause to be determined personally by the judge after examination under oath or affirmation of the complainant and the witnesses he may produce, and particularly describing the place to be searched and the persons or things to be seized. DOCTRINE:What the Constitution requires is that the issuing judge must satisfy himself first with the criteria in finding probable cause. And to satisfy himself doesn't mean to he is required to personally examine the complainant and his witnesses. The Constitution mandates that he shall: (1) personally evaluate the report and the supporting documents submitted by the fiscal regarding the existence of probable cause and, on the basis thereof, issue a warrant of arrest; or(2) if on the basis thereof he finds no probable cause, he may disregard the fiscal's report and require the submission of supporting affidavits of witnesses to aid him in arriving at a conclusion as to the existence of probable cause. Silva vs. Presiding Judge of RTC, Negros FACTS: l RTC judge Nickarter Ontal issued a search warrant filed by M/Sgt. Ranulfo Villamor, chief of the PC NARCOM Detachment in Dumaguete City, Negros Oriental, to be served against Petitioner Nicomedes Silva. l The application was accompanied by “deposition of witness” executed by Arthur Alcoran and Pat. Leon Quindo. l The search warrant stated that “You are hereby commanded to make an immediate search at any time of the day (night) of the room of Tama Silva residence of his father Comedes Silva to open aparadors, lockers,

cabinets, cartoons, containers, forthwith seize and take possession of the following property Marijuana dried leaves, cigarettes, joint and bring the said property to the undersigned to be dealt with as the law directs.” l During the raid, the officers seized money belonging to Antonieta Silva in the amount of P1,231.40. l Petitioners alleged that the enforcement of the search warrant was illegal because it was issued on the sole basis of mimeographed and the judge failed to personally examine the complainant and witness by searching questions and answers.

Lim v. Felix, G.R. No. 94054-7, February 19, 1991 FACTS:l Congressman Moises Espinosa, Sr., together with his security escorts were attacked and killed by a lone assassin at the airport vicinity in Masbate. l Dante Siblante another security escort of Congressman Espinosa, Sr. survived the assassination plot, although, he himself suffered a gunshot wound. l Herein petitioners were alleged to be behind the crime of multiple murder and frustrated murder in connection with the airport incident.

l Antoinette Silva also filed a motion the return of the said amount because her name is not included in the search warrant. Thus, her belongings shouldn’t be subject of the warrant.

l After conducting the preliminary investigation, the court issued an order finding probable cause for the issuance of a warrant of arrest of herein petitioners.

l Acting on the said motion to return the money, Judge Ontal issued an order stating that the court “holds in abeyance the disposition of the said amount pending the filing of appropriate charges in connection with the search warrant.

l In the same Order, the court ordered the arrest of the petitioners and recommended the amount of P200,000.00 as bail for the provisional liberty of each of the accused.

l RTC’s new judge, replacing judge Ontal, ruled against petitioners. l MR was likewise denied by Judge Cruz (new judge). l Hence, this special civil action for certiorari. ISSUE: WON the search warrant is validly issued by then Judge Ontal HELD: NO. In the case at bar, we have carefully examined the questioned search warrant as well as the "Application for Search Warrant" and "Deposition of Witness", and found that Judge Ontal failed to comply with the legal requirement that he must examine the applicant and his witnesses in the form of searching questions and answers in order to determine the existence of probable cause. The joint "Deposition of Witness" executed by Pfc. Alcoran and Pat. Quindo, which was submitted together with the "Application for Search Warrant" contained, for the most part, suggestive questions answerable by merely placing "yes" or "no" in the blanks provided thereon. The above deposition did not only contain leading questions but it was also very broad. The questions propounded to the witnesses were in fact, not probing but were merely routinary. The deposition was already mimeographed and all that the witnesses had to do was fill in their answers on the blanks provided. Thus, in issuing a search warrant, the judge must strictly comply with the constitutional and statutory requirement that he must determine the existence of probable cause by personally examining the applicant and his witnesses in the form of searching questions and answers. His failure to comply with this requirement constitutes grave abuse of discretion. The officers implementing the search warrant clearly abused their authority when they seized the money of Antonieta Silva. This is highly irregular considering that Antonieta Silva was not even named as one of the respondents, that the warrant did not indicate the seizure of money but only of marijuana leaves, cigarettes and joints, and that the search warrant was issued for the seizure of personal property (a) subject of the offense and (b) used or intended to be used as means of committing an offense and NOT for personal property stolen or embezzled or other proceeds of fruits of the offense. Thus, the then presiding Judge Ontal likewise abused his discretion when he rejected the motion of petitioner Antonieta Silva seeking the return of her seized money.

l Respondent Acting Fiscal Antonio C. Alfane was designated to review the case containing 261 pages. l Fiscal Alfane issued a Resolution which affirmed the finding of a prima facie case against the petitioners but differed in the designation of the crime in that the ruled that ". . . all of the accused should not only be charged with Multiple Murder With Frustrated Murder" but for a case of MURDER for each of the killing of the four victims and a physical injuries case for inflicting gunshot wound on the buttocks of Dante Siblante." l MR’s of the petitioner’s Lim was also denied. l Fiscal Alfane filed with the Regional Trial Court of Masbate, four (4) separate information of murder against the twelve (12) accused with a recommendation of no bail. l Petitioners Vicente Lim, Sr. and Susana Lim filed with us a verified petition for change of venue and was granted to avoid a miscarriage of justice. (from Masbate to Makati RTC) l The cases were raffled to Branch 56 presided by respondent Judge Nemesio S. Felix. l Petitioners questioned the validity of the warrant of arrest because it was not personally determined by the judge as he relied solely on the certification or recommendation of a prosecutor that a probable cause exists. l RTC dismissed their petition upholding the validity of the arrest warrants. ISSUE: WON a judge may issue a warrant of arrest without bail by simply relying on the prosecution's certification and recommendation that a probable cause exists. HELD: NO. If a Judge relies solely on the certification of the Prosecutor as in this case where all the records of the investigation are in Masbate, he or she has not personally determined probable cause. The determination is made by the Provincial Prosecutor. The constitutional requirement has not been satisfied. The Judge commits a grave abuse of discretion. The records of the preliminary investigation conducted by the Municipal Court of Masbate and reviewed by the respondent Fiscal were still in Masbate when the respondent Fiscal issued the warrants of arrest against the petitioners. There was no basis for the respondent Judge to make his own personal determination regarding the existence of a probable cause for the issuance of a warrant of arrest as mandated by the Constitution. He could not possibly have known what transpired in Masbate as he had nothing but a certification. Significantly, the respondent Judge denied the petitioners' motion for the transmittal of

the records on the ground that the mere certification and recommendation of the respondent Fiscal that a probable cause exists is sufficient for him to issue a warrant of arrest.

Particularity of description

Hence, the Judge must go beyond the Prosecutor's certification and investigation report whenever necessary. He should call for the complainant and witnesses themselves to answer the court's probing questions when the circumstances of the case so require.

THE FACTS:Petitioner 20th Century Fox Film Corporation sought the assistance of the NBI in conducting searches and seizures in connection with the NBI’s anti-film piracy campaign. Petitioner alleged that certain videotape outlets all over Metro Manila are engaged in the unauthorized sale and renting out of copyrighted films in violation of PD No. 49 (the old Intellectual Property Law).

Mata v. Bayona , 128 SCRA 388 (1984) FACTS: Soriano Mata was accused under Presidential Decree (PD) 810, as amended by PD 1306, the information against him alleging that Soriano Mata offered, took and arranged bets on the Jai Alai game by “selling illegal tickets known as ‘Masiao tickets’ without any authority from the Philippine Jai Alai & Amusement Corporation or from the government authorities concerned.” Mata claimed that during the hearing of the case, he discovered that nowhere from the records of the said case could be found the search warrant and other pertinent papers connected to the issuance of the same, so that he had to inquire from the City Fiscal its whereabouts, and to which inquiry Judge Josephine K. Bayona, presiding Judge of the City Court of Ormoc replied, “it is with the court”. The Judge then handed the records to the Fiscal who attached them to the records. This led Mata to file a motion to quash and annul the search warrant and for the return of the articles seized, citing and invoking, among others, Section 4 of Rule 126 of the Revised Rules of Court. The motion was denied by the Judge on 1 March 1979, stating that the court has made a thorough investigation and examination under oath of Bernardo U. Goles and Reynaldo T. Mayote, members of the Intelligence Section of 352nd PC Co./Police District II INP; that in fact the court made a certification to that effect; and that the fact that documents relating to the search warrant were not attached immediately to the record of the criminal case is of no moment, considering that the rule does not specify when these documents are to be attached to the records. Mata’s motion for reconsideration of the aforesaid order having been denied, he came to the Supreme Court, with the petition for certiorari, praying, among others, that the Court declare the search warrant to be invalid for its alleged failure to comply with the requisites of the Constitution and the Rules of Court, and that all the articles confiscated under such warrant as inadmissible as evidence in the case, or in any proceedings on the matter. ISSUE: WON the judge must before issuing the warrant personally examine on oath or affirmation the complainant and any witnesses he may produce and take their depositions in writing, and attach them to the record, in addition to any affidavits presented to him? HELD:YES. Under the Constitution “no search warrant shall issue but upon probable cause to be determined by the Judge or such other responsible officer as may be authorized by law after examination under oath or affirmation of the complainant and the witnesses he may produce”. More emphatic and detailed is the implementing rule of the constitutional injunction, The Rules provide that the judge must before issuing the warrant personally examine on oath or affirmation the complainant and any witnesses he may produce and take their depositions in writing, and attach them to the record, in addition to any affidavits presented to him. Mere affidavits of the complainant and his witnesses are thus not sufficient. The examining Judge has to take depositions in writing of the complainant and the witnesses he may produce and to attach them to the record. Such written deposition is necessary in order that the Judge may be able to properly determine the existence or nonexistence of the probable cause, to hold liable for perjury the person giving it if it will be found later that his declarations are false. We, therefore, hold that the search warrant is tainted with illegality by the failure of the Judge to conform with the essential requisites of taking the depositions in writing and attaching them to the record, rendering the search warrant invalid.

20th Century Fox Film Corp. v. CA, 164 SCRA 655 (1988)

The NBI conducted surveillance and investigation of the outlets pinpointed by the petitioner and subsequently filed three (3) applications for search warrants against the video outlets owned by the private respondents. The lower court issued the desired search warrants. The NBI, accompanied by the petitioner's agents, raided the video outlets and seized the items described in the three warrants. Private respondents later filed a motion to lift the search warrants and release the seized properties, which was granted by the lower court. Petitioner’s motion for reconsideration was denied by the lower court. The CA affirmed the trial court. II. THE ISSUE:Did the judge properly lift the search warrants he issued earlier? III. THE RULING:[The Court DISMISSED the petition and AFFIRMED the questioned decision and resolution of the CA.] YES, the judge properly lifted the search warrants he issued earlier. The lower court lifted the three (3) questioned search warrants in the absence of probable cause that the private respondents violated P.D. 49. NBI agents who acted as witnesses during the application for search warrant did not have personal knowledge of the subject matter of their testimony, which was the alleged commission of the offense of piracy by the private respondents. Only the petitioner’s counsel who was also a witness during the application stated that he had personal knowledge that the confiscated tapes owned by the private respondents were pirated tapes taken from master tapes belonging to the petitioner. The lower court lifted the warrants, declaring that the testimony of petitioner’s counsel did not have much credence because the master tapes of the allegedly pirated tapes were not shown to the court during the application. The presentation of the master tapes of the copyrighted films, from which the pirated films were allegedly copied, was necessary for the validity of search warrants against those who have in their possession the pirated films. The petitioner's argument to the effect that the presentation of the master tapes at the time of application may not be necessary as these would be merely evidentiary in nature and not determinative of whether or not a probable cause exists to justify the issuance of the search warrants is not meritorious. The court cannot presume that duplicate or copied tapes were necessarily reproduced from master tapes that it owns. The essence of a copyright infringement is the similarity or at least substantial similarity of the purported pirated works to the copyrighted work. Hence, the applicant must present to the court the copyrighted films to compare them with the purchased evidence of the video tapes allegedly pirated to determine whether the latter is an unauthorized reproduction of the former. This linkage of the copyrighted films to the pirated films must be established to satisfy the requirements of probable cause. Mere allegations as to the existence of the copyrighted films cannot serve as basis for the issuance of a search warrant. Nolasco v. Cruz Pano, 132 SCRA 152 (1985) FACTS: Milagros Aguilar-Roque was arrested together with Cynthia Nolasco by the Constabulary Security Group (CSG). Milagros had been wanted as a high ranking officer of the CPP. The arrest took place at 11:30 a.m. of August 6, 1984. At noon of the same day, her premises

were searched and 428 documents, a portable typewriter and 2 boxes were seized. Earlier that day, Judge Cruz Paño issued a search warrant to be served at Aguilar-Roque’s leased residence allegedly an underground house of the CPP/NPA. On the basis of the documents seized, charges of subversion and rebellion by the CSG were filed by but the fiscal’s office merely charged her and Nolasco with illegal possession of subversive materials. Aguilar-Roque asked for suppression of the evidence on the ground that it was illegally obtained and that the search warrant is void because it is a general warrant since it does not sufficiently describe with particularity the things subject of the search and seizure, and that probable cause has not been properly established for lack of searching questions propounded to the applicant’s witness. ISSUE: WON the search warrant was valid? HELD: NO. Section 3, Article IV of the Constitution, guarantees the right of the people to be secure in their persons, houses, papers and effects against unreasonable searches and seizures of whatever nature and for any purpose. It also specifically provides that no Search Warrant shall issue except upon probable cause to be determined by the Judge or such other responsible officer as may be authorized by law, after examination under oath or affirmation of the complainant and the witnesses he may produce, and particularly describing the place to be searched and the things to be seized. It is at once evident that the foregoing Search Warrant authorizes the seizure of personal properties vaguely described and not particularized. It is an all- embracing description which includes everything conceivable regarding the Communist Party of the Philippines and the National Democratic Front. It does not specify what the subversive books and instructions are; what the manuals not otherwise available to the public contain to make them subversive or to enable them to be used for the crime of rebellion. There is absent a definite guideline to the searching team as to what items might be lawfully seized thus giving the officers of the law discretion regarding what articles they should seize as, in fact, taken also were a portable typewriter and 2 wooden boxes. It is thus in the nature of a general warrant and infringes on the constitutional mandate requiring particular description of the things to be seized. In the recent rulings of this Court, search warrants of similar description were considered null and void for being too general. Valid Warrantless searches People v. Malmstedt, 198 SCRA 401 (1991) Facts:Captain Alen Vasco, the commanding officer of the first regional command (NARCOM) stationed at camp Dangwa, ordered his men to set up a temporary checkpoint for the purpose of checking all vehicles coming from the Cordillera Region. The order to establish a checkpoint was prompted by persistent reports that vehicles coming from Sagada were transporting marijuana and other prohibited drugs. And an information also was received about a Caucasian coming from Sagada had in his possession prohibited drugs. In the afternoon the bus where accused was riding stopped. Sgt. Fider and CIC Galutan boarded the bus and announced that they were members of the NARCOM and that they would conduct an inspection. During the inspection CIC Galutan noticed a bulge on accused waist. Suspecting the bulge on accused waist to be a gun, the officer asked for accused’s passport and other identification papers. When accused failed to comply, the officer required him to bring out whatever it was that was bulging o his waist. And it turned out to be a pouched bag and when accused opened the same bag the officer noticed four suspicious looking objects wrapped in brown packing tape. It contained hashish, a derivative of marijuana.

Thereafter, the accused was invited outside the bus for questioning. But before he alighted from the bus accused stopped to get two travelling bags. The officer inspects the bag. It was only after the officers had opened the bags that the accused finally presented his passport. The two bags contained a stuffed toy each, upon inspection the stuff toy contained also hashish. Issue: Whether or not there is a violation of the constitutional right against unreasonable search and seizure Ruling: The Supreme Court held that under Section 5 Rule 113 of the Rules of Court provides: “Arrest without warrant; when lawful – a peace officer or a private person may, without a warrant, arrest a person: a) When, in the presence, the person to be arrested has committed, is actually committing, or is attempting to commit an offense; b) When an offense has in fact just been committed, and he has personal knowledge of facts indicating that the person to be arrested has committed it; and c) When the person to be arrested is a prisoner who has escaped from a penal establishment or place where he is serving final judgment or temporary confined while his case is pending, or has escaped while being transferred from one confinement to another” Accused was searched and arrested while transporting prohibited drugs. A crime was actually being committed by the accused and he was caught in flagrante delicto, thus the search made upon his personal effects falls squarely under paragraph 1 of the foregoing provision of law, which allows a warrantless search incident to a lawful arrest. Probable cause has been defined as such facts and circumstances which could lead a reasonable, discreet and prudent man to believe that an offense has been committed, and that the object sought in connection with the offense are in the placed sought to be searched. When NARCOM received the information that a Caucasian travelling from Sagada to Baguio City was carrying with him a prohibited drug, there was no time to obtain a search warrant. Nolasco v. Cruz Pano, G.R. No. L-69803, January 30, 1987 FACTS: Milagros Aguilar-Roque was arrested together with Cynthia Nolasco by the Constabulary Security Group (CSG). Milagros had been wanted as a high ranking officer of the CPP. The arrest took place at 11:30 a.m. of August 6, 1984. At noon of the same day, her premises were searched and 428 documents, a portable typewriter and 2 boxes were seized. Earlier that day, Judge Cruz Paño issued a search warrant to be served at Aguilar-Roque’s leased residence allegedly an underground house of the CPP/NPA. On the basis of the documents seized, charges of subversion and rebellion by the CSG were filed by but the fiscal’s office merely charged her and Nolasco with illegal possession of subversive materials. Aguilar-Roque asked for suppression of the evidence on the ground that it was illegally obtained and that the search warrant is void because it is a general warrant since it does not sufficiently describe with particularity the things subject of the search and seizure, and that probable cause has not been properly established for lack of searching questions propounded to the applicant’s witness. Held: The majority pronouncement that "as an incident to (petitioner Mila Aguilar- Roque's) arrest, her dwelling at No. 239-B Mayon Street, Quezon City could have been searched, even without a warrant, for evidence of rebellion" is patently against the constitutional proscription and settled law and jurisprudence. Mr. Justice Cuevas amply discusses this in the dissenting portion of his separate opinion. Suffice it to add and stress that the arresting CSG Group themselves knew that they needed a search warrant and obtained the void warrant

in question. The exception of Rule 126, sec. 12 which allows a warrantless search of a person who is lawfully arrested is absolutely limited to his person, at the time of and incident to his arrest and to dangerous weapons or anything which may be used as proof of the commission of the offense." Such warrantless search obviously cannot be made in a place other than the place of arrest. In this case, petitioner Aguilar-Roque was arrested at 11:30 a.m. on board a public vehicle on the road (at Mayon and P. Margall Streets). To hold that her dwelling could "later on the same day" be searched without warrant is to sanction an untenable violation, if not nullification, of the cited basic constitutional rights against unreasonable searches and seizures. Plain view doctrine People v. Musa, 217 SCRA 597 (1993) Facts: A civilian informer gave the information that Mari Musa was engaged in selling marijuana in Suterville, Zamboanga City. Sgt. Ani was ordered by NARCOM leader T/Sgt. Belarga, to conduct a surveillance and test buy on Musa. The civilian informer guided Ani to Musa’s house and gave the description of Musa. Ani was able to buy one newspaper-wrapped dried marijuana for P10.00. The next day, a buy-bust was planned. Ani was to raise his right hand if he successfully buys marijuana from Musa. As Ani proceeded to the house, the NARCOM team positioned themselves about 90 to 100 meters away. From his position, Belarga could see what was going on. Musa came out of the house and asked Ani what he wanted. Ani said he wanted more marijuana and gave Musa the P20.00 marked money. Musa went into the house and came back, giving Ani two newspaper wrappers containing dried marijuana. Ani opened and inspected it. He raised his right hand as a signal to the other NARCOM agents, and the latter moved in and arrested Musa inside the house. Belarga frisked Musa in the living room but did not find the marked money (gave it to his wife who slipped away). T/Sgt. Belarga and Sgt. Lego went to the kitchen and found a ‘cellophane colored white and stripe hanging at the corner of the kitchen.’ They asked Musa about its contents but failed to get a response. So they opened it and found dried marijuana leaves inside. Musa was then placed under arrest. Issue: Whether or Not the seizure of the plastic bag and the marijuana inside it is unreasonable, hence, inadmissible as evidence. Held: Yes. It constituted unreasonable search and seizure thus it may not be admitted as evidence. The warrantless search and seizure, as an incident to a suspect’s lawful arrest, may extend beyond the person of the one arrested to include the premises or surroundings under his immediate control. Objects in the ‘plain view’ of an officer who has the right to be in the position to have that view are subject to seizure and may be presented as evidence. The ‘plain view’ doctrine is usually applied where a police officer is not searching for evidence against the accused, but nonetheless inadvertently comes across an incriminating object. It will not justify the seizure of the object where the incriminating nature of the object is not apparent from the ‘plain view’ of the object. In the case at bar, the plastic bag was not in the ‘plain view’ of the police. They arrested the accused in the living room and moved into the kitchen in search for other evidences where they found the plastic bag. Furthermore, the marijuana inside the plastic bag was not immediately apparent from the ‘plain view’ of said object. Therefore, the ‘plain view’ does not apply. The plastic bag was seized illegally and cannot be presented in evidence pursuant to Article III Section 3 (2) of the Constitution. Notes: Plain view Doctrine Requisites: 1. There must have been a prior valid intrusion based on the warrantless arrest in which the police are legally present in the pursuit of their official duties

2. The evidence was inadvertently discovered by the police who had the right to be where they are 3. The evidence must be immediately apparent 4. There was no need for further search “Stop and frisk” Terry v. Ohio, 392 U.S. 1 Facts: Officer McFadden observed two men outside of a store walking up to the window then away several times. A third man met up with the initial two and engaged in conversation. The plainly clothed officer developed suspicion that the men may be planning to rob the store. McFadden approached the men and after identifying himself as an officer asked what they were doing. The men mumbled back a response. McFadden then grabbed Terry, turned him around and patted him down to determine if he was armed. The search revealed a gun in Terry’s coat pocket. After conducting the same search on the second man, another gun was revealed. Once at trial, the officer testified he thought the men may have been armed. Procedural History: An Ohio trial court convicted Terry with carrying a concealed weapon. Terry appealed to the Supreme Court of the United States. Issues and Holding: May a police officer detain an individual on the street absent probable cause and conduct a limited search to find weapons? Yes. Judgment:Officers may conduct a search limited for weapons when they observe unusual conduct leading them to reasonably suspect criminal activity is afoot and the individual(s) involved is/are armed. An officer may identify himself as the police and make initial inquiries. This conduct is proper when the officer observes conduct leading him to develop reasonable suspicion that a crime is occurring or about to occur. If the officer believes a threat to himself of others still exists after such an inquiry, a limited search may be performed to find weapons. The Court held that an individual is seized when stopped by a police officer on the street because he is not free to walk away. As a result, the Fourth Amendment protections prohibiting unreasonable searches and seizures apply. The Court also stated that a “pat down” of outer clothing constitutes a search under the Fourth Amendment. The Court held that the constitutionality of the search depended on whether or not the scope of the search was reasonable in light of the circumstances. The test used to determine reasonableness is that the interest in officer safety must outweigh the suspect’s Fourth Amendment right to privacy. The Court held that an officer’s interest in the safety of himself and others outweighs an individual’s Fourth Amendment right. In addition, when an individual is stopped on the street, the police may conduct a proper search for weapons if based on the facts and circumstances, the officer reasonably believes the person is armed. Here, considering the circumstances, it was reasonable for the experienced officer to suspect the two men were planning a robbery. In addition, the government’s interest in law enforcement trumps any minimal invasion of privacy each may have experienced when approached by the officer. The Court also determined the pat-down was reasonable as the officer’s initial concerns were not abated as a result of the responses given. The officer stated that the pat down was conducted under the belief either men could have been armed. As a result, it was proper for the officer to conduct a searched for weapons, as the interest in the safety of the officer and the public

outweighed any privacy right the individuals had under the 4th Amendment. People v. Solayao, 262 SCRA 255 (1996) FACTS:l SPO3 Nio and his team of CAFGU went to Brgy. Caulangohan, Caibiran, Biliran to conduct an investigation regarding reports on the presence of armed men roaming around barangays of Caibiran.

The prosecution should have presented a certification from the Firearms and Explosives Unit of the Philippine National Police that accused-appellant was not a licensee of a firearm of any kind or caliber would have sufficed for the prosecution to prove beyond reasonable doubt the second element of the crime of illegal possession of firearm. Malacat v. Court of Appeals, 283 SCRA 159 (1997)

l Upon arriving in Brgy. Onion, the agents became suspicious to the group of Solayao because the accused-appellant himself is drunk and wearing a camouflage uniform or a jungle suit. What’s more suspicious is when they noticed the team of SPO3 Nio, the group fled leaving behind Solayao, herein accused-appellant.

Facts:Petitioner was arrested for having in his possession a hand grenade after he was searched by a group of policemen when he was said to be acting suspiciously when he was hanging around Plaza Miranda with his eyes moving fast together with other Muslim-looking men. When the policemen approached the group of men, they scattered in all directions which prompted the police to give chase and petitioner was then apprehended and a search was made on his person.

l According to Solayao, he’s not aware that he is carrying a “latong” (49-inch firearm) wrapped in dried coconut leaves. He thought that it’s only a torch which Hermogenes Cenining gave to him and that he is not aware that there’s a concealed weapon inside. He further claimed that this was the third torch handed to him after the others had been used up.

He was then convicted under PD 1866 in the lower court. Hence, the present petition wherein petitioner contended that the lower court erred in holding that the search made on him and the seizure of the hand grenade from him was an appropriate incident to his arrest and that it erred in admitting the hand grenade as evidence since it was admissible because it was a product of an unreasonable and illegal search.

l Accused-appellant Nilo Solayao was charged before the RTC of Biliran, with the crime of illegal possession of firearm and ammunition defined and penalized under PD No. 1866.

Issue: WON the search and seizure conducted by the police was valid.

l The lower court found that accused-appellant did not contest the fact that SPO3 Nino confiscated the firearm from him and that he had no permit or license to possess the same. It hardly found credible accusedappellant's submission that he was in possession of the firearm only by accident and that upon reaching Barangay Onion, he followed four persons, namely, Hermogenes Cenining, Antonio Sevillano, Willie Regir and Jovenito Jaro when he earlier claimed that he did not know his companions.

Held:The general rule as regards arrests, searches and seizures is that a warrant is needed in order to validly effect the same. 31 The Constitutional prohibition against unreasonable arrests, searches and seizures refers to those effected without a validly issued warrant, 32 subject to certain exceptions. As regards valid warrantless arrests, these are found in Section 5, Rule 113 of the Rules of Court, which reads, in part: Sec. 5. — Arrest, without warrant; when lawful — A peace officer or a private person may, without a warrant, arrest a person:

ISSUE: WON the trial court erred in admitting the subject firearm in evidence as it was the product of an unlawful warrantless search.

(a) When, in his presence, the person to be arrested has committed, is actually committing, or is attempting to commit an offense;

HELD:NO. There was no error on the part of the trial court when it admitted the homemade firearm as evidence nor violation of the constitutional guarantee against unreasonable searches and seizures.

(b) When an offense has in fact just been committed, and he has personal knowledge of facts indicating that the person to be arrested has committed it; and

The SC ruled that the search and seizure conducted in this case be likened to the Posadas case where the suspicious conduct of Posadas himself can be likened to a "stop and frisk" situation. There was a probable cause to conduct a search even before an arrest could be made.

(c) When the person to be arrested is a prisoner who has escaped . . .

In the present case, after SPO3 Nino told accused-appellant not to run away, the former identified himself as a government agent.[16] The peace officers did not know that he had committed, or was actually committing, the offense of illegal possession of firearm. Tasked with verifying the report that there were armed men roaming around in the barangays surrounding Caibiran, their attention was understandably drawn to the group that had aroused their suspicion. They could not have known that the object wrapped in coconut leaves which accusedappellant was carrying hid a firearm.

Turning to valid warrantless searches, they are limited to the following: (1) customs searches; (2) search of moving vehicles; (3) seizure of evidence in plain view; (4) consent searches; 33 (5) a search incidental to a lawful arrest;34 and (6) a "stop and frisk.’

As with Posadas, the case at bar constitutes an instance where a search and seizure may be effected without first making an arrest. There was justifiable cause to "stop and frisk" accused-appellant when his companions fled upon seeing the government agents. Under the circumstances, the government agents could not possibly have procured a search warrant first. However, the prosecution failed to produce evidence that the accusedappellant has no license to carry the firearm by merely relying on the lone witness’ (SPO3 Nio) testimony that accused-appellant admitted to them during the time he was apprehended that he has no license to carry such weapon.

A warrantless arrest under the circumstances contemplated under Section 5(a) has been denominated as one "in flagrante delicto," while that under Section 5(b) has been described as a "hot pursuit" arrest.

At the outset, we note that the trial court confused the concepts of a "stop-and-frisk" and of a search incidental to a lawful arrest. These two types of warrantless searches differ in terms of the requisite quantum of proof before they may be validly effected and in their allowable scope. In a search incidental to a lawful arrest, as the precedent arrest determines the validity of the incidental search, the legality of the arrest is questioned in a large majority of these cases, e.g., whether an arrest was merely used as a pretext for conducting a search. 36 In this instance, the law requires that there first be a lawful arrest before a search can be made — the process cannot be reversed. 37 At bottom, assuming a valid arrest, the arresting officer may search the person of the arrestee and the area within which the latter may reach for a weapon or for evidence to destroy, and seize any money or property found which was used in the commission of the crime, or the fruit of the crime, or that which may be used as evidence, or which might furnish the arrestee with the means of escaping or committing violence.

We now proceed to the justification for and allowable scope of a "stopand-frisk" as a "limited protective search of outer clothing for weapons," as laid down in Terry, thus: We merely hold today that where a police officer observes unusual conduct which leads him reasonably to conclude in light of his experience that criminal activity may be afoot and that the persons with whom he is dealing may be armed and presently dangerous, where in the course of investigating this behavior he identifies himself as a policeman and makes reasonable inquiries, and where nothing in the initial stages of the encounter serves to dispel his reasonable fear for his own or others' safety, he is entitled for the protection of himself and others in the area to conduct a carefully limited search of the outer clothing of such persons in an attempt to discover weapons which might be used to assault him. Such a search is a reasonable search under the Fourth Amendment . . Other notable points of Terry are that while probable cause is not required to conduct a "stop and frisk," it nevertheless holds that mere suspicion or a hunch will not validate a "stop and frisk." A genuine reason must exist, in light of the police officer's experience and surrounding conditions, to warrant the belief that the person detained has weapons concealed about him. Finally, a "stop-and-frisk" serves a two-fold interest: (1) the general interest of effective crime prevention and detection, which underlies the recognition that a police officer may, under appropriate circumstances and in an appropriate manner, approach a person for purposes of investigating possible criminal behavior even without probable cause; and (2) the more pressing interest of safety and self-preservation which permit the police officer to take steps to assure himself that the person with whom he deals is not armed with a deadly weapon that could unexpectedly and fatally be used against the police officer. Search of moving vehicles Papa v. Mago, 22 SCRA 857 (1968) Facts: Petitioner Martin Alagao, head of the counterintelligence unit of the Manila Police Department, acting upon a reliable information received on November 3, 1966 to the effect that a certain shipment of personal effects, allegedly misdeclared and undervalued, would be released the following day from the customs zone of the port of Manila and loaded on two trucks, and upon orders of petitioner Ricardo Papa, Chief of Police of Manila and a duly deputized agent of the Bureau of Customs, conducted surveillance at gate No. 1 of the customs zone. When the trucks left gate No. 1 at about 4:30 in the afternoon of November 4, 1966, elements of the counterintelligence unit went after the trucks and intercepted them at the Agrifina Circle, Ermita, Manila. The load of the two trucks consisting of nine bales of goods, and the two trucks, were seized on instructions of the Chief of Police. Upon investigation, a person claimed ownership of the goods and showed to the policemen a “Statement and Receipts of Duties Collected in Informal Entry No. 147-5501”, issued by the Bureau of Customs in the name of a certain Bienvenido Naguit. Claiming to have been prejudiced by the seizure and detention of the two trucks and their cargo, Private Respondents filed with the Court of First Instance of Manila a petition “for mandamus with restraining order. That the goods were seized by members of the Manila Police Department without search warrant issued by a competent court; Respondent Judge Hilarion Jarencio issued an order ex parte restraining the petitioners from opening the nine bales in question, and at the same time set the hearing of the petition for preliminary injunction on November 16, 1966. However, when the restraining order was received by herein petitioners, some bales had already been opened by the examiners of the Bureau of Customs in the presence of officials of the Manila Police Department, an assistant city fiscal and a representative of herein respondent Remedios Mago. Issue: Whether a warrant issued by a competent court is required to search and seize a moving cargo or vehicle.

Held: No, The Tariff and Customs Code does not require said warrant in the instant case. The Code authorizes persons having police authority under Section 2203 of the Tariff and Customs Code to enter, pass through or search any land, enclosure, warehouse, store or building, not being a dwelling house; and also to inspect, search and examine any vessel or aircraft and any trunk, package, or envelope or any person on board, or to stop and search and examine any vehicle, beast or person suspected of holding or conveying any dutiable or prohibited article introduced into the Philippines contrary to law, without mentioning the need of a search warrant in said cases. But in the search of a dwelling house, the Code provides that said “dwelling house may be entered and searched only upon warrant issued by a judge or justice of the peace. The court view, therefor, that except in the case of the search of a dwelling house, persons exercising police authority under the customs law may effect search and seizure without a search warrant in the enforcement of customs laws. the court defined the difference made as to the necessity for a search warrant between goods subject to forfeiture, when concealed in a dwelling house of similar place, and like goods in course of transportation and concealed in a movable vessel, where readily they could be put out of reach of a search warrant. In the instant case, we note that petitioner Martin Alagao and his companion policemen did not have to make any search before they seized the two trucks and their cargo. In their original petition, and amended petition, in the court below Remedios Mago and Valentin Lanopa did not even allege that there was a search. But even if there was a search, there is still authority to the effect that no search warrant would be needed under the circumstances obtaining in the instant case. he guaranty of freedom from unreasonable searches and seizures is construed as recognizing a necessary difference between a search of a dwelling house or other structure in respect of which a search warrant may readily be obtained and a search of a ship, motorboat, wagon, or automobile for contraband goods, where it is not practicable to secure a warrant because the vehicle can be quickly moved out of the locality or jurisdiction in which the warrant must be sought. Emergency circumstances People v. De Gracia 233 SCRA 716 (1994) Facts: The records show that in the early morning of December 1, 1989, Maj. Efren Soria of the Intelligence Division, National Capital Region Defense Command, was on board a brown Toyota car conducting a surveillance of the Eurocar Sales Office located at EDSA, together with his team and was conducted pursuant to an intelligence report received by the division that said establishment was being occupied by elements of the RAM-SFP as a communication command post. After a while, a group of five men disengaged themselves from the crowd and walked towards the car of the surveillance team drew their guns and fired at the team, which attack resulted in the wounding of Sgt. Sagario on the right thigh. Nobody in the surveillance team was able to retaliate because they sought cover inside the car and they were afraid that civilians or bystanders might be caught in the cross-fire. As a consequence, at around 6:30 A.M. of December 5, 1989, a searching team raided the Eurocar Sales Office. They were able to find and confiscate contrabands inside one of the rooms belonging to a certain Col. Matillano which is located at the right portion of the building. On February 22, 1991, the trial court rendered judgment found him guilty beyond reasonable doubt of the offense of illegal possession of firearms in furtherance of rebellion and sentenced him to serve the penalty of reclusion perpetua. That judgment of conviction is now challenged before us in this appeal. Issue: Whether in a state of emergency circumstances exist, the arrest of the accused involved in rebellious act is valid without securing a arrest and search warrant. Held: Yes, Under the foregoing circumstances, it is our considered opinion that the instant case falls under one of the exceptions to the prohibition against a warrantless search. In the first place, the military

operatives, taking into account the facts obtaining in this case, had reasonable ground to believe that a crime was being committed. There was consequently more than sufficient probable cause to warrant their action. Furthermore, under the situation then prevailing, the raiding team had no opportunity to apply for and secure a search warrant from the courts. The trial judge himself manifested that on December 5, 1989 when the raid was conducted, his court was closed. Under such urgency and exigency of the moment, a search warrant could lawfully be dispensed with. While it is true that the officers were not armed with a search warrant when the search was made over the personal effects of accused, however, under the circumstances of the case, there was sufficient probable cause for said officers to believe that accused was then and there committing a crime. Probable cause has been defined as such facts and circumstances which would lead a reasonable, discreet and prudent man to believe that an offense has been committed, and that the objects sought in connection with the offense are in the place sought to be searched. The required probable cause that will justify a warrantless search and seizure is not determined by any fixed formula but is resolved according to the facts of each case. Checkpoints

the public. Checkpoints may also be regarded as measures to thwart plots to destabilize the government, in the interest of public security. In this connection, the Court may take judicial notice of the shift to urban centers and their suburbs of the insurgency movement, so clearly reflected in the increased killings in cities of police and military men by NPA “sparrow units,” not to mention the abundance of unlicensed firearms and the alarming rise in lawlessness and violence in such urban centers, not all of which are reported in media, most likely brought about by deteriorating economic conditions – which all sum up to what one can rightly consider, at the very least, as abnormal times. Between the inherent right of the state to protect its existence and promote public welfare and an individual's right against a warrantless search which is however reasonably conducted, the former should prevail. True, the manning of checkpoints by the military is susceptible of abuse by the men in uniform, in the same manner that all governmental power is susceptible of abuse. But, at the cost of occasional inconvenience, discomfort and even irritation to the citizen, the checkpoints during these abnormal times, when conducted within reasonable limits, are part of the price we pay for an orderly society and a peaceful community.

Valmonte v. De Villa G.R. No. 83988, May 24, 1990 Facts: On 20 January 1987, the National Capital Region District Command (NCRDC) was activated pursuant to Letter of Instruction 02/87 of the Philippine General Headquarters, AFP, with the mission of conducting security operations within its area of responsibility and peripheral areas, for the purpose of establishing an effective territorial defense, maintaining peace and order, and providing an atmosphere conducive to the social, economic and political development of the National Capital Region. As part of its duty to maintain peace and order, the NCRDC installed checkpoints in various parts of Valenzuela, Metro Manila. Petitioners aver that because of the institution of said checkpoints, the Velanzuela residents are worried of being harassed and of their safety being placed at the arbitrary, capricious and whimsical disposition of the military manning the checkpoints considering that their cars and vehicles are being subjected to regular searches and check-ups especially at night or at dawn without a search warrant or a court order. Their alleged fear for their safety increased when Benjamin Parpon was gunned down allegedly in cold blood by members of the NCRDC for ignoring and/or continuing to speed off inspite of warning shots fired in the air. Petitioners Atty. Ricardo Valmonte, who is a resident of Valenzuela, Metro Manila, and the Union of Lawyers and Advocates For People’s Rights (ULAP) sought the declaration of checkpoints in Valenzuela, Metro Manila and elsewhere as unconstitutional. In the alternative, they prayed that respondents Renato De Villa and the National Capital Region District Command (NCRDC) be directed to formulate guidelines in the implementation of checkpoints for the protection of the people. Petitioners contended that the checkpoints gave the respondents blanket authority to make searches and seizures without search warrant or court order in violation of the Constitution. Held: Not all searches and seizures are prohibited. Those which are reasonable are not forbidden. A reasonable search is not to be determined by any fixed formula but is to be resolved according to the facts of each case. Where, for example, the officer merely draws aside the curtain of a vacant vehicle which is parked on the public fair grounds, or simply looks into a vehicle, or flashes a light therein, these do not constitute unreasonable search. The setting up of the questioned checkpoints in Valenzuela (and probably in other areas) may be considered as a security measure to enable the NCRDC to pursue its mission of establishing effective territorial defense and maintaining peace and order for the benefit of

Aniag v. Comelec, 237 SCRA 424 (1994) Facts: In preparation for the synchronized national and local elections, the COMELEC issued Resolution No. 2323, “Gun Ban”, promulgating rules and regulations on bearing, carrying and transporting of firearm or other deadly weapons on security personnel or bodyguards, on bearing arms by members of security agencies or police organizations, and organization or maintenance of reaction forces during the election period. COMELEC also issued Resolution No. 2327 providing for the summary disqualification of candidates engaged in gunrunning, using and transporting of firearms, organizing special strike forces, and establishing spot checkpoints. Pursuant to the “Gun Ban”, Mr. Serrapio Taccad, Sergeant at Arms of the House of Representatives, wrote petitioner for the return of the two firearms issued to him by the House of Representatives. Petitioner then instructed his driver, Arellano, to pick up the firearms from petitioner’s house and return them to Congress. The PNP set up a checkpoint. When the car driven by Arellano approached the checkpoint, the PNP searched the car and found the firearms. Arellano was apprehended and detained. He then explained the order of petitioner. Petitioner also explained that Arellano was only complying with the firearms ban, and that he was not a security officer or a bodyguard. Later, COMELEC issued Resolution No.92-0829 directing the filing of information against petitioner and Arellano for violation of the Omnibus Election Code, and for petitioner to show cause why he should not be disqualified from running for an elective position. Petitioner then questions the constitutionality of Resolution No. 2327. He argues that “gunrunning, using or transporting firearms or similar weapons” and other acts mentioned in the resolution are not within the provisions of the Omnibus Election Code. Thus, according to petitioner, Resolution No. 2327 is unconstitutional. The issue on the disqualification of petitioner from running in the elections was rendered moot when he lost his bid for a seat in Congress in the elections. Issue: Whether or Not petitioner can be validly prosecuted for instructing his driver to return the firearms issued to him on the basis of the evidence gathered from the warrant less search of his car Held: A valid search must be authorized by a search warrant issued by an appropriate authority. However, a warrantless search is not violative of the Constitution for as long as the vehicle is neither searched nor its occupants subjected to a body search, and the inspection of the vehicle is merely limited to a visual search. In the case at bar, the guns were not tucked in Arellano’s waist nor placed within his reach, as they were neatly packed in gun cases and placed inside a bag at the back of the car. Given these circumstances, the PNP could not have thoroughly searched the car lawfully as well as the package without violating the

constitutional injunction. Absent any justifying circumstance specifically pointing to the culpability of petitioner and Arellano, the search could not have been valid. Consequently, the firearms obtained from the warrantless search cannot be admitted for any purpose in any proceeding. It was also shown in the facts that the PNP had not informed the public of the purpose of setting up the checkpoint. Petitioner was also not among those charged by the PNP with violation of the Omnibus Election Code. He was not informed by the City Prosecutor that he was a respondent in the preliminary investigation. Such constituted a violation of his right to due process. Hence, it cannot be contended that petitioner was fully given the opportunity to meet the accusation against him as he was not informed that he was himself a respondent in the case. Thus, the warrantless search conducted by the PNP is declared illegal and the firearms seized during the search cannot be used as evidence in any proceeding against the petitioner. Resolution No. 92-0829 is unconstitutional, and therefore, set aside. Warrantless arrests Espano v. Court of Appeals, 288 SCRA 558 (1998) Facts: Pat. Pagilagan together with other police officers went to Zamora and Pandacan Streets, Manila to confirm reports of drug pushing in the area. They saw petitioner selling something to another person. After the alleged buyer left, they approached petitioner, identified themselves as policemen, and frisked him. The search yielded two plastic cellophane tea bags of marijuana. When asked if he had more marijuana, he replied that there was more in his house. The policemen went to his residence where they found ten more cellophane tea bags of marijuana. Petitioner was brought to the police headquarters where he was charged of possession of prohibited drugs. Issue: Whether or not the pieces of evidence were inadmissible Ruling: The Supreme Court held that Section 5 Rule 113 of the Rules of Court provides: “Arrest without warrant; when lawful – a peace officer or a private person may, without a warrant, arrest a person: When, in the presence, the person to be arrested has committed, is actually committing, or is attempting to commit an offense . . . “ Petitioner’s arrest falls squarely under the aforecited rule. He was caught in flagrante as a result of a buy bust operation conducted by police officers on the basis of information received regarding the illegal trade of drugs within the area. The police officer saw petitioner handling over something to an alleged buyer. After the buyer left, they searched him and discovered two cellophane of marijuana. His arrest was, therefore, lawful and the two cellophane bag of marijuana seized were admissible in evidence, being fruits of the crime. People vs. Del Rosario [G.R. No. 109633, July 20, 1994] A raiding team was organized to buy shabu from appellant and after buying from appellants’ house, the raiding team will implement search warrant. PO1 Luna with a companion proceeded to appellant's house to implement the search warrant. Barangay Capt. Maigue, Norma del Rosario and appellant witnessed the search at appellant's house. They found black canister constaining shabu and a paltik .22 caliber. At police station, the seized items were taped and initialed. In addition, the search warrant implemented by the raiding party authorized only the search and seizure of shabu and paraphernalia for the use thereof and no other. ISSUE: Whether or not police officers can seize items not mentioned in search warrant. RULING: NO. The Constitution itself (Section 2, Article III) and the Rules of Court (Section 3, Rule 126) specifically mandate that the search warrant must particularly describe the things to be seized. Thus, the search warrant was no authority for the police officers to seize the

firearm which was not mentioned, much less described with particularity, in the search warrant. Neither may it be maintained that the gun was seized in the course of an arrest, for as earlier observed, accused-appellant's arrest was far from regular and legal. Said firearm, having been illegally seized, the same is not admissible in evidence. The Constitution expressly ordains the exclusion in evidence of illegally seized articles. Umil vs. Ramos [G.R. No. 81567, July 9, 1990] Facts: on 1 February 1988, the Regional Intelligence Operations Unit of the Capital Command (RIOU-CAPCOM) received confidential information about a member of the NPA Sparrow Unit (liquidation squad) being treated for a gunshot wound at the St. Agnes Hospital in Roosevelt Avenue, Quezon City. Upon verification, it was found that the wounded person, who was listed in the hospital records as Ronnie Javelon, is actually Rolando Dural, a member of the NPA liquidation squad, responsible for the killing of two (2) CAPCOM soldiers the day before, or on 31 January 1988, in Macanining Street, Bagong Barrio, Caloocan City. In view of this verification, Rolando Dural was transferred to the Regional Medical Services of the CAPCOM, for security reasons. While confined thereat, or on 4 February 1988, Rolando Dural was positively identified by eyewitnesses as the gunman who went on top of the hood of the CAPCOM mobile patrol car, and fired at the two (2) CAPCOM soldiers seated inside the car. As a consequence of this positive identification, Rolando Dural was referred to the Caloocan City Fiscal who conducted an inquest and thereafter filed with the Regional Trial Court of Caloocan City an information charging Rolando Dural alias Ronnie Javelon with the crime of “Double Murder with Assault Upon Agents of Persons in Authority.” The case was docketed therein as Criminal Case No. C30112 and no bail was recommended. On 15 February 1988, the information was amended to include, as defendant, Bernardo Itucal, Jr. who, at the filing of the original information, was still unidentified. As to Rolando Dural, it clearly appears that he was not arrested while in the act of shooting the two (2) CAPCOM soldiers aforementioned. Nor was he arrested just after the commission of the said offense for his arrest came a day after the said shooting incident. Seemingly, his arrest without warrant is unjustified. However, Rolando Dural was arrested for being a member of the New Peoples Army (NPA), an outlawed subversive organization. Subversion being a continuing offense, the arrest of Rolando Dural without warrant is justified as it can be said that he was committing an offense when arrested. The crimes of rebellion, subversion, conspiracy or proposal to commit such crimes, and crimes or offenses committed in furtherance thereof or in connection therewith constitute direct assaults against the State and are in the nature of continuing crimes Issue: Whether an arrest and search warrant is required for the crimes of rebellion, subversion, conspiracy or proposal to commit such crimes, and crimes or offenses committed in furtherance thereof or in connection therewith constitute direct assaults against the State. Held: No, the claim of the petitioners that they were initially arrested illegally is, therefore, without basis in law and in fact. The crimes of insurrection or rebellion, subversion, conspiracy or proposal to commit such crimes, and other crimes and offenses committed in the furtherance, on the occasion thereof, or incident thereto, or in connection therewith under Presidential Proclamation No. 2045, are all in the nature of continuing offenses which set them apart from the common offenses, aside from their essentially involving a massive conspiracy of nationwide magnitude. Clearly then, the arrest of the herein detainees was well within the bounds of the law and existing jurisprudence in our jurisdiction. The arrest of persons involved in the rebellion whether as its fighting armed elements, or for committing non-violent acts but in furtherance of the rebellion, is more an act of capturing them in the course of an armed conflict, to quell the rebellion, than for the purpose of immediately prosecuting them in court for a statutory offense. The arrest, therefore, need not follow the usual procedure in the prosecution of offenses which requires the

determination by a judge of the existence of probable cause before the issuance of a judicial warrant of arrest and the granting of bail if the offense is bailable. Obviously, the absence of a judicial warrant is no legal impediment to arresting or capturing persons committing overt acts of violence against government forces, or any other milder acts but equally in pursuance of the rebellious movement. The arrest or capture is thus impelled by the exigencies of the situation that involves the very survival of society and its government and duly constituted authorities. If killing and other acts of violence against the rebels find justification in the exigencies of armed hostilities which is of the essence of waging a rebellion or insurrection, most assuredly so in case of invasion, merely seizing their persons and detaining them while any of these contingencies continues cannot be less justified. In this case, whatever may be said about the manner of his arrest, the fact remains that the defendant was actually in court in the custody of the law on March 29, when a complaint sufficient in form and substance was read to him. To this he pleaded not guilty. The trial followed, in which, and in the judgment of guilty pronounced by the court, we find no error. Whether, if there were irregularities in bringing him personally before the court, he could have been released on a writ of habeas corpus or now has a civil action for damages against the person who arrested him we need not inquire. It is enough to say that such irregularities are not sufficient to set aside a valid judgment rendered upon a sufficient complaint and after a trial free from error.

also assail the prejudicial publicity that attended their preliminary investigation.

Webb v. De Leon 247 SCRA 652

REASONS:1. The Court ruled that the DOJ Panel did not gravely abuse its discretion when it found probable cause against the petitioners. A probable cause needs only to rest on evidence showing that more likely than not, a crime has been committed and was committed by the suspects. Probable cause need not be based on clear and convincing evidence of guilt, neither on evidence establishing guilt beyond reasonable doubt and definitely, not on evidence establishing absolute certainty of guilt.

Hubert Webb was one of the accused in the high-profile case Vizconde massacre. Preliminary investigation was provided by NBI and the case was raffled to Judge Zosimo Escano who inhibited himself from the case for being employed with NBI before. His pair Judge Escano issued warrant of arrest to defendants. The case was re-raffled to Branch 274, presided by Judge Amelita Tolentino who issued new warrants of arrest. Webb and the others voluntarily surrendered. They files before the court petition of certiorari, prohibition and mandamus. They contend that (1) respondent Judges de Leon and Tolentino gravely abused their discretion when they failed to conduct a preliminary examination before issuing warrants of arrest against them: (2) the DOJ Panel likewise gravely abused its discretion in holding that there is probable cause to charge them with the crime of rape with homicide; (3) the DOJ Panel denied them their constitutional right to due process during their preliminary investigation; and (4) the DOJ Panel unlawfully intruded into judicial prerogative when it failed to charge Jessica Alfaro in the Information as an accused. FACTS: On June 19, 1994, the National Bureau of Investigation (NBI) filed with the Department of Justice a letter-complaint charging petitioners Hubert Webb, Michael Gatchalian, Antonio J. Lejano and six (6) other persons with the crime of Rape and Homicide of Carmela N. Vizconde, her mother Estrellita Nicolas-Vizconde, and her sister Anne Marie Jennifer in their home at Number 80 W. Vinzons, St., BF Homes Paranaque, Metro Manila on June 30, 1991. Forthwith, the Department of Justice formed a panel of prosecutors headed by Assistant Chief State Prosecutor Jovencio R. Zuno to conduct the preliminary investigation. ARGUMENTS: Petitioners fault the DOJ Panel for its finding of probable cause. They assail the credibility of Jessica Alfaro as inherently weak and uncorroborated due to the inconsistencies between her April 28, 1995 and May 22, 1995 sworn statements. They criticize the procedure followed by the DOJ Panel when it did not examine witnesses to clarify the alleged inconsistencies. Petitioners charge that respondent Judge Raul de Leon and, later, respondent Judge Amelita Tolentino issued warrants of arrest against them without conducting the required preliminary examination. Petitioners complain about the denial of their constitutional right to due process and violation of their right to an impartial investigation. They

ISSUES:1. Whether or not the DOJ Panel likewise gravely abused its discretion in holding that there is probable cause to charge them with the crime of rape and homicide 2. Whether or not respondent Jud”””ges de Leon and Tolentino gravely abused their discretion when they failed to conduct a preliminary examination before issuing warrants of arrest against them 3. Whether or not the DOJ Panel denied them their constitutional right to due process during their preliminary investigation 4. Whether or not the DOJ Panel unlawfully intruded into judicial prerogative when it failed to charge Jessica Alfaro in the information as an accused. HELD:1. NO. 2. NO. 3. NO. There is no merit in this contention because petitioners were given all the opportunities to be heard. 4. NO.

2. The Court ruled that respondent judges did not gravely abuse their discretion. In arrest cases, there must be a probable cause that a crime has been committed and that the person to be arrested committed it. Section 6 of Rule 112 simply provides that “upon filing of an information, the Regional Trial Court may issue a warrant for the accused. Clearly the, our laws repudiate the submission of petitioners that respondent judges should have conducted “searching examination of witnesses” before issuing warrants of arrest against them. 3. The DOJ Panel precisely ed the parties to adduce more evidence in their behalf and for the panel to study the evidence submitted more fully. 4. Petitioner’s argument lacks appeal for it lies on the faulty assumption that the decision whom to prosecute is a judicial function, the sole prerogative of the courts and beyond executive and legislative interference. In truth, the prosecution of crimes appertains to the executive department of government whose principal power and responsibility is to see that our laws are faithfully executed. A necessary component of this power is the right to prosecute their violators (See R.A. No. 6981 and section 9 of Rule 119 for legal basis). With regard to the inconsistencies of the sworn statements of Jessica Alfaro, the Court believes that these have been sufficiently explained and there is no showing that the inconsistencies were deliberately made to distort the truth. With regard to the petitioners’ complaint about the prejudicial publicity that attended their preliminary investigation, the Court finds nothing in the records that will prove that the tone and content of the publicity that attended the investigation of petitioners fatally infected the fairness and impartiality of the DOJ Panel. Petitioners cannot just rely on the subliminal effects of publicity on the sense of fairness of the DOJ Panel, for these are basically unbeknown and beyond knowing

People vs. Sucro [G.R. No. 93239, March 18, 1991] Facts: On March 21, 1989, Pat. Roy Fulgencio, a member of the INP, Kalibo, Aklan, was instructed by P/Lt. Vicente Seraspi, Jr. (Station Commander of the INP Kalibo, Aklan) to monitor the activities of appellant Edison Sucro, because of information gathered by Seraspi that Sucro was selling marijuana. As planned, at about 5:00 P.M. on said date, Pat. Fulgencio Positioned himself under the house of a certain Arlie Regalado at C. Quimpo Street. Adjacent to the house of Regalado, about 2 meters away, was a chapel. Thereafter, Pat. Fulgencio saw appellant enter the chapel, taking something which turned out later to be marijuana from the compartment of a cart found inside the chapel, and then return to the street where he handed the same to a buyer, Aldie Borromeo. After a while appellant went back to the chapel and again came out with marijuana which he gave to a group of persons. It was at this instance that Pat. Fulgencio radioed P/Lt. Seraspi and reported the activity going on. P/Lt. Seraspi instructed Pat. Fulgencio to continue monitoring developments. At about 6:30 P.M., Pat. Fulgencio again called up Seraspi to report that a third buyer later Identified as Ronnie Macabante, was transacting with appellant. At that point, the team of P/Lt. Seraspi proceeded to the area and while the police officers were at the Youth Hostel at Maagma St., Pat. Fulgencio told P/Lt. Seraspi to intercept Macabante and appellant. P/Lt. Seraspi and his team caught up with Macabante at the crossing of Mabini and Maagma Sts. in front of the Aklan Medical Center. Upon seeing the police, Macabante threw something to the ground which turned out to be a tea bag of marijuana. When confronted, Macabante readily admitted that he bought the same from appellant (Edison Sucro) in front of the chapel. The police team was able to overtake and arrest appellant at the corner of C. Quimpo and Veterans Sts. The police recovered 19 sticks and 4 teabags of marijuana from the cart inside the chapel and another teabag from Macabante, The teabags of marijuana were sent to the PC-INP Crime Laboratory Service, at Camp Delgado, Iloilo City for analysis. The specimens were all found positive of marijuana. Issue: Whether the police officer can arrest the accused without any arrest and search warrant when the latter committed the crime in front of the former. Held: Yes, Section 5, Rule 113 of the Rules on Criminal Procedure provides for the instances where arrest without warrant is considered lawful. The rule states that arrest without warrant, when lawful. Is when a peace officer or private person may, without warrant, arrest a person (a)When in his presence, the person to be arrested has committed, is actually committing, or is attempting to commit an offense; (b) When an offense has in fact just been committed, and he has personal knowledge of facts indicating that the person to be arrested has committed it; When an offense is committed in the presence or within the view of an officer, within the meaning of the rule authorizing an arrest without a warrant, when the officer sees the offense, although at a distance, or hears the disturbances created thereby and proceeds at once to the scene thereof. The records show that Fulgencio went to Arlie Regalado’s house at C. Quimpo Street to monitor the activities of the accused who was earlier reported to be selling marijuana at a chapel two (2) meters away from Regalado’s house. Fulgencio, within a distance of two meters saw Sucro conduct his nefarious activity. He saw Sucro talk to some persons, go inside the chapel, and return to them and exchange some things. These, Sucro did three times during the time that he was being monitored. Fulgencio would then relay the on-going transaction to P/Lt. Seraspi. People vs. Rodrigueza [G.R. No. 95902, February 4, 1992] Facts: NARCOM agents staged a buy-bust operation, after gaining information that there was an ongoing illegal traffic of prohibited drugs in Tagas, Albay. The participating agents were given money treated with ultraviolet powder. One of the agents went to said location, asked

for a certain Don. Thereafter, the Don, herein accused, met with him and “a certain object wrapped in a plastic” later identified as marijuana was given in exchange for P200. The agent went back to headquarters and made a report, based on which, a team was subsequently organized and a raid was conducted in the house of the father of the accused. During the raid, the NARCOM agents were able to confiscate dried marijuana leaves and a plastic syringe among others. There was no authorization by any search warrant. The accused was found positive of ultraviolet powder. The lower court, considering the evidences obtained and testimonies from the prosecution, found him guilty of violating the Dangerous Drugs Act of 1972 and sentenced him to reclusion perpetua. Issue: Whether or Not the lower court was correct in its judgment. Held: The NARCOM agents’ procedure in the entrapment of the accused failed to meet the qualification that the suspected drug dealer must be caught red-handed in the act of selling marijuana to a person posing as a buyer, since the operation was conducted after the actual exchange. Said raid also violated accused’ right against unreasonable search and seizure, as the situation did not fall in the circumstances wherein a search may be validly made even without a search warrant, i.e. when the search is incidental to a lawful arrest; when it involves prohibited articles in plain view. The NARCOM agents could not have justified their act by invoking the urgency and necessity of the situation because the testimonies of the prosecution witnesses reveal that the place had already been put under surveillance for quite some time. Had it been their intention to conduct the raid, then they should, because they easily could, have first secured a search warrant during that time. The Court further notes the confusion and ambiguity in the identification of the confiscated marijuana leaves and other prohibited drug paraphernalia presented as evidence against appellant: CIC Taduran, who acted as the poseur buyer, testified that appellant sold him 100 grams of dried marijuana leaves wrapped in a plastic bag. Surprisingly, and no plausible explanation has been advanced therefor, what were submitted to and examined by the PCCL and thereafter utilized as evidence against the appellant were the following items: One (1) red and white colored plastic bag containing the following: Exh. "A"—Thirty (30) grams of suspected dried marijuana fruiting tops contained inside a transparent plastic bag. Exh. "B"— Fifty (50) grams of suspected dried marijuana leaves and seeds contained inside a white colored plastic labelled "Robertson". Exh. "C"— Four (4) aluminum foils each containing suspected dried marijuana fruiting tops having a total weight of seven grams then further wrapped with a piece of aluminum foil. Exh. "D"— Five (5) small transparent plastic bags each containing suspected dried marijuana fruiting tops having a total weight of seventeen grams. Exh. "E"— One plastic syringe. Evidently, these prohibited articles were among those confiscated during the so-called follow-up raid in the house of Rodrigueza’s father. The unanswered question then arises as to the identity of the marijuana leaves that became the basis of appellant's conviction. In People vs. Rubio, this Court had the occasion to rule that the plastic bag and the dried marijuana leaves contained therein constitute the corpus delicti of the crime. As such, the existence thereof must be proved with certainty and conclusiveness. Failure to do so would be fatal to the cause of the prosecution. Conviction is reversed and set aside and accused is acquitted. Go. vs. Court of Appeals [G.R. No. 101837, February 11, 1992] Facts: Rolito Go while traveling in the wrong direction on a one-way street, nearly bumped Eldon Maguan’s car. Go alighted from his car,

shot Maguan and left the scene. A security guard at a nearby restaurant was able to take down petitioner’s car plate number. The police arrived shortly thereafter at the scene of the shooting. A manhunt ensued. Six days after, petitioner presented himself before the San Juan Police Station to verify news reports that he was being hunted by the police; he was accompanied by two (2) lawyers. The police forthwith detained him. An eyewitness to the shooting, who was at the police station at that time, positively identified petitioner as the gunman. Petitioner posted bail, the prosecutor filed the case to the lower court, setting and commencing trial without preliminary investigation. Prosecutor reasons that the petitioner has waived his right to preliminary investigation as bail has been posted and that such situation, that petitioner has been arrested without a warrant lawfully, falls under Section 5, Rule 113 and Section 7, Rule 112 of The 1985 Rules of Criminal Procedure which provides for the rules and procedure pertaining to situations of lawful warrantless arrests. Petitioner argues that he was not lawfully arrested without warrant because he went to the police station six (6) days after the shooting which he had allegedly perpetrated. Thus, petitioner argues, the crime had not been “just committed” at the time that he was arrested. Moreover, none of the police officers who arrested him had been an eyewitness to the shooting of Maguan and accordingly none had the “personal knowledge” required for the lawfulness of a warrantless arrest. Since there had been no lawful warrantless arrest, Section 7, Rule 112 of the Rules of Court which establishes the only exception to the right to preliminary investigation, could not apply in respect of petitioner. Issue/s:Whether or not a lawful warrantless arrest had been effected by the San Juan Police in respect of petitioner Go; Whether petitioner had effectively waived his right to preliminary investigation Held:1. No. The Court does not believe that the warrantless “arrest” or detention of petitioner in the instant case falls within the terms of Section 5 of Rule 113 of the 1985 Rules on Criminal Procedure which provides as follows: “Sec. 5. Arrest without warrant; when lawful. — A peace officer or a private person may, without a warrant, arrest a person; (a) When, in his presence, the person to be arrested has committed, is actually committing, or is attempting to commit an offense; (b) When an offense has in fact just been committed, and he has personal knowledge of facts indicating that the person to be arrested has committed it; and (c) When the person to be arrested is a prisoner who has escaped from a penal establishment or place where he is serving final judgment or temporarily confined while his case is pending, or has escaped while being transferred from one confinement to another. In cases falling under paragraphs (a) and (b) hereof, the person arrested without a warrant shall be forthwith delivered to the nearest police station or jail, and he shall be proceeded against in accordance with Rule 112, Section 7.” Petitioner’s “arrest” took place six (6) days after the shooting of Maguan. The “arresting” officers obviously were not present, within the meaning of Section 5(a), at the time petitioner had allegedly shot Maguan. Neither could the “arrest” effected six (6) days after the shooting be reasonably regarded as effected “when [the shooting had] in fact just been committed” within the meaning of Section 5 (b). Moreover, none of the “arresting” officers had any “personal knowledge” of facts indicating that petitioner was the gunman who had shot Maguan. The information upon which the police acted had been derived from statements made by alleged eyewitnesses to the shooting

— one stated that petitioner was the gunman; another was able to take down the alleged gunman’s car’s plate number which turned out to be registered in petitioner’s wife’s name. That information did not, however, constitute “personal knowledge.” It is thus clear to the Court that there was no lawful warrantless arrest of petitioner within the meaning of Section 5 of Rule 113. 2. No. In the circumstances of this case, the Court does not believe that by posting bail, petitioner had waived his right to preliminary investigation. In People v. Selfaison, the Court held that appellants there had waived their right to preliminary investigation because immediately after their arrest, they filed bail and proceeded to trial “without previously claiming that they did not have the benefit of a preliminary investigation.” In the instant case, petitioner Go asked for release on recognizance or on bail and for preliminary investigation in one omnibus motion. He had thus claimed his right to preliminary investigation before respondent Judge approved the cash bond posted by petitioner and ordered his release on 12 July 1991. Accordingly, the Court cannot reasonably imply waiver of preliminary investigation on the part of petitioner. In fact, when the Prosecutor filed a motion in court asking for leave to conduct preliminary investigation, he clearly if impliedly recognized that petitioner’s claim to preliminary investigation was a legitimate one. Posadas vs. Court of Appeals [G.R. No. 89139, August 2, 1990] FACTS: While conducting a surveillance, the two policemen spotted petitioner carrying a “buri” bag and acting suspiciously. He attempted to flee when the policemen approached him and identified themselves but his attempt was thwarted. Found inside the “buri” bag were one (1) caliber .38 Smith & Wesson revolver, a smoke (tear gas) grenade, and two (2) rounds live ammunitions for a .22 caliber gun. Petitioner failed to show the necessary license or authority to possess firearms and ammunitions found in his possession. He was subsequently prosecuted for and found guilty of illegal possession of firearms and ammunitions ISSUE(S):Whether or not there was probable cause to justify the warrantless search and arrest of the petitioner. RULING:YES. The probable cause is that when the petitioner acted suspiciously and attempted to flee with the buri bag there was a probable cause that he was concealing something illegal in the bag and it was the right and duty of the police officers to inspect the same. People v. Mengote, G.R. No. 87059, June 22, 1992 Facts: a telephone call was by Western Police district that here were three suspicious-looking persons at the corner of Juan Luna and North Bay Boulevard in Tondo Manila. A surveillanve team of plainclothesmen was dispatch to the place. They saw two men “looking from side to side” one of whom is holding his abdomen. They approached these persons and identified themselves as policemen, whereupon the two tried to run away but were unable to escape because the other lawmen had surrounded them. The suspects were then searched. One of them, who turned out to be the accused was found with a .38 caliber Smith and Wesson revolver with six live bullets in the chamber. His companion had a fan knife. The weapons were taken from them. Issue: Whether or not the accused constitutional right against unreasonable search and seizure is violated Ruling: The Supreme court held that par(a) section 5 Rule 113 of rules of court requires that a person be arrested 1 After he has committed or while he is actually committing or is at least attempting to commit an offense 2 In the presence of the arresting officer.

These requirements have not been established in the case at bar at bar. At the time of the arrest in question, the accused was merely “looking from side to side” and “holding his abdomen”. There was apparently no offense that has just been committed or was being actually committed or at least being attempted by Mengote in their presence.

he will be so declared even if his defense is weak as long as the prosecution is not strong enough to convict him.

Par. B. is no less applicable because it’s no less stringent requirements have not been satisfied. The prosecution has not shown that at the time of arrest an offense had in fact just been committed and that the arresting officer had personal knowledge of facts indicating that Mengote had committed it. All they had was hearsay information from the telephone caller, and about a crime that had yet to be committed.

Ramirez vs. Court of Appeals [G.R. No. 93833, September 28 1995]

People vs. Aminnudin, 163 SCRA 402 (1988) Facts: The PC (Philippine Constabulary) officer received a tip from one of their informers that the accused was on board a vessel bound for Iloilo City and was carrying marijuana. He was identified by name. Acting on this tip, they waited for him in the evening and approached him as he descended from the gangplank after the informer pointed at him. They detained him and inspected the bag he was carrying. It was found to contained three kilos of what were later analyzed as marijuana leaves by the NBI forensic examiner. On the basis of the finding, the corresponding charge was then filed against Aminnudin. Wilcon 9 The PC officers who were in fact waiting for him simply accosted him, inspected his bag and finding what looked liked marijuana leaves took him to their headquarters for investigation. The two bundles of suspect articles were confiscated from him and later taken to the NBI laboratory for examination. When they were verified as marijuana leaves, an information for violation of the Dangerous Drugs Act was filed against him. However, and it is Aminnudin’s claim that he was arrested and searched without warrant, making the marijuana allegedly found in his possession inadmissible in evidence against him under the Bill of Rights. Issue: Whether the accused was caught in flagrante delicto hence justifies the warrantless arrest Held: No, the accused-appellant was not caught in flagrante nor was a crime about to be committed or had just been committed to justify the warrantless arrest allowed under the Rules of Court. The present case presented no such urgency. It is clear that they had at least two days within which they could have obtained a warrant to arrest and search Aminnudin who was coming to Iloilo on the M/V Wilcon 9. His name was known. The vehicle was Identified. The date of its arrival was certain. And from the information they had received, they could have persuaded a judge that there was probable cause, indeed, to justify the issuance of a warrant. Yet they did nothing. No effort was made to comply with the law. The Bill of Rights was ignored altogether because the PC lieutenant who was the head of the arresting team, had determined on his own authority that a “search warrant was not necessary.” In the case at bar, the accused-appellant was not, at the moment of his arrest, committing a crime nor was it shown that he was about to do so or that he had just done so. What he was doing was descending the gangplank of the M/V Wilcon 9 and there was no outward indication that called for his arrest. To all appearances, he was like any of the other passengers innocently disembarking from the vessel. It was only when the informer pointed to him as the carrier of the marijuana that he suddenly became suspect and so subject to apprehension. It was the furtive finger that triggered his arrest. The Identification by the informer was the probable cause as determined by the officers (and not a judge) that authorized them to pounce upon Aminnudin and immediately arrest him. While this is not to say that the accused-appellant is innocent, for indeed his very own words suggest that he is lying, that fact alone does not justify a finding that he is guilty. The constitutional presumption is that he is innocent, and

Rules 113 and 126 of the Revised Rules of Court Section 3 - Privacy of Communication and Correspondence

Facts: A civil case damages was filed by petitioner in the RTC alleging that the private respondent in a confrontation in the latter’s office, allegedly vexed, insulted and humiliated her in a “hostile and furious mood” and in a manner offensive to petitioner’s dignity and personality,” contrary to morals, good customs and public policy.” In support of her claim, petitioner produced a verbatim transcript of the event and sought moral damages, attorney’s fees and other expenses of litigation in the amount of P610,000.00, in addition to costs, interests and other reliefs awardable at the trial court’s discretion. The transcript on which the civil case was based was culled from a tape recording of the confrontation made by petitioner.

As a result of petitioner’s recording of the event and alleging that the said act of secretly taping the confrontation was illegal, private respondent filed a criminal case before the Regional Trial Court of Pasay City for violation of Republic Act 4200, entitled “An Act to prohibit and penalize wire tapping and other related violations of private communication, and other purposes. Petitioner filed a Motion to Quash the Information on the ground that the facts charged do not constitute an offense, particularly a violation of R.A. 4200. the RTC granted the Motion. From the RTC’s order, the private respondent filed a Petition for Review on Certiorari with this Court, which forthwith referred the case to the CA. Respondent CA declared the RTC’s order null and void, and holding that the allegations sufficiently constitute an offense punishable under Section 1 of R.A. 4200. Petitioner filed a MR which the CA denied. Hence, the instant petition.

Issue: Whether the recording of a “Private Conversation” without the consent of both of the party is a violation of R.A. 4200. Held: Yes, Section 1 of R.A. 4200 entitled, ” An Act to Prohibit and Penalized Wire Tapping and Other Related Violations of Private Communication and Other Purposes,” provides that it shall be unlawful for any person, not being authorized by all the parties to any private communication or spoken word, to tap any wire or cable, or by using any other device or arrangement, to secretly overhear, intercept, or record such communication or spoken word by using a device commonly known as a dictaphone or dictagraph or detectaphone or walkie-talkie or tape recorder, or however otherwise described. The aforestated provision clearly and unequivocally makes it illegal for any person, not authorized by all the parties to any private communication to secretly record such communication by means of a tape recorder. The law makes no distinction as to whether the party sought to be penalized by the statute ought to be a party other than or different from those involved in the private communication. The statute’s intent to penalize all persons unauthorized to make such recording is underscored by the use of the qualifier “any”. Consequently, as respondent Court of Appeals correctly concluded, “even a (person) privy to a communication who records his private conversation with another without the knowledge of the latter (will) qualify as a violator. The unambiguity of the express words of the provision, therefore plainly supports the view held by the respondent court that the provision seeks to penalize even those privy to the private communications. Where the law makes no distinctions, one does not distinguish.

Zulueta vs. Court of Appeals [G.R. No. 107383, February 20 1996] Facts: Cecilia Zulueta is the Petitioner who offset the private papers of his husband Dr. Alfredo Martin. Dr. Martin is a doctor of medicine while he is not in his house His wife took the 157 documents consisting of diaries, cancelled check, greeting cards, passport and photograph, private respondents between her Wife and his alleged paramours, by means of forcibly opened the drawers and cabinet. Cecilia Zulueta filed the papers for the evidence of her case of legal separation and for disqualification from the practice of medicine against her husband. Dr. Martin brought the action for recovery of the documents and papers and for damages against Zulueta, with the Regional Trial Court of Manila, Branch X. the trial court rendered judgment for Martin, declaring him the capital/exclusive owner of the properties described in paragraph 3 of Martin’s Complaint or those further described in the Motion to Return and Suppress and ordering Zulueta and any person acting in her behalf to a immediately return the properties to Dr. Martin and to pay him P5,000.00, as nominal damages; P5,000.00, as moral damages and attorney’s fees; and to pay the costs of the suit. On appeal, the Court of Appeals affirmed the decision of the Regional Trial Court. Zulueta filed the petition for review with the Supreme Court.

Issue: The papers and other materials obtained from forcible entrusion and from unlawful means are admissible as evidence in court regarding marital separation and disqualification from medical practice.

Ruling/Held: The documents and papers are inadmissible in evidence. The constitutional injunction declaring “the privacy of communication and correspondence to be inviolable is no less applicable simply because it is the wife who thinks herself aggrieved by her husband’s infidelity, who is the party against whom the constitutional provision is to be enforced. The only exception to the prohibition in the Constitution is if there is a lawful order from a court or when public safety or order requires otherwise, as prescribed by law. Any violation of this provision renders the evidence obtained inadmissible for any purpose in any proceeding. The intimacies between husband and wife do not justify any one of them in breaking the drawers and cabinets of the other and in ransacking them for any telltale evidence of marital infidelity. A person, by contracting marriage, does not shed his/her integrity or his right to privacy as an individual and the constitutional protection is ever available to him or to her. The law insures absolute freedom of communication between the spouses by making it privileged. Neither husband nor wife may testify for or against the other without the consent of the affected spouse while the marriage subsists. Neither may be examined without the consent of the other as to any communication received in confidence by one from the other during the marriage, save for specified exceptions. But one thing is freedom of communication; quite another is a compulsion for each one to share what one knows with the other. And this has nothing to do with the duty of fidelity that each owes to the other.

policeman Navarro who was then having drinks outside the headquarters, lead to a fisticuffs. The victim was hit with the handle of the accused's gun below the left eyebrow, followed by a fist blow, resulted the victim to fell and died under treatment. The exchange of words was recorded on tape, specifically the frantic exclamations made by Navarro after the altercation that it was the victim who provoked the fight. During the trial, Jalbuena, the other media man , testified. Presented in evidence to confirm his testimony was a voice recording he had made of the heated discussion at the police station between the accused police officer Navarro and the deceased, Lingan, which was taken without the knowledge of the two.

ISSUES: 1. Whether or not the voice recording is admissible in evidence in view of RA 4200, which prohibits wire tapping.

2. Whether the mitigating circumstances of sufficient provocation or threat on the part of the offended party and lack of intention to commit so grave a wrong may be appreciated in favor of the accused.

HELD: 1. The answer is affirmative, the tape is admissible in view of RA 4200, which prohibits wire tapping. Jalbuena's testimony is confirmed by the voice recording he had made.

The law prohibits the overhearing, intercepting, or recording of private communications (Ramirez v Cpourt of Appeals, 248 SCRA 590 [1995]). Snce the exchange between petitioner Navarro and Lingan was not private, its tape recording is not prohibited. Tape Recording in the present case is not prohibited SECTION 1. It shall be unlawful for any person, not being authorized by all the parties to any private communication or spoken word, to tap any wire or cable, or by using any other device or arrangement, to secretly overhear, intercept, or record such communication or spoken word by using a device commonly known as a dictaphone or dictagraph or detectaphone or walkie-talkie or tape-recorder, or however otherwise described:

It shall also be unlawful for any person, be he a participant or not in the act or acts penalized in the next preceding sentence, to knowingly possess any tape record, wire record, disc record, or any other such record, or copies thereof, of any communication or spoken word secured either before or after the effective date of this Act in the manner prohibited by this law; or to replay the same for any other person or persons; or to communicate the contents thereof, either verbally or in writing, or to furnish transcriptions thereof, whether complete or partial, to any other person: Provided, That the use of such record or any copies thereof as evidence in any civil, criminal investigation or trial of offenses mentioned in section 3 hereof, shall not be covered by this prohibition.

Navarro vs. Court of Appeals [G.R. NO. 121087, August 26, 1999]

....

FACTS:

SEC. 4. Any communication or spoken word, or the existence, contents, substance, purport, effect, or meaning of the same or any part thereof, or any information therein contained obtained or secured by any person in violation of the preceding sections of this Act shall not be admissible in evidence in any judicial, quasi-judicial, legislative or administrative hearing or investigation.

Two local media men, Stanley Jalbuena, Enrique Lingan, in Lucena City wnet to the police station to report alledged indecent show in one of the night establishment shows in the City. At the station, a heated confrontation followed between victim Lingan and accused

Thus, the law prohibits the overhearing, intercepting, or recording of private communications. Since the exchange between petitioner Navarro and Lingan was not private, its tape recording is not prohibited. Authentication of a voice recording

"Sec. 3. (1) The privacy of communication and correspondence shall be inviolable except upon lawful order of the court, or when public safety or order requires otherwise as prescribed by law."

Other facets of the right to privacy are protected in various provisions of the Bill of Rights, viz:[34]

Nor is there any question that it was duly authenticated. A voice recording is authenticated by the testimony of a witness (1)

that he personally recorded the conversation;

(2)

that the tape played in court was the one he recorded; and

(3) that the voices on the tape are those of the persons such are claimed to belong. In the instant case, Jalbuena testified that he personally made the voice recording; that the tape played in court was the one he recorded; and that the speakers on the tape were petitioner Navarro and Lingan. A sufficient foundation was thus laid for the authentication of the tape presented by the prosecution.

"Sec. 1. No person shall be deprived of life, liberty, or property without due process of law, nor shall any person be denied the equal protection of the laws.

Sec. 2. The right of the people to be secure in their persons, houses, papers, and effects against unreasonable searches and seizures of whatever nature and for any purpose shall be inviolable, and no search warrant or warrant of arrest shall issue except upon probable cause to be determined personally by the judge after examination under oath or affirmation of the complainant and the witnesses he may produce, and particularly describing the place to be searched and the persons or things to be seized.

Ople vs. Torres {GR No, 127685, July 23, 1998] FACTS: A.O. No. 308 was issued by President Fidel V. Ramos on December 12, 1996 for the Adoption of a National Computerized Identification Reference System. It was published in four newspapers of general circulation on January. Petitioner filed the instant petition against respondents, on the grounds that: 1. it is a usurpation of the power of Congress to legislate, 2. it impermissibly intrudes on our citizenry’s protected zone of privacy. ISSUE: Whether there is a violation of the Right to Privacy as enshrined in the Bill of Rights. HELD: The essence of privacy is the “right to be left alone.” The right to privacy as such is accorded recognition independently of its identification with liberty; in itself, it is fully deserving of constitutional protection. The Court prescind from the premise that the right to privacy is a fundamental right guaranteed by the Constitution, hence, it is the burden of government to show that A.O. No. 308 is justified by some compelling state interest and that it is narrowly drawn. A.O. No. 308 is predicated on two considerations: 1. the need to provides our citizens and foreigners with the facility to conveniently transact business with basic service and social security providers and other government instrumentalities and ; 2. the need to reduce, if not totally eradicate, fraudulent transactions and misrepresentations by persons seeking basic services. It is debatable whether the interests are compelling enough to warrant the issuance of the said order. The broadness, vagueness, and overbreadth of A.O. No. 308 which if implemented will put our people’s right to privacy in clear and present danger. In the case at bar, the threat comes from which by issuing A.O. No. 308 pressures the people to surrender their privacy by giving information about themselves on the pretext that it will facilitate delivery of basic services. Petition is granted. A.O. No. 308 is unconstitutional. Doctrines: Section 3(1) of the Bill of Rights:

x x x.

Sec. 6. The liberty of abode and of changing the same within the limits prescribed by law shall not be impaired except upon lawful order of the court. Neither shall the right to travel be impaired except in the interest of national security, public safety, or public health, as may be provided by law.

x x x.

Sec. 8. The right of the people, including those employed in the public and private sectors, to form unions, associations, or societies for purposes not contrary to law shall not be abridged.

Sec. 17. No person shall be compelled to be a witness against himself."

Zones of privacy are likewise recognized and protected in our laws. The Civil Code provides that "[e]very person shall respect the dignity, personality, privacy and peace of mind of his neighbors and other persons" and punishes as actionable torts several acts by a person of meddling and prying into the privacy of another.[35] It also holds a public officer or employee or any private individual liable for damages for any violation of the rights and liberties of another person,[36] and recognizes the privacy of letters and other private communications.[37] The Revised Penal Code makes a crime the violation of secrets by an officer,[38] the revelation of trade and industrial secrets,[39] and trespass to dwelling.[40] Invasion of privacy is an offense in special laws like the Anti-Wiretapping Law,[41] the Secrecy of Bank Deposit Act[42] and the Intellectual Property Code.[43] The Rules of Court on privileged communication likewise recognize the privacy of certain information.[44] Unlike the dissenters, we prescind from the premise that the right to privacy is a fundamental right guaranteed by the Constitution, hence, it is the burden of government to show that A.O. No. 308 is justified

by some compelling state interest and that it is narrowly drawn. A.O. No. 308 is predicated on two considerations: (1) the need to provide our citizens and foreigners with the facility to conveniently transact business with basic service and social security providers and other government instrumentalities and (2) the need to reduce, if not totally eradicate, fraudulent transactions and misrepresentations by persons seeking basic services. It is debatable whether these interests are compelling enough to warrant the issuance of A.O. No. 308. But what is not arguable is the broadness, the vagueness, the overbreadth of A.O. No. 308 which if implemented will put our people's right to privacy in clear and present danger. The reasonableness of a person's expectation of privacy depends on a two-part test: (1) whether by his conduct, the individual has exhibited an expectation of privacy; and (2) whether this expectation is one that society recognizes as reasonable.[67] The factual circumstances of the case determines the reasonableness of the expectation.[68] However, other factors, such as customs, physical surroundings and practices of a particular activity, may serve to create or diminish this expectation.[69] The use of biometrics and computer technology in A.O. No. 308 does not assure the individual of a reasonable expectation of privacy.[70] As technology advances, the level of reasonably expected privacy decreases.[71] The measure of protection granted by the reasonable expectation diminishes as relevant technology becomes more widely accepted.[72] The security of the computer data file depends not only on the physical inaccessibility of the file but also on the advances in hardware and software computer technology. A.O. No. 308 is so widely drawn that a minimum standard for a reasonable expectation of privacy, regardless of technology used, cannot be inferred from its provisions. Pollo vs Constantino-David G.R. No. 181881, October 18 2011 Facts: Respondent CSC Chair Constantino-David received an anonymous letter complaint alleging of an anomaly taking place in the Regional Office of the CSC. The respondent then formed a team and issued a memo directing the team “to back up all the files in the computers found in the Mamamayan Muna (PALD) and Legal divisions.” Several diskettes containing the back-up files sourced from the hard disk of PALD and LSD computers were turned over to Chairperson David. The contents of the diskettes were examined by the CSC’s Office for Legal Affairs (OLA). It was found that most of the files in the 17 diskettes containing files copied from the computer assigned to and being used by the petitioner, numbering about 40 to 42 documents, were draft pleadings or lettersin connection with administrative cases in the CSC and other tribunals. On the basis of this finding, Chairperson David issued the Show-Cause Order, requiring the petitioner, who had gone on extended leave, to submit his explanation or counter-affidavit within five days from notice. In his Comment, petitioner denied the accusations against him and accused the CSC Officials of “fishing expedition” when they unlawfully copied and printed personal files in his computer. He was charged of violating R.A. No. 6713 (Code of Conduct and Ethical Standards for Public Officials and Employees). He assailed the formal charge and filed an Omnibus Motion ((For Reconsideration, to Dismiss and/or to Defer) assailing the formal charge as without basis having proceeded from an illegal search which is beyond the authority of the CSC Chairman, such power pertaining solely to the court. The CSC denied the omnibus motion and treated the motion as the petitioner’s answer to the charge. In view of the absence of petitioner and his counsel, and upon the motion of the prosecution, petitioner was deemed to have waived his right to the formal investigation which then proceeded ex parte. The petitioner was dismissed from service. He filed a petition to the CA which was dismissed by the latter on the ground that it found no

grave abuse of discretion on the part of the respondents. He filed a motion for reconsideration which was further denied by the appellate court. Hence, this petition. Issue: WON the search conducted by the CSC on the computer of the petitioner constituted an illegal search and was a violation of his constitutional right to privacy Ruling: The search conducted on his office computer and the copying of his personal files was lawful and did not violate his constitutional right. Ratio Decidendi: In this case, the Court had the chance to present the cases illustrative of the issue raised by the petitioner. Katz v. United States 389 U.S. 437 (1967), the US Supreme Court held that the act of FBI agents in electronically recording a conversation made by petitioner in an enclosed public telephone booth violated his right to privacy and constituted a “search and seizure”. Because the petitioner had a reasonable expectation of privacy in using the enclosed booth to make a personal telephone call, the protection of the Fourth Amendment extends to such area. Moreso, the concurring opinion of Mr. Justice Harlan noted that the existence of privacy right under prior decisions involved a two-fold requirement: first, that a person has exhibited an actual (subjective) expectation of privacy; and second, that the expectation be one that society is prepared to recognize as reasonable (objective). Mancusi v. DeForte 392 U.S. 364, 88 S.Ct. 2120, 20 L.Ed2d 1154 (1968),thus “recognized that employees may have a reasonable expectation of privacy against intrusions by police.” O’Connor v. Ortega 480 U.S. 709 (1987), the Court categorically declared that “[i]ndividuals do not lose Fourth Amendment rights merely because they work for the government instead of a private employer.” In O’Connor the Court recognized that “special needs” authorize warrantless searches involving public employees for workrelated reasons. The Court thus laid down a balancing test under which government interests are weighed against the employee’s reasonable expectation of privacy. This reasonableness test implicates neither probable cause nor the warrant requirement, which are related to law enforcement. Social Justice Society (SJS) v. Dangerous Drugs Board G.R. Nos. 157870, 158633 and 161658, November 3, 2008, 570 SCRA 410, 427, (citing Ople v. Torres, G.R. No. 127685, July 23, 1998, 293 SCRA 141, 169), recognized the fact that there may be such legitimate intrusion of privacy in the workplace. The Court ruled that the petitioner did not have a reasonable expectation of privacy in his office and computer files. As to the second point of inquiry, the Court answered in the affirmative. The search authorized by the CSC Chair, the copying of the contents of the hard drive on petitioner’s computer reasonable in its inception and scope. The Court noted that unlike in the case of Anonymous LetterComplaint against Atty. Miguel Morales, Clerk of Court, Metropolitan Trial Court of Manila A.M. Nos. P-08-2519 and P-08-2520, November 19, 2008, 571 SCRA 361, the case at bar involves the computer from which the personal files of the petitioner were retrieved is a government-issued computer, hence government property the use of which the CSC has absolute right to regulate and monitor. Doctrines: Existence of privacy right under prior decisions involved a two-fold requirement: first, that a person has exhibited an actual (subjective) expectation of privacy; and second, that the expectation be one that society is prepared to recognize as reasonable (objective). the correct analysis has two steps: first, because some government offices may be so open to fellow employees or the public that no

expectation of privacy is reasonable, a court must consider [t]he operational realities of the workplace in order to determine whether an employees Fourth Amendment rights are implicated; and next, where an employee has a legitimate privacy expectation, an employers intrusion on that expectation for noninvestigatory, work-related purposes, as well as for investigations of work-related misconduct, should be judged by the standard of reasonableness under all the circumstances.[36] On the matter of government employees reasonable expectations of privacy in their workplace, OConnor teaches: x x x Public employees expectations of privacy in their offices, desks, and file cabinets, like similar expectations of employees in the private sector, may be reduced by virtue of actual office practices and procedures, or by legitimate regulation. x x x The employees expectation of privacy must be assessed in the context of the employment relation. An office is seldom a private enclave free from entry by supervisors, other employees, and business and personal invitees. Instead, in many cases offices are continually entered by fellow employees and other visitors during the workday for conferences, consultations, and other work-related visits. Simply put, it is the nature of government offices that others such as fellow employees, supervisors, consensual visitors, and the general public may have frequent access to an individuals office. We agree with JUSTICE SCALIA that [c]onstitutional protection against unreasonable searches by the government does not disappear merely because the government has the right to make reasonable intrusions in its capacity as employer, x x x but some government offices may be so open to fellow employees or the public that no expectation of privacy is reasonable. x x x Given the great variety of work environments in the public sector, the question of whether an employee has a reasonable expectation of privacy must be addressed on a case-by-case basis. In sum, we conclude that the special needs, beyond the normal need for law enforcement make theprobable-cause requirement impracticable, x x x for legitimate, work-related noninvestigatory intrusions as well as investigations of work-related misconduct. A standard of reasonableness will neither unduly burden the efforts of government employers to ensure the efficient and proper operation of the workplace, nor authorize arbitrary intrusions upon the privacy of public employees. We hold, therefore, that public employer intrusions on the constitutionally protected privacy interests of government employees for noninvestigatory, work-related purposes, as well as for investigations of work-related misconduct, should be judged by the standard of reasonableness under all the circumstances. Under this reasonableness standard, both the inception and the scope of the intrusion must be reasonable: Determining the reasonableness of any search involves a twofold inquiry: first, one must consider whether theaction was justified at its inception, x x x ; second, one must determine whether the search as actually conducted was reasonably related in scope to the circumstances which justified the interference in the first place, x x x Ordinarily, a search of an employees office by a supervisor will be justified at its inception when there are reasonable grounds for suspecting that the search will turn up evidence that the employee is guilty of work-related misconduct, or that the search is necessary for a noninvestigatory work-related purpose such as to retrieve a needed file. x x x The search will be permissible in its scope when the measures adopted are reasonably related to the objectives of the search and not excessively intrusive in light of the nature of the [misconduct] In this inquiry, the relevant surrounding circumstances to consider include (1) the employees relationship to the item seized; (2) whether the item was in the immediate control of the employee when it was seized; and (3) whether the employee took actions to maintain his privacy in the item. These factors are relevant to both the subjective and objective prongs of the reasonableness inquiry, and we consider

the two questions together.[44] Thus, where the employee used a password on his computer, did not share his office with co-workers and kept the same locked, he had a legitimate expectation of privacy and any search of that space and items located therein must comply with the Fourth Amendment.[45]

We answer the first in the negative. Petitioner failed to prove that he had an actual (subjective) expectation of privacy either in his office or government-issued computer which contained his personal files. Petitioner did not allege that he had a separate enclosed office which he did not share with anyone, or that his office was always locked and not open to other employees or visitors. Neither did he allege that he used passwords or adopted any means to prevent other employees from accessing his computer files. On the contrary, he submits that being in the public assistance office of the CSC-ROIV, he normally would have visitors in his office like friends, associates and even unknown people, whom he even allowed to use his computer which to him seemed a trivial request. He described his office as full of people, his friends, unknown people and that in the past 22 years he had been discharging his functions at the PALD, he is personally assisting incoming clients, receiving documents, drafting cases on appeals, in charge of accomplishment report, in the absence of allegation or proof of the aforementioned factual circumstances, that petitioner had at least a subjective expectation of privacy in his computer as he claims, such is negated by the presence of policy regulating the use of office computers, as in Simons. The CSC in this case had implemented a policy that put its employees on notice that they have no expectation of privacy in anything they create, store, send or receive on the office computers, and that the CSC may monitor the use of the computer resources using both automated or human means. This implies that on-the-spot inspections may be done to ensure that the computer resources were used only for such legitimate business purposes. One of the factors stated in OConnor which are relevant in determining whether an employees expectation of privacy in the workplace is reasonable is the existence of a workplace privacy policy Vivares vs. St. Theresa's College G.R. No. 202666, September 29 2014 FACTS:Julia and Julienne, both minors, were graduating high school students at St. Theresa’s College (STC), Cebu City. Sometime in January 2012, while changing into their swimsuits for a beach party they were about to attend, Julia and Julienne, along with several others, took digital pictures of themselves clad only in their undergarments. These pictures were then uploaded by Angela on her Facebook profile. At STC, Mylene Escudero, a computer teacher at STC’s high school department, learned from her students that some seniors at STC posted pictures online, depicting themselves from the waist up, dressed only in brassieres. Escudero then asked her students if they knew who the girls in the photos are. In turn, they readily identified Julia and Julienne, among others. Using STC’s computers, Escudero’s students logged in to their respective personal Facebook accounts and showed her photos of the identified students, which include: (a) Julia and Julienne drinking hard liquor and smoking cigarettes inside a bar; and (b) Julia and Julienne along the streets of Cebu wearing articles of clothing that show virtually the entirety of their black brassieres. Also, Escudero’s students claimed that there were times when access to or the availability of the identified students’ photos was not confined to the girls’ Facebook friends, but were, in fact, viewable by any Facebook user.

Investigation ensued. Then Julia, Julienne and other students involved were barred from joining the commencement exercises.

Meaning of “engaged” in the gathering, collecting or storing of data or information

Petitioners, who are the respective parents of the minors, filed a Petition for the Issuance of a Writ of Habeas Data. RTC dismissed the petition for habeas data on the following grounds:

Habeas data is a protection against unlawful acts or omissions of public officials and of private individuals or entities engaged in gathering, collecting, or storing data about the aggrieved party and his or her correspondences, or about his or her family. Such individual or entity need not be in the business of collecting or storing data.

Petitioners failed to prove the existence of an actual or threatened violation of the minors’ right to privacy, one of the preconditions for the issuance of the writ of habeas data. The photos, having been uploaded on Facebook without restrictions as to who may view them, lost their privacy in some way. STC gathered the photographs through legal means and for a legal purpose, that is, the implementation of the school’s policies and rules on discipline. ISSUE:Whether or not there was indeed an actual or threatened violation of the right to privacy in the life, liberty, or security of the minors involved in this case. (Is there a right to informational privacy in online social network activities of its users?) HELD: (Note that you can skip the preliminary discussions and check the ruling at the latter part) Nature of Writ of Habeas Data It is a remedy available to any person whose right to privacy in life, liberty or security is violated or threatened by an unlawful act or omission of a public official or employee, or of a private individual or entity engaged in the gathering, collecting or storing of data or information regarding the person, family, home and correspondence of the aggrieved party. It is an independent and summary remedy designed to protect the image, privacy, honor, information, and freedom of information of an individual, and to provide a forum to enforce one’s right to the truth and to informational privacy. It seeks to protect a person’s right to control information regarding oneself, particularly in instances in which such information is being collected through unlawful means in order to achieve unlawful ends.

In developing the writ of habeas data, the Court aimed to protect an individual’s right to informational privacy, among others. A comparative law scholar has, in fact, defined habeas data as “a procedure designed to safeguard individual freedom from abuse in the information age.” Issuance of writ of habeas data; requirements The existence of a person’s right to informational privacy An actual or threatened violation of the right to privacy in life, liberty or security of the victim (proven by at least substantial evidence) Note that the writ will not issue on the basis merely of an alleged unauthorized access to information about a person. The writ of habeas data is not only confined to cases of extralegal killings and enforced disappearances The writ of habeas data can be availed of as an independent remedy to enforce one’s right to privacy, more specifically the right to informational privacy. The remedies against the violation of such right can include the updating, rectification, suppression or destruction of the database or information or files in possession or in control of respondents. Clearly then, the privilege of the Writ of Habeas Data may also be availed of in cases outside of extralegal killings and enforced disappearances.

To “engage” in something is different from undertaking a business endeavour. To “engage” means “to do or take part in something.” It does not necessarily mean that the activity must be done in pursuit of a business. What matters is that the person or entity must be gathering, collecting or storing said data or information about the aggrieved party or his or her family. Whether such undertaking carries the element of regularity, as when one pursues a business, and is in the nature of a personal endeavour, for any other reason or even for no reason at all, is immaterial and such will not prevent the writ from getting to said person or entity. As such, the writ of habeas data may be issued against a school like STC. Right to informational privacy Right to informational privacy is the right of individuals to control information about themselves. Several commentators regarding privacy and social networking sites, however, all agree that given the millions of OSN users, “in this Social Networking environment, privacy is no longer grounded in reasonable expectations, but rather in some theoretical protocol better known as wishful thinking.” So the underlying question now is: Up to what extent is the right to privacy protected in OSNs? Facebook Privacy Tools To address concerns about privacy, but without defeating its purpose, Facebook was armed with different privacy tools designed to regulate the accessibility of a user’s profile as well as information uploaded by the user. In H v. W, the South Gauteng High Court recognized this ability of the users to “customize their privacy settings,” but did so with this caveat: “Facebook states in its policies that, although it makes every effort to protect a user’s information, these privacy settings are not foolproof.” For instance, a Facebook user can regulate the visibility and accessibility of digital images (photos), posted on his or her personal bulletin or “wall,” except for the user’s profile picture and ID, by selecting his or her desired privacy setting: Public – the default setting; every Facebook user can view the photo; Friends of Friends – only the user’s Facebook friends and their friends can view the photo; Friends – only the user’s Facebook friends can view the photo; Custom – the photo is made visible only to particular friends and/or networks of the Facebook user; and Only Me – the digital image can be viewed only by the user. The foregoing are privacy tools, available to Facebook users, designed to set up barriers to broaden or limit the visibility of his or her specific profile content, statuses, and photos, among others, from another user’s point of view. In other words, Facebook extends its users an avenue to make the availability of their Facebook activities reflect their choice as to “when and to what extent to disclose facts about themselves – and to put others in the position of receiving such confidences.” LONE ISSUE:NONE. The Supreme Court held that STC did not violate petitioners’ daughters’ right to privacy as the subject digital

photos were viewable either by the minors’ Facebook friends, or by the public at large.

instead of being broadcasted to the public at large or all the user’s friends en masse, becomes more manifest and palpable.

Without any evidence to corroborate the minors’ statement that the images were visible only to the five of them, and without their challenging Escudero’s claim that the other students were able to view the photos, their statements are, at best, self-serving, thus deserving scant consideration.

Doctrines: The Common Right to Privacy,20 where he explained the three strands of the right to privacy, viz: (1) locational or situational privacy;21 (2) informational privacy; and (3) decisional privacy.22 Of the three, what is relevant to the case at bar is the right to informational privacy––usually defined as the right of individuals to control information about themselves.23

It is well to note that not one of petitioners disputed Escudero’s sworn account that her students, who are the minors’ Facebook “friends,” showed her the photos using their own Facebook accounts. This only goes to show that no special means to be able to view the allegedly private posts were ever resorted to by Escudero’s students, and that it is reasonable to assume, therefore, that the photos were, in reality, viewable either by (1) their Facebook friends, or (2) by the public at large.

Considering that the default setting for Facebook posts is “Public,” it can be surmised that the photographs in question were viewable to everyone on Facebook, absent any proof that petitioners’ children positively limited the disclosure of the photograph. If such were the case, they cannot invoke the protection attached to the right to informational privacy. US v. Gines-Perez: A person who places a photograph on the Internet precisely intends to forsake and renounce all privacy rights to such imagery, particularly under circumstances such as here, where the Defendant did not employ protective measures or devices that would have controlled access to the Web page or the photograph itself. United States v. Maxwell: The more open the method of transmission is, the less privacy one can reasonably expect. Messages sent to the public at large in the chat room or e-mail that is forwarded from correspondent to correspondent loses any semblance of privacy. The Honorable Supreme Court continued and held that setting a post’s or profile detail’s privacy to “Friends” is no assurance that it can no longer be viewed by another user who is not Facebook friends with the source of the content. The user’s own Facebook friend can share said content or tag his or her own Facebook friend thereto, regardless of whether the user tagged by the latter is Facebook friends or not with the former. Also, when the post is shared or when a person is tagged, the respective Facebook friends of the person who shared the post or who was tagged can view the post, the privacy setting of which was set at “Friends.” Thus, it is suggested, that a profile, or even a post, with visibility set at “Friends Only” cannot easily, more so automatically, be said to be “very private,” contrary to petitioners’ argument.

Alejano vs. Cabuay G.R. No. 160792, August 25 2005

Facts: Early morning of 27 July 2003, some 321 armed soldiers, led by the now detained junior officers, entered and took control of the Oakwood Premier Luxury Apartments (“Oakwood”). The soldiers disarmed the security officers of Oakwood and planted explosive devices in its immediate surroundings. The junior officers publicly renounced their support for the administration and called for the resignation of President Gloria Macapagal-Arroyo and several cabinet members. Around 7:00 p.m. of the same date, the soldiers voluntarily surrendered to the authorities after several negotiations with government emissaries. The soldiers later defused the explosive devices they had earlier planted. The soldiers then returned to their barracks. Gen. Abaya, as the Chief of Staff of the AFP, issued a directive to all the Major Service Commanders to turn over custody of ten junior officers to the ISAFP Detention Center. The transfer took place while military and civilian authorities were investigating the soldiers’ involvement in the Oakwood incident. Government prosecutors filed an Information for coup d’etat with the RTC against the soldiers involved in the Oakwood incident. the CA rendered its decision ordered Gen. Cabuay, who was in charge of implementing the regulations in the ISAFP Detention Center, to uphold faithfully the rights of the detainees in accordance with Standing Operations Procedure No. 0263-04. The appellate court directed Gen. Cabuay to adhere to his commitment made in court regarding visiting hours and the detainees’ right to exercise for two hours a day. The appellate court declared that while the opening and reading of Trillanes’ letter is an abhorrent violation of his right to privacy of communication, this does not justify the issuance of a writ of habeas corpus. The violation does not amount to illegal restraint, which is the proper subject of habeas corpus proceedings.

Issue: Whether the opening, inspection and reading of the letter of the detainees is an infringement of a citizen’s privacy rights.

No privacy invasion by STC; fault lies with the friends of minors Respondent STC can hardly be taken to task for the perceived privacy invasion since it was the minors’ Facebook friends who showed the pictures to Tigol. Respondents were mere recipients of what were posted. They did not resort to any unlawful means of gathering the information as it was voluntarily given to them by persons who had legitimate access to the said posts. Clearly, the fault, if any, lies with the friends of the minors. Curiously enough, however, neither the minors nor their parents imputed any violation of privacy against the students who showed the images to Escudero. Different scenario of setting is set on “Me Only” or “Custom” Had it been proved that the access to the pictures posted were limited to the original uploader, through the “Me Only” privacy setting, or that the user’s contact list has been screened to limit access to a select few, through the “Custom” setting, the result may have been different, for in such instances, the intention to limit access to the particular post,

Held: No, the SC do not agree with the CA that the opening and reading of the detainees’ letters violated the detainees’ right to privacy of communication. The letters were not in a sealed envelope. The inspection of the folded letters is a valid measure as it serves the same purpose as the opening of sealed letters for the inspection of contraband. The letters alleged to have been read by the ISAFP authorities were not confidential letters between the detainees and their lawyers. The petitioner who received the letters from detainees Trillanes and Maestrecampo was merely acting as the detainees’ personal courier and not as their counsel when he received the letters for mailing. In the present case, since the letters were not confidential communication between the detainees and their lawyers, the officials of the ISAFP Detention Center could read the letters. If the letters are marked confidential communication between the detainees and their lawyers, the detention officials should not read the letters but only open

the envelopes for inspection in the presence of the detainees. The right to privacy of those detained is subject to Section 4 of RA 7438, as well as to the limitations inherent in lawful detention or imprisonment. By the very fact of their detention, pre-trial detainees and convicted prisoners have a diminished expectation of privacy rights. The detainees in the present case are junior officers accused of leading 300 soldiers in committing coup d’etat, a crime punishable with reclusion perpetua. The junior officers are not ordinary detainees but visible leaders of the Oakwood incident involving an armed takeover of a civilian building in the heart of the financial district of the country. As members of the military armed forces, the detainees are subject to the Articles of War. Moreover, the junior officers are detained with other high-risk persons from the Abu Sayyaf and the NPA. Thus, we must give the military custodian a wider range of deference in implementing the regulations in the ISAFP Detention Center. The military custodian is in a better position to know the security risks involved in detaining the junior officers, together with the suspected Abu Sayyaf and NPA members. Since the appropriate regulations depend largely on the security risks involved, we should defer to the regulations adopted by the military custodian in the absence of patent arbitrariness. Doctrines: The letters alleged to have been read by the ISAFP authorities were not confidential letters between the detainees and their lawyers. The petitioner who received the letters from detainees Trillanes and Maestrecampo was merely acting as the detainees personal courier and not as their counsel when he received the letters for mailing. In the present case, since the letters were not confidential communication between the detainees and their lawyers, the officials of the ISAFP Detention Center could read the letters. If the letters are marked confidential communication between the detainees and their lawyers, the detention officials should not read the letters but only open the envelopes for inspection in the presence of the detainees. That a law is required before an executive officer could intrude on a citizens privacy rights[62] is a guarantee that is available only to the public at large but not to persons who are detained or imprisoned. The right to privacy of those detained is subject to Section 4 of RA 7438, as well as to the limitations inherent in lawful detention or imprisonment. By the very fact of their detention, pre-trial detainees and convicted prisoners have a diminished expectation of privacy rights. In assessing the regulations imposed in detention and prison facilities that are alleged to infringe on the constitutional rights of the detainees and convicted prisoners, U.S. courts balance the guarantees of the Constitution with the legitimate concerns of prison administrators.[63] The deferential review of such regulations stems from the principle that: [s]ubjecting the day-to-day judgments of prison officials to an inflexible strict scrutiny analysis would seriously hamper their ability to anticipate security problems and to adopt innovative solutions to the intractable problems of prison administration.[64] The detainees in the present case are junior officers accused of leading 300 soldiers in committing coup detat, a crime punishable with reclusion perpetua.[65] The junior officers are not ordinary detainees but visible leaders of the Oakwood incident involving an armed takeover of a civilian building in the heart of the financial district of the country. As members of the military armed forces, the detainees are subject to the Articles of War.[66] Moreover, the junior officers are detained with other high-risk persons from the Abu Sayyaf and the NPA. Thus, we must give the military custodian a wider range of deference in implementing the regulations in the ISAFP Detention Center. The military custodian is in a better position to know the security risks involved in detaining the junior officers, together with the suspected Abu Sayyaf and NPA members. Since the appropriate regulations depend largely on the security risks

involved, we should defer to the regulations adopted by the military custodian in the absence of patent arbitrariness. Rules on Habeas Data Section 1. Habeas Data. - The writ of habeas data is a remedy available to any person whose right to privacy in life, liberty or security is violated or threatened by an unlawful act or omission of a public official or employee, or of a private individual or entity engaged in the gathering, collecting or storing of data or information regarding the person, family, home and correspondence of the aggrieved party.

Sec. 2. Who May File. - Any aggrieved party may file a petition for the writ of habeas data. However, in cases of extralegal killings and enforced disappearances, the petition may be filed by: chanrobles virtual law library

(a) Any member of the immediate family of the aggrieved party, namely: the spouse, children and parents; or chanrobles virtual law library

(b) Any ascendant, descendant or collateral relative of the aggrieved party within the fourth civil degree of consanguinity or affinity, in default of those mentioned in the preceding paragraph; or chanrobles virtual law library

Sec. 3. Where to File. - The petition may be filed with the Regional Trial Court where the petitioner or respondent resides, or that which has jurisdiction over the place where the data or information is gathered, collected or stored, at the option of the petitioner. chanrobles virtual law library

The petition may also be filed with the Supreme Court or the Court of Appeals or the Sandiganbayan when the action concerns public data files of government offices.

Sec. 4. Where Returnable; Enforceable. - When the writ is issued by a Regional Trial Court or any judge thereof, it shall be returnable before such court or judge.

When issued by the Court of Appeals or the Sandiganbayan or any of its justices, it may be returnable before such court or any justice thereof, or to any Regional Trial Court of the place where the petitioner or respondent resides, or that which has jurisdiction over the place where the data or information is gathered, collected or stored.

When issued by the Supreme Court or any of its justices, it may be returnable before such Court or any justice thereof, or before the Court of Appeals or the Sandiganbayan or any of its justices, or to any Regional Trial Court of the place where the petitioner or respondent resides, or that which has jurisdiction over the place where the data or information is gathered, collected or stored.

The writ of habeas data shall be enforceable anywhere in the Philippines.

Sec. 5. Docket Fees. - No docket and other lawful fees shall be required from an indigent petitioner. The petition of the indigent shall be docked and acted upon immediately, without prejudice to subsequent submission of proof of indigency not later than fifteen (15) days from the filing of the petition. chanrobles virtual law library

Sec. 6. Petition. - A verified written petition for a writ of habeas data should contain:

Sec. 9. How the Writ is Served. - The writ shall be served upon the respondent by a judicial officer or by a person deputized by the court, justice or judge who shall retain a copy on which to make a return of service. In case the writ cannot be served personally on the respondent, the rules on substituted service shall apply.

Sec. 10. Return; Contents. - The respondent shall file a verified written return together with supporting affidavits within five (5) working days from service of the writ, which period may be reasonably extended by the Court for justifiable reasons. The return shall, among other things, contain the following:

(a) The personal circumstances of the petitioner and the respondent;

(b) The manner the right to privacy is violated or threatened and how it affects the right to life, liberty or security of the aggrieved party; chanrobles virtual law library

(a) The lawful defenses such as national security, state secrets, privileged communications, confidentiality of the source of information of media and others;

(b) In case of respondent in charge, in possession or in control of the data or information subject of the petition; (c) The actions and recourses taken by the petitioner to secure the data or information; (i) a disclosure of the data or information about the petitioner, the nature of such data or information, and the purpose for its collection; (d) The location of the files, registers or databases, the government office, and the person in charge, in possession or in control of the data or information, if known;

(e) The reliefs prayed for, which may include the updating, rectification, suppression or destruction of the database or information or files kept by the respondent.

In case of threats, the relief may include a prayer for an order enjoining the act complained of; and

(f) Such other relevant reliefs as are just and equitable.

Sec. 7. Issuance of the Writ. - Upon the filing of the petition, the court, justice or judge shall immediately order the issuance of the writ if on its face it ought to issue. The clerk of court shall issue the writ under the seal of the court and cause it to be served within three (3) days from the issuance; or, in case of urgent necessity, the justice or judge may issue the writ under his or her own hand, and may deputize any officer or person serve it.

The writ shall also set the date and time for summary hearing of the petition which shall not be later than ten (10) work days from the date of its issuance. chanrobles virtual law library

Sec. 8. Penalty for Refusing to Issue or Serve the Writ. - A clerk of court who refuses to issue the writ after its allowance, or a deputized person who refuses to serve the same, shall be punished by the court, justice or judge for contempt without prejudice to other disciplinary actions. chanrobles virtual law library

(ii) the steps or actions taken by the respondent to ensure the security and confidentiality of the data or information; and chanrobles virtual law library

(iii) the currency and accuracy of the data or information held; and,

(c) Other allegations relevant to the resolution of the proceeding.

A general denial of the allegations in the petition shall not be allowed.

Sec. 11. Contempt. - The court, justice or judge may punish with imprisonment or fine a respondent who commits contempt by making a false return, or refusing to make a return; or any person who otherwise disobeys or resist a lawful process or order of the court.

Sec. 12. When Defenses May be Heard in Chambers. - A hearing in chambers may be conducted where the respondent invokes the defense that the release of the data or information in question shall compromise national security or state secrets, or when the data or information cannot be divulged to the public due to its nature or privileged character.

Sec. 13. Prohibited Pleadings and Motions. - The following pleadings and motions are prohibited:

(a) Motion to dismiss;

(b) Motion for extension of time to file return, opposition, affidavit, position paper and other pleadings; chanrobles virtual law library

Upon its finality, the judgment shall be enforced by the sheriff or any lawful officers as may be designated by the court, justice or judge within five (5) working days.

(c) Dilatory motion for postponement;

(d) Motion for a bill of particulars;

(e) Counterclaim or cross-claim;

(f) Third-party complaint;

Sec. 17. Return of Service. - The officer who executed the final judgment shall, within three (3) days from its enforcement, make a verified return to the court. The return shall contain a full statement of the proceedings under the writ and a complete inventory of the database or information, or documents and articles inspected, updated, rectified, or deleted, with copies served on the petitioner and the respondent.

The officer shall state in the return how the judgment was enforced and complied with by the respondent, as well as all objections of the parties regarding the manner and regularity of the service of the writ.

(g) Reply;

(h) Motion to declare respondent in default;

(i) Intervention;

Sec. 18. Hearing on Officer-s Return. - The court shall set the return for hearing with due notice to the parties and act accordingly. chanrobles virtual law library

Sec. 19. Appeal. - Any party may appeal from the final judgment or order to the Supreme Court under Rule 45. The appeal may raise questions of fact or law or both. chanrobles virtual law library

(j) Memorandum;

(k) Motion for reconsideration of interlocutory orders or interim relief orders; and

(l) Petition for certiorari, mandamus or prohibition against any interlocutory order.

Sec. 14. Return; Filing. - In case the respondent fails to file a return, the court, justice or judge shall proceed to hear the petition ex parte, granting the petitioner such relief as the petition may warrant unless the court in its discretion requires the petitioner to submit evidence. chanrobles virtual law library

Sec. 15. Summary Hearing. - The hearing on the petition shall be summary. However, the court, justice or judge may call for a preliminary conference to simplify the issues and determine the possibility of obtaining stipulations and admissions from the parties. chanrobles virtual law library

Sec. 16. Judgment. - The court shall render judgment within ten (10) days from the time the petition is submitted for decision. If the allegations in the petition are proven by substantial evidence, the court shall enjoin the act complained of, or order the deletion, destruction, or rectification of the erroneous data or information and grant other relevant reliefs as may be just and equitable; otherwise, the privilege of the writ shall be denied.

The period of appeal shall be five (5) working days from the date of notice of the judgment or final order.

The appeal shall be given the same priority as in habeas corpus and amparo cases.

Sec. 20. Institution of Separate Actions. - The filing of a petition for the writ of habeas data shall not preclude the filing of separate criminal, civil or administrative actions.

Sec. 21. Consolidation. - When a criminal action is filed subsequent to the filing of a petition for the writ, the latter shall be consolidated with the criminal action.

When a criminal action and a separate civil action are filed subsequent to a petition for a writ of habeas data, the petition shall be consolidated with the criminal action.

After consolidation, the procedure under this Rule shall continue to govern the disposition of the reliefs in the petition.

Sec. 22. Effect of Filing of a Criminal Action. - When a criminal action has been commenced, no separate petition for the writ shall be filed. The relief under the writ shall be available to an aggrieved party by motion in the criminal case.

The procedure under this Rule shall govern the disposition of the reliefs available under the writ of habeas data.

well. These purposes would be defeated if the card were to be mutilated or destroyed.

Sec. 23. Substantive Rights. - This Rule shall not diminish, increase or modify substantive rights.

No. Judgment of the Court of Appeals reversed. The purpose of Congress is not a basis for declaring this legislation unconstitutional. Therefore, the 1965 Amendment is constitutional as enacted.

Section 4 - Freedom of Expression US v. O’Brien 391 US 367 (1963) Brief Fact Summary. The Defendant, O’Brien (Defendant), was convicted for symbolically burning his draft card under a federal statute forbidding the altering of a draft card. His conviction was upheld after the Supreme Court of the United States (Supreme Court) found the law constitutional.

Synopsis of Rule of Law. First, a government regulation is sufficiently justified if it is within the constitutional power of the government. Second, if it furthers a substantial or important governmental interest. Third, if the governmental interest is unrelated to the suppression of free expression. Fourth, if the incidental restriction on alleged First Amendment constitutional freedoms is no greater than is essential to the furtherance of that interest.

Facts. The Defendant was convicted under Section:462(b)(3) of the Universal Military Training and Service Act (UMTSA) of 1948, amended in 1965 to include the applicable provision that made it an offense to “alter, knowingly destroy, knowingly mutilate”� a Selective Service registration certification. Defendant knowingly burned his draft card on the front steps of the local courthouse. The Court of Appeals held the 1965 amendment unconstitutional as a law abridging the freedom of speech.

Issue. Whether the 1965 Amendment is unconstitutional as applied to Defendant because his act of burning the draft card was protected “symbolic speech”� within the First Amendment? Whether the draft cards are merely pieces of paper designed only to notify registrants of their registration or classification, to be retained or tossed into the waste basket according to the convenience of the registrant? Whether the 1965 Amendment is unconstitutional as enacted because it was intended to “suppress freedom of speech?” Held. No. Judgment of the Court of Appeals reversed. It cannot be accepted that there is an endless and limitless variety of conduct that constitutes “speech”� whenever the person engaging in the conduct intends to express an idea. However, even if the alleged communicative element of Defendant’s conduct is sufficient to bring into play the First Amendment of the United States Constitution (Constitution), it does not necessarily follow that the destruction of a draft card is constitutionally protected activity. First, a government regulation is sufficiently justified if it is within the constitutional power of the government. Second, if it furthers a substantial or important governmental interest. Third, if the governmental interest is unrelated to the suppression of free expression. Fourth, if the incidental restriction on alleged First Amendment constitutional freedoms is no greater than is essential to the furtherance of that interest. The 1965 Amendment meets all these requirement s. Therefore, the 1965 Amendment is constitutional as applied to Defendant. No. Judgment of the Court of Appeals reversed. Although the initial purpose of the draft card is to notify, it serves many other purposes as

Doctrines: Warren wrote that when a regulation prohibits conduct that combines "speech" and "nonspeech" elements, "a sufficiently important governmental interest in regulating the nonspeech element can justify incidental limitations on First Amendment freedoms". The regulation must 1) be within the constitutional power of the government to enact, 2) further an important or substantial government interest, 3) that interest must be unrelated to the suppression of speech (or "content neutral", as later cases have phrased it), and 4) prohibit no more speech than is essential to further that interest. The Court ruled that § 462(b)(3) satisfied this test.[10] Schenck v. US 249 US 47 (1919) Facts: During World War I, socialists Charles Schenck and Elizabeth Baer distributed leaflets declaring that the draft violated the Thirteenth Amendment prohibition against involuntary servitude. The leaflets urged the public to disobey the draft, but advised only peaceful action. Schenck was charged with conspiracy to violate the Espionage Act of 1917 by attempting to cause insubordination in the military and to obstruct recruitment. Schenck and Baer were convicted of violating this law and appealed on the grounds that the statute violated the First Amendment. Issue: Did Schenck's conviction under the Espionage Act for criticizing the draft violate his First Amendment right to freedom of speech? Ruling: The Court held that the Espionage Act did not violate the First Amendment and was an appropriate exercise of Congress’ wartime authority. Writing for a unanimous Court, Justice Oliver Wendell Holmes concluded that courts owed greater deference to the government during wartime, even when constitutional rights were at stake. Articulating for the first time the “clear and present danger test,” Holmes concluded that the First Amendment does not protect speech that approaches creating a clear and present danger of a significant evil that Congress has power to prevent. Holmes reasoned that the widespread dissemination of the leaflets was sufficiently likely to disrupt the conscription process. Famously, he compared the leaflets to shouting “Fire!” in a crowded theatre, which is not permitted under the First Amendment. United States vs. Bustos [G.R. No. L-12592, March 8, 1918] Facts: In 1915, numerous citizens of Pampanga assembled, and prepared and signed a petition to the Executive Secretary (privileged communication) through the law office of Crossfield and O'Brien, and five individuals signed affidavits, charging Roman Punsalan, justice of the peace of Macabebe and Masantol, Pampanga, with malfeasance in office and asking for his removal. The petition transmitted by these attorneys was signed by thirty-four citizensThe specific charges: o Francisca Polintan asked for money and kept her in the house for four days as a servant and took from her two chickens and twelve "gandus;" o

Valentin Sunga asked for P50

o Leoncio Quiambao: Punsalan gave him P30 and his complaint was shelved. Now, Punsalan alleged that accused published a writing which was false, scandalous, malicious, defamatory, and libelous against him. Issue: WON accused is entitled to constitutional protection by virtue of his right to free speech and free press.

Held: Yes. The guaranties of a free speech and a free press include the right to criticize judicial conduct. The administration of the law is a matter of vital public concern. Whether the law is wisely or badly enforced is, therefore, a fit subject for proper comment. If the people cannot criticize a justice of the peace or a judge the same as any other public officer, public opinion will be effectively suppressed. It is a duty which every one owes to society or to the State to assist in the investigation of any alleged misconduct. It is further the duty of all who know of any official dereliction on the part of a magistrate or the wrongful act of any public officer to bring the facts to the notice of those whose duty it is to inquire into and punish them. Doctrines: The interest of society and the maintenance of good government demand a full discussion of public affairs. Completely liberty to comment on the conduct of public men is a scalpel in the case of free speech. The sharp incision of its probe relieves the abscesses of officialdom. Men in public life may suffer under a hostile and an unjust accusation; the wound can be assuaged with the balm of a clear conscience. A public officer must not be too thin-skinned with reference to comment upon his official acts. Only thus can the intelligence and the dignity of the individual be exalted. Of course, criticism does not authorize defamation. Nevertheless, as the individual is less than the State, so must expected criticism be born for the common good. Rising superior to any official or set of officials, to the Chief of Executive, to the Legislature, to the Judiciary — to any or all the agencies of Government — public opinion should be the constant source of liberty and democracy. The guaranties of a free speech and a free press include the right to criticize judicial conduct. The administration of the law is a matter of vital public concern. Whether the law is wisely or badly enforced is, therefore, a fit subject for proper comment. If the people cannot criticize a justice of the peace or a judge the same as any other public officer, public opinion will be effectively muzzled. Attempted terrorization of public opinion on the part of the judiciary would be tyranny of the basest sort. The sword of Damocles in the hands of a judge does not hang suspended over the individual who dares to assert his prerogative as a citizen and to stand up bravely before any official. On the contrary, it is a duty which every one owes to society or to the State to assist in the investigation of any alleged misconduct. The right to assemble and petition is the necessary consequence of republican institutions and the complement of the part of free speech. Assembly means a right on the part of citizens to meet peaceably for consultation in respect to public affairs. Petition means that any person or group of persons can apply, without fear of penalty, to the appropriate branch or office of the government for a redress of grievances. The persons assembling and petitioning must, of course, assume responsibility for the charges made. Public policy, the welfare of society, and the orderly administration of government have demanded protection for public opinion. The inevitable and incontestable result has been the development and adoption of the doctrine of privilege. The doctrine of privileged communications rests upon public policy, 'which looks to the free and unfettered administration of justice, though, as an incidental result, it may in some instances afford an immunity to the evil-disposed and malignant slanderer.' (Abbott vs. National Bank of Commerce, Tacoma [1899], 175 U. S., 409, 411.) Privilege is classified as either absolute or qualified. With the first, we are not concerned. As to qualified privilege, it is as the words suggest a prima facie privilege which may be lost by proof of malice. The rule is thus stated by Lord Campbell, C. J. A communication made bona fide upon any subject-matter in which the party communicating has an interest, or in reference to which has a duty, is privileged, if made to a person having a corresponding interest or duty, although it contained criminatory matter which without this privilege would be slanderous and actionable.

A pertinent illustration of the application of qualified privilege is a complaint made in good faith and without malice in regard to the character or conduct of a public official when addressed to an officer or a board having some interest or duty in the matter. Even when the statements are found to be false, if there is probable cause for belief in their truthfulness and the charge is made in good faith, the mantle of privilege may still cover the mistake of the individual. But the statements must be made under an honest sense of duty; a self-seeking motive is destructive. Personal injury is not necessary. the usual case malice can be presumed from defamatory words. Privilege destroy that presumption. The onus of proving malice then lies on the plaintiff. The plaintiff must bring home to the defendant the existence of malice as the true motive of his conduct. Falsehood and the absence of probable cause will amount to proof of malice. (See White vs. Nicholls [1845], 3 How., 266 A privileged communication should not be subjected to microscopic examination to discover grounds of malice or falsity. Such excessive scrutiny would defeat the protection which the law throws over privileged communications. The ultimate test is that of bona fides. People vs. Alarcon [G.R. No. 46551, December 12, 1939] As an aftermath of the decision rendered by the Court of first Instance of Pampanga in criminal case No. 5733, The People of the Philippines vs. Salvador Alarcon, et al., convicting the accused therein — except one — of the crime of robbery committed in band, a denunciatory letter, signed by Luis M. Taruc, was addressed to His Excellency, the President of the Philippines. A copy of said letter found its way to the herein respondent, Federico Mañgahas who, as columnist of the Tribune, a newspaper of general circulation in the Philippines, quoted the letter in an article published by him in the issue of that paper of September 23, 1937. The objectionable portion is inserted in the following petition of the provincial fiscal of Pampanga, filed with the Court of First Instance of that province on September 29, 1937 Ruling: Elements of Contempt by Newspaper Publications The elements of contempt by newspaper publications are well defined by the cases adjudicated in this as in other jurisdictions. Newspaper publications tending to impede, obstruct, embarrass, or influence the courts in administering justice in a pending suit or proceeding constitutes criminal contempt which is summarily punishable by the courts. The rule is otherwise after the cause is ended. It must, however, clearly appear that such publications do impede, interfere with, and embarrass the administration of justice before the author of the publications should be held for contempt. What is thus sought to be shielded against the influence of newspaper comments is the all-important duty of the court to administer justice in the decision of a pending case. There is no pending case to speak of when and once the court has come upon a decision and has lost control either to reconsider or amend it. That, we believe, is the case at bar, for here we have a concession that the letter complained of was published after the Court of First Instance of Pampanga had decided the aforesaid criminal case for robbery in band, and after that decision had been appealed to the Court of Appeals. The fact that a motion to reconsider its order confiscating the bond of the accused therein was subsequently filed may be admitted; but, the important consideration is that it was then without power to reopen or modify the decision which it had rendered upon the merits of the case, and could not have been influenced by the questioned publication. Lagunzad vs. Gonzales, G.R. No. L-32066 August 6, 1979 FACTS Petitioner Manuel Lagunzad, a newspaperman, began the production of a movie entitled "The Moises Padilla Story" portraying the life of

Moises Padilla, a mayoralty candidate of the Nacionalista Party for the Municipality of Magallon, Negros Occidental and for whose murder, Governor Rafael Lacson, a member of the Liberal Party then in power and his men were tried and convicted. The emphasis of the movie was on the public life of Moises Padilla, there were portions which dealt with his private and family life including the portrayal in some scenes, of his mother, Maria Soto, private respondent herein, and of one "Auring" as his girl friend. Padilla’s half sister, for and in behalf of her mother, Vda.de Gonzales, objected to the "exploitation" of his life and demanded in writing for certain changes, corrections and deletions in the movie. After some bargaining as to the amount to be paid Lagunzad and Vda. de Gonzales, executed a "Licensing Agreement" whereby the latter as LICENSOR granted Lagunzad authority and permission to exploit, use, and develop the life story of Moises Padilla for purposes of producing the picture for consideration of P20,000.00.Lagunzad paid Vda. de Gonzales the amount of P5,000.00. Subsequently, the movie was shown indifferent theaters all over the country. Because petitioner refused to pay any additional amounts pursuant to the Agreement, Vda. de Gonzales instituted the present suit against him praying for judgment in her favor ordering petitioner 1) to pay her the balance of P15,000.00, with legal interest from of the Complaint; and 2) to render an accounting of the proceeds from the picture and to pay the corresponding 2-1/2% royalty there from, among others. Petitioner contended in his Answer that the episodes in life of Moises Padilla depicted in the movie were matters of public knowledge and occurred at or about the same time that the deceased became and was a public figure; that private respondent has no property right over those incidents; that the Licensing Agreement was without valid cause or consideration and constitutes an infringement on the constitutional right of freedom of speech and of the press; and that he paid private respondent the amount of P5,000.00 only because of the coercion and threat employed upon him. As a counterclaim, petitioner sought for the nullification of the Licensing Agreement, Both the trial court and the CA ruled in favor of Vda. deGonzales. ISSUES Whether or not the fictionalized representation of Moises Padilla is an intrusion upon his right to privacy notwithstanding that he was a public figure. Whether or not Vda. de Gonzales., the mother, has any property right over the life of Moises Padilla considering that the latter was a public figure. Whether or not the Licensing Agreement constitutes an infringement on the constitutional right of freedom of speech and of the press.

HELD YES, being a public figure ipso facto does not automatically destroy in toto a person's right to privacy. The right to invade as person's privacy to disseminate public information does not extend to a fictional or novelized representation of a person, no matter how public a figure he or she may be. In the case at bar, while it is true that petitioner exerted efforts to present a true-to-life story of Moises Padilla, petitioner admits that he included a little romance in the film because without it, it would be a drab story of torture and brutality. YES, Lagunzad cannot dispense with the need for prior consent and authority from the deceased heirs to portray publicly episodes in said deceased's life and in that of his mother and the members of his family. As held in Schuyler v. Curtis" a privilege may be given the surviving relatives of a deceased person to protect his memory, but the privilege exists for the benefit of the living, to protect their feelings and to prevent a violation of their own rights in the character and memory of the deceased."

NO, Lagunzad claims that as a citizen and as a newspaperman, he had the right to express his thoughts in film on the public life of Moises Padilla without prior restraint. The right of freedom of expression, indeed, occupies a preferred position in the "hierarchy of civil liberties." It is not, however, without limitations. One criterion for permissible limitation on freedom of speech and of the press is the "balancing-of-interests test." The principle requires a court to take conscious and detailed consideration of the interplay of interests observable in a given situation or type of situation." In the case at bar, the interest’s observable are the right to privacy asserted by respondent and the right of -freedom of expression invoked by petitioner. Taking into account the interplay of those interests, and considering the obligations assumed in the Licensing Agreement entered into by petitioner, the validity of such agreement will have to be upheld particularly because the limits of freedom of expression are reached when expression touches upon matters of essentially private concern Ayer Productions PTY Ltd. vs. Capulong [G.R. No. L—82380, April 29, 1988] Facts: Petitioner McElroy an Australian film maker, and his movie production company, Ayer Productions, envisioned, sometime in 1987, for commercial viewing and for Philippine and international release, the historic peaceful struggle of the Filipinos at EDSA. The proposed motion picture entitled "The Four Day Revolution" was endorsed by the MTRCB as and other government agencies consulted. Ramos also signified his approval of the intended film production.

It is designed to be viewed in a six-hour mini-series television play, presented in a "docu-drama" style, creating four fictional characters interwoven with real events, and utilizing actual documentary footage as background. David Williamson is Australia's leading playwright and Professor McCoy (University of New South Wales) is an American historian have developed a script.

Enrile declared that he will not approve the use, appropriation, reproduction and/or exhibition of his name, or picture, or that of any member of his family in any cinema or television production, film or other medium for advertising or commercial exploitation. petitioners acceded to this demand and the name of Enrile was deleted from the movie script, and petitioners proceeded to film the projected motion picture. However, a complaint was filed by Enrile invoking his right to privacy. RTC ordered for the desistance of the movie production and making of any reference to plaintiff or his family and from creating any fictitious character in lieu of plaintiff which nevertheless is based on, or bears substantial or marked resemblance to Enrile. Hence the appeal.

Issue: Whether or Not freedom of expression was violated.

Held: Yes. Freedom of speech and of expression includes the freedom to film and produce motion pictures and exhibit such motion pictures in theaters or to diffuse them through television. Furthermore the circumstance that the production of motion picture films is a commercial activity expected to yield monetary profit, is not a disqualification for availing of freedom of speech and of expression.

The projected motion picture was as yet uncompleted and hence not exhibited to any audience. Neither private respondent nor the respondent trial Judge knew what the completed film would precisely look like. There was, in other words, no "clear and present danger" of any violation of any right to privacy. Subject matter is one of public interest and concern. The subject thus relates to a highly critical stage in the history of the country.

public figure and the information sought to be elicited from him or to be published about him constitute of a public character. Succinctly put, the right of privacy cannot be invoked resist publication and dissemination of matters of public interest.

At all relevant times, during which the momentous events, clearly of public concern, that petitioners propose to film were taking place, Enrile was a "public figure:" Such public figures were held to have lost, to some extent at least, their right to privacy.

1.

from unwarranted publicity,

2.

from the wrongful publicizing of the private affairs

The line of equilibrium in the specific context of the instant case between the constitutional freedom of speech and of expression and the right of privacy, may be marked out in terms of a requirement that the proposed motion picture must be fairly truthful and historical in its presentation of events. Doctrines: Freedom of Speech and of Expression includes the freedom to film and produce motion pictures and to exhibit such motion pictures Considering first petitioners' claim to freedom of speech and of expression the Court would once more stress that this freedom includes the freedom to film and produce motion pictures and to exhibit such motion pictures in theaters or to diffuse them through television. In our day and age, motion pictures are a univesally utilized vehicle of communication and medium Of expression. Along with the press, radio and television, motion pictures constitute a principal medium of mass communication for information, education and entertainment

Motion Pictures Motion pictures are important both as a medium for the communication of Ideas and the expression of the artistic impulse. Their effect on the perception by our people of issues and public officials or public figures as well as the pre cultural traits is considerable. Nor as pointed out in BURSTYN V. WILSON is the Importance of motion pictures as an organ of public opinion lessened by the fact that they are designed to entertain as well as to inform' . There is no clear dividing line between what involves knowledge and what affords pleasure. If such a distinction were sustained, there is a diminution of the basic right to free expression. Monetary profit is not a disqualification This freedom is available in our country both to locally-owned and to foreign-owned motion picture companies. Furthermore, the circumstance that the production of motion picture films is a commercial activity expected to yield monetary profit, is not a disqualification for availing of freedom of speech and of expression. In our community as in many other countries, media facilities are owned either by the government or the private sector but the private sector-owned media facilities commonly require to be sustained by being devoted in whole or in part to revenue producing activities. Indeed, commercial media constitute the bulk of such facilities available in our country and hence to exclude commercially owned and operated media from the exercise of constitutionally protected om of speech and of expression can only result in the drastic contraction of such constitutional liberties in our country.

The interest sought to be protected by the right of privacy is the right to be free

3. and activities of an individual which are outside the realm of legitimate public concern.

Tests for Permissible Limitation 1. The prevailing doctrine is that the clear and present danger rule is such a limitation. 2. Another criterion for permissible limitation on freedom of speech and the press, which includes such vehicles of the mass media as radio, television and the movies, is the "balancing of interest test" The principle "requires a court to take conscious and detailed consideration of the interplay of interests observable in given situation or type of situation"

Whether the "balancing of interests test" or the clear and present danger test" be applied in respect of the instant Petitions, the Court believes that a different conclusion must here be reached: The production and filming by petitioners of the projected motion picture "The Four Day Revolution" does not, in the circumstances of this case, constitute an unlawful intrusion upon private respondent's "right of privacy." Borjal vs. Court of Appeals [G.R. No. 126466, January 14, 1999] Facts: A civil action for damages based on libel was filed before the court against Borjal and Soliven for writing and publishing articles that are allegedly derogatory and offensive against Francisco Wenceslao, attacking among others the solicitation letters he send to support a conference to be launch concerning resolving matters on transportation crisis that is tainted with anomalous activities. Wenceslao however was never named in any of the articles nor was the conference he was organizing. The lower court ordered petitioners to indemnify the private respondent for damages which was affirmed by the Court of Appeals. A petition for review was filed before the SC contending that private respondent was not sufficiently identified to be the subject of the published articles.

Issue: Whether or not there are sufficient grounds to constitute guilt of petitioners for libel.

The Right to Privacy is not an Absolute Right The right of privacy or "the right to be let alone," like the right of free expression, is not an absolute right. A limited intrusion into a person's privacy has long been regarded as permissible where that person is a

Held: In order to maintain a libel suit, it is essential that the victim be identifiable although it is not necessary that he be named. It is also not sufficient that the offended party recognized himself as the person

attacked or defamed, but it must be shown that at least a third person could identify him as the object of the libelous publication. These requisites have not been complied with in the case at bar. The element of identifiability was not met since it was Wenceslaso who revealed he was the organizer of said conference and had he not done so the public would not have known.

denial of the permit is a violation of the constitutional right of the freedom of speech and expression.

Issue The concept of privileged communications is implicit in the freedom of the press and that privileged communications must be protective of public opinion. Fair commentaries on matters of public interest are privileged and constitute a valid defense in an action for libel or slander. The doctrine of fair comment means that while in general every discreditable imputation publicly made is deemed false, because every man is presumed innocent until his guilt is judicially proved, and every false imputation is deemed malicious, nevertheless, when the discreditable imputation is directed against a public person in his public capacity, it is not necessarily actionable. In order that such discreditable imputation to a public official may be actionable, it must either be a false allegation of fact or a comment based on a false supposition. If the comment is an expression of opinion, based on established facts, then it is immaterial that the opinion happens to be mistaken, as long as it might reasonably be inferred from the facts. The questioned article dealt with matters of public interest as the declared objective of the conference, the composition of its members and participants, and the manner by which it was intended to be funded no doubt lend to its activities as being genuinely imbued with public interest. Respondent is also deemed to be a public figure and even otherwise is involved in a public issue. The court held that freedom of expression is constitutionally guaranteed and protected with the reminder among media members to practice highest ethical standards in the exercise thereof. A privileged communication may be either: 1. Absolutely privileged communication à those which are not actionable even if the author has acted in bad faith. An example is found in Sec. 11, Art.VI, of the 1987 Constitution which exempts a member of Congress from liability for any speech or debate in the Congress or in any Committee thereof. 2. Qualifiedly privileged communications à those containing defamatory imputations are not actionable unless found to have been made without good intention justifiable motive. To this genre belong "private communications" and "fair and true report without any comments or remarks."

Whether or not the Anti-Bases Coalition should be allowed to hold a peaceful protest rally in front of the US Embassy

Ruling The Supreme Court ruled to allow the rally in front of the US Embassy to protect the exercise of the rights to free speech and peaceful assembly and on the ground that there was no showing of the existence of a clear and present danger of a substantive evil that could justify the denial of the permit. These rights are not only assured by our constitution but also provided for in the Universal Declaration of Human Rights. Between the two generally accepted principles of diplomatic relations and human rights, the former takes higher ground. The right of the freedom of expression and peaceful assembly is highly ranked in the scheme of constitutional values. Miller v. California 413 US 15 (1973) In 1971, Marvin Miller, an owner/operator of a California mail-order business specializing in pornographic films and books, sent out a brochure advertising books and a film that graphically depicted sexual activity between men and women. The brochure used in the mailing contained graphic images from the books and the film. Five of the brochures were mailed to a restaurant in Newport Beach, California. The owner and his mother opened the envelope and seeing the brochures, called the police.[3] Miller was arrested and charged with violating California Penal Code 666.2(a) which says in part, "Every person who knowingly sends or causes to be sent, or brings or causes to be brought, into this state for sale or distribution, or in this state possesses, prepares, publishes, produces, or prints, with intent to distribute or to exhibit to others, or who offers to distribute, distributes, or exhibits to others, any obscene matter is for a first offense, guilty of a misdemeanor."[4] California lawmakers wrote the statute based on two previous Supreme Court obscenity cases, Memoirs v. Massachusetts and Roth v. United States.[5]

Reyes vs. Bagatsing [G.R. No. L-65366, November 9, 1983] Facts Retired Justice Jose B.L. Reyes, in behalf of the Anti-Bases Coalition, sought for a permit from the City of Manila to hold a peaceful march and rally on October 26, 1983 starting from Luneta to the gates of the United States embassy. The objective of the rally was to peacefully protest the removal of all foreign military bases and to present a petition containing such to a representative of the Embassy so it may be delivered to the United States Ambassador. This petition was to initially compel the Mayor of the City of Manila to make a decision on the application for a permit but it was discovered that a denial has already been sent through mail. It also included a provision that if it be held somewhere else, permit may be issued. The respondent mayor alleges that holding the rally in front of the US Embassy is a violation of the resolutions during the Vienna Convention on Diplomatic Relations adopted in 1961 and of which the Philippines is a signatory. In the doctrine of incorporation, the Philippines has to comply with such generally accepted principles of international law as part of the law of the land. The petitioner, on the other hand, contends that the

Miller was tried by jury in the Superior Court of Orange County. At the conclusion of the evidence phase, the judge instructed the jury to evaluate the evidence by the community standards of California, i.e., as defined by the statute.[6] The jury returned a guilty verdict. Miller appealed to the Appellate Division of the Superior Court, arguing that the jury instructions did not use the standard set in Memoirs v. Massachusetts which said that in order to be judged obscene, materials must be "utterly without redeeming social value". Miller argued that only a national standard for obscenity could be applied.[7] The appellate division rejected the argument and affirmed the jury verdict. Miller then filed an appeal with the California Court of Appeal for the Third District, which declined to review. Miller applied to the Supreme Court for certiorari, which was granted. Oral arguments were heard in January 1972.[7] Issue: Is the sale and distribution of obscene materials by mail protected under the First Amendment's freedom of speech guarantee? Ruling: No. Judgment of the lower court vacated and remanded for further proceedings. In determining whether speech is obscene, the

basic guidelines for the trier of fact must be: (a) whether “the average person, applying contemporary community standards” would find the material, taken as a whole, appeals to the prurient interest of sex, (b) whether the work depicts or describes, in a patently offensive way, sexual conduct specifically defined by the applicable state law, and (c) whether the work, taken as a whole, lacks serious literacy, artistic, political, or scientific value. The Supreme Court of the Untied States (Supreme Court) does not adopt as a constitutional standard the “utterly without redeeming social value” test. If a state law that regulates obscene material is thus limited, as written or construed, First Amendment constitutional values are adequately protected by the ultimate power of appellate courts to conduct an independent review of constitutional claims when necessary. Pita vs. Court of Appeals [G.R. No. 80806, October 5, 1989] Facts: In 1983, elements of the Special Anti-Narcotics Group, and the Manila Police, seized and confiscated from dealers along Manila sidewalks, magazines believed to be obscene. These were later burned. One of the publications was Pinoy Playboy published by Leo Pita. He filed an injunction case against the mayor of manila to enjoin him from confiscating more copies of his magazine and claimed that this was a violation of freedom of speech. The court ordered him to show cause. He then filed an Urgent Motion for issuance of a temporary restraining order against indiscriminate seizure. Defendant Mayor Bagatsing admitted the confiscation and burning of obscence reading materials but admitted that these were surrendered by the stall owners and the establishments were not raided. The other defendant, WPD Superintendent, Narcisco Cabrera, filed no answer. On January 11, 1984, the trial court issued an Order setting the case for hearing on January 16, 1984 "for the parties to adduce evidence on the question of whether the publication 'Pinoy Playboy Magazine alleged (sic) seized, confiscated and/or burned by the defendants, are obscence per se or not". On February 3, 1984, the trial court promulgated the Order appealed from denying the motion for a writ of preliminary injunction, and dismissing the case for lack of merit The CA also dismissed the appeal due to the argument that freedom of the press is not without restraint. In the SC, the petitioner claimed that: 1. The CA erred in holding that the police officers could without any court warrant or order seize and confiscate petitioner's magazines on the basis simply of their determination that they are obscene.

Test for obscenity: "whether the tendency of the matter charged as obscene, is to deprave or corrupt those whose minds are open to such immoral influences and into whose hands a publication or other article charged as being obscene may fall Also, "whether a picture is obscene or indecent must depend upon the circumstances of the case, and that ultimately, the question is to be decided by the "judgment of the aggregate sense of the community reached by it." (Kottinger) When does a publication have a corrupting tendency, or when can it be said to be offensive to human sensibilities? The issue is a complicated one, in which the fine lines have neither been drawn nor divided. Katigbak- "Whether to the average person, applying contemporary standards, the dominant theme of the material taken as a whole appeals to prurient interest." Kalaw-Katigbak represented a marked departure from Kottinger in the sense that it measured obscenity in terms of the "dominant theme" of the work, rather than isolated passages, which were central to Kottinger (although both cases are agreed that "contemporary community standards" are the final arbiters of what is "obscene"). Kalaw-Katigbak undertook moreover to make the determination of obscenity essentially a judicial question and as a consequence, to temper the wide discretion Kottinger had given unto law enforcers. The latest say on American jurisprudence was Miller v. California, which expressly abandoned Massachusettes, and established "basic guidelines," to wit: "(a) whether 'the average person, applying contemporary standards' would find the work, taken as a whole, appeals to the prurient interest . . .; (b) whether the work depicts or describes, in a patently offensive way, sexual conduct specifically defined by the applicable state law; and (c) whether the work, taken as a whole, lacks serious literary, artistic, political, or scientific value. The lack of uniformity in American jurisprudence as to what constitutes "obscenity" has been attributed to the reluctance of the courts to recognize the constitutional dimension of the problem. Apparently, the courts have assumed that "obscenity" is not included in the guaranty of free speech, an assumption that, as we averred, has allowed a climate of opinions among magistrates predicated upon arbitrary, if vague theories of what is acceptable to society. In the case at bar, there is no challenge on the right of the State, in the legitimate exercise of police power, to suppress smut provided it is smut. For obvious reasons, smut is not smut simply because one insists it is smut. So is it equally evident that individual tastes develop, adapt to wide-ranging influences, and keep in step with the rapid advance of civilization. What shocked our forebears, say, five decades ago, is not necessarily repulsive to the present generation.

2. The Court of Appeals erred in affirming the decision of the trial court and, in effect, holding that the trial court could dismiss the case on its merits without any hearing thereon when what was submitted to it for resolution was merely the application of petitioner for the writ of preliminary injunction.

But neither should we say that "obscenity" is a bare (no pun intended) matter of opinion. As we said earlier, it is the divergent perceptions of men and women that have probably compounded the problem rather than resolved it.

Issue: Was the seizure constitutional?

Undoubtedly, "immoral" lore or literature comes within the ambit of free expression, although not its protection. In free expression cases, this Court has consistently been on the side of the exercise of the right, barring a "clear and present danger" that would warrant State interference and action. But the burden to show this lies with the authorities.

Held: No. Petition granted "There must be objective and convincing, not subjective or conjectural, proof of the existence of such clear and present danger." Ratio:

As we so strongly stressed in Bagatsing, a case involving the delivery of a political speech, the presumption is that the speech may validly be

said. The burden is on the State to demonstrate the existence of a danger, a danger that must not only be: (1) clear but also, (2) present, to justify State action to stop the speech. The Court is not convinced that the private respondents have shown the required proof to justify a ban and to warrant confiscation of the literature for which mandatory injunction had been sought below. First of all, they were not possessed of a lawful court order: (1) finding the said materials to be pornography, and (2) authorizing them to carry out a search and seizure, by way of a search warrant. Has petitioner been found guilty for publishing obscene works under Presidential Decrees Nos. 960 and 969? This not answered, one can conclude that the fact that the former respondent Mayor's act was sanctioned by "police power" is no license to seize property in disregard of due process. The PD’s don’t give the authorities the permission to execute high-handed acts. It is basic that searches and seizures may be done only through a judicial warrant, otherwise, they become unreasonable and subject to challenge. There is of course provision for warrantless searches under the Rules of Court but as the provision itself suggests, the search must have been an incident to a lawful arrest and it must be on account fo a crime committed. The Court rejected the argument that "[t]here is no constitutional nor legal provision which would free the accused of all criminal responsibility because there had been no warrant, and there is no "accused" here to speak of, who ought to be "punished". Second, to say that the respondent Mayor could have validly ordered the raid (as a result of an anti-smut campaign) without a lawful search warrant because, in his opinion, "violation of penal laws" has been committed, is to make the respondent Mayor judge, jury, and executioner rolled into one. Iglesia ni Cristo vs. CA [G.R. No. 119673, July 26 1996] Facts: Petitioner has a television program entitled "Ang Iglesia ni Cristo" aired on Channel 2 every Saturday and on Channel 13 every Sunday. The program presents and propagates petitioner's religious beliefs, doctrines and practices often times in comparative studies with other religions. Petitioner submitted to the respondent Board of Review for Moving Pictures and Television the VTR tapes of its TV program Series Nos. 116, 119, 121 and 128. The Board classified the series as "X" or not for public viewing on the ground that they "offend and constitute an attack against other religions which is expressly prohibited by law." On November 28, 1992, it appealed to the Office of the President the classification of its TV Series No. 128 which allowed it through a letter of former Executive Secretary Edelmiro A. Amante, Sr., addressed for Henrietta S. Mendez reversing the decision of the respondent Board. According to the letter the episode in is protected by the constitutional guarantee of free speech and expression and no indication that the episode poses any clear and present danger. Petitioner also filed Civil Case. Petitioner alleged that the respondent Board acted without jurisdiction or with grave abuse of discretion in requiring petitioner to submit the VTR tapes of its TV program and in x-rating them. It cited its TV Program Series Nos. 115, 119, 121 and 128. In their Answer, respondent Board invoked its power under PD No. 19861 in relation to Article 201 of the Revised Penal Code. The Iglesia ni Cristo insists on the literal translation of the bible and says that our (Catholic) veneration of the Virgin Mary is not to be condoned because nowhere it is found in the bible. The board contended that it outrages Catholic and Protestant's beliefs. RTC ruled in favor of petitioners. CA however reversed it hence this petition.

Issue: Whether or Not the "ang iglesia ni cristo" program is not constitutionally protected as a form of religious exercise and expression.

Held: Yes. Any act that restrains speech is accompanied with presumption of invalidity. It is the burden of the respondent Board to overthrow this presumption. If it fails to discharge this burden, its act of censorship will be struck down. This is true in this case. So-called "attacks" are mere criticisms of some of the deeply held dogmas and tenets of other religions. RTC’s ruling clearly suppresses petitioner's freedom of speech and interferes with its right to free exercise of religion. “attack” is different from “offend” any race or religion. The respondent Board may disagree with the criticisms of other religions by petitioner but that gives it no excuse to interdict such criticisms, however, unclean they may be. Under our constitutional scheme, it is not the task of the State to favor any religion by protecting it against an attack by another religion. Religious dogmas and beliefs are often at war and to preserve peace among their followers, especially the fanatics, the establishment clause of freedom of religion prohibits the State from leaning towards any religion. Respondent board cannot censor the speech of petitioner Iglesia ni Cristo simply because it attacks other religions, even if said religion happens to be the most numerous church in our country. The basis of freedom of religion is freedom of thought and it is best served by encouraging the marketplace of dueling ideas. It is only where it is unavoidably necessary to prevent an immediate and grave danger to the security and welfare of the community that infringement of religious freedom may be justified, and only to the smallest extent necessary to avoid the danger. There is no showing whatsoever of the type of harm the tapes will bring about especially the gravity and imminence of the threatened harm. Prior restraint on speech, including religious speech, cannot be justified by hypothetical fears but only by the showing of a substantive and imminent evil. It is inappropriate to apply the clear and present danger test to the case at bar because the issue involves the content of speech and not the time, place or manner of speech. Allegedly, unless the speech is first allowed, its impact cannot be measured, and the causal connection between the speech and the evil apprehended cannot be established. The determination of the question as to whether or not such vilification, exaggeration or fabrication falls within or lies outside the boundaries of protected speech or expression is a judicial function which cannot be arrogated by an administrative body such as a Board of Censors." A system of prior restraint may only be validly administered by judges and not left to administrative agencies. Social Weather Stations, Inc. vs. COMELEC [G.R. No. 147571, May 5, 2001] Facts : Petitioner, Social Weather Stations, Inc. (SWS), is a private non-stock, non-profit social research institution conducting surveys in various fields, including economics, politics, demography, and social development, and thereafter processing, analyzing, and publicly reporting the results thereof. On the other hand, petitioner Kamahalan Publishing Corporation publishes the Manila Standard, a newspaper of general circulation, which features news- worthy items of information including election surveys Petitioners brought this action for prohibition to enjoin the Commission on Elections from enforcing §5.4 of RA. No.9006 (Fair Election Act), which provides: Surveys affecting national candidates shall not be published fifteen (15) days before an election and surveys affecting local candidates shall not be published seven (7) days be- fore an election. Petitioner SWS states that it wishes to conduct an election survey throughout the period of the elections both at the national and local levels and release to the media the results of such survey as well as publish them directly. Petitioner Kamahalan Publishing Corporation, on the other hand, states that it intends to publish election survey results up to the last day of the elections on May 14,2001

Issue : WON §5.4 of R.A. No. 9006 constitutes an unconstitutional abridgment of freedom of speech, expression, and the press.

Issue: Whether the constitutional guarantees of freedom of the press and right to information of public concern be given more weight than the fundamental rights of the accused.

Ratio: The petition is denied. HELD : What test should then be employed to determine the constitutional validity of §5.4? The United States Supreme Court, through Chief Justice Warren, held in United States v. O 'Brien: [A] Government regulation is sufficiently justified [1] if it is within the constitutional power of the Government; [2] if it furthers an important or substantial governmental interest; [3] if the governmental interest is unrelated to the suppression of free expression; and [4] if the incidental restriction on alleged First Amendment freedoms [of speech, expression and press] is no greater than is essential to the furtherance of that interest. This is so far the most influential test for distinguishing content-based from content neutral regulations and is said to have "become canonical in the review of such laws." is noteworthy that the O 'Brien test has been applied by this Court in at least two cases First. Sec. 5.4 fails to meet criterion [3] of the O 'Brien test because the causal connection of expression to the asserted governmental interest makes such interest "not related to the suppression of free expression." By prohibiting the publication of election survey results because of the possibility that such publication might undermine the integrity of the election, §5.4 actually suppresses a whole class of expression, while allowing the expression of opinion concerning the same subject matter by newspaper columnists, radio and TV commentators, armchair theorists, and other opinion takers Even if the governmental interest sought to be promoted is unrelated to the suppression of speech and the resulting restriction of free expression is only incidental, §5.4 nonetheless fails to meet criterion [4] of the O'Brien test, namely, that the restriction be not greater than is necessary to further the governmental interest. As already stated, §5.4 aims at the prevention of last-minute pressure on voters, the creation of bandwagon effect, "junking" of weak or "losing" candidates, and resort to the form of election cheating called "dagdag-bawas." Praiseworthy as these aims of the regulation might be, they cannot be attained at the sacrifice of the fundamental right of expression, when such aim can be more narrowly pursued by punishing unlawful acts, rather than speechbecause of apprehension that such speech creates the danger of such evils To summarize then, we hold that §5.4 is invalid because (1) it imposes a prior restraint on the freedom of expression, (2) it is a direct and total suppression of a category of expression even though such suppression is only for a limited period, and (3) the governmental interest sought to be promoted can be achieved by means other than suppression of freedom of expression. Re: Request for TV Coverage of the Trial of President Estrada [A.M. No. 01-4-03-SC, June 29 2001] Facts: On 13 March 2001, the Kapisanan ng mga Brodkaster ng Pilipinas (KBP) sent a letter requesting this Court to allow live media coverage of the anticipated trial of the plunder and other criminal cases filed against former President Joseph E. Estrada before the Sandiganbayan. The petitioners invoked other than the freedom of the press, the constitutional right of the people to be informed of matters of public concern which could only be recognized, served and satisfied by allowing live radio and television coverage of the court proceedings. Moreover, the live radio and television coverage of the proceedings will also serve the dual purpose of ensuring the desired transparency in the administration of justice. However, in the Resolution of the Court on October 1991, in a case for libel filed by then President Corazon C. Aquino read that the Court resolved to prohibit live radio and television coverage of court proceedings in view of protecting the parties’ right to due process, to prevent distraction of the participants in the proceedings and to avoid miscarriage of justice.

The courts recognize the constitutionally embodied freedom of the press and the right to public information. It also approves of media's exalted power to provide the most accurate and comprehensive means of conveying the proceedings to the public and in acquainting the public with the judicial process in action; nevertheless, within the courthouse, the overriding consideration is still the paramount right of the accused to due process which must never be allowed to suffer diminution in its constitutional proportions.

Due process guarantees the accused a presumption of innocence until the contrary is proved in a trial that is not lifted above its individual settings nor made an object of public's attention and where the conclusions reached are induced not by any outside force or influence but only by evidence and argument given in open court, where fitting dignity and calm ambiance is demanded."Television can work profound changes in the behavior of the people it focuses on."The conscious or unconscious effect that such coverage may have on the testimony of witnesses and the decision of judges cannot be evaluated but, it can likewise be said, it is not at all unlikely for a vote of guilt or innocence to yield to it. Although an accused has a right to a public trial but it is a right that belongs to him, more than anyone else, where his life or liberty can be held critically in balance. A public trial aims to ensure that he is fairly dealt with and would not be unjustly condemned and that his rights are not compromised. A public trial is not synonymous with publicized trial; it only implies that the court doors must be open to those who wish to come, sit in the available seats, conduct themselves with decorum and observe the trial process. In the constitutional sense, a courtroom should have enough facilities for a reasonable number of the public to observe the proceedings, not too small as to render the openness negligible and not too large as to distract the trial participants from their proper functions, who shall then be totally free to report what they have observed during the proceedings. Ruling: WHEREFORE, an audio-visual recording of the trial of former President Estrada before the Sandiganbayan is hereby ordered to be made, for the account of the Sandiganbayan, under the following conditions: (a) the trial shall be recorded in its entirety, excepting such portions thereof as the Sandiganbayan may determine should not be held public under Rule 119, 21 of the Rules of Criminal Procedure; (b) cameras shall be installed inconspicuously inside the courtroom and the movement of TV crews shall be regulated consistent with the dignity and solemnity of the proceedings; (c) the audio-visual recordings shall be made for documentary purposes only and shall be made without comment except such annotations of scenes depicted therein as may be necessary to explain them; (d) the live broadcast of the recordings before the Sandiganbayan shall have rendered its decision in all the cases against the former President shall be prohibited under pain of contempt of court and other sanctions in case of violations of the prohibition; (e) to ensure that the conditions are observed, the audio-visual recording of the proceedings shall be made under the supervision and control of the Sandiganbayan or its Division concerned and shall be made pursuant to rules promulgated by it; and (f) simultaneously with the release of the audio-visual recordings for public broadcast, the original thereof shall be deposited in the National

Museum and the Records Management and Archives Office for preservation and exhibition in accordance with law.

its people, or with a dangerous tendency to encourage the commission of violence or of a wrong or crime, such as but not limited to:

MTRCB vs. ABS-CBN [G.R. No. 155282, January 17 2005]

d) To supervise, regulate, and grant, deny or cancel, permits for the importation, exportation, production, copying, distribution, sale, lease, exhibition, and/or television broadcast of all motion pictures, television programs and publicity materials, to the end and that no such pictures, programs and materials as are determined by the BOARD to be objectionable in accordance with paragraph (c) hereof shall be imported, exported, produced, copied, reproduced, distributed, sold, leased, exhibited and/or broadcast by television

Facts: Respondent abs-cbn aired “Prosti-tuition”, an episode of the TV program “The Inside Story” produced and hosted by respondent Legarda. It depicted female students moonlighting as prostitutes to enable them to pay for their tuition fees. PWU was named as the school of some of the students involved and the façade of the PWU building served as the background of the episode. This caused upsoar in the PWU community and they filed a letter-complaint to the MTRCB. MTRCB alleged that respondents 1) Did not submit “the inside story” to petitioner for review 2) Exhibited the same without its permission, thus violating sec 7 of PD 1986 and some sections of MTRCB rules and regulations ABS-CBN averred: 1) The Inside Story is a public affairs program, news documentary and socio-political editorial, its airing is protected by the constitutional provision on freedom of expression and of the press 2) Petitioners has no power, authority and jurisdiction to impose any form of prior restraint upon respondents. After hearing and submission of the parties’ memoranda, MTRCB investigating committee ordered the respondents to pay P20,000 for non-submission of the program MTRCB affirmed the ruling Respondents filed a special civil action for certiorari with RTC QC. RTC rendered a decision in favor of respondents, annulling and setting aside the decision and resolution of the MTRCB and declaring and decreeing that certain sections of PD 1986 & MTRCB do not cover the TV program “Inside Story”, they being a public affairs programs which can be equated to a newspaper Hence, this petition Doctrines: The present controversy brings into focus the provisions of Section 3 of P. D. No. 1986, partly reproduced as follows: SEC. 3. Powers and Functions. The BOARD shall have the following functions, powers and duties: b) To screen, review and examine all motion pictures as herein defined, television programs, including publicity materials such as advertisements, trailers and stills, whether such motion pictures and publicity materials be for theatrical or non-theatrical distribution, for television broadcast or for general viewing, imported or produced in the Philippines, and in the latter case, whether they be for local viewing or for export. c) To approve or disapprove, delete objectionable portions from and/or prohibit the importation, exportation, production, copying, distribution, sale, lease exhibition and/or television broadcast of the motion pictures, television programs and publicity materials subject of the preceding paragraph, which, in the judgment of the BOARD applying contemporary Filipino cultural values as standard, are objectionable for being immoral, indecent, contrary to law and/or good customs, injurious to the prestige of the Republic of the Philippines or

Vis-a-vis the foregoing provisions, our task is to decide whether or not petitioner has the power to review the television program The Inside Story. The task is not Herculean because it merely resurrects this Court En Bancs ruling in Iglesia ni Cristo vs. Court of Appeals.[26] There, the Iglesia ni Cristo sought exception from petitioners review power contending that the term television programs under Sec. 3 (b) does not include religious programs which are protected under Section 5, Article III of the Constitution.[27] This Court, through Justice Reynato Puno, categorically ruled that P.D. No. 1986 gives petitioner the power to screen, review and examine all television programs, emphasizing the phrase all television programs, thus: The law gives the Board the power to screen, review and examine all television programs. By the clear terms of the law, the Board has the power to approve, delete x x x and/or prohibit the x x x exhibition and/or television broadcast of x x x television programs x x x. The law also directs the Board to apply contemporary Filipino cultural values as standard to determine those which are objectionable for being immoral, indecent, contrary to law and/or good customs, injurious to the prestige of the Republic of the Philippines and its people, or with a dangerous tendency to encourage the commission of violence or of a wrong or crime. Settled is the rule in statutory construction that where the law does not make any exception, courts may not except something therefrom, unless there is compelling reason apparent in the law to justify it.[28] Ubi lex non distinguit nec distinguere debemos. Thus, when the law says all television programs, the word all covers all television programs, whether religious, public affairs, news documentary, etc.[29] The principle assumes that the legislative body made no qualification in the use of general word or expression.[30] It then follows that since The Inside Story is a television program, it is within the jurisdiction of the MTRCB over which it has power of review. Here, respondents sought exemption from the coverage of the term television programs on the ground that the The Inside Story is a public affairs program, news documentary and socio-political editorial protected under Section 4,[31] Article III of the Constitution. Albeit, respondents basis is not freedom of religion, as in Iglesia ni Cristo,[32] but freedom of expression and of the press, the ruling in Iglesia ni Cristo applies squarely to the instant issue. It is significant to note that in Iglesia ni Cristo, this Court declared that freedom of religion has been accorded a preferred status by the framers of our fundamental laws, past and present, designed to protect the broadest possible liberty of conscience, to allow each man to believe as his conscience directs x x x. Yet despite the fact that freedom of religion has been accorded a preferred status, still this Court, did not exempt the Iglesia ni Cristos religious program from petitioners review power. Respondents claim that the showing of The Inside Story is protected by the constitutional provision on freedom of speech and of the press. However, there has been no declaration at all by the framers of the Constitution that freedom of expression and of the press has a preferred status. If this Court, in Iglesia ni Cristo, did not exempt religious programs from the jurisdiction and review power of petitioner MTRCB, with

more reason, there is no justification to exempt therefrom The Inside Story which, according to respondents, is protected by the constitutional provision on freedom of expression and of the press, a freedom bearing no preferred status. The only exceptions from the MTRCBs power of review are those expressly mentioned in Section 7 of P. D. No. 1986, such as (1) television programs imprinted or exhibited by the Philippine Government and/or its departments and agencies, and (2) newsreels. Thus: SEC. 7. Unauthorized showing or exhibition. It shall be unlawful for any person or entity to exhibit or cause to be exhibited in any moviehouse, theatre, or public place or by television within the Philippines any motion picture, television program or publicity material, including trailers, and stills for lobby displays in connection with motion pictures, not duly authorized by the owner or his assignee and passed by the BOARD; or to print or cause to be printed on any motion picture to be exhibited in any theater or public place or by television a label or notice showing the same to have been officially passed by the BOARD when the same has not been previously authorized, except motion pictures, television programs or publicity material imprinted or exhibited by the Philippine Government and/or its departments and agencies, and newsreels. Still in a desperate attempt to be exempted, respondents contend that the The Inside Story falls under the category of newsreels. Their contention is unpersuasive. P. D. No. 1986 does not define newsreels. Websters dictionary defines newsreels as short motion picture films portraying or dealing with current events.[33] A glance at actual samples of newsreels shows that they are mostly reenactments of events that had already happened. Some concrete examples are those of Dziga Vertovs Russian KinoPravda newsreel series (Kino-Pravda means literally film-truth, a term that was later translated literally into the French cinema verite) and Frank Capras Why We Fight series.[34] Apparently, newsreels are straight presentation of events. They are depiction of actualities. Correspondingly, the MTRCB Rules and Regulations[35] implementing P. D. No. 1986 define newsreels as straight news reporting, as distinguished from news analyses, commentaries and opinions. Talk shows on a given issue are not considered newsreels.[36] Clearly, the The Inside Story cannot be considered a newsreel. It is more of a public affairs program which is described as a variety of news treatment; a cross between pure television news and news-related commentaries, analysis and/or exchange of opinions.[37] Certainly, such kind of program is within petitioners review power.

Issues: (1) Will a purported violation of law such as the AntiWiretapping Law justify straitjacketing the exercise of freedom of speech and of the press? (2) Did the mere press statements of respondents DOJ Secretary and the NTC constitute a form of contentbased prior restraint that has transgressed the Constitution?

Held: (1) No, a purported violation of law such as the AntiWiretapping Law will not justify straitjacketing the exercise of freedom of speech and of the press. A governmental action that restricts freedom of speech or of the press based on content is given the strictest scrutiny, with the government having the burden of overcoming the presumed unconstitutionality by the clear and present danger rule. This rule applies equally to all kinds of media, including broadcast media. Respondents, who have the burden to show that these acts do not abridge freedom of speech and of the press, failed to hurdle the clear and present danger test. For this failure of the respondents alone to offer proof to satisfy the clear and present danger test, the Court has no option but to uphold the exercise of free speech and free press. There is no showing that the feared violation of the antiwiretapping law clearly endangers the national security of the State.

(2) Yes, the mere press statements of respondents DOJ Secretary and the NTC constituted a form of content-based prior restraint that has transgressed the Constitution. It is not decisive that the press statements made by respondents were not reduced in or followed up with formal orders or circulars. It is sufficient that the press statements were made by respondents while in the exercise of their official functions. Any act done, such as a speech uttered, for and on behalf of the government in an official capacity is covered by the rule on prior restraint. The concept of an “act” does not limit itself to acts already converted to a formal order or official circular. Otherwise, the non formalization of an act into an official order or circular will result in the easy circumvention of the prohibition on prior restraint. Doctrines: When the issue involves freedom of expression, as in the present case, any citizen has the right to bring suit to question the constitutionality of a government action in violation of freedom of expression, whether or not the government action is directed at such citizen.

Overview of Freedom of Expression, Prior Restraint and Subsequent Punishment

Chavez vs. Gonzales [G.R. No. 168338, February 15 2008] Facts: As a consequence of the public release of copies of the “Hello Garci” compact disc audiotapes involving a wiretapped mobile phone conversation between then-President Gloria Arroyo and Comelec Commissioner Virgilio Garcillano, respondent DOJ Secretary Gonzales warned reporters that those who had copies of the CD and those broadcasting or publishing its contents could be held liable under the Anti-Wiretapping Act. He also stated that persons possessing or airing said tapes were committing a continuing offense, subject to arrest by anybody. Finally, he stated that he had ordered the NBI to go after media organizations “found to have caused the spread, the playing and the printing of the contents of a tape.” Meanwhile, respondent NTC warned TV and radio stations that their broadcast/airing of such false information and/or willful misrepresentation shall be a just cause for the suspension, revocation and/or cancellation of the licenses or authorizations issued to the said media establishments. Petitioner Chavez filed a petition under Rule 65 against respondents Secretary Gonzales and the NTC directly with the Supreme Court.

Freedom of expression is the foundation of a free, open and democratic society. Freedom of expression is an indispensable condition to the exercise of almost all other civil and political rights. No society can remain free, open and democratic without freedom of expression. Freedom of expression guarantees full, spirited, and even contentious discussion of all social, economic and political issues. To survive, a free and democratic society must zealously safeguard freedom of expression.

Freedom of expression allows citizens to expose and check abuses of public officials. Freedom of expression allows citizens to make informed choices of candidates for public office. Freedom of expression crystallizes important public policy issues, and allows citizens to participate in the discussion and resolution of such issues. Freedom of expression allows the competition of ideas, the clash of claims and counterclaims, from which the truth will likely emerge. Freedom of expression allows the airing of social grievances, mitigating sudden eruptions of violence from marginalized groups who otherwise would not be heard by government. Freedom of expression

provides a civilized way of engagement among political, ideological, religious or ethnic opponents for if one cannot use his tongue to argue, he might use his fist instead.

Section 4, Article III of the Constitution prohibits the enactment of any law curtailing freedom of expression: “No law shall be passed abridging the freedom of speech, of expression, or the press, or the right of the people peaceably to assemble and petition the government for redress of grievances.” Thus, the rule is that expression is not subject to any prior restraint or censorship because the Constitution commands that freedom of expression shall not be abridged. Over time, however, courts have carved out narrow and well defined exceptions to this rule out of necessity. Exceptions: The exceptions, when expression may be subject to prior restraint, apply in this jurisdiction to only four categories of expression namely: pornography,

Party List Buhay Gabriela, Akbayan, Bayan Muna, Anak Pawis Party List Ang Pamilya Respondent Atty. Mavil V. Majarucon, as Election Officer of Bacolod City, issued a Notice to Remove Campaign Materials addressed to petitioner Most Rev. Bishop Vicente M. Navarra, otherwise, COMELEC will be constrained to file an election offense against the petitioners. Doctrines: Exhaustion of administrative remedies will prolong the violation of freedom of expression in the case at bar; Hierarchy of Protected Expressions Political speech enjoys preferred protection within our constitutional order. In Chavez v. Gonzales, Justice Carpio in a separate opinion emphasized: "[i]f ever there is a hierarchy of protected expressions, political expression would occupy the highest rank, and among different kinds of political expression, the subject of fair and honest elections would be at the top." Sovereignty resides in the people. Political speech is a direct exercise of the sovereignty. The principle of exhaustion of administrative remedies yields in order to protect this fundamental right.

false or misleading advertisement, advocacy of imminent lawless action, and danger to national security. All other expression is not subject to prior restraint. The history of press freedom has been a constant struggle against the censor whose weapon is the suspension or cancellation of licenses to publish or broadcast. The NTC warning resurrects the weapon of the censor. In sum, the NTC press release constitutes an unconstitutional prior restraint on protected expression. There can be no content-based prior restraint on protected expression. This rule has no exception.

Even assuming that the principle of exhaustion of administrative remedies is applicable, the current controversy is within the exceptions to the principle.

When is prior exhaustion of administrative remedies may be dispensed with and judicial action may be validly resorted to immediately? (a)

when there is a violation of due process;

(b)

when the issue involved is purely a legal question;

Diocese of Bacolod vs Comelec [G.R. No. 2015] FACTS:On February 2013, petitioners posted two (2) tarpaulins within the compound of San Sebastian Cathedral of Bacolod. Each tarpaulin was approximately 6×10 in size. They were posted on the front walls of the cathedral within public view. The first tarpaulin contains the message “IBASURA RH Law” referring to the Reproductive Health Law of 2012 or Republic Act No. 10354. The second tarpaulin is the subject of the present case. This tarpaulin contains the heading “Conscience Vote” and lists candidates as either “(Anti-RH)/ Team Buhay” or “(Pro-RH)/Team Patay”. The electoral candidates were classified according to their vote on the adoption of the RH Law. Those who voted for the passing of the law were classified by petitioners as comprising “Team Patay,” while those who voted against it form “Team Buhay”: TEAM BUHAY Estrada, JV

TEAM PATAY Angara, Juan Edgardo

Honasan, Gregorio

Casiño, Teddy

Magsaysay, Mitos

Cayetano, Alan Peter

Pimentel, Koko

Enrile, Jackie

Trillanes, Antonio

Escudero, Francis

Villar, Cynthia

Hontiveros, Risa

*Party List

Legarda, Loren

(c) when the administrative action is patently illegal amounting to lack or excess of jurisdiction; (d) when there is estoppel on the part ofthe administrative agency concerned; (e)

when there is irreparable injury;

(f) when the respondent is a department secretary whose acts as analter ego of the President bear the implied and assumed approval of the latter; (g) when to require exhaustion of administrative remedies would be unreasonable; (h)

when it would amount to a nullification of a claim;

(i) when the subject matter is a private land in land case proceedings; (j) when the rule does not provide a plain, speedy and adequate remedy; or (k) when there are circumstances indicating the urgency of judicial intervention.

COMELEC had no legal basis to regulate expressions made by private citizens Respondents cite the Constitution, laws, and jurisprudence to support their position that they had the power to regulate the tarpaulin.

However, all of these provisions pertain to candidates and political parties. Petitioners are not candidates. Neither do theybelong to any political party. COMELEC does not have the authority to regulate the enjoyment of the preferred right to freedom of expression exercised by a non-candidate in this case.

not, add the words AND EXPRESSION after the word "speech," because it is more expansive, it has a wider scope, and it would refer to means of expression other than speech.

THE PRESIDING OFFICER (Mr.Bengzon): What does the Committee say? Tarpaulin Respondents considered the tarpaulin as a campaign material in their issuances. The above provisions regulating the posting of campaign materials only apply to candidates and political parties, and petitioners are neither of the two.

FR. BERNAS: "Expression" is more broad than speech. We accept it.

MR. BROCKA: Thank you. The tarpaulin was not paid for by any candidate or political party. There was no allegation that petitioners coordinated with any of the persons named in the tarpaulin regarding its posting. On the other hand, petitioners posted the tarpaulin as part of their advocacy against the RH Law.

THE PRESIDING OFFICER (Mr.Bengzon): Is it accepted?

FR. BERNAS: Yes. True, there is no mention whether election campaign is limited only to the candidates and political parties themselves. The focus of the definition is that the act must be "designed to promote the election or defeat of a particular candidate or candidates to a public office."

THE PRESIDING OFFICER (Mr.Bengzon): Is there any objection? (Silence) The Chair hears none; the amendment is approved.

In this case, the tarpaulin contains speech on a matter of public concern, that is, a statement of either appreciation or criticism on votes made in the passing of the RH law. Thus, petitioners invoke their right to freedom of expression.

FR. BERNAS: So, that provision will now read: "No law shall be passed abridging the freedom of speech, expression or of the press . . . ." Speech may be said to be inextricably linked to freedom itself as "[t]he right to think is the beginning of freedom, and speech must be protected from the government because speech is the beginning of thought."

Does Article III Section 4 of the Constitution also apply to other governmental acts even if it expressly states that “NO LAW…”? YES

Communication

While it is true that the present petition assails not a law but an opinion by the COMELEC Law Department, this court has applied Article III, Section 4 of the Constitution even to governmental acts.

Communication is an essential outcome of protected speech. Communication exists when " (1) a speaker, seeking to signal others, uses conventional actions because he or she reasonably believes that such actions will be taken by the audience in the manner intended; and

. . . shall be passed abridging. . . (2) All regulations will have an impact directly or indirectly on expression. The prohibition against the abridgment of speech should not mean an absolute prohibition against regulation. The primary and incidental burden on speech must be weighed against a compelling state interest clearly allowed in the Constitution. The test depends on the relevant theory of speech implicit in the kind of society framed by our Constitution.

Of Expression explained Our Constitution has also explicitly included the freedom of expression, separate and in addition to the freedom of speech and of the press provided in the US Constitution. The word "expression" was added in the 1987 Constitution by Commissioner Brocka for having a wider scope:

MR. BROCKA: This is a very minor amendment, Mr. Presiding Officer. On Section 9, page 2, line 29, it says: "No law shall be passed abridging the freedom of speech." I would like to recommend to the Committee the change of the word "speech" to EXPRESSION; or if

the audience so takes the actions." "

[I]n communicative action[,] the hearer may respond to the claims by . . . either accepting the speech act’s claims or opposing them with criticism or requests for justification." Speech Speech is not limited to vocal communication. "[C]onduct is treated as a form of speech sometimes referred to as ‘symbolic speech[,]’"such that "‘when ‘speech’ and ‘nonspeech’ elements are combined in the same course of conduct,’ the ‘communicative element’ of the conduct may be ‘sufficient to bring into play the [right to freedom of expression].’"

The right to freedom of expression, thus, applies to the entire continuum of speech from utterances made to conduct enacted, and even to inaction itself as a symbolic manner of communication.

Freedom of Expression includes the Right to be Silent Freedom of speech includes the right to be silent. Aptly has it been said that the Bill of Rights that guarantees to the individual the liberty to utter what is in his mind also guarantees to him the liberty not to utter what is not in his mind.

Citing EBRALINAG vs. DIVISION SUPERINTENDENT OF CEBU: The democratic system provides for the accommodation of diverse ideas, including the unconventional and even the bizarre or eccentric. The will of the majority prevails, but it cannot regiment thought by prescribing the recitation by rote of its opinions or proscribing the assertion of unorthodox or unpopular views as inthis case. The conscientious objections of the petitioners, no less than the impatience of those who disagree with them, are protected by the Constitution. The State cannot make the individual speak when the soul within rebels. Size Matters

Borrowing the words of Justice Brandeis, "it is hazardous to discourage thought, hope and imagination; that fear breeds repression; that repression breeds hate; that hate menaces stable government; that the path of safety lies in the opportunity to discuss freely supposed grievances and proposed remedies."

In this jurisdiction, this court held that "[t]he interest of society and the maintenance of good government demand a full discussion of public affairs." This court has, thus, adopted the principle that "debate on public issues should be uninhibited, robust,and wide open . . . [including even] unpleasantly sharp attacks on government and public officials.”

Second, free speech should be encouraged under the concept of a market place of ideas. This theory was articulated by Justice Holmes in that "the ultimate good desired is better reached by [the] free trade in ideas:"

Size does matter The form of expression is just as important as the information conveyed that it forms part of the expression. The present case is in point. It is easy to discern why size matters. First, it enhances efficiency in communication. A larger tarpaulin allows larger fonts which make it easier to view its messages from greater distances. Second, the size of the tarpaulin may underscore the importance of the message to the reader. From an ordinary person’s perspective, those who post their messages in larger fonts care more about their message than those who carry their messages in smaller media. Third, larger spaces allow for more messages. Larger spaces, therefore, may translate to more opportunities to amplify, explain, and argue points which the speakers might want to communicate. Large tarpaulins, therefore, are not analogous to time and place. They are fundamentally part of expression protected under Article III, Section 4 of the Constitution. Theories and Schools of Thought for the protection of this right First, this relates to the right of the people to participate in public affairs, including the right to criticize government actions. Proponents of the political theory on "deliberative democracy" submit that "substantial, open, [and] ethical dialogue is a critical, and indeed defining, feature of a good polity." This theory may be considered broad, but it definitely "includes [a] collective decision making with the participation of all who will be affected by the decision." It anchors on the principle that the cornerstone of every democracy is that sovereignty resides in the people. To ensure order in running the state’s affairs, sovereign powers were delegated and individuals would be elected or nominated in key government positions to represent the people. On this note, the theory on deliberative democracy may evolve to the right of the people to make government accountable. Necessarily, this includes the right of the people to criticize acts made pursuant to governmental functions.

Speech that promotes dialogue on public affairs, or airs out grievances and political discontent, should thus be protected and encouraged.

When men have realized that time has upset many fighting faiths, they may come to believe even more than they believe the very foundations of their own conduct that the ultimate good desired is better reached by free trade in ideas - that the best test of truth is the power of the thought to get itself accepted in the competition of the market, and that truth is the only ground upon which their wishes safely can be carried out. The way it works, the exposure to the ideas of others allows one to "consider, test, and develop their own conclusions." A free, open, and dynamic market place of ideas is constantly shaping new ones. This promotes both stability and change where recurring points may crystallize and weak ones may develop. Of course, free speech is more than the right to approve existing political beliefs and economic arrangements as it includes, "[t]o paraphrase Justice Holmes, [the] freedom for the thought that we hate, no less than for the thought that agrees with us." In fact, free speech may "best serve its high purpose when it induces a condition of unrest, creates dissatisfaction with conditions as they are, or even stirs people to anger." It is in this context that we should guard against any curtailment of the people’s right to participate in the free trade of ideas. Third, free speech involves self-expression that enhances human dignity. This right is "a means of assuring individual self-fulfillment," among others. In PHILIPPINE BLOOMING MILLS EMPLOYEES ORGANIZATION V. PHILIPPINE BLOOMING MILLS CO., INC, this court discussed as follows: The rights of free expression, free assembly and petition, are not only civil rights but also political rights essential to man's enjoyment of his life, to his happiness and to his full and complete fulfillment. Thru these freedoms the citizens can participate not merely in the periodic establishment of the government through their suffrage but also in the administration of public affairs as well as in the discipline of abusive public officers. The citizen is accorded these rights so that he can appeal to the appropriate governmental officers or agencies for redress and protection as well as for the imposition of the lawful sanctions on erring public officers and employees. (Emphasis supplied) Fourth, expression is a marker for group identity. For one, "[v]oluntary associations perform [an] important democratic role [in providing] forums for the development of civil skills, for deliberation, and for the formation of identity and community spirit[,] [and] are largely immune from [any] governmental interference." They also "provide a buffer between individuals and the state - a free space for the development of individual personality, distinct group identity,

and dissident ideas - and a potential source of opposition to the state." Free speech must be protected as the vehicle to find those who have similar and shared values and ideals, to join together and forward common goals.

Are all speech treated the same? NO

Fifth, the Bill of Rights, free speech included, is supposed to "protect individuals and minorities against majoritarian abuses perpetrated through [the] framework [of democratic governance]."

Some types of speech may be subjected to some regulation by the State under its pervasive police power, in order that it may not be injurious to the equal right of others or those of the community or society. The difference in treatment is expected because the relevant interests of one type of speech, e.g., political speech, may vary from those of another, e.g., obscene speech. Distinctions have therefore been made in the treatment, analysis, and evaluation of the permissible scope of restrictions on various categories of speech. We have ruled, for example, that in our jurisdiction slander or libel, lewd and obscene speech, as well as "fighting words" are not entitled to constitutional protection and may be penalized.

Federalist framers led by James Madison were concerned about two potentially vulnerable groups: 1. "the citizenry at large - majorities - who might be tyrannized or plundered by despotic federal officials" and 2. the minorities who may be oppressed by "dominant factions of the electorate [that] capture [the] government for their own selfish ends[.]"

Not all speech are treated the same. In CHAVEZ V. GONZALES, this court discussed that some types of speech may be subject to regulation:

Political and Commercial Speech distinguished

According to Madison, "[i]t is of great importance in a republic not only to guard the society against the oppression of its rulers, but to guard one part of the society against the injustice of the other part." We should strive to ensure that free speech is protected especially in light of any potential oppression against those who find themselves in the fringes on public issues.

Political speech refers to speech "both intended and received as a contribution to public deliberation about some issue," "foster[ing] informed and civicminded deliberation."

Lastly, free speech must be protected under the safety valve theory.

In the case at bar: The expression resulting from the content of the tarpaulin is, however, definitely political speech.

This provides that "nonviolent manifestations of dissent reduce the likelihood of violence[.]" "[A] dam about to burst . . . resulting in the ‘banking up of a menacing flood of sullen anger behind the walls of restriction’" has been used to describe the effect of repressing nonviolent outlets. In order to avoid this situation and prevent people from resorting to violence, there is a need for peaceful methods in making passionate dissent. This includes "free expression and political participation"in that they can "vote for candidates who share their views, petition their legislatures to [make or] change laws, . . . distribute literature alerting other citizens of their concerns[,]"and conduct peaceful rallies and other similar acts. Free speech must, thus, be protected as a peaceful means of achieving one’s goal, considering the possibility that repression of nonviolent dissent may spill over to violent means just to drive a point.

On the other hand, commercial speech has been defined as speech that does "no more than propose a commercial transaction."

In Justice Brion’s dissenting opinion, he discussed that "[t]he content of the tarpaulin, as well as the timing of its posting, makes it subject of the regulations in RA 9006 and Comelec Resolution No. 9615." He adds that "[w]hile indeed the RH issue, by itself,is not an electoralmatter, the slant that the petitioners gave the issue converted the non-election issue into a live election one hence, Team Buhay and Team Patay and the plea to support one and oppose the other." Debate on public issues We have adopted the principle that debate on public issues should be uninhibited, robust, and wide open and that it may well include vehement, caustic and sometimes unpleasantly sharp attacks on government and public officials. Too many restrictions will deny to people the robust, uninhibited, and wide open debate, the generating of interest essential if our elections will truly be free, clean and honest. Speech with political consequences

Free speech and other intellectual freedoms This court has held free speech and other intellectual freedoms as "highly ranked in our scheme of constitutional values.” These rights enjoy precedence and primacy. Are Human Rights can be lost through prescriptions? Property and property rights can be lost thru prescription; but human rights are imprescriptible. If human rights are extinguished by the passage of time, then the Bill of Rights is a useless attempt to limit the power of government and ceases to be an efficacious shield against the tyranny of officials, of majorities, of the influential and powerful, and of oligarchs - political, economic or otherwise. Hierarchy of Civil Liberties In the hierarchy of civil liberties, the rights of free expression and of assembly occupy a preferred position as they are essential to the preservation and vitality of our civil and political institutions; and such priority "gives these liberties the sanctity and the sanction not permitting dubious intrusions." This primordial right calls for utmost respect, more so "when what may be curtailed is the dissemination of information to make more meaningful the equally vital right of suffrage."

Speech with political consequences is at the core of the freedom of expression and must be protected by this court. Justice Brion pointed out that freedom of expression "is not the god of rights to which all other rights and even government protection of state interest must bow." Is the Freedom of Expression absolute? NO The right to freedom of expression is indeed not absolute. Even some forms of protected speech are still subject to some restrictions. The degree of restriction may depend on whether the regulation is contentbased or content-neutral. Content-based regulations can either be based on the viewpoint of the speaker or the subject of the expression. Content-based Regulation Size limitations during elections hit at a core part of expression. The content of the tarpaulin is not easily divorced from the size of its medium. Content-based regulation bears a heavy presumption of invalidity, and this court has used the clear and present danger rule as measure. Thus, in CHAVEZ V. GONZALES:

A content-based regulation, however, bears a heavy presumption of invalidity and is measured against the clear and present danger rule. The latter will pass constitutional muster only if justified by a compelling reason, and the restrictions imposed are neither overbroad nor vague. Under this rule, "the evil consequences sought to be prevented must be substantive, ‘extremely serious and the degree of imminence extremely high.’" "Only when the challenged act has overcome the clear and present danger rule will it pass constitutional muster, with the government having the burden of overcoming the presumed unconstitutionality." Even with the clear and present danger test, respondents failed to justify the regulation. There is no compelling and substantial state interest endangered by the posting of the tarpaulinas to justify curtailment of the right of freedom of expression. There is no reason for the state to minimize the right of non-candidate petitioners to post the tarpaulin in their private property. The size of the tarpaulin does not affect anyone else’s constitutional rights.

there can be underhanded, covert, or illicit dealings so as to hide the candidate’s real levels of expenditures. However, labelling all expressions of private parties that tend to have an effect on the debate in the elections as election paraphernalia would be too broad a remedy that can stifle genuine speech like in this case. Instead, to address this evil, better and more effective enforcement will be the least restrictive means to the fundamental freedom. Satirical elements of the tarpaulins The twin tarpaulins consist of satire of political parties. Satire is a "literary form that employs such devices as sarcasm, irony and ridicule to deride prevailing vices or follies," and this may target any individual or group in society, private and government alike. It seeks to effectively communicate a greater purpose, often used for "political and social criticism""because it tears down facades, deflates stuffed shirts, and unmasks hypocrisy. . . . Nothing is more thoroughly democratic than to have the high-and-mighty lampooned and spoofed." Northrop Frye, well known in this literary field, claimed that satire had two defining features:

Content-based restraint or censorship Content-based restraint or censorship refers to restrictions "based on the subject matter of the utterance or speech." In contrast, contentneutral regulation includes controls merely on the incidents of the speech such as time, place, or manner of the speech.

"one is wit or humor founded on fantasy or a sense of the grotesque and absurd, the other is an object of attack." Thus, satire frequently uses exaggeration, analogy, and other rhetorical devices.

Intermediate Approach & Content-neutral government regulation

Two paradigms of free speech

This intermediate approach is based on the test that we have prescribed in several cases.

1.

Equality

2.

Liberty

A content-neutral government regulation is sufficiently justified: (1)

if it is within the constitutional power of the Government;

(2) if it furthers an important or substantial governmental interest; (3) if the governmental interest is unrelated to the suppression of free expression; and (4) if the incident restriction on alleged [freedom of speech & expression] is no greater than is essential to the furtherance of that interest. On the first requisite, it is not within the constitutional powers of the COMELEC to regulate the tarpaulin. As discussed earlier, this is protected speech by petitioners who are non-candidates. On the second requirement, not only must the governmental interest be important or substantial, it must also be compelling as to justify the restrictions made. The third requisite is likewise lacking. We look not only at the legislative intent or motive in imposing the restriction, but more so at the effects of such restriction, if implemented. The restriction must not be narrowly tailored to achieve the purpose. It must be demonstrable. It must allow alternative avenues for the actor to make speech. In this case, the size regulation is not unrelated to the suppression of speech. Limiting the maximum sizeof the tarpaulin would render ineffective petitioners’ message and violate their right to exercise freedom of expression. The restriction in the present case does not pass even the lower test of intermediate scrutiny for content-neutral regulations. In relation to the possibility of abuse The guarantee of freedom of expression to individuals without any relationship to any political candidate should not be held hostage by the possibility of abuse by those seeking to be elected. It is true that

Speech and Equality In an equality-based approach, "politically disadvantaged speech prevails over regulation[,] but regulation promoting political equality prevails over speech." This view allows the government leeway to redistribute or equalize ‘speaking power,’ such as protecting, even implicitly subsidizing, unpopular or dissenting voices often systematically subdued within society’s ideological ladder. This view acknowledges that there are dominant political actors who, through authority, power, resources, identity, or status, have capabilities that may drown out the messages of others. This is especially true in a developing or emerging economy that is part of the majoritarian world like ours. Speech and Libertarian Tolerance This balance between equality and the ability to express so as to find one’s authentic self or to participate in the self determination of one’s communities is not new only to law. It has always been a philosophical problematique. In his seminal work, Repressive Tolerance, philosopher and social theorist Herbert Marcuse recognized how institutionalized inequality exists as a background limitation, rendering freedoms exercised within such limitation as merely "protect[ing] the already established machinery of discrimination." In his view, any improvement "in the normal course of events" within an unequal society, without subversion, only strengthens existing interests of those in power and control. In other words, abstract guarantees of fundamental rights like freedom of expression may become meaningless if not taken in a real context. This tendency to tackle rights in the abstract compromises liberties. In his words: Liberty is self-determination, autonomy—this is almost a tautology, but a tautology which results from a whole series of synthetic judgments. It stipulates the ability to determine one’s own life: to be

able to determine what to do and what not to do, what to suffer and what not. But the subject of this autonomy is never the contingent, private individual as that which he actually is or happens to be; it is rather the individual as a human being who is capable of being free with the others. And the problem of making possible such a harmony between every individual liberty and the other is not that of finding a compromise between competitors, or between freedom and law, between general and individual interest, common and private welfare in an established society, but of creating the society in which man is no longer enslaved by institutions which vitiate self-determination from the beginning. In other words, freedom is still to be created even for the freest of the existing societies.

RULING: VI. Claims of press freedom and religious liberty. We have held that, as a general proposition, the press is not exempt from the taxing power of the State and that what the constitutional guarantee of free press prohibits are laws which single out the press or target a group belonging to the press for special treatment or which in any way discriminate against the press on the basis of the content of the publication, and R.A. No. 7716 is none of these.

Now it is contended by the PPI that by removing the exemption of the press from the VAT while maintaining those granted to others, the law discriminates against the press. At any rate, it is averred, "even nondiscriminatory taxation of constitutionally guaranteed freedom is unconstitutional."

Regulation of speech made by non-candidates Regulation of speech in the context of electoral campaigns made by persons who are not candidates or who do not speak as members of a political party which are, taken as a whole, principally advocacies of a social issue that the public must consider during elections is unconstitutional. Such regulation is inconsistent with the guarantee of according the fullest possible range of opinions coming from the electorate including those that can catalyze candid, uninhibited, and robust debate in the criteria for the choice of a candidate. Regulation of election paraphernalia will still be constitutionally valid if it reaches into speech of persons who are not candidates or who do not speak as members of a political party if they are not candidates, only if what is regulated is declarative speech that, taken as a whole, has for its principal object the endorsement of a candidate only. The regulation (a)

should be provided by law,

(b)

reasonable,

(c) narrowly tailored to meet the objective of enhancing the opportunity of all candidates to be heard and considering the primacy of the guarantee of free expression, and (d) object.

demonstrably the least restrictive means to achieve that

The regulation must only be with respect to the time, place, and manner of the rendition of the message. In no situation may the speech be prohibited or censored on the basis of its content. For this purpose, it will not matter whether the speech is made with or on private property. Tarpaulins in this case do not pertain to religious speech With all due respect to the Catholic faithful, the church doctrines relied upon by petitioners are not binding upon this court. The position of the Catholic religion in the Philippines as regards the RH Law does not suffice to qualify the posting by one of its members of a tarpaulin as religious speech solely on such basis. The enumeration of candidates on the face of the tarpaulin precludes any doubt as to its nature as speech with political consequences and not religious speech. Tolentino v. Sec of Finance GR No. 115455, Aug 25 1994 FACTS: The valued-added tax (VAT) is levied on the sale, barter or exchange of goods and properties as well as on the sale or exchange of services. It is equivalent to 10% of the gross selling price or gross value in money of goods or properties sold, bartered or exchanged or of the gross receipts from the sale or exchange of services. Republic Act No. 7716 seeks to widen the tax base of the existing VAT system and enhance its administration by amending the National Internal Revenue Code.

With respect to the first contention, it would suffice to say that since the law granted the press a privilege, the law could take back the privilege anytime without offense to the Constitution. The reason is simple: by granting exemptions, the State does not forever waive the exercise of its sovereign prerogative.

Indeed, in withdrawing the exemption, the law merely subjects the press to the same tax burden to which other businesses have long ago been subject. It is thus different from the tax involved in the cases invoked by the PPI. The license tax in Grosjean v. American Press Co., 297 U.S. 233, 80 L. Ed. 660 (1936) was found to be discriminatory because it was laid on the gross advertising receipts only of newspapers whose weekly circulation was over 20,000, with the result that the tax applied only to 13 out of 124 publishers in Louisiana. These large papers were critical of Senator Huey Long who controlled the state legislature which enacted the license tax. The censorial motivation for the law was thus evident.

On the other hand, in Minneapolis Star & Tribune Co. v. Minnesota Comm'r of Revenue, 460 U.S. 575, 75 L. Ed. 2d 295 (1983), the tax was found to be discriminatory because although it could have been made liable for the sales tax or, in lieu thereof, for the use tax on the privilege of using, storing or consuming tangible goods, the press was not. Instead, the press was exempted from both taxes. It was, however, later made to pay a special use tax on the cost of paper and ink which made these items "the only items subject to the use tax that were component of goods to be sold at retail." The U.S. Supreme Court held that the differential treatment of the press "suggests that the goal of regulation is not related to suppression of expression, and such goal is presumptively unconstitutional." It would therefore appear that even a law that favors the press is constitutionally suspect. (See the dissent of Rehnquist, J. in that case)

Nor is it true that only two exemptions previously granted by E.O. No. 273 are withdrawn "absolutely and unqualifiedly" by R.A. No. 7716. Other exemptions from the VAT, such as those previously granted to PAL, petroleum concessionaires, enterprises registered with the Export Processing Zone Authority, and many more are likewise totally withdrawn, in addition to exemptions which are partially withdrawn, in an effort to broaden the base of the tax.

The PPI says that the discriminatory treatment of the press is highlighted by the fact that transactions, which are profit oriented, continue to enjoy exemption under R.A. No. 7716. An enumeration of some of these transactions will suffice to show that by and large this is not so and that the exemptions are granted for a purpose. As the

Solicitor General says, such exemptions are granted, in some cases, to encourage agricultural production and, in other cases, for the personal benefit of the end-user rather than for profit. The exempt transactions are: (a) Goods for consumption or use which are in their original state (agricultural, marine and forest products, cotton seeds in their original state, fertilizers, seeds, seedlings, fingerlings, fish, prawn livestock and poultry feeds) and goods or services to enhance agriculture (milling of palay, corn, sugar cane and raw sugar, livestock, poultry feeds, fertilizer, ingredients used for the manufacture of feeds). (b) Goods used for personal consumption or use (household and personal effects of citizens returning to the Philippines) or for professional use, like professional instruments and implements, by persons coming to the Philippines to settle here. (c) Goods subject to excise tax such as petroleum products or to be used for manufacture of petroleum products subject to excise tax and services subject to percentage tax. (d) Educational services, medical, dental, hospital and veterinary services, and services rendered under employer-employee relationship. (e) Works of art and similar creations sold by the artist himself. (f) Transactions exempted under special laws, or international agreements. (g) Export-sales by persons not VAT-registered. (h) Goods or services with gross annual sale or receipt not exceeding P500,000.00. (Respondents' Consolidated Reconsideration, pp. 58-60)

Comment

on

the

Motions

for

The PPI asserts that it does not really matter that the law does not discriminate against the press because "even nondiscriminatory taxation on constitutionally guaranteed freedom is unconstitutional." PPI cites in support of this assertion the following statement in Murdock v. Pennsylvania, 319 U.S. 105, 87 L. Ed. 1292 (1943): The fact that the ordinance is "nondiscriminatory" is immaterial. The protection afforded by the First Amendment is not so restricted. A license tax certainly does not acquire constitutional validity because it classifies the privileges protected by the First Amendment along with the wares and merchandise of hucksters and peddlers and treats them all alike. Such equality in treatment does not save the ordinance. Freedom of press, freedom of speech, freedom of religion are in preferred position. The Court was speaking in that case of a license tax, which, unlike an ordinary tax, is mainly for regulation. Its imposition on the press is unconstitutional because it lays a prior restraint on the exercise of its right. Hence, although its application to others, such those selling goods, is valid, its application to the press or to religious groups, such as the Jehovah's Witnesses, in connection with the latter's sale of religious books and pamphlets, is unconstitutional. As the U.S. Supreme Court put it, "it is one thing to impose a tax on income or property of a preacher. It is quite another thing to exact a tax on him for delivering a sermon."

A similar ruling was made by this Court in American Bible Society v. City of Manila, 101 Phil. 386 (1957) which invalidated a city ordinance requiring a business license fee on those engaged in the sale of general merchandise. It was held that the tax could not be imposed on the sale of bibles by the American Bible Society without restraining the free exercise of its right to propagate.

The VAT is, however, different. It is not a license tax. It is not a tax on the exercise of a privilege, much less a constitutional right. It is imposed on the sale, barter, lease or exchange of goods or properties or the sale or exchange of services and the lease of properties purely for revenue purposes. To subject the press to its payment is not to burden the exercise of its right any more than to make the press pay income tax or subject it to general regulation is not to violate its freedom under the Constitution. Additionally, the Philippine Bible Society, Inc. claims that although it sells bibles, the proceeds derived from the sales are used to subsidize the cost of printing copies which are given free to those who cannot afford to pay so that to tax the sales would be to increase the price, while reducing the volume of sale. Granting that to be the case, the resulting burden on the exercise of religious freedom is so incidental as to make it difficult to differentiate it from any other economic imposition that might make the right to disseminate religious doctrines costly. Otherwise, to follow the petitioner's argument, to increase the tax on the sale of vestments would be to lay an impermissible burden on the right of the preacher to make a sermon. On the other hand the registration fee of P1,000.00 imposed by §107 of the NIRC, as amended by §7 of R.A. No. 7716, although fixed in amount, is really just to pay for the expenses of registration and enforcement of provisions such as those relating to accounting in §108 of the NIRC. That the PBS distributes free bibles and therefore is not liable to pay the VAT does not excuse it from the payment of this fee because it also sells some copies. At any rate whether the PBS is liable for the VAT must be decided in concrete cases, in the event it is assessed this tax by the Commissioner of Internal Revenue. DATA PRIVACY ACT. [REPUBLIC ACT NO. 10173] SECTION 1. Short Title. – This Act shall be known as the “Data Privacy Act of 2012”. SEC. 3. Definition of Terms. – Whenever used in this Act, the following terms shall have the respective meanings hereafter set forth:

(a) Commission shall refer to the National Privacy Commission created by virtue of this Act.

(b) Consent of the data subject refers to any freely given, specific, informed indication of will, whereby the data subject agrees to the collection and processing of personal information about and/or relating to him or her. Consent shall be evidenced by written, electronic or recorded means. It may also be given on behalf of the data subject by an agent specifically authorized by the data subject to do so.

(c) Data subject refers to an individual whose personal information is processed.

(d) Direct marketing refers to communication by whatever means of any advertising or marketing material which is directed to particular individuals.

(e) Filing system refers to any act of information relating to natural or juridical persons to the extent that, although the information is not processed by equipment operating automatically in response to instructions given for that purpose, the set is structured, either by reference to individuals or by reference to criteria relating to

individuals, in such a way that specific information relating to a particular person is readily accessible.

have been committed by such person, the disposal of such proceedings, or the sentence of any court in such proceedings;

(f) Information and Communications System refers to a system for generating, sending, receiving, storing or otherwise processing electronic data messages or electronic documents and includes the computer system or other similar device by or which data is recorded, transmitted or stored and any procedure related to the recording, transmission or storage of electronic data, electronic message, or electronic document.

(3) Issued by government agencies peculiar to an individual which includes, but not limited to, social security numbers, previous or current health records, licenses or its denials, suspension or revocation, and tax returns; and

(g) Personal information refers to any information whether recorded in a material form or not, from which the identity of an individual is apparent or can be reasonably and directly ascertained by the entity holding the information, or when put together with other information would directly and certainly identify an individual.

(h) Personal information controller refers to a person or organization who controls the collection, holding, processing or use of personal information, including a person or organization who instructs another person or organization to collect, hold, process, use, transfer or disclose personal information on his or her behalf. The term excludes:

(1) A person or organization who performs such functions as instructed by another person or organization; and

(2) An individual who collects, holds, processes or uses personal information in connection with the individual’s personal, family or household affairs.

(i) Personal information processor refers to any natural or juridical person qualified to act as such under this Act to whom a personal information controller may outsource the processing of personal data pertaining to a data subject.

(j) Processing refers to any operation or any set of operations performed upon personal information including, but not limited to, the collection, recording, organization, storage, updating or modification, retrieval, consultation, use, consolidation, blocking, erasure or destruction of data.

(k) Privileged information refers to any and all forms of data which under the Rules of Court and other pertinent laws constitute privileged communication.

(4) Specifically established by an executive order or an act of Congress to be kept classified.

SEC. 4. Scope. – This Act applies to the processing of all types of personal information and to any natural and juridical person involved in personal information processing including those personal information controllers and processors who, although not found or established in the Philippines, use equipment that are located in the Philippines, or those who maintain an office, branch or agency in the Philippines subject to the immediately succeeding paragraph: Provided, That the requirements of Section 5 are complied with.

This Act does not apply to the following:

(a) Information about any individual who is or was an officer or employee of a government institution that relates to the position or functions of the individual, including:

(1) The fact that the individual is or was an officer or employee of the government institution;

(2) The title, business address and office telephone number of the individual;

(3) The classification, salary range and responsibilities of the position held by the individual; and

(4) The name of the individual on a document prepared by the individual in the course of employment with the government;

(b) Information about an individual who is or was performing service under contract for a government institution that relates to the services performed, including the terms of the contract, and the name of the individual given in the course of the performance of those services;

(l) Sensitive personal information refers to personal information:

(1) About an individual’s race, ethnic origin, marital status, age, color, and religious, philosophical or political affiliations;

(2) About an individual’s health, education, genetic or sexual life of a person, or to any proceeding for any offense committed or alleged to

(c) Information relating to any discretionary benefit of a financial nature such as the granting of a license or permit given by the government to an individual, including the name of the individual and the exact nature of the benefit;

(d) Personal information processed for journalistic, artistic, literary or research purposes;

(c) The entity has other links in the Philippines such as, but not limited to: (e) Information necessary in order to carry out the functions of public authority which includes the processing of personal data for the performance by the independent, central monetary authority and law enforcement and regulatory agencies of their constitutionally and statutorily mandated functions. Nothing in this Act shall be construed as to have amended or repealed Republic Act No. 1405, otherwise known as the Secrecy of Bank Deposits Act; Republic Act No. 6426, otherwise known as the Foreign Currency Deposit Act; and Republic Act No. 9510, otherwise known as the Credit Information System Act (CISA);

(f) Information necessary for banks and other financial institutions under the jurisdiction of the independent, central monetary authority or Bangko Sentral ng Pilipinas to comply with Republic Act No. 9510, and Republic Act No. 9160, as amended, otherwise known as the AntiMoney Laundering Act and other applicable laws; and

(g) Personal information originally collected from residents of foreign jurisdictions in accordance with the laws of those foreign jurisdictions, including any applicable data privacy laws, which is being processed in the Philippines.

SEC. 5. Protection Afforded to Journalists and Their Sources. – Nothing in this Act shall be construed as to have amended or repealed the provisions of Republic Act No. 53, which affords the publishers, editors or duly accredited reporters of any newspaper, magazine or periodical of general circulation protection from being compelled to reveal the source of any news report or information appearing in said publication which was related in any confidence to such publisher, editor, or reporter.

SEC. 6. Extraterritorial Application. – This Act applies to an act done or practice engaged in and outside of the Philippines by an entity if:

(a) The act, practice or processing relates to personal information about a Philippine citizen or a resident;

(b) The entity has a link with the Philippines, and the entity is processing personal information in the Philippines or even if the processing is outside the Philippines as long as it is about Philippine citizens or residents such as, but not limited to, the following:

(1) A contract is entered in the Philippines;

(2) A juridical entity unincorporated in the Philippines but has central management and control in the country; and

(3) An entity that has a branch, agency, office or subsidiary in the Philippines and the parent or affiliate of the Philippine entity has access to personal information; and

(1) The entity carries on business in the Philippines; and

(2) The personal information was collected or held by an entity in the Philippines. Back To Top

CHAPTER II THE NATIONAL PRIVACY COMMISSION

SEC. 7. Functions of the National Privacy Commission. – To administer and implement the provisions of this Act, and to monitor and ensure compliance of the country with international standards set for data protection, there is hereby created an independent body to be known as the National Privacy Commission, winch shall have the following functions:

(a) Ensure compliance of personal information controllers with the provisions of this Act;

(b) Receive complaints, institute investigations, facilitate or enable settlement of complaints through the use of alternative dispute resolution processes, adjudicate, award indemnity on matters affecting any personal information, prepare reports on disposition of complaints and resolution of any investigation it initiates, and, in cases it deems appropriate, publicize any such report: Provided, That in resolving any complaint or investigation (except where amicable settlement is reached by the parties), the Commission shall act as a collegial body. For this purpose, the Commission may be given access to personal information that is subject of any complaint and to collect the information necessary to perform its functions under this Act;

(c) Issue cease and desist orders, impose a temporary or permanent ban on the processing of personal information, upon finding that the processing will be detrimental to national security and public interest;

(d) Compel or petition any entity, government agency or instrumentality to abide by its orders or take action on a matter affecting data privacy;

(e) Monitor the compliance of other government agencies or instrumentalities on their security and technical measures and recommend the necessary action in order to meet minimum standards for protection of personal information pursuant to this Act;

(f) Coordinate with other government agencies and the private sector on efforts to formulate and implement plans and policies to strengthen the protection of personal information in the country;

(g) Publish on a regular basis a guide to all laws relating to data protection;

(h) Publish a compilation of agency system of records and notices, including index and other finding aids;

(i) Recommend to the Department of Justice (DOJ) the prosecution and imposition of penalties specified in Sections 25 to 29 of this Act;

(j) Review, approve, reject or require modification of privacy codes voluntarily adhered to by personal information controllers:Provided, That the privacy codes shall adhere to the underlying data privacy principles embodied in this Act: Provided, further,That such privacy codes may include private dispute resolution mechanisms for complaints against any participating personal information controller. For this purpose, the Commission shall consult with relevant regulatory agencies in the formulation and administration of privacy codes applying the standards set out in this Act, with respect to the persons, entities, business activities and business sectors that said regulatory bodies are authorized to principally regulate pursuant to the law: Provided, finally. That the Commission may review such privacy codes and require changes thereto for purposes of complying with this Act;

(k) Provide assistance on matters relating to privacy or data protection at the request of a national or local agency, a private entity or any person;

(l) Comment on the implication on data privacy of proposed national or local statutes, regulations or procedures, issue advisory opinions and interpret the provisions of this Act and other data privacy laws;

(m) Propose legislation, amendments or modifications to Philippine laws on privacy or data protection as may be necessary;

(n) Ensure proper and effective coordination with data privacy regulators in other countries and private accountability agents, participate in international and regional initiatives for data privacy protection;

(o) Negotiate and contract with other data privacy authorities of other countries for cross-border application and implementation of respective privacy laws;

(p) Assist Philippine companies doing business abroad to respond to foreign privacy or data protection laws and regulations; and

SEC. 8. Confidentiality. – The Commission shall ensure at all times the confidentiality of any personal information that comes to its knowledge and possession.

SEC. 9. Organizational Structure of the Commission. – The Commission shall be attached to the Department of Information and Communications Technology (DICT) and shall be headed by a Privacy Commissioner, who shall also act as Chairman of the Commission. The Privacy Commissioner shall be assisted by two (2) Deputy Privacy Commissioners, one to be responsible for Data Processing Systems and one to be responsible for Policies and Planning. The Privacy Commissioner and the two (2) Deputy Privacy Commissioners shall be appointed by the President of the Philippines for a term of three (3) years, and may be reappointed for another term of three (3) years. Vacancies in the Commission shall be filled in the same manner in which the original appointment was made.

The Privacy Commissioner must be at least thirty-five (35) years of age and of good moral character, unquestionable integrity and known probity, and a recognized expert in the field of information technology and data privacy. The Privacy Commissioner shall enjoy the benefits, privileges and emoluments equivalent to the rank of Secretary.

The Deputy Privacy Commissioners must be recognized experts in the field of information and communications technology and data privacy. They shall enjoy the benefits, privileges and emoluments equivalent to the rank of Undersecretary.

The Privacy Commissioner, the Deputy Commissioners, or any person acting on their behalf or under their direction, shall not be civilly liable for acts done in good faith in the performance of their duties. However, he or she shall be liable for willful or negligent acts done by him or her which are contrary to law, morals, public policy and good customs even if he or she acted under orders or instructions of superiors: Provided, That in case a lawsuit is filed against such official on the subject of the performance of his or her duties, where such performance is lawful, he or she shall be reimbursed by the Commission for reasonable costs of litigation.

SEC. 10. The Secretariat. – The Commission is hereby authorized to establish a Secretariat. Majority of the members of the Secretariat must have served for at least five (5) years in any agency of the government that is involved in the processing of personal information including, but not limited to, the following offices: Social Security System (SSS), Government Service Insurance System (GSIS), Land Transportation Office (LTO), Bureau of Internal Revenue (BIR), Philippine Health Insurance Corporation (PhilHealth), Commission on Elections (COMELEC), Department of Foreign Affairs (DFA), Department of Justice (DOJ), and Philippine Postal Corporation (Philpost). Back To Top

CHAPTER III (q) Generally perform such acts as may be necessary to facilitate crossborder enforcement of data privacy protection.

PROCESSING OF PERSONAL INFORMATION

SEC. 11. General Data Privacy Principles. – The processing of personal information shall be allowed, subject to compliance with the requirements of this Act and other laws allowing disclosure of

information to the public and adherence to the principles of transparency, legitimate purpose and proportionality.

(d) The processing is necessary to protect vitally important interests of the data subject, including life and health;

Personal information must, be:,

(e) The processing is necessary in order to respond to national emergency, to comply with the requirements of public order and safety, or to fulfill functions of public authority which necessarily includes the processing of personal data for the fulfillment of its mandate; or

(a) Collected for specified and legitimate purposes determined and declared before, or as soon as reasonably practicable after collection, and later processed in a way compatible with such declared, specified and legitimate purposes only;

(b) Processed fairly and lawfully;

(f) The processing is necessary for the purposes of the legitimate interests pursued by the personal information controller or by a third party or parties to whom the data is disclosed, except where such interests are overridden by fundamental rights and freedoms of the data subject which require protection under the Philippine Constitution.

(c) Accurate, relevant and, where necessary for purposes for which it is to be used the processing of personal information, kept up to date; inaccurate or incomplete data must be rectified, supplemented, destroyed or their further processing restricted;

SEC. 13. Sensitive Personal Information and Privileged Information. – The processing of sensitive personal information and privileged information shall be prohibited, except in the following cases:

(d) Adequate and not excessive in relation to the purposes for which they are collected and processed;

(a) The data subject has given his or her consent, specific to the purpose prior to the processing, or in the case of privileged information, all parties to the exchange have given their consent prior to processing;

(e) Retained only for as long as necessary for the fulfillment of the purposes for which the data was obtained or for the establishment, exercise or defense of legal claims, or for legitimate business purposes, or as provided by law; and

(f) Kept in a form which permits identification of data subjects for no longer than is necessary for the purposes for which the data were collected and processed: Provided, That personal information collected for other purposes may lie processed for historical, statistical or scientific purposes, and in cases laid down in law may be stored for longer periods: Provided, further,That adequate safeguards are guaranteed by said laws authorizing their processing.

The personal information controller must ensure implementation of personal information processing principles set out herein.

SEC. 12. Criteria for Lawful Processing of Personal Information. – The processing of personal information shall be permitted only if not otherwise prohibited by law, and when at least one of the following conditions exists:

(a) The data subject has given his or her consent;

(b) The processing of personal information is necessary and is related to the fulfillment of a contract with the data subject or in order to take steps at the request of the data subject prior to entering into a contract;

(c) The processing is necessary for compliance with a legal obligation to which the personal information controller is subject;

(b) The processing of the same is provided for by existing laws and regulations: Provided, That such regulatory enactments guarantee the protection of the sensitive personal information and the privileged information: Provided, further, That the consent of the data subjects are not required by law or regulation permitting the processing of the sensitive personal information or the privileged information;

(c) The processing is necessary to protect the life and health of the data subject or another person, and the data subject is not legally or physically able to express his or her consent prior to the processing;

(d) The processing is necessary to achieve the lawful and noncommercial objectives of public organizations and their associations: Provided, That such processing is only confined and related to the bona fide members of these organizations or their associations: Provided, further, That the sensitive personal information are not transferred to third parties: Provided, finally, That consent of the data subject was obtained prior to processing;

(e) The processing is necessary for purposes of medical treatment, is carried out by a medical practitioner or a medical treatment institution, and an adequate level of protection of personal information is ensured; or

(f) The processing concerns such personal information as is necessary for the protection of lawful rights and interests of natural or legal persons in court proceedings, or the establishment, exercise or defense of legal claims, or when provided to government or public authority.

SEC. 14. Subcontract of Personal Information. – A personal information controller may subcontract the processing of personal information: Provided, That the personal information controller shall

be responsible for ensuring that proper safeguards are in place to ensure the confidentiality of the personal information processed, prevent its use for unauthorized purposes, and generally, comply with the requirements of this Act and other laws for processing of personal information. The personal information processor shall comply with all the requirements of this Act and other applicable laws.

SEC. 15. Extension of Privileged Communication. – Personal information controllers may invoke the principle of privileged communication over privileged information that they lawfully control or process. Subject to existing laws and regulations, any evidence gathered on privileged information is inadmissible.

Any information supplied or declaration made to the data subject on these matters shall not be amended without prior notification of data subject: Provided, That the notification under subsection (b) shall not apply should the personal information be needed pursuant to a subpoena or when the collection and processing are for obvious purposes, including when it is necessary for the performance of or in relation to a contract or service or when necessary or desirable in the context of an employer-employee relationship, between the collector and the data subject, or when the information is being collected and processed as a result of legal obligation;

(c) Reasonable access to, upon demand, the following:

Back To Top (1) Contents of his or her personal information that were processed; CHAPTER IV RIGHTS OF THE DATA SUBJECT

(2) Sources from which personal information were obtained;

SEC. 16. Rights of the Data Subject. – The data subject is entitled to:

(3) Names and addresses of recipients of the personal information;

(a) Be informed whether personal information pertaining to him or her shall be, are being or have been processed;

(4) Manner by which such data were processed;

(5) Reasons for the disclosure of the personal information to recipients; (b) Be furnished the information indicated hereunder before the entry of his or her personal information into the processing system of the personal information controller, or at the next practical opportunity:

(6) Information on automated processes where the data will or likely to be made as the sole basis for any decision significantly affecting or will affect the data subject;

(1) Description of the personal information to be entered into the system; (7) Date when his or her personal information concerning the data subject were last accessed and modified; and (2) Purposes for which they are being or are to be processed;

(3) Scope and method of the personal information processing;

(4) The recipients or classes of recipients to whom they are or may be disclosed;

(5) Methods utilized for automated access, if the same is allowed by the data subject, and the extent to which such access is authorized;

(6) The identity and contact details of the personal information controller or its representative;

(7) The period for which the information will be stored; and

(8) The existence of their rights, i.e., to access, correction, as well as the right to lodge a complaint before the Commission.

(8) The designation, or name or identity and address of the personal information controller;

(d) Dispute the inaccuracy or error in the personal information and have the personal information controller correct it immediately and accordingly, unless the request is vexatious or otherwise unreasonable. If the personal information have been corrected, the personal information controller shall ensure the accessibility of both the new and the retracted information and the simultaneous receipt of the new and the retracted information by recipients thereof: Provided, That the third parties who have previously received such processed personal information shall he informed of its inaccuracy and its rectification upon reasonable request of the data subject;

(e) Suspend, withdraw or order the blocking, removal or destruction of his or her personal information from the personal information controller’s filing system upon discovery and substantial proof that the personal information are incomplete, outdated, false, unlawfully obtained, used for unauthorized purposes or are no longer necessary for the purposes for which they were collected. In this case, the personal information controller may notify third parties who have previously received such processed personal information; and

(f) Be indemnified for any damages sustained due to such inaccurate, incomplete, outdated, false, unlawfully obtained or unauthorized use of personal information.

(1) Safeguards to protect its computer network against accidental, unlawful or unauthorized usage or interference with or hindering of their functioning or availability;

SEC. 17. Transmissibility of Rights of the Data Subject. – The lawful heirs and assigns of the data subject may invoke the rights of the data subject for, which he or she is an heir or assignee at any time after the death of the data subject or when the data subject is incapacitated or incapable of exercising the rights as enumerated in the immediately preceding section.

(2) A security policy with respect to the processing of personal information;

SEC. 18. Right to Data Portability. – The data subject shall have the right, where personal information is processed by electronic means and in a structured and commonly used format, to obtain from the personal information controller a copy of data undergoing processing in an electronic or structured format, which is commonly used and allows for further use by the data subject. The Commission may specify the electronic format referred to above, as well as the technical standards, modalities and procedures for their transfer.

SEC. 19. Non-Applicability. – The immediately preceding sections are not applicable if the processed personal information are used only for the needs of scientific and statistical research and, on the basis of such, no activities are carried out and no decisions are taken regarding the data subject: Provided, That the personal information shall be held under strict confidentiality and shall be used only for the declared purpose. Likewise, the immediately preceding sections are not applicable to processing of personal information gathered for the purpose of investigations in relation to any criminal, administrative or tax liabilities of a data subject.

(3) A process for identifying and accessing reasonably foreseeable vulnerabilities in its computer networks, and for taking preventive, corrective and mitigating action against security incidents that can lead to a security breach; and

(4) Regular monitoring for security breaches and a process for taking preventive, corrective and mitigating action against security incidents that can lead to a security breach.

(d) The personal information controller must further ensure that third parties processing personal information on its behalf shall implement the security measures required by this provision.

(e) The employees, agents or representatives of a personal information controller who are involved in the processing of personal information shall operate and hold personal information under strict confidentiality if the personal information are not intended for public disclosure. This obligation shall continue even after leaving the public service, transfer to another position or upon termination of employment or contractual relations.

Back To Top

CHAPTER V SECURITY OF PERSONAL INFORMATION

SEC. 20. Security of Personal Information. – (a) The personal information controller must implement reasonable and appropriate organizational, physical and technical measures intended for the protection of personal information against any accidental or unlawful destruction, alteration and disclosure, as well as against any other unlawful processing.

(b) The personal information controller shall implement reasonable and appropriate measures to protect personal information against natural dangers such as accidental loss or destruction, and human dangers such as unlawful access, fraudulent misuse, unlawful destruction, alteration and contamination.

(c) The determination of the appropriate level of security under this section must take into account the nature of the personal information to be protected, the risks represented by the processing, the size of the organization and complexity of its operations, current data privacy best practices and the cost of security implementation. Subject to guidelines as the Commission may issue from time to time, the measures implemented must include:

(f) The personal information controller shall promptly notify the Commission and affected data subjects when sensitive personal information or other information that may, under the circumstances, be used to enable identity fraud are reasonably believed to have been acquired by an unauthorized person, and the personal information controller or the Commission believes (bat such unauthorized acquisition is likely to give rise to a real risk of serious harm to any affected data subject. The notification shall at least describe the nature of the breach, the sensitive personal information possibly involved, and the measures taken by the entity to address the breach. Notification may be delayed only to the extent necessary to determine the scope of the breach, to prevent further disclosures, or to restore reasonable integrity to the information and communications system.

(1) In evaluating if notification is unwarranted, the Commission may take into account compliance by the personal information controller with this section and existence of good faith in the acquisition of personal information.

(2) The Commission may exempt a personal information controller from notification where, in its reasonable judgment, such notification would not be in the public interest or in the interests of the affected data subjects.

(3) The Commission may authorize postponement of notification where it may hinder the progress of a criminal investigation related to a serious breach. Back To Top

CHAPTER VI ACCOUNTABILITY INFORMATION

FOR

TRANSFER

OF

PERSONAL

SEC. 21. Principle of Accountability. – Each personal information controller is responsible for personal information under its control or custody, including information that have been transferred to a third party for processing, whether domestically or internationally, subject to cross-border arrangement and cooperation.

(a) The personal information controller is accountable for complying with the requirements of this Act and shall use contractual or other reasonable means to provide a comparable level of protection while the information are being processed by a third party.

(b) The personal information controller shall designate an individual or individuals who are accountable for the organization’s compliance with this Act. The identity of the individual(s) so designated shall be made known to any data subject upon request.

is submitted and approved by the head of the agency in accordance with the following guidelines:

(1) Deadline for Approval or Disapproval – In the case of any request submitted to the head of an agency, such head of the agency shall approve or disapprove the request within two (2) business days after the date of submission of the request. In case there is no action by the head of the agency, then such request is considered disapproved;

(2) Limitation to One thousand (1,000) Records – If a request is approved, the head of the agency shall limit the access to not more than one thousand (1,000) records at a time; and

(3) Encryption – Any technology used to store, transport or access sensitive personal information for purposes of off-site access approved under this subsection shall be secured by the use of the most secure encryption standard recognized by the Commission.

The requirements of this subsection shall be implemented not later than six (6) months after the date of the enactment of this Act.

CHAPTER VII

SEC. 24. Applicability to Government Contractors. – In entering into any contract that may involve accessing or requiring sensitive personal information from one thousand (1,000) or more individuals, an agency shall require a contractor and its employees to register their personal information processing system with the Commission in accordance with this Act and to comply with the other provisions of this Act including the immediately preceding section, in the same manner as agencies and government employees comply with such requirements.

SECURITY OF SENSITIVE PERSONAL

Back To Top

Back To Top

INFORMATION IN GOVERNMENT CHAPTER VIII SEC. 22. Responsibility of Heads of Agencies. – All sensitive personal information maintained by the government, its agencies and instrumentalities shall be secured, as far as practicable, with the use of the most appropriate standard recognized by the information and communications technology industry, and as recommended by the Commission. The head of each government agency or instrumentality shall be responsible for complying with the security requirements mentioned herein while the Commission shall monitor the compliance and may recommend the necessary action in order to satisfy the minimum standards.

SEC. 23. Requirements Relating to Access by Agency Personnel to Sensitive Personal Information. – (a) On-site and Online Access – Except as may be allowed through guidelines to be issued by the Commission, no employee of the government shall have access to sensitive personal information on government property or through online facilities unless the employee has received a security clearance from the head of the source agency.

(b) Off-site Access – Unless otherwise provided in guidelines to be issued by the Commission, sensitive personal information maintained by an agency may not be transported or accessed from a location off government property unless a request for such transportation or access

PENALTIES

SEC. 25. Unauthorized Processing of Personal Information and Sensitive Personal Information. – (a) The unauthorized processing of personal information shall be penalized by imprisonment ranging from one (1) year to three (3) years and a fine of not less than Five hundred thousand pesos (Php500,000.00) but not more than Two million pesos (Php2,000,000.00) shall be imposed on persons who process personal information without the consent of the data subject, or without being authorized under this Act or any existing law.

(b) The unauthorized processing of personal sensitive information shall be penalized by imprisonment ranging from three (3) years to six (6) years and a fine of not less than Five hundred thousand pesos (Php500,000.00) but not more than Four million pesos (Php4,000,000.00) shall be imposed on persons who process personal information without the consent of the data subject, or without being authorized under this Act or any existing law.

SEC. 26. Accessing Personal Information and Sensitive Personal Information Due to Negligence. – (a) Accessing personal information due to negligence shall be penalized by imprisonment ranging from

one (1) year to three (3) years and a fine of not less than Five hundred thousand pesos (Php500,000.00) but not more than Two million pesos (Php2,000,000.00) shall be imposed on persons who, due to negligence, provided access to personal information without being authorized under this Act or any existing law.

(b) Accessing sensitive personal information due to negligence shall be penalized by imprisonment ranging from three (3) years to six (6) years and a fine of not less than Five hundred thousand pesos (Php500,000.00) but not more than Four million pesos (Php4,000,000.00) shall be imposed on persons who, due to negligence, provided access to personal information without being authorized under this Act or any existing law.

SEC. 27. Improper Disposal of Personal Information and Sensitive Personal Information. – (a) The improper disposal of personal information shall be penalized by imprisonment ranging from six (6) months to two (2) years and a fine of not less than One hundred thousand pesos (Php100,000.00) but not more than Five hundred thousand pesos (Php500,000.00) shall be imposed on persons who knowingly or negligently dispose, discard or abandon the personal information of an individual in an area accessible to the public or has otherwise placed the personal information of an individual in its container for trash collection.

(b) The improper disposal of sensitive personal information shall be penalized by imprisonment ranging from one (1) year to three (3) years and a fine of not less than One hundred thousand pesos (Php100,000.00) but not more than One million pesos (Php1,000,000.00) shall be imposed on persons who knowingly or negligently dispose, discard or abandon the personal information of an individual in an area accessible to the public or has otherwise placed the personal information of an individual in its container for trash collection.

SEC. 28. Processing of Personal Information and Sensitive Personal Information for Unauthorized Purposes. – The processing of personal information for unauthorized purposes shall be penalized by imprisonment ranging from one (1) year and six (6) months to five (5) years and a fine of not less than Five hundred thousand pesos (Php500,000.00) but not more than One million pesos (Php1,000,000.00) shall be imposed on persons processing personal information for purposes not authorized by the data subject, or otherwise authorized under this Act or under existing laws.

The processing of sensitive personal information for unauthorized purposes shall be penalized by imprisonment ranging from two (2) years to seven (7) years and a fine of not less than Five hundred thousand pesos (Php500,000.00) but not more than Two million pesos (Php2,000,000.00) shall be imposed on persons processing sensitive personal information for purposes not authorized by the data subject, or otherwise authorized under this Act or under existing laws.

SEC. 29. Unauthorized Access or Intentional Breach. – The penalty of imprisonment ranging from one (1) year to three (3) years and a fine of not less than Five hundred thousand pesos (Php500,000.00) but not more than Two million pesos (Php2,000,000.00) shall be imposed on persons who knowingly and unlawfully, or violating data confidentiality and security data systems, breaks in any way into any system where personal and sensitive personal information is stored.

SEC. 30. Concealment of Security Breaches Involving Sensitive Personal Information. – The penalty of imprisonment of one (1) year and six (6) months to five (5) years and a fine of not less than Five hundred thousand pesos (Php500,000.00) but not more than One million pesos (Php1,000,000.00) shall be imposed on persons who, after having knowledge of a security breach and of the obligation to notify the Commission pursuant to Section 20(f), intentionally or by omission conceals the fact of such security breach.

SEC. 31. Malicious Disclosure. – Any personal information controller or personal information processor or any of its officials, employees or agents, who, with malice or in bad faith, discloses unwarranted or false information relative to any personal information or personal sensitive information obtained by him or her, shall be subject to imprisonment ranging from one (1) year and six (6) months to five (5) years and a fine of not less than Five hundred thousand pesos (Php500,000.00) but not more than One million pesos (Php1,000,000.00).

SEC. 32. Unauthorized Disclosure. – (a) Any personal information controller or personal information processor or any of its officials, employees or agents, who discloses to a third party personal information not covered by the immediately preceding section without the consent of the data subject, shall he subject to imprisonment ranging from one (1) year to three (3) years and a fine of not less than Five hundred thousand pesos (Php500,000.00) but not more than One million pesos (Php1,000,000.00).

(b) Any personal information controller or personal information processor or any of its officials, employees or agents, who discloses to a third party sensitive personal information not covered by the immediately preceding section without the consent of the data subject, shall be subject to imprisonment ranging from three (3) years to five (5) years and a fine of not less than Five hundred thousand pesos (Php500,000.00) but not more than Two million pesos (Php2,000,000.00).

SEC. 33. Combination or Series of Acts. – Any combination or series of acts as defined in Sections 25 to 32 shall make the person subject to imprisonment ranging from three (3) years to six (6) years and a fine of not less than One million pesos (Php1,000,000.00) but not more than Five million pesos (Php5,000,000.00).

SEC. 34. Extent of Liability. – If the offender is a corporation, partnership or any juridical person, the penalty shall be imposed upon the responsible officers, as the case may be, who participated in, or by their gross negligence, allowed the commission of the crime. If the offender is a juridical person, the court may suspend or revoke any of its rights under this Act. If the offender is an alien, he or she shall, in addition to the penalties herein prescribed, be deported without further proceedings after serving the penalties prescribed. If the offender is a public official or employee and lie or she is found guilty of acts penalized under Sections 27 and 28 of this Act, he or she shall, in addition to the penalties prescribed herein, suffer perpetual or temporary absolute disqualification from office, as the case may be.

SEC. 35. Large-Scale. – The maximum penalty in the scale of penalties respectively provided for the preceding offenses shall be imposed when the personal information of at least one hundred (100) persons is

harmed, affected or involved as the result of the above mentioned actions. SEC. 36. Offense Committed by Public Officer. – When the offender or the person responsible for the offense is a public officer as defined in the Administrative Code of the Philippines in the exercise of his or her duties, an accessory penalty consisting in the disqualification to occupy public office for a term double the term of criminal penalty imposed shall he applied. SEC. 37. Restitution. – Restitution for any aggrieved party shall be governed by the provisions of the New Civil Code. BP880 AN ACT ENSURING THE FREE EXERCISE BY THE PEOPLE OF THEIR RIGHT PEACEABLY TO ASSEMBLE AND PETITION THE GOVERNMENT FOR OTHER PURPOSES

Section 1.Title - This Act shall be known as "The Public Assembly Act of 1985."

Section 2.Declaration of policy - The constitutional right of the people peaceably to assemble and petition the government for redress of grievances is essential and vital to the strength and stability of the State. To this end, the State shall ensure the free exercise of such right without prejudice to the rights of others to life, liberty and equal protection of the law.

Section 3.Definition of terms - For purposes of this Act:

(a) "Public assembly" means any rally, demonstration, march, parade, procession or any other form of mass or concerted action held in a public place for the purpose of presenting a lawful cause; or expressing an opinion to the general public on any particular issue; or protesting or influencing any state of affairs whether political, economic or social; or petitioning the government for redress of grievances.

The processions, rallies, parades, demonstrations, public meetings and assemblages for religious purposes shall be governed by local ordinances: Provided, however, That the declaration of policy as provided in Section 2 of this Act shall be faithfully observed.

(d) "Modification of permit" shall include the change of the place and time of the public assembly, rerouting of the parade or street march, the volume of loud-speakers or sound system and similar changes.

Section 4.Permit when required and when not required - A written permit shall be required for any person or persons to organize and hold a public assembly in a public place. However, no permit shall be required if the public assembly shall be done or made in a freedom park duly established by law or ordinance or in private property, in which case only the consent of the owner or the one entitled to its legal possession is required, or in the campus of a government-owned and operated educational institution which shall be subject to the rules and regulations of said educational institution. Political meetings or rallies held during any election campaign period as provided for by law are not covered by this Act.

Section 5.Application requirements - All applications for a permit shall comply with the following guidelines:

(a) The applications shall be in writing and shall include the names of the leaders or organizers; the purpose of such public assembly; the date, time and duration thereof, and place or streets to be used for the intended activity; and the probable number of persons participating, the transport and the public address systems to be used.

(b) The application shall incorporate the duty and responsibility of applicant under Section 8 hereof.

(c) The application shall be filed with the office of the mayor of the city or municipality in whose jurisdiction the intended activity is to be held, at least five (5) working days before the scheduled public assembly.

(d) Upon receipt of the application, which must be duly acknowledged in writing, the office of the city or municipal mayor shall cause the same to immediately be posted at a conspicuous place in the city or municipal building.

Section 6.Action to be taken on the application The definition herein contained shall not include picketing and other concerted action in strike areas by workers and employees resulting from a labor dispute as defined by the Labor Code, its implementing rules and regulations, and by the Batas Pambansa Bilang 227.

(b) "Public place" shall include any highway, boulevard, avenue, road, street, bridge or other thoroughfare, park, plaza, square, and/or any open space of public ownership where the people are allowed access.

(c) "Maximum tolerance" means the highest degree of restraint that the military, police and other peace keeping authorities shall observe during a public assembly or in the dispersal of the same.

(a) It shall be the duty of the mayor or any official acting in his behalf to issue or grant a permit unless there is clear and convincing evidence that the public assembly will create a clear and present danger to public order, public safety, public convenience, public morals or public health.

(b) The mayor or any official acting in his behalf shall act on the application within two (2) working days from the date the application was filed, failing which, the permit shall be deemed granted. Should for any reason the mayor or any official acting in his behalf refuse to accept the application for a permit, said application shall be posted by the applicant on the premises of the office of the mayor and shall be deemed to have been filed.

(c) To confer with local government officials concerned and law enforcers to the end that the public assembly may be held peacefully; (c) If the mayor is of the view that there is imminent and grave danger of a substantive evil warranting the denial or modification of the permit, he shall immediately inform the applicant who must be heard on the matter.

(d) The action on the permit shall be in writing and served on the application within twenty-four hours.

(e) If the mayor or any official acting in his behalf denies the application or modifies the terms thereof in his permit, the applicant may contest the decision in an appropriate court of law.

(f) In case suit is brought before the Metropolitan Trial Court, the Municipal Trial Court, the Municipal Circuit Trial Court, the Regional Trial Court, or the Intermediate Appellate Court, its decisions may be appealed to the appropriate court within forty-eight (48) hours after receipt of the same. No appeal bond and record on appeal shall be required. A decision granting such permit or modifying it in terms satisfactory to the applicant shall, be immediately executory.

(g) All cases filed in court under this Section shall be decided within twenty-four (24) hours from date of filing. Cases filed hereunder shall be immediately endorsed to the executive judge for disposition or, in his absence, to the next in rank.

(h) In all cases, any decision may be appealed to the Supreme Court.

(i) Telegraphic appeals to be followed by formal appeals are hereby allowed.

Section 7.Use of public thoroughfare - Should the proposed public assembly involve the use, for an appreciable length of time, of any public highway, boulevard, avenue, road or street, the mayor or any official acting in his behalf may, to prevent grave public inconvenience, designate the route thereof which is convenient to the participants or reroute the vehicular traffic to another direction so that there will be no serious or undue interference with the free flow of commerce and trade.

Section 8.Responsibility of applicant - It shall be the duty and responsibility of the leaders and organizers of a public assembly to take all reasonable measures and steps to the end that the intended public assembly shall be conducted peacefully in accordance with the terms of the permit. These shall include but not be limited to the following:

(a) To inform the participants of their responsibility under the permit;

(b) To police the ranks of the demonstrators in order to prevent nondemonstrators from disrupting the lawful activities of the public assembly;

(d) To see to it that the public assembly undertaken shall not go beyond the time stated in the permit; and

(e) To take positive steps that demonstrators do not molest any person or do any act unduly interfering with the rights of other persons not participating in the public assembly.

Section 9.Non-interference by law enforcement authorities - Law enforcement agencies shall not interfere with the holding of a public assembly. However, to adequately ensure public safety, a law enforcement contingent under the command of a responsible police officer may be detailed and stationed in a place at least one hundred (100) meter away from the area of activity ready to maintain peace and order at all times.

Section 10. Police assistance when requested - It shall be imperative for law enforcement agencies, when their assistance is requested by the leaders or organizers, to perform their duties always mindful that their responsibility to provide proper protection to those exercising their right peaceably to assemble and the freedom of expression is primordial. Towards this end, law enforcement agencies shall observe the following guidelines:

(a) Members of the law enforcement contingent who deal with the demonstrators shall be in complete uniform with their nameplates and units to which they belong displayed prominently on the front and dorsal parts of their uniform and must observe the policy of "maximum tolerance" as herein defined;

(b) The members of the law enforcement contingent shall not carry any kind of firearms but may be equipped with baton or riot sticks, shields, crash helmets with visor, gas masks, boots or ankle high shoes with shin guards;

(c) Tear gas, smoke grenades, water cannons, or any similar anti-riot device shall not be used unless the public assembly is attended by actual violence or serious threats of violence, or deliberate destruction of property.

Section 11. Dispersal of public assembly with permit - No public assembly with a permit shall be dispersed. However, when an assembly becomes violent, the police may disperse such public assembly as follows:

(a) At the first sign of impending violence, the ranking officer of the law enforcement contingent shall call the attention of the leaders of the public assembly and ask the latter to prevent any possible disturbance;

(b) If actual violence starts to a point where rocks or other harmful objects from the participants are thrown at the police or at the nonparticipants, or at any property causing damage to such property, the

ranking officer of the law enforcement contingent shall audibly warn the participants that if the disturbance persists, the public assembly will be dispersed;

(c) If the violence or disturbances prevailing as stated in the preceding subparagraph should not stop or abate, the ranking officer of the law enforcement contingent shall audibly issue a warning to the participants of the public assembly, and after allowing a reasonable period of time to lapse, shall immediately order it to forthwith disperse;

3 the malicious burning of any object in the streets or thoroughfares;

4. the carrying of firearms by members of the law enforcement unit;

5. the interfering with or intentionally disturbing the holding of a public assembly by the use of a motor vehicle, its horns and loud sound systems.

(d) No arrest of any leader, organizer or participant shall also be made during the public assembly unless he violates during the assembly a law, statute, ordinance or any provision of this Act. Such arrest shall be governed by Article 125 of the Revised Penal Code, as amended:

Section 14. Penalties - Any person found guilty and convicted of any of the prohibited acts defined in the immediately preceding Section shall be punished as follows:

(e) Isolated acts or incidents of disorder or branch of the peace during the public assembly shall not constitute a group for dispersal.

(a) violation of subparagraph (a) shall be punished by imprisonment of one month and one day to six months;

Section 12. Dispersal of public assembly without permit When the public assembly is held without a permit where a permit is required, the said public assembly may be peacefully dispersed.

(b) violations of subparagraphs (b), (c), (d), (e), (f), and item 4, subparagraph (g) shall be punished by imprisonment of six months and one day to six years;

Section 13. Prohibited acts - The following shall constitute violations of this Act:

(c) violation of item 1, subparagraph (g) shall be punished by imprisonment of six months and one day to six years without prejudice to prosecution under Presidential Decree No. 1866;

(a) The holding of any public assembly as defined in this Act by any leader or organizer without having first secured that written permit where a permit is required from the office concerned, or the use of such permit for such purposes in any place other than those set out in said permit: Provided, however, That no person can be punished or held criminally liable for participating in or attending an otherwise peaceful assembly; (b) Arbitrary and unjustified denial or modification of a permit in violation of the provisions of this Act by the mayor or any other official acting in his behalf. (c) The unjustified and arbitrary refusal to accept or acknowledge receipt of the application for a permit by the mayor or any official acting in his behalf; (d) Obstructing, impeding, disrupting or otherwise denying the exercise of the right to peaceful assembly; (e) The unnecessary firing of firearms by a member of any law enforcement agency or any person to disperse the public assembly; (f) Acts in violation of Section 10 hereof;

(g) Acts described hereunder if committed within one hundred (100) meters from the area of activity of the public assembly or on the occasion thereof;

1. the carrying of a deadly or offensive weapon or device such as firearm, pillbox, bomb, and the like;

2. the carrying of a bladed weapon and the like;

(d) violations of item 2, item 3, or item 5 of subparagraph (g) shall be punished by imprisonment of one day to thirty days.

Section 15. Freedom parks - Every city and municipality in the country shall within six months after the effectivity of this Act establish or designate at least one suitable "freedom park" or mall in their respective jurisdictions which, as far as practicable, shall be centrally located within the poblacion where demonstrations and meetings may be held at any time without the need of any prior permit.

Related Documents


More Documents from ""